You are on page 1of 187

01.

Constitutional Development
of India
1. Regulating Act 1773 was the first step due to Ans. (a) Charter Act of 1813 asserted the Crown’s
which- sovereignty on British India controlled by East India
(a) Emphasized the British Parliament to make Company. Hence statement 2 is correct. Company’s
legislature for India rule and trade in India was extended for another twenty
(b) Separation of Executive from Legislature years. Hence statement 3 is incorrect. Charter Act of
(c) Separation of Executive from Judiciary 1813 ended the trade monopoly of the East India
(d) Centralization of legislation Company in India except for trade in tea and trade with
IAS (Pre.) G.S. 1993 China. Hence statement 1 is also correct. Charter Act of
Ans. (a) Regulating Act, 1773, a legislation passed by 1813 granted permission to the Missionaries to come to
the British Parliament for the regulation of the India. The 1813 Act provided for a financial grant
British East India Company’s Indian territories. It was towards the revival of Indian literature and the
the first intervention by the British Government in the promotion of science. The East India Company was also
company’s territorial affairs mainly due to East India to take up a greater role in the education of the Indians
Company’s misgovarnance in Bengal. The main under them. It was to set aside one lakh for this purpose.
provisions of the Act were the appointment of a 4. Which Act of British Government granted One
Governor-General of Fort William in Bengal with lakh for education in India for the first time?
supervisory powers over the presidencies of Madras and (a) Wood's dispatch 1854
Bombay. Under this Act, a Supreme Court of four (b) Charter Act 1813
British Judges (one Chief Justice and 3 other Judges) (c) Charter Act 1853
was set up in Calcutta in 1774. (d) Indian Council Act 1892
2. Under which of the following Act, Supreme UPPCS (Main) G.S. IInd 2009
Court was established in Calcutta? Ans. (b) Kindly refer the explanation of the above
(a) Regulating Act -1773 question.
(b) Pitts India Act, 1784
5. Which Act for the first time made it possible
(c) Charter Act, 1813 for Indians to take some share in the
(d) Charter Act, 1833 administration of their country?
UPPCS (Main) G.S. IInd 2010 (a) Charter Act, 1833
Ans. (a) Kindly refer the explanation of the above (b) Charter Act, 1853
question. (c) Government of India Act, 1858
3. Consider the following statements about 'the (d) Indian Councils Act, 1861
Charter Act of 1813': UPPCS (Pre.) G.S. 2012
1. It ended the trade monopoly of the East India Ans. (a) Charter Act of 1833 was the final step towards
Company in India except for trade in tea and
centralization in British India. This was also called the
trade with China.
Saint Helena Act, 1833. It made the Governor-General
2. It asserted the sovereignty of the British of Bengal as the Governor-General of India and granted
Crown over the Indian territories held by the him all civil and military powers. The Governor-
Company. General of India was given exclusive legislative powers
3. The revenues of India were now controlled for the entire British India. Under this Act, first Law
by the British Parliament. Commission was set up under the chairmanship of Lord
Which of the statements given above are Macaulay. The Charter Act of 1833 attempted to
correct? introduce a system of open competition for selection of
(a) 1 and 2 only (b) 2 and 3 only civil servants and stated that the Indians should not be
(c) 1 and 3 only (d) 1, 2 and 3 debarred from holding any place, office and
IAS (Pre) G.S. 2019 employment under the Company.
Indian Polity & Constitution 6 YCT
6. Which of the following is not the main then India’s Viceroy. Government of India Act, 1919
provisions of Morley-Minto Reform? introduced Diarchy into provincial administration in
(a) To increase the number of seats in Legislative India. A bicameral legislature was set up with two
Councils houses; Legislative Assembly and the Council of State
(b) Government majority in Imperial legislative Under this plan, responsibility for some key areas
Council devolved to the provincial level. This Act provided the
(c) Communal and separate electoral system establishment of a public service commission in India
(d) Elected members were not permitted at for the first time. The act also provided that after 10
Imperial legislative Council years, a statutory commission would be set up to study
the working of the government. Simon Commission was
MPPCS (Pre) G.S. 2017
set up under this Act. It also created an office of the
Ans. (d) The Indian Councils Act, 1909, commonly High Commissioner for India in London.
known as the Morley-Minto Reforms was an Act of
8. The Government of India Act of 1919 clearly
the Parliament of the United Kingdom that brought
defined
about a limited increase in the involvement of Indians in
the governance of British India. The main provisions of (a) The separation of power between the
the Morley-Minto Reforms are : judiciary and the legislature
(i) The number of elected members in the Imperial (b) The jurisdiction of the central and provincial
Legislative Council and the Provincial Legislative governments
Councils was increased. In the Provincial Councils, (c) The powers of the Secretary of State for India
non-official majority was introduced but since and the Viceroy
some of these non-officials were nominated and not (d) None of the above
elected, the overall non-elected majority remained IAS (Pre.) G.S. 2015
the same. Ans. (b) Kindly refer the explanation of the above
(ii) In the Imperial Legislative Council, of the total 68 question.
members, 36 were to be the officials and of the 32 9. Which of the following is/are the principal
non-officials, 5 were to be nominated. feature(s) of the Government of India Act, 1919?
(iii) The elected members were to be indirectly elected. 1. Introduction of diarchy in the executive
The local bodies were to elect an electoral college government of the provinces
which in turn would elect members of provincial 2. Introduction of separate communal
legislatures, who in turn would elect members of electorates for Muslims
the central legislature. 3. Devolution of legislative authority by the
(iv) The act introduced separate electorates for the centre to the provinces
Muslims, representation in excess of the strength of Select the correct answer using the codes given
their population was accorded to the Muslims. below :
(v) The Act legalized communalism and Lord Minto (a) 1 only (b) 2 and 3 only
came to be known as the 'Father of Communal (c) 1 and 3 only (d) 1, 2 and 3
Electorate'. IAS (Pre.) G.S. 2012
(v) Powers of legislatures, both at the centre and in
Ans. (c) Kindly refer the explanation of the above
provinces were enlarged and the legislatures could
question.
now pass resolutions (which may not be accepted),
ask questions and supplementaries, vote separate 10. ‘Diarchy’ was established in India by which of
items in the budget but the budget as a whole could the following?
not be voted upon. (a) The Indian Council Act, 1909
(vi) One Indian was to be appointed to the Viceroy’s (b) The Government of India Act, 1935
executive Council (Satyendra Sinha was the first to (c) The Government of India Act, 1919
be appointed as a law member in 1909). (d) None of the above
7. The Indian Legislature was made bi-cameral Maharashtra PSC (Pre) G.S 2013
for the first time by- Ans. (c) Kindly refer the explanation of the above
(a) Indian council Act of 1892 question.
(b) Indian council Act of 1909 11. The Govemment of India Act, 1919 was based
(c) The Government of India Act of 1919 upon:
(d) The Government of India Act of 1935 (a) Morley-Minto Reforms
Himanchal PSC (Pre) G.S 2010 (b) Montagu-Chelmsford Report
Ans. (c) The Government of India Act, 1919 was based (c) Ramsay MacDonald Award
on the recommendations of a report by Edwin Montagu, (d) Nehru Report
then Secretary of State for India and Lord Chelmsford, Haryana PSC (Pre) G.S. 2014
Indian Polity & Constitution 7 YCT
Ans. (b) Kindly refer the expalanation of the above (vii) Burma was separated from India with effect from
question. April 1937. The 1935 Act provided for the creation
12. Which of the following Acts introduced the ‘the of two new provinces of Sindh and Orissa.
Principle of Constitutional Autocracy’? 14. By which of the following Acts was the India
(a) The Indian Councils Act of 1909 Council abolished?
(b) The Government of India Act of 1919 (a) Morley Minto Reform 1909
(c) The Government of India Act of 1935 (b) The Government of India Act 1919
(d) The Indian Independence Act of 1947
(c) The Government of India Act 1935
MPPCS (Pre) G.S. 2017
(d) The Indian Independence Act 1947
Ans. (c) The Principle of ‘Constitutional Autocracy’
UPPSC RO/ARO (Pre) G.S. 2016
was introduced by the Act of 1935. By this Act, so
many provisions were introduced in the field of the (Held on 20/09/2020)
governance that made the whole governance system Ans. (c) Kindly refer the explanation of the above
relying on the Constitution and being directed by it. question.
13. Which among the following was not a salient 15. In the Federation established by the Govern-
feature of the Government of India Act 1935? ment of India Act of 1935, residuary powers
(a) It marked the beginning of the Provincial were given to the?
Autonomy (a) Federal Legislature
(b) The Act provided for an All India Federation
(b) Governor General
(c) It abolished Diarchy at the Provincial level
(c) Provincial Legislature
and introduced at the Centre
(d) Residuary subjects were allocated to (d) Provincial Governors
Provincial Legislatures IAS (Pre) G.S. 2018
RAS/RTS (Pre.) G.S. 2016 UPPCS (Main) G.S. IInd 2008
Ans. (d) The Government of India Act, 1935 Ans. (b) Kindly refer the explanation of the above
introduced federal system at the Centre. It abolished question.
India Council. Residually subject were given to Viceroy 16. Which one of the following is NOT a feature of
not to the Provincial legislature. Its salient features are
the Government of India Act of 1935?
as follows:
(a) Diarchy at the Centre as well as in the
(i) The India Act, 1935 proposed to set up All Indian
Federation, comprising of the British Indian provinces
Provinces and Princely States. The States were (b) A bicameral legislature
absolutely free to join or not to join the proposed (c) Provincial autonomy
Federation. (d) An All-India Federation
(ii) Diarchy was abolished in the provinces. Proper IAS (Pre.) G.S. 2000
division of subjects between the Centre and the
Ans. (a) Kindly refer the explanation of the above
Provinces under 1919 Act was revised and the 1935
question.
Act contained three lists namely Federal (Center),
Provincial and Concurrent (both for Center and 17. By which one of the following Acts was created
Province) legislative lists. The Viceroy was vested the Federal Court in India?
with residual powers which were to be exercised by (a) Indian Council Act, 1861
him through Governor General. (b) Government of India Act, 1909
(iii) The India Act, 1935 introduced Diarchy at the (c) Government of India Act, 1919
Centre. The Federal Subjects were divided into two
(d) None of the above
categories namely ‘reserved’ and ‘transferred’.
(iv) A very significant provision of the 1935 Act was UPPCS (Pre) G.S. 2014
the safeguards and protective amours for the Ans. (d) Kindly refer the explanation of the above
minorities. The Anglo-Indians and the Indo- question.
Christians were also given separate electorate. 18. Which of the following Acts provided for a
(v) The India Act, 1935 provided for the establishment federal polity in India?
of a Federal Court. (a) Government of India Act, 1909
(vi) The supremacy of the British Parliament remained
(b) Government of India Act, 1919
intact under the Government of India Act, 1935.
The Government of India Act, 1935 abolished the (c) Government of India Act, 1935
Council of the Secretary of State for India, which (d) India Independence Act, 1947
was created in 1858. UPUDA/LDA (Pre.) G.S. 2010
Indian Polity & Constitution 8 YCT
Ans. (c) Kindly refer the explanation of the above Ans. (a) The provision for the written Constitution for
question. India was first mentioned during Cripps Mission in
19. A “federal system” and “diarchy” at the March 1942.Rest three provisions were made in
“center “was introduced in India by Government of India Act, 1935.
(a) The Act of 1909 (b) The Act of 1919
(c) The Act of 1935 (d) None of the above 25. Which of the following is correct?
UPPCS (Main) G.S. IInd 2013 (a) The Nehru Report (1928) had advocated the
Ans. (c) Kindly refer the explanation of the above inclusion of Fundamental Rights in the
question. Constitution of India
20. The provision for the establishment of All (b) The Government of India Act, 1935 referred
India Federation was included in the to Fundamental Rights.
(a) Government of India Act, 1935 (c) The August Offer. 1940 included the
(b) August offer, 1940 Fundamental Rights.
(c) Government of India Act, 1919 (d) The Cripps Mission 1942 referred to
(d) Cabinet Mission Proposal, 1946 Fundamental Rights.
UPPCS (Main) G.S. IInd 2012
UPPCS (Main) G.S. IInd 2009
Ans. (a) Kindly refer the explanation of the above
question. Ans. (a) The Draft Constitution prepared by the
21. Which one of the following Acts led to the committee was called the Nehru Committee Report or
separation of Burma from India? Nehru Report. This committee was headed by Motilal
(a) The Indian Council Act, 1909 Nehru with Jawaharlal Nehru as the Secretary. Other
(b) Government of India Act, 1919 members were Ali Imam, Tej Bahadur Sapru, Mangal
(c) Government of India Act, 1935 Singh, M S Aney, Subhas Chandra Bose, Shuaib
(d) India Independence Act, 1947 Qureshi and G R Pradhan. The report was submitted at
UPPCS (Main) G.S. IInd 2011 the Lucknow session of the all-party conference on
Ans. (c) Kindly refer the explanation of the above August 28, 1928.This was the first major attempt by
question. Indians to draft a constitution for them.
22. The Provincial were constituted Act of Recommendations of the report were; dominion status
Governments under the for India (like Canada, Australia, etc.) within the British
(a) 1935 (b) 1932 Commonwealth, nineteen Fundamental Rights,
(c) 1936 (d) 1947
including right to vote for men and women above 21
BPSC (Pre.) 2011
years of age, no State religion, no separate electorates
Ans. (a) Kindly refer the explanation of the above
for any community etc.
question.
23. The distribution of powers between the Centre 26. The British Parliament passed the Indian
and the States in the Indian Constitution is Independence Act in :
based on the scheme provided in the (a) January1947 (b) June1947
(a) Morley-Minto Reforms, 1909 (c) July1947 (d) August 1947
(b) Montagu-Chelmsford Act, 1919 UPPCS (Main) G.S. IInd 2012
(c) Government of India Act, 1935
Ans. (c) The Parliament of the United Kingdom passed
(d) Indian Independence Act, 1947
IAS (Pre.) G.S. 2012 the ‘Indian Independence Act’ 1947, which was later
sanctioned by the British crown on July 18, 1947. The
Ans. (c) Kindly refer the explanation of the above
question. Act was one of the final formalities that had to be
24. Which one the following is not an important completed before the inevitable split of British India.
and permanent constituent in the This split gave rise to Pakistan on August 14 and to the
Constitutional History of India as adopted in dominion of India on August 15. This Act was the
the Government of India Act, 1935? direct result of the ‘3rd June Plan,’ which was proposed
(a) A written constitution for the country by then Viceroy, Lord Mountbatten.
(b) Elected representatives responsible to the 27. When was interim government formed under
Legislature
the leadership of Pandit Jawahar Lal Nehru?
(c) Envisaging a scheme of Federation
(d) Nomination of official members to the (a) July 1946 (b) August 1946
legislature (c) September 1946 (d) October 1946
UPPCS (Main) G.S. IInd 2010 UPPCS (Main) G.S. IInd 2006
Indian Polity & Constitution 9 YCT
Ans. (c) The Interim Government under the leadership A B C D
of Pandit Jawahar Lal Nehru was formed on September (a) III IV II I
02, 1946, from the newly elected Constituent Assembly. (b) IV III II I
Sixty Nine percent seats were allocated to Indian (c) III IV I II
National Congress. Other members of the Government
(d) IV II I III
were from the All India Muslim League, which had
initially refused to join the cabinet. While Muhammad Maharashtra PSC (Pre) G.S. 2017
Ali Jinnah was not a member of the cabinet, Liaquat Ali Ans. (a) For the first time separate representation to the
Khan, who went on to become Pakistan's first Prime Muslim community was provided by Indian Council
Minister was part of the Government and held the Act, 1909. Dyarchy at the Provinces was introduced by
portfolio of finance. Dr Rajendra Prasad held the the Government of India Act, 1919. Government of
Agriculture portfolio in the cabinet while Sardar India Act, 1935 made the provision for diarchy at
Vallabhai Patel was in charge of Home Affairs. Center while India Independence Act, 1947 made the
28. Match list I with list II and select the correct provision that the British Crown is no longer source of
answer from the code given below the list: authority in India.
List I List II 30. Who has termed Constitution a sacred
A. Indian Council 1. Beginningof document?
Act, 1909 Diarchy (a) B.R. Ambedkar
B. Indian Council 2. 1916 (b) Mahatma Gandhi
Act, 1935 (c) Deendayal Upadhaya
C. Lucknow Pact 3. Lord Minto
(d) Mohammad Ali Jinnah
D. Indian Council 4. Provencial
MPPCS (Pre.) G.S. 2015
Act, 1919 Autonomy
Ans. (a) Dr.B.R.Ambedkar termed Constitution as
Code:
sacred document. The Constituent Assembly on 29th
A B C D
August, 1947 formed a 7 member drafting committee in
(a) 1 2 3 4
the leadership of B.R. Ambedkar.
(b) 4 3 2 1
(c) 2 4 1 3 31. The first general elections under the Indian
(d) 3 4 2 1 constitution was held in
UP-UDA/LDA Spl. (Main) G.S. 2010 (a) 1949 (b) 1950
Ans. (d) Indian Council Act, 1909 is also known as (c) 1952 (d) 1953
Morley-Minto reform. Provincial autonomy was MPPCS (Pre.) G.S. 2015
provided by Indian Council Act, 1935. The Lucknow Ans. (c) The first general elections under Indian
Pact was an agreement reached between the Indian Constitution were held between October 25, 1951 to
National Congress and the Muslim League at the joint March 27, 1952. The parties contested for 489
session of both the parties held in Lucknow in seats. About 1874 candidates and 53 parties contested
December 1916. Indian Council Act, 1919 Introduced for the elections.
diarchy in the executive government of the provinces.
32. Who was the first Indian Governor General of
29. Match the Pairs : India?
Act Provision (a) Mountbatten
A. Indian Councils I. The Crown no (b) C.Rajgopalachari
Act, 1909 longer the source
(c) Dr.Rajendra Prasad
of authority
(d) Sardar Ballabh Bhai Patel
B. Govt. of India Act, II. Dyarchy at the
MPPCS (Pre.) G.S. 2010
1919 Centre
C. Govt. of India Act, III. First time, separate Ans. (b) C. Rajagopalachari (1948-1950) was the first
1935 representation to Indian Governor General of India. After that the post of
the Muslim Governor General was abolished.
community Lord Mountbatten was the last British Governor general
D. India Independence IV. Dyarchy in the of India and the first Governor General of Independent
Act, 1947 Provinces of India.

Indian Polity & Constitution 10 YCT


02.
Constituent Assembly, its
functions and related committees
1. On which of the following dates, Constituent 4. The total strength of the Constituent
Assembly of India adopted the National Flag? Assembly came down to 299 as against 389.
(a) 22 January, 1950 (b) 24 January, 1950 Answer Options:
(c) 22 July, 1947 (d) 22 July, 1948 (a) Statements 1, 2 and 3 are correct
UPPCS (Pre) G.S. 2020 (b) Statements 2, 3 and 4 are correct
Ans. (c) The National Flag of India was adopted in its (c) Statements 1, 2 and 4 are correct
present form during the meeting of Constituent (d) Statements 1, 3 and 4 are correct
Assembly held on the 22 July 1947. Maharashtra PSC (Pre) G.S, 2019
On 24 January 1950, the National Anthem & National Ans : (c) Elections were held for the Constituent
song were adopted by Constituent Assembly. Assembly in accordance with the Cabinet Mission Plan
2. Constituent Assembly of India was set up- of 1946. So Constituent Assembly was a sovereign
(a) Under the recommendations of Kripps mission body. Hence statement 1 is correct. The Constitution of
(b) Under the recommendations of Cabinet India came into force on 26th January, 1950. On that
mission day, the Constituent Assembly ceased to exist,
(c) Under the recommendations of Indian transforming itself into the Provisional Parliament of
Independence Act India until a new Parliament was constituted in 1952.
(d) Under the recommendations of Mountbatten Hence statement 2 is also correct. The First Session of
plan the Constituent Assembly of India was held on
UPPCS (Main) Spl. G.S. IInd 2004, 2008 December 9, 1946.Its temporary President was
Uttarakhand PCS (Pre) 1st 2014 Dr.Sacchidanand Sinha. The second meeting of
U.P. Lower (Pre.) G.S 2009 Constituent Assembly was held on December11,
1946.Its President was Dr.Rajendra Prasad. Therefore
Ans. (b) Fristly the idea of constituent Assembly was
statement 3 is not correct. As per the Cabinet Mission
given by M.N. Roy in 1934. In September 1945, the
Plan, 292 members of the Constituent Assembly were to
newly elected Labour Government in Britain expressed
be elected from Provincial Legislative Assemblies, 93
its intention to create a Constituent Assembly for India
members from Indian Princely States and 4 members
to frame India’s Constitution. The Cabinet Mission
from the Chief Commissioners Provinces. However, as
(three Cabinet Ministers namely Lord Pethick
a result of the partition under the Mountbatten Plan of 3
Lawrence, Sir Stafford Cripps and Mr. A. V.
June, 1947, a separate Constituent Assembly was set up
Alexander) was sent to India in March 1946 to process
for Pakistan and representatives of some Provinces
the formalities. The Cabinet Mission Plan was a
ceased to be members of the Assembly. As a result, the
statement made by the Cabinet Mission and then
membership of the Assembly was reduced to 299.
Viceroy, Lord Wavell on May 16, 1946, which
Hence statement 4 is correct.
contained proposals regarding the constitutional future
of India. 4. With reference to Indian History, the Members
of the Constituent Assembly from the
3. Consider the following statements regarding
Provinces were
changes made by the Indian Independence Act
of 1947 in the position of the Constituent (a) Directly elected by the people of those
Assembly: Provinces
1. The Constituent Assembly was made a fully (b) Nominated by the Indian National Congress
sovereign body. and the Muslim League
2. The Constituent Assembly became the first (c) Elected by the Provincial Legislative
Parliament of free India. Assemblies
3. When the Constituent Assembly met as a (d) Selected by the Government for their
Legislative body it was chaired by Dr. expertise in constitutional matters
Rajendra Prasad. IAS (Pre) G.S., 2013
Indian Polity & Constitution 11 YCT
Ans. (c) Kindly refer the explanation of the above Ans: (c) Kindly refer the explanation of the above
question. question.
5. When did the Constituent Assembly for 9. Indian Constitution was adopted by
Undivided India meet for the first time? Constituent Assembly on-
(a) 9th Nov 1946 (b) 9th Dec 1946 (a) 26 November, 1949 (b) 15 August, 1949
th
(c) 9 Oct 1946 (d) 9th Sep 1946 (c) 2 October, 1949 (d) 15 November, 1949
TNPSC (Pre) G.S. 2016 (UPPCS (Pre) G.S. 2000, 2002, 2006)
Ans. (b) : Kindly refer the explanation of the above BPSC (Pre) 1996
question.
Ans. (a) Kindly refer the explanation of the above
6. The members of the Constituent Assembly
question.
which drafted the Constitution of India were
(a) Nominated by the British parliament 10. The Constitution of India was enacted on 26
(b) Nominated by the Governor General November 1949 by the
(c) Elected by the Legislative Assemblies of (a) Constituent Assembly
various provinces (b) Governor General of India
(d) Elected by the Indian national Congress and (c) Parliament of India
Muslim League (d) British Parliament
IAS (Pre) G.S., 2002 UPPCS (Main) G.S. IInd 2012
UPPCS (Pre.) G.S., 1993 Ans. (a) Kindly refer the explanation of the above
Ans. (c) Kindly refer the explanation of the above question.
question. 11. Who presided over the first meeting of Indian
7. Which one of the following statements is Constituent Assembly?
correct? (a) Dr. Rajendra Prasad
(a) The Constitution Assembly of India was
(b) Pandit Jawahar Lal Nehru
elected by the Provincial Assemblies in the
year 1946. (c) B.R. Ambedkar
(b) Jawaharlal Nehru, M.A. Jinnah and Sardar (d) Sachchidananda Sinha
Vallabhbhai Patel were members of the UPPCS (Pre.) G.S. 2006
Constituent Assembly of India. Ans. (d) Kindly refer the explanation of the above
(c) The First Session of the Constituent question.
Assembly of India was held in January, 1947. 12. The first Session of the Constituent Assembly
(d) The Constitution of India was adopted on was held on
26th January, 1950. (a) 16th August, 1947
IAS (Pre) G.S., 2004
(b) 26th January, 1948
Ans. (a) The Constitution Assembly of India was elected
(c) 9th December, 1946
by the Provincial Assemblies in the year 1946.Except
Mahatma Gandhi and M.A.Jinnah, all the prominent (d) 26th November, 1946
political leaders of that time were members of the UPPCS (Main) G.S. IInd 2009, 2011
Constituent Assembly. The First Session of the UPPCS (Pre.) G.S., 1995
Constituent Assembly of India was held on December 09, Ans. (c) Kindly refer the explanation of the above
1946. Two hundred and seven representatives, including question.
nine women were present at the inaugural session. Its
13. President of the Constituent Assembly was-
temporary President was Dr.Sacchidanand Sinha.The
second meeting of Constituent Assembly was held on (a) Dr.B.R.Ambedkar (b) Dr.K.N.Katzu
11th December, 1946.Its President was Dr.Rajendra (c) Dr.Rajendra Prasad (d) C.Rajagopalachari
Prasad. The Constitution of India was adopted on 26th (U.P.PCS., 1998, 2000)
November, 1949 and the members appended their Ans. (c) Kindly refer the explanation of the above
signatures to it on the last meeting of the Constituent question.
Assembly on 24th January, 1950. In all, 284 members
14. The inaugural session of Constituent Assembly
actually signed the Constitution. The Constitution of
India came to force on 26th January, 1950. was presides by-
(a) Dr.Rajendra Prasad
8. Mention the correct date of the last meeting of
the Constituent Assembly of India. (b) Dr.Sachhidanand Sinha
(a) 26 Nov. 1949 (b) 5 Dec. 1949 (c) Dr.B.R.Ambedkar
(c) 24 Jan. 1950 (d) 25 Jan. 1950 (d) Pandit Jawahar Lal Nehru
UPPCS (Pre) G.S, 2018 UPPCS (Main) G.S. IInd Paper 2013
Indian Polity & Constitution 12 YCT
Ans. (b) Kindly refer the explanation of the above (c) It was elected on the basis of universal adult
question. franchise
15. Which of the following items is wrongly (d) Its electoral process was based on 6th
matched? Schedule of the act of 1935 which allowed for
(a) 11th December, 1946 : Jawaharlal Nehru a restricted franchise based on tax, property
moved the Objective Resolution in the and education qualification
Constituent Assembly. MPPSC (Pre) GS 1st Paper 2012
(b) 29th August, 1947 : Drafting committee was Ans. (c) Kindly refer the explanation of the above
set up question.
(c) 26th November, 1949 : The people of India 19. Who was the Chairman of the Provincial
adopted, enacted and gave to themselves the Constitution Committee of the Constituent
Constitution Assembly?
(d) 24th January, 1950 : Constitution was finally (a) Dr. B.R. Ambedkar
signed by the members of the Constituent (b) Pt. Jawahar Lal Nehru
Assembly. (c) Dr. Rajendra Prasad
Maharashtra PSC (Pre) G.S, 2019 (d) Sardar Patel
Ans. (a) : On 13th December, 1946, Pandit Jawaharlal UPPCS (Main) G.s. IInd Paper 2008
Nehru moved the Objectives Resolution. Hence (a) is Ans. (d) Sardar Vallabai Patel was the Chairman of
not correctly matched. Rest is correctly matched. the Provincial Constituion Committee of the
16. Which of the following Bills was drafted by Dr. Constituent Assembly.The Constituent Assembly
B.R Ambedkar as the first law minister of selected 22 committees to deal with diverse task of
India? Constitution-making. Out of these, 10 were on
(a) Hindu Code Bill procedural affairs and 12 were on substantive affairs.
(b) Indian Code Bill List of the major committees is given below:-
(c) Hindu Marriage Bill Name of the Committee Chairman
(d) Human Rights Bills Committee on the Rules of
Rajendra Prasad
Himanchal PSC (Pre) G.S., 2018 Procedure
Ans. (a) As a first law minister of India, Dr. B.R Steering Committee Rajendra Prasad
Ambedkar drafted Hindu Code Bill on 11th April, 1947. Finance and Staff Committee Rajendra Prasad
17. Which of the following statements regarding Alladi Krishnaswami
Credential Committee
the Constituent Assembly are not true? Ayyar
I. It was not based on adult franchise. B. Pattabhi
House Committee
II. It resulted from direct elections. Sitaramayya
III. It was a multiparty body. Order of Business Committee K.M. Munsi
IV. It worked through several committees. Ad hoc Committee on the
Rajendra Prasad
Select the correct answer from the codes given National Flag
below. Committee on the Functions of
G.V. Mavalankar
Codes : the Constituent Assembly
(a) I and II (b) II and III States Committee Jawaharlal Nehru
(c) I and IV (d) I, II, III and IV Advisory Committee on
IAS (Pre) G.S., 1993 Fundamental Rights, Minorities Vallabhbhai Patel
and Tribal and Excluded Areas
Ans. (a) Members of Constituent Assembly were
indirectly elected. The members of the Constituent Minorities Sub-Committee H.C. Mookherjee
Assembly were elected by the members of the Fundamental Rights Sub-
J.B. Kripalani
Provincial Assemblies by method of single transferable Committee
vote system of proportional representations. Hence it North-East Frontier Tribal Areas
was not based on adult franchise. and Assam Exluded & Partially Gopinath Bardoloi
18. Which among the following is INCORRECT Excluded Areas Sub-Committee
statement about constituent assembly? Excluded and Partially Excluded
(a) It worked with the help of a large number of Areas (Other than those in Assam) A.V. Thakkar
committees, the most important among them Sub-Committee
being the drafting committee Union Powers Committee Jawaharlal Nehru
(b) Minorities communities were adequately Union Constitution Committee Jawaharlal Nehru
represented in the assembly Drafting Committee B.R. Ambedkar
Indian Polity & Constitution 13 YCT
20. Who among the following was the chairman of 26. Who among the following was the chairman of
the Advisory Committee on Fundamental the Union Constitution Committee of the
Rights of the Constituent Assembly? Constitutional Assembly?
(a) Maulan Abul Kalam Azad (a) B.R.Ambedkar
(b) Rajendra Prasad (b) J.B.Kriplani
(c) Jawaharlal Nehru (c) Jawahar Lal Nehru
(d) Aladi Krishnaswami Ayyar
(d) Vallabhbhai Patel
IAS (Pre) G.S., 2005
UPPSC ACF-RFO Mains (IInd Paper) 2019
Ans. (c) Kindly refer the above chart.
Ans. (d) : Kindly refer the above chart.
27. Who among the following was appointed
21. Who headed Provincial Constituion Committee constitutional adviser to the Constituent
of Constituent Assembly? Assembly?
(a) J.B. Kriplani (b) H.C. Mukherjee (a) B.R.Ambedkar (b) K.T.Shah
(c) A.V. Thakkar (d) Sardar Vallabai Patel (c) B.N.Rau (d) A.K.Ayyar
TNPSC (Pre) G.S. 2019 U.P.P.C.S (Pre.) G.S.2014
Ans. (d) : Kindly refer the above chart. Ans. (c) Jurist B. N. Rau was appointed constitutional
22. As a member of the Constituent Assembly, adviser to the Constitutent Assembly.B.N. Rau prepared
Sardar Patel did not lead which of the the original draft of the Constitution, which later was
scrutinized by Drafting Committee. Drafting committee
following committees?
was set up on 29th August, 1947 under the chairmanship
1. Committee on Tribal and Excluded Areas of Dr. B R Ambedkar. The Drafting Committee had
2. Committee on Minoriry Affairs seven members namely Alladi Krishna swami Ayyar,
3. Committee on Fundamental Rights N. Gopalaswami, B.R. Ambedkar, K.M Munshi,
4. Committee on Devolution of Power to the Mohammad Saadulla, B.L. Mitter and D.P. Khaitan.
Union Later on B.L. Mitter & D.P. Khaitan was replaced by N.
(a) 1, 2 and 3 (b) 2, 3 and 4 Madhav Rau & T.T. Krishnamachari respectively.
(c) 4 only (d) 2 and 4 only 28. Who was the constitutional advisor during the
Punjab PSC (Pre) G.S 2015 drafting of Indian Constitution?
(a) Dr.B.R.Ambedkar (b) Dr.Rajendra Prasad
Ans. (c) : Kindly refer the above chart.
(c) B.N. Rao (d) K.M.Munshi
23. Who headed Steering Committee of UP Lower (Pre) G.S., 1998
Constituent Assembly? Ans. (c) Kindly refer the explanation of the above
(a) Jawaharlan Nehru (b) K.M. Mushsi question.
(c) H.C. Mookerjee (d) Rajendra Prasad 29. First draft of the Indian Constitution was
TNPSC (Pre) G.S. 2016 prepared by
Ans. (d) : Kindly refer the above chart. (a) B.R.Ambedkar (b) B.N.Rao
24. Who was the chairman of the Ad hoc (c) K.Santhanam (d) K.M.Munshi
Committee on the National flag? U.P. Lower (Pre.) G.S. 2009
(a) C.Rajgopalachari Ans. (b) Kindly refer the explanation of the above
(b) Dr.Rajendra Prasad question.
(c) J.B.Kriplani 30. The number of members included in the
(d) Dr.B.R.Ambedkar Constitution Drafting Committee was
(a) Seven (b) Nine
U.P. Lower (Pre.) G.S.2008
(c) Eleven (d) Thirteen
Ans. (b) Kindly refer the above chart.
UPPCS (Main) G.S. IInd Paper, 2006
25. Who was the Chairman of Advisory Committee Ans. (a) Kindly refer the explanation of the above
on Fundamental Rights and Minorities question.
constituted by the Constituent Assembly? 31. Who amongst the following was not a member
(a) Pandit Nehru of the Drafting Committee of the Constitution?
(b) Sardar Patel (a) Mohammad Sadullah
(c) B.N.Rao (b) K.M.Munshi
(d) Ambedkar (c) A.K.Ayyar
MPPSC (Pre) GS 1st Paper 2014 (d) Jawaharlal Nehru
Ans. (b) Kindly refer the above chart. UPPCS (Main) G.S. IInd Paper 2012
Indian Polity & Constitution 14 YCT
Ans. (d) Kindly refer the explanation of the above (c) (A)-(i) (B)-(ii) (C)-(iii) (D)-(iv)
question. (d) (A)-(iii) (B)-(iv) (C)-(i) (D)-(ii)
32. Who among the following were the members of RAS/RTS (Pre) G.S., 2013
the drafting committee of the Constitution? Ans. (b) First Vice President of the Constituent
(i) N. Gopalaswami Assembly was H.C.Mookerjee. K.M.Munshi was
(ii) Jawaharlal Nehru originally the only Congress member of Draft
(iii) Alladi Krishnaswami Ayyar Committee while V.T.Krishnamachari was member of
(iv) Sardar Patel the Constituent Assembly representing Princely State of
Select the correct answer using the codes given Rajasthan. Jawaharlal Nehru was the Chairman of
below : Union Constitution Committee.
Codes : 36. How much time the Constituent Assembly took
(a) (i), (iii) and (iv) (b) (i) and (iv) to frame the Constitution of India?
(c) (i) and (iii) (d) (ii), (iii) and (iv) (a) 2 years 11 months 18 days
UPPCS (Pre.) G.S., 2014 (b) 2 years 7 months 23 days
(c) 3 years 4 months 14 days
Ans. (c) Kindly refer the explanation of the above
question. (d) 3 years 11 months 5 days
UPPCS (Main) G.S. IInd Paper 2007
33. Who was the chairman of the drafting
committee of the Indian Constitution? Ans. (a) Constitution Assembly took 2 years, 11
(a) K.M.Munshi (b) Dr.Rajendra Prasad months and 18 days to frame the Constitution of India.
(c) B.R.Ambedkar (d) Dr.Sampurnanand 37. How many Sessions of the Indian Constituent
Chhatisgarh PSC (Pre) G.S., 2003 Assembly were conducted for the formulation
of Indian Constitution?
Ans. (c) Dr. B.R. Ambedkar was the Chairman of the
(a) 7 (b) 11
drafting committee of the Indian Constitution.
(c) 12 (d) 15
34. The Constituent Assembly set-up a Drafting
UPPCS (Main) G.S. IInd Paper 2005
Committee under the Chairmanship of Dr.
B.R. Ambedkar on Ans. (b) The Constituent Assembly took almost three
(a) 13th December, 1946 years (two years, eleven months and eighteen days) to
complete its historic task of drafting the Constitution for
(b) 22nd January, 1947
Independent India. During this period, it held eleven
(c) 3rd June, 1947
sessions, covering a total of 165 days. Of these, 114
(d) 29th August, 1947 days were spent on the consideration of the Draft
UPPCS (Main) G.S. IInd Paper 2008 Constitution.
Ans. (d) Drafting committee was set up on 29th August, 38. Who said “The Assembly was the Congress
1947 under the chairmanship of Dr. B R Ambedkar. and the Congress was India”?
35. Match list-X with list-Y and select the correct (a) Austin (b) C.R Attlee
answer using the codes given below the list : (c) Winston Churchil (d) Lord Mountbatten
List-X List-Y UP RO/ARO (Pre) G.S., 2013
(A) First Vice President (i) V.T.Krishnachri Ans. (a) Granville Austin in his book ‘The Indian
of the Constituent Constitution’ wrote that “The Assembly was the
Assembly Congress and the Congress was India”.
(B) Originally the only (ii) Jawahar Lal 39. Which one of these was the first law minister of
Congress Member of Nehru India?
Draft Committee (a) Jawahar Lal Nehru
(C) Member of the (iii) K.M.Munshi (b) Dr.B.R.Ambedkar
Constituent (c) Maulana Abdul Kalam Azad
Assembly (d) T.T.Krishnamachari
Representing UPPCS (Main) G.s. Ist Paper 2012
Rajasthan’s princely
state Ans. (b) B.R.Ambedkar was the first law minister of
India after independence.
(D) Chairman of Union (iv) H.C.Mookerjee
Jawaharlal Nehru was the first Prime Minister and
Constitution
Foreign Minister of India.
Committee
Maulana Abdul Kalam Azad was the First Education
Code : Minister of India.
(a) (A)-(i), (B)-(iv) (C)-(ii) (D)-(iii) T.T. Krishnamachari was Indian finance minister from
(b) (A)-(iv) (B)-(iii) (C)-(i) (D)-(ii) 1956 and from 1964-1966.
Indian Polity & Constitution 15 YCT
03.
Influence of other Constitutions
1. The concept of Concurrent List in Indian 2. Match List-I with list-II and select the correct
Constitution was borrowed from answer:
(a) Canada (b) Australia List–I List-II
(c) U.S.A (d) Great Britain (A) Directive Principle of State 1. Australia
UPPCS (Main) G.S. IInd Paper 2011, 12, 14, 16 Policy
Jharkhand PSC (Pre) G.S. 2003 (B) Fundamental Rights 2. Canada
Ans. (b) Features that have been borrowed from the (C) Concurrent List of Union- 3. Ireland
Constitution of other countries are- State relations
(D) India as Union of States with 4. United
Countries Borrowed features greater powers to center Kindom
5. U.S.A
Canada Idea of federation with a strong Code:
Center, Distribution of powers A B C D
between Center and the States, (a) 5 4 1 2
Residuary powers with the Center (b) 3 5 2 1
(c) 5 4 1 2
Japan Concept of Procedure Established by (d) 3 5 1 2
law
IAS (Pre) G.S., 2003
United President as nominal head, Ans. (d) :
Kingdom Designation of Prime Minister, Directive Principle of State Ireland
Cabinet System of Ministers, Policy
Parliamentary type of Government , Fundamental Rights USA
Bicameral Parliament( Rajya Sabha
and Lok Sabha), Speaker in Lok Concurrent List of Union- Australia
Sabha, Council of Ministers State relations
responsible to Lok Sabha India as Union of States with Canada
greater powers to center
Germany Suspension of Fundamental Rights 3. The concept of fundamental rights has been
during the emergency taken from the constitution of which country?
(a) France (b) Britain
France Concept of Republic, Ideals of (c) United States of America (d) Russia
Liberty, Equality and Fraternity UPUDA/LDA (Pre.) G.S., 2006, 2010
UP RO/ ARO (Pre) G.S., 2014
Australia Concurrent list, Language of the Ans. (c) Kindly refer the explanation of the above
Preamble, Provision regarding question.
commerce and trade 4. Concept of Directive Principles of State Policy
of the Indian Constitution has been borrowed
Ireland Concept of Directive Principles of from the constitution of which country?
State Policy, Election of President, (a) Britain (b) U.S.A
Nomination of members in the
Rajya Sabha (c) Ireland (d) Japan
UPPCS (Main) G.S. IInd Paper 2005
Russia USSR) Five year Plan, Fundamental Duties Ans. (c) Kindly refer the explanation of the above
question.
United States Preamble ,Written Constitution, 5. Assertion (A) The Constitution of India has
Fundamental Rights, Functions of become the longest one.
the President and the Vice Reason (R) The chapter on fundamental rights
President ,Independence of has been borrowed from the model of American
Judiciary, udicial review ,Removal Constitution.
of Supreme Court and High Court Choose the correct answer using the following
Judges codes
(a) Both A and R are correct and R is the correct
South Africa Election of members of the Rajya explanation of A
Sabha, Amendment of the (b) Both A and R are correct but R is not the
Constitution correct explanation of A
Indian Polity & Constitution 16 YCT
(c) A is true, but R is false Ans. (c) The concept of ‘A Union of States’ in the
(d) A is false, but R is true Indian Constitution has been derived from the British
UPPCS (Pre) G.S., 2015 North – American Act (Canada).
Ans. (b) Constitution of India is the longest written 11. The system of judicial review exist in
Constitution in the world. So assertion A is correct. (a) India only (b) U.S.A only
Fundamental Rights has been borrowed from the model (c) India and U.S.A (d) U.K only
of American Constitution. So reason R is also true but UPPCS (Pre) G.S., 1998, 2008
reason R is not the correct explanation of assertion A. UPPCS (Main) G.S. IInd Paper 2006, 2012, 2013
Ans. (c) Judicial review is the power of the Supreme
6. In giving representation to talent, experience Court and the High Courts to examine the
and service in the composition of the Council of constitutionality of the Acts of the Parliament and
States, the Constitution makers of India were State Legislatures, executive orders of both Centre
influenced by the example of – and State Governments. The system of Judicial
(a) Irish Republic (b) Canada review exist in India and U.S.A. The concept of
(c) U.S.A (d) Australia judicial review in Indian Constitution has been
UPPCS (Pre) G.S., 1998 adopted from the Constitution of U.S.A.
12. The concept of Fundamental Duties in Indian
Ans. (a) In giving representation to talent, experience Constitution has been taken from-
and service in the composition of the Council of States (a) American Constitution
(Rajya Sabha), the Constitution makers of India were (b) British Constitution
influenced by Irish Republic. (c) Russian Constitution
7. The concept of ‘The Directive Principles of (d) French Constitution
State policy’ in the Indian Constitution has UP Lower (Pre) G.S., 2003
been adopted from the constitution of- Ans. (c) India borrowed the concept of Fundamental
(a) France (b) Ireland Duties from Russian Constitution. Fundamental Duties
(c) Japan (d) U.S.S.R are based upon the recommendation of Sardar Swaran
UPPCS (Pre.) G.S., 1994, 1988, 1999, 2000 Singh Committee in 1976. Article 51A of the Indian
U.P. Lower (Pre.) Spl. G.S. 2003, 2010 Constitution deals with Fundamental Duties.
Fundamental Duties have no legal binding but it is the
Jharkhand PSC (Pre) G.S. 2003, 2010 moral duty of the every citizen of India to perform the
UPPCS (Main) Spl. G.S. IInd Paper 2008 Fundamental Duties.
Ans. (b) The concept of ‘The Directive Principles of 13. Match List-I with list-II and select the correct
State policy’ in the Indian Constitution has been answer:
adopted from the Constitution of Ireland. Part IV List – I List – II
(Articles 36-51) of Indian Constitution deals with (Constitutional (Source)
Directive Principles of State Policy. The provisions Porvision)
contained in this Part cannot be enforced by any court (A) Fundamental Rights 1. British
but these principles are fundamental in the governance Constitution
of the country and it shall be the duty of the State to (B) Directive Principle 2. Canadian
apply these principles in making laws. of State Policy Constitution
8. Concept of Federation in India is based on (C) Cabinet Government 3. Irish Constitution
which country? (D) Union-State 4. United Kindom
(a) U.S.A (b) Canada relations
(c) Australia (d) Germany Code :
UPPCS (Main) G.S. IInd 2017 A B C D
(a) 4 3 2 1
Ans. (b) The concept of the Federal system of the (b) 4 2 3 1
Indian Constitution is inspired by the Constitution of (c) 4 1 3 2
Canada. The Constitution of India gives a federal (d) 4 3 1 2
structure to the Republic of India, declaring it to be a
RAS/RTS (Pre) G.S. 2015
‘Union of States’.
Ans. (d) The correct match is-
9. Indian Federation closely resembles the Fundamental Rights - U.S Bills of Rights
Federation of Directive Principle - Irish Costituion
(a) U.S.A (b) Australia of State Policy
(c) Switzerland (d) Canada Cabinet Government - British Constitution
Uttarakhand RO/ARO (M) 2016 Union-State Relation - Canadian Constitution
Ans. (d) Kindly refer the explanation of the above 14. Who has said it: ‘I have not to beg pardon in
question. connection with the allegation that in the draft
10. The concept of ‘A Union of States” in the of the Constitution, a major part of the
Indian Constitution has been derived from: Government of India Act, 1935, has again been
reproduced’?
(a) The American Declaration of Independence (a) Dr. Rajendra Prasad (b) Sardar Patel
(b) The Australian Constitution (c) Jawaharlal Nehru (d) Dr. B. R. Ambedkar
(c) The British North – American Act UPPCS (Main) G.S. IInd Paper 2015
(d) The Swiss Constitution Ans. (d) The above statement was quoted by Dr. B. R.
UPPCS (Pre) G.S, 2017 Ambedkar.
Indian Polity & Constitution 17 YCT
04.
Features of Preamble
1. The Preamble to the Constitution of India is 3. Who proposed the Preamble before the
(a) A part of the Constitution but has no legal Drafting Committee of the Constitution?
effect (a) Jawaharlal Nehru (b) B.R. Ambedkar
(b) Not a part of the Constitution and has no legal (c) B.N. Rau (d) Mahatama Gandhi
effect either Uttarakhand PCS (M) GS 1st 2006
(c) A part of the Constitution and has the same Ans. (a) The Preamble of the Indian Constitution was
legal effect as any other part drafted and moved by Pandit Jawahar Lal Nehru, which
(d) A part of the Constitution but has no legal is based on the 'objective resolutions'.
effect independently of other parts 4. Which among the following introduced
IAS (Pre) G.S. 2020 Preamble in the Constituent Assembly?
Ans: (d) The legislative intent of the Preamble of the (a) Dr.B.R.Ambedkar
Constitution was moved by Jawaharlal Nehru and (b) Pt.Jawaharlal Nehru
adopted by the Constituent Assembly on 26th (c) Dr.Rajendra Prasad
November, 1949. The Preamble of the Constitution of (d) Dr.C.D.Deshmukh
India is part of the Constitution but is not enforceable UPPCS (Main) G.S. IInd Paper, 2007
by courts. The Preamble is non-Justifiable which means Ans. (b) Kindly refer the explanation of the above
that courts cannot pass orders against the Government question.
of India to implement the ideas in the Preamble. In the 5. Which one of the following objectives is not
Kesavananda Bharati case, the Supreme Court said that embodied in the Preamble to the
the Preamble of the Constitution can be amended Constitution of India?
according to Article 368 (Power of Parliament to amend (a) Liberty of thought
the Constitution) of the Constitution, but the
(b) Economic liberty
amendment is subject to certain conditions such as it
(c) Liberty of expression
cannot alter the ‘Basic Structure’ of the Constitution.
(d) Liberty of belief
Preamble cannot override the specific provision of the
Constitution. In case of any conflict between the two, IAS (Pre.)G.S, 2017
the latter shall prevail. Ans. (b) The objective of the Preamble of Indian
Constitution is Justice, Equality, Liberty and Fraternity.
2. The mind of the makers of the Constitution of
Liberty in Preamble reflects liberty of thought,
India is reflected in which of the following?
expression, belief, faith and worship. Hence economic
(a) The Preamble liberty is not embodied in the Preamble of the
(b) The Fundamental Rights Constituion of India.
(c) The Directive Principles of State Policy 6. How many types of liberty are mentioned in
(d) The Fundamental Duties the Preamble of the Indian Constitution?
IAS (Pre)G.S, 2017 (a) Seven (b) Three
Ans. (a) The Preamble embodies the fundamental (c) Five (d) None of these
values and the philosophy on which the Constitution is UPPSC ACF-RFO Mains (IInd Paper), 2019
based. The Preamble contains the ideals and Ans. (c) : Kindly refer the explanation of the above
aspirations of the people. It reflects the objects which question.
the Constitution seeks to establish and promote. It also
7. Which one of the following liberty is not
aids the legal interpretation of the Constitution. It
embodied in The Preamble of the Constitution
reflects the minds of the framers of the Constitution.
of India?
Where the words were found to be vague or their
(a) Liberty of Thought
meaning are unclear, help of the Preamble could be
(b) Liberty of Expression
taken to understand the intention of the framers of the
Constitution and find out whether a particular word was (c) Liberty of Belief
used in a wide or narrow context. Hence Preamble is (d) Economic Liberty
known as the ‘Soul of the Indian Constitution’. UPPCS (Main) G.S. IInd 2017
Indian Polity & Constitution 18 YCT
Ans. (d) Kindly refer the explanation of the above 11. Which of the following statements about the
question. framing of the Indian Constitutions is correct?
8. The ideals of Liberty, Equality and Fraternity 1. Pt. Nehru's objective resolution adopted by
enshrined in the Preamble of the Constitution the Constituent Assembly had a bearing on
of India were adopted under inspiration from the framing of the Constitution
the- 2. The Preamble serves very important
(a) American Declaration of Independence purposes.
(b) French Revolution 3. The Constitution has been ordained by the
(c) Russian Revolution people of India.
(d) UN charter 4. The Head of the State is elected directly by
TNPSC (Pre) G.S. 2017 the people.
Ans. (b) : The concept of Liberty, Equality and Select the correct answer from the codes given
Fraternity in Preamble of the Constitution of India was below:
adopted from the French Motto of the French
(a) 1, 2 and 3 (b) 1, 3 and 4
Revolution.
(c) 2, 3 and 4 (d) All the four
9. Consider the following statements in regard to
The Preamble of Constitution and give correct UPPCS (Main) G.S. IInd Paper 2010
answer using codes given below- Ans. (a) As per the Constitution of India, Head of the
1. The objectives Resolution by Jawaharlal State is President of India which has been elected by the
Nehru finally became Preamble. members of an electoral college consisting of the elected
2. It is non-Justifiable. members of both the Houses of Parliament and the
3. It can’t be amended. elected members of the Legislative Assemblies of States
4. Preamble cannot override the specific and the Union Territories of Delhi and Pondicherry.
provision of the Constitution. Hence President of India which is head of the State is not
Code : directly elected by the people. Hence statement 4 is
(a) Only 1 and 2 (b) Only 1, 2 and 4 incorrect. Rest all three statements are correct.
(c) Only 1, 2 and 3 (d) Only 2, 3 and 4 12. In which form India has been declared in the
UPPCS (Pre.) G.S., 2009 Preamble of Indian Constitution?
Ans. (b) The legislative intent of the Preamble of the (a) A Sovereign, Democratic, Republic
Constitution was moved by Jawaharlal Nehru and (b) A Socialist, Democratic, Republic
adopted by the Constituent Assembly on 26th (c) Sovereign, Social, Secular, Democratic,
November, 1949. The Preamble of the Constitution of Republic
India is part of the Constitution but is not enforceable
(d) None of the above
by courts. The Preamble is non-Justifiable which means
that courts cannot pass orders against the Government BPSC (Pre) 1997-98
of India to implement the ideas in the Preamble. In the Ans. (c) The Preamble of the Indian Constitution
Kesavananda Bharati case, the Supreme Court said that declares; We, The people of India, having solemnly
the Preamble of the Constitution can be amended resolved to constitute India into a Sovereign, Socialist,
according to Article 368 (Power of Parliament to amend Secular, Democratic Republic.
the Constitution) of the Constitution, but the 13. The sequence in which the given terms are
amendment is subject to certain conditions such as it
mentioned in the Preamble of the Constituion
cannot alter the ‘Basic Structure’ of the Constitution.
of India is:
Preamble cannot override the specific provision of the
Constitution. In case of any conflict between the two, (a) Sovereign, Socialist, Secular, Democratic,
the latter shall prevail. Hence (b) is the correct answer. Republic
10. What is the legal nature of the Preamble of the (b) Socialist, Secular, Sovereign, Democratic,
Constitution? Republic
(a) It is enforceable (c) Secular, Sovereign, Democratic, Socialist,
(b) It is not enforceable Republic
(c) It may be enforced in special circumstances (d) Sovereign, Democratic, Secular, Socialist,
(d) None of the above Republic
UPPCS (Pre) G.S, 2019 TNPSC (Pre) G.S. 2019
Ans: (b) Kindly refer the explanation of the above Ans. (a) Kindly refer the explanation of the above
question. question.
Indian Polity & Constitution 19 YCT
14. What was the exact constitutional status of the Ans. (c) : Kindly refer the explanation of the above
Indian Republic on 26th January 1950? question.
(a) A Democratic Republic 19. The word ‘Secularism’ was inserted in the
(b) A Sovereign Democratic Republic Preamble of the Indian Constitution by
(c) A Sovereign Secular Democratic Republic (a) 25th Amendment (b) 42nd Amendment
(d) A Sovereign Socialist Secular Democratic (c) 44th Amendment (d) 52nd Amendment
Republic UPPCS (Main) G.S. IInd Paper 2013
Himanchal PSC (Pre), G.S, 2016 Ans. (b) Kindly refer the explanation of the above
UPPCS (Main) G.S. IInd Paper, 2009 question.
Ans. (b) The exact Constitutional states of Indian 20. Preamble to the Cosititution of India provides
Republic on 26 January 1950 was a Sovereign for equality of-
Democratic Republic. 42nd Constitutional Amendment
(a) Wages (b) Status
Act (1976) added three new words Socialist, Secular
and integrity. (c) Opportunity
Select the correct anwer from the codes given
15. India has peoples's sovereignty because the
below-
Preamble of the Constitution begins with the
words: (a) 1 and 2 (b) 1 and 3
(a) Democratic India (c) 2 and 3 (d) None of the above
(b) Replubic of India UPPSC ACF-RFO Mains (IInd Paper), 2019
(c) Democracy of People Ans. (c) : Preamble of the Consititution of India
(d) We, the People of India provides for equality of Status and Opportunity.
BPSC (Pre.)G.S, 1994 21. 'Economic Justice' as one of the objectives of
Ans. (d) Kindly refer the explanation of the above Indian Constitution has been provided in
question. (a) The Preamble and Fundamental Rights
16. In the Preamble of Constitution adopted on 26 (b) The Preamble and the Directive Principles of
November, 1949, which word/words was/were State Policy
not present- (c) The Fundamental Rights and the Directive
1. Socialist 2. Integrity Principles of State Policy
3. Secular 4. Republic (d) None of the above
Choose the correct answer by following code: IAS (Pre) G.S., 2013
(a) 1, 2 and 3 (b) 2, 3 and 4 Ans. (b) Preamble, the arch of the Indian Constitution
(c) 1, 2 and 4 (d) 3 and 4 provides socio-economic justice to all the India citizens.
UPPCS (Pre) G.S., 2009 According to Article 38(1) under the Directive
Principles of State Policy, State shall strive to promote
Ans. (a) The Constitution (Forty-Second Amendment) the welfare of the people by securing and protecting as
Act, 1976 amended the Preamble of the Constitution.
effectively as it may a social order in which justice;
As per the 42nd Amendment, for the words ‘Sovereign
social, economic and political shall inform all the
Democratic Republic’ the words ‘Sovereign Socialist
institutions of the national life.
Secular Democratic Republic’ was substituted and for
the words ‘Unity of the Nation’, the words ‘Unity and 22. How many types of justice have been enshrined
Integrity of the Nation’ was substituted. in the Preamble of the Constitution of India?
17. These two words ‘Secular’ and ‘Socialist ‘have (a) Two (b) Three
been added to the Preamble of the Constitution. (c) One (d) Four
Which was this Constitutional Amendment? UPPCS (Main) G.S. IInd Paper, 2016
(a) 25th Amendment (b) 4th Amendment Ans. (b) Kindly refer the explanation of the above
(c) 42nd Amendment (d) 44th Amendment question.
UPPCS (Main) G.S. IInd 2017 23. How many types of justice, liberty, equality
Ans. (c) Kindly refer the explanation of the above and fraternity in that order have been
question. mentioned in the preamble of constitution of
India?
18. The word 'Secularism' was inserted in the
preamble of the constitution after Supreme (a) 3, 5, 2, 1 (b) 1, 3, 5, 2
Court of India observed 'secularism' was a (c) 2, 5, 3, 1 (d) 5, 2, 1, 3
basic feature of Constitution in. UPPCS (Pre) G.S, 2018
(a) 1975 (b) 1973 Ans: (a) In the Preamble of the Constituion, three types
(c) 1976 (d) 1961 of justice namely social, economic and political; five
TNPSC (Pre) G.S. 2017 types of Liberty namely thought, expression, belief,
Indian Polity & Constitution 20 YCT
faith and worship; two type of Equality namely status political horoscope of Indian Constitution by KM
and opportunity; one type of Fraternity namely assuring Munshi, soul of the constitution by Thakurdas Bhargav
the dignity of the indivisual has been mentioned. and identity card of the constitution by NA Palkhiwala.
24. In the verdict of which case Supreme Court 28. Which one of the following said the Preamble of
declared first time that Preamble is not a part the Indian Constitution as ‘Political Horoscope’?
of the Constitution? (a) Jawaharlal Nehru (b) Vallabh Bhai Patel
(a) Berubari (b) Sajjan Singh (c) B.R.Ambedkar (d) K.M.Munshi
(c) Golaknath (d) Keshwanand Bharti (e) Mahatma Gandhi
UPPCG (Main) 2017 G.S. IInd Paper Chaatisgarh PSC (Pre) 1st Paper G.S., 2013
Ans. (a) Berubari Case, 1960 was regarding the power Ans. (d) Kindly refer the explanation of the above
of the Parliament to transfer the territory of Berubari to question.
Pakistan. In this case Supreme Court ruled that the 29. Which one of the following describes India as a
Preamble is not a part of the Constitution and not secular State?
amendable under Article 368 (Power of Parliament to (a) Fundamental Rights
amend the Constitution) of the Constitution. Later in
(b) Preamble of the Constitution
Kesavananda Bharati case, 1973, the Supreme Court
(c) 9th Schedule
ruled that the Preamble is the part of the Constitution. In
the Bommai Vs.Union of India case, 1994, the Supreme (d) Directive Principles of State Policy
Court ruled that the Preamble is part of the Constitution UPPCS (Main) G.S. IInd Paper 2005
and can be amended according to Article 368 of the Ans. (b) Preamble of the Indian Constitution describes
Constitution but the amendment is subject to certain India as Sovereign Socialist Secular Democratic
conditions such as it cannot alter the ‘Basic Structure’ Republic State. The Fundamental Rights are defined as
of the Constitution. the basic human rights of all citizens. These rights,
25. In which one of the following cases has the defined in Part III (Article 14 to 30) of the Indian
Preamble been accepted as a part of the Indian Constitution apply irrespective of race, place of birth,
Constitution? religion, caste, creed or sex. 9th Schedule of the Indian
(a) A. K. Gopalan case Constitution is related to the land reforms. Part IV
(Articles 36-51) of Indian Constitution deals with
(b) Keshavanand Bharti case
Directive Principles of State Policy. The provisions
(c) Golaknathe case contained in this Part cannot be enforced by any court,
(d) Minerva Mills case but these principles are fundamental in the governance
Himanchal PSC (Pre) G.S. 2010 of the country and it shall be the duty of the State to
Ans. (b) Kindly refer the explanation of the above apply these principles in making laws.
question. 30. Which among the following is the correct
26. In which of the following cases Supreme Court expression of the term ’Secular ‘in India?
held that the Preamble forms part of the (a) India has many religions
Constitution? (b) Indians have religious freedom.
(a) Union of India Vs.Dr.Kohli (c) Following the religion depends upon the will
(b) BanarsidasVs.State of U.P of an individual.
(c) Bommai Vs.Union of India (d) There is no religion of the State in India.
(d) Malak Singh Vs.State of Punjab UPPCS (Main) G.S. IInd Paper 2005
UPPCS (Pre.) G.S., 2012 Ans. (d) The word ‘Secular’ was added into the
Ans. (c) Kindly refer the explanation of the above Preamble of the Constitution by the 42nd Amendment
question. Act (1976). Secularism in India means, there would be
27. Which part of the Indian Constitution has been equality of all religions in India along with religious
described as the ‘Soul’ of the Constitution? tolerance and respect. In other words, there is no
(a) Fundamental Rights religion of the State in India.
(b) Directive Principles of State Policy 31. The objective of Indian Constitution is
(c) The Preamble reflected in which of the following?
(d) Right to Constitutional Remedies (a) Fundamental Rights
UPPCS (Pre) G.S 2008 (b) Preamble of the Constitution
UPPCG (Main) G.S. IInd Paper 2013, 2015 (c) Directive Principles of the State Policy
UP UDA/LDA (Pre) G.S 2006 (d) Parliament
Uttarakhand PCS (Pre) Ist Paper 2014 U.P. Lower (Pre.) G.S. 2013
Ans. (c) The Preamble has been described as the ‘Soul’ Ans. (b) The objective of Indian Constitution is
of the Indian Constitution. It has been also called reflected in the Preamble of the Constitution.

Indian Polity & Constitution 21 YCT


05.
Articles, Schedules & Parts
of the Constitution
1. Which one of the following is NOT correctly Ans : (*) Article 263 of the Constituion is related to
atched? Interstate Council, Article 280 is regarding Finance
(a) Article 39 A - Equal Justice and free legal Commission, Article 323A is regarding Administrative
aid
Tribunals and Article 315 is regarding Public Service
(b) Article 40 - Organization of the Village
Panchayats Commissions for the Union and for the States.
(c) Article 44 - Uniform Civil Code So all pairs are correctly matched.
(d) Article 48 - Separation of Judiciary from 4. Which of the following is NOT correctly
Executive matched?
UPPCS (Pre) G.S 2020 Fundamental Right Article
Ans : (d) Article 48 of the Constituition is related to (a) Abolition of – Article 17
organization of agriculture and animal husbandary
Untouchability
while Article 50 is related to separation of Judiciary
from Executive. Rest is correctly matched. (b) Protection against arrest – Article 23
2. Match list I with list II and select the correct and detention
answer from the code given below the lists: (c) Freedom of Religion – Article 25
List I (Article) List II (Provisions) (d) Protection of Interest – Article 29
A. Article 61 1. Removal of Deputy Minorities
Chairman of Rajya UPPSC RO/ARO (Pre), 2016
Sabha
(Held on 20/09/2020)
B. Article 67(b) 2. Impeachment of
President Ans : (b) Article 22 of the Constitution is protection
C. Article 94 3. Removal of Vice- against arrest and detention in certain cases wereas
President Article 23 has provision of Prohibition of traffic in
D. Article 90 4. Removal of Speaker human beings and forced Labour. Rest is correctly
Code: matched.
A B C D 5. Which Article of the Constitution of India
(a) 2 4 3 1 safeguards one's right to marry the person of
(b) 3 2 4 1 one's choice?
(c) 2 3 4 1 (a) Article 19 (b) Article 21
(d) 4 1 3 2 (c) Article 25 (d) Article 29
UPPCS (Pre) G.S. 2020 IAS (Pre) G.S, 2019
Ans : (c) Correct match is-
Ans : (b) Supreme Court in Hadiya case judgement
Article 61 - Impeachment of the President
Article 67(b) - Removal of Vice-President ruled that the right to marry a person of one’s choice is
Article 94 - Removal of Speaker integral to Article 21 (Protection of life and personal
Article 90 - Removal of Deputy liberty) of the Constitution.
Chairman of Rajya Sabha 6. Which of the following is correct about the
3. Which of the following pairs is NOT correctly heading of the Article 162 of the Constitution of
matched? India?
(a) Interstate Council – Article 263 (a) Executive Power of State
(b) Finance Commission – Article 280
(b) Conditions of Governor’s Office
(c) Administrative Tribunal – Article 323A
(d) Union Public Service – Article 315 (c) Term of the Office of Governor
Commission (d) Extension of Executive Power of State
UPPCS (Pre) G.S 2020 UPPCS (Pre) G.S, 2019
Indian Polity & Constitution 22 YCT
Ans: (d) Article 162 of the Constituion is regarding a medium of expression for all the elements of the
executive power of the State shall extends to matters composite culture of India and to secure its enrichment
with respect to which the Legislature of the State has by assimilating without interfering with its genius, the
power to make laws. forms, style and expressions used in Hindustani and in
7. Consider the following statements: the other languages of India specified in the Eighth
1. Article 2 of the Indian Constitution relates to Schedule and by drawing, wherever necessary or
admission or establishment of new States desirable for its vocabulary, primarily on Sanskrit and
which are not part of the Union of India. secondarily on other languages.
2. Article 3 of the Indian Constitution provides 10. The Government of India Instituted: Bharat
for the formation of or changes in the existing Ratna and Padma Shri award under-
States including Union territories. (a) Article 14 of the Indian Constitution
3. A Bill under Article 3 of the Indian (b) Article 18 of the Indian Constitution
Constitution shall be introduced only in the (c) Article 25 of the Indian Constitution
Lok Sabha. (d) Article 32 of the Indian Constitution
Which of the above statements is/are correct? TNPSC (Pre) G.S. 2017
(a) All of the above (b) 1 and 2 Ans. (b) Government of India instituted Bharat Ratna
(c) 2 and 3 (d) Only 3 and Padma Shri award under Article 18 of the
Maharashtra PSC (Pre) G.S, 2019 Constituion. The Government of India instituted two
Ans: (b) Article 2 of the Constituion asserts that the civilian awards namely Bharat Ratna and Padma
Parliament may by law admit into the Union or Vibhushan in 1954. The latter had three classes namely
establish new States on such terms and conditions, as Pahela Varg, Dusra Varg and Tisra Varg. These were
it thinks fit. Hence statement 1 is correct. Article 3 of subsequently renamed as Padma Vibhushan, Padma
the Constitution provides power to the Parliament for Bhushan and Padma Shri vide Presidential Notification
the formation of new States and alteration of areas, issued on January 8, 1955. The Padma Awards are one
boundaries or names of existing States. Hence of the highest civilian honours of India announced
statement 2 is also correct. A Bill under Article 3 of annually on the eve of Republic Day.
the Constituion shall be introduced in either house of 11. Indian Parliament has the power to create a
the parliament only after recommendation of the new State under which of the following
President after the Bill has been reffered by the Constitutional provisions?
President to the Legislature of the affected States. (a) Article 1 (b) Article 2
Hence statement 3 is not correct. (c) Article 3 (d) Article 4
8. Which of the following is NOT possible by a (e) Article 5
law of Parliament under Article 3 of the Chhatisgarh PSC (Pre) 1st Paper G.S., 2014
Constitution?
Ans. (c) Article 3 of the Constitution provides power to
(a) Formation of new States
the Parliament for the formation of new States and
(b) Alteration of areas of States
alteration of areas, boundaries or names of existing
(c) Alteration of boundaries of States
States.
(d) Admission of new States
UPPCS (Pre) G.S 2020 12. Article related to the abolition of untoucha-
bility in the Indian Constitution is-
Ans. (d) Kindly refer the explanation of the above
question. (a) Article 15 (b) Article 16
(c) Article 17 (d) Article 18
9. Which Article of the Constitution provides that
it shall be the duty of the Union to promote the UP PSC RO/ARO (Pre) 2017
spread of the Hindi language, to develop is so UPPCS (Pre.) G.S.Spl., 2004
that it may become the 'Lingua France' of the UPPCS (Pre.) G.S., 1994
composite culture of India? Uttarakhand UDA/LDA (Pre) 2006
(a) Article 343 (b) Article 348 Uttarkhand PCS (Pre) 2004-05
(c) Article 350 (d) Article 351 MPPSC (Pre) GS 1st Paper 2013
Maharashtra PSC (Pre) G.S 2018 Ans. (c) Article 17 of the Constitution is a Fundamental
Ans. (d) Article 351 of the Constitution asserts that it Right, related to the abolition of untouchability. Article
shall be the duty of the Union to promote the spread of 15 of the Constitution is Prohibition of Discrimination
the Hindi language to develop it so that it may serve as on ground of religion, race, caste sex or place of birth.
Indian Polity & Constitution 23 YCT
Article 16 has provision of equality of opportunity in Codes:
matters of public employment. (a) Both (A) and (R) are true and (R) is the
Article 18 is related to abolition of titles. correct explanation of (A)
(b) Both (A) and (R) are true but (R) is not the
13. By which Article of the Constitution,
correct explanation of (A)
possession of Kripan by the Sikhs is part of
(c) (A) is true, but (R) is false
their religious freedom?
(d) (A) is false, but (R) is true
(a) Article 24 (b) Article 25
UPPCS (Pre) G.S, 2019
(c) Article 26 (d) Article 27
Ans: (b) Constituion of India does not define the term
Jharkand PCS (Pre) G.S 2009
‘minorities’ any where but used the term in some
Ans. (b) Article 25 of the Constitution is related to the articles such as Aricle 29, Article 30, Article 350 A and
freedom of conscience and free profession, practice and Article 350 B. Constituion of India referes the term
propagation of religion. Article 25 (2) (b) asserts that minorities based on religion or language. Therefore the
the wearing and carrying of kirpans shall be deemed to Constitution recognises only linguistic and religious
be included in the profession of the Sikh religion. minorities. Hence both (A) and (R) are true but (R) is
14. In which Article of the Indian Constitution not the correct explanation of (A)
provision has been made for all minorities shall 17. Which Article of the Indian Constitution is
have the right to establish and administer related to the Fundamental Rights against
educational institutions of their choice? child exploitation?
(a) Article 28 (b) Article 29 (a) Article 17 (b) Article 19
(c) Article 23 (d) Article 24
(c) Article 30 (d) Article 31
UPPCS (Main) G.S. IInd 2017
UPPCS (Main) G.S. IInd 2017
Ans. (d) According to Article 24 of the Constitution, no
Ans. (c) Article 30 (1) asserts that all minorities, whether
child below the age of fourteen years shall be employed
based on religion or language, shall have the right to to work in any factory or mine or engaged in any other
establish and administer educational institutions of their hazardous employment. Article 17 is related to the
choice. Article 28 is related to freedom as to attendance abolition of untouchability. Article 19 is related to the
at religious instruction or religious worship in certain protection of certain rights regarding freedom of speech.
educational institutions while Article 29 is related to the Article 23 is related to prohibition of traffic in human
protection of interests of minorities. Article 31 with sub beings and forced labour.
heading ‘Right to Property’ has been omitted by the 18. Which Article of the Indian Constitution
Constitution 44th Amendment Act, 1978. prohibits child to work in any factory?
15. Article 28 of the Constitution prohibits: (a) Article 19 (b) Article 17
(a) Discrimination on the basis of caste and (c) Article 23 (d) Article 24
religion UPPUDA/LDA Special (Pre) G.S., 2010
(b) Spread of communal hatred Ans. (d) Kindly refer the explanation of the above
(c) Religious Education in the government question.
educational institutions 19. Which Article of the Indian Constitution has
(d) Untouchability the provision for freedom of speech and
Himanchal PSC (Pre) G.S. 2012 expression?
(a) Article 14 (b) Article 19
Ans. (c) According to Article 28(1) of the
(c) Article 21 (d) Article 22
Constitution, no religion instruction shall be provided
Uttarakhand UDA/LDA (Pre) 2006
in any educational institution wholly maintained out of
Ans. (b) Article 19(1) (a) asserts that all citizens shall
State funds.
have the right to freedom of speech and expression with
16. Given below are two statements, one is labelled some reasonable restrictions on the exercise of right in
as Assertion (A) and the other as Reason (R) the interests of the sovereignty and integrity of India, the
Assertion (A) : Article 30 of the Constitution of security of the State, friendly relations with foreign
India does not define the term ‘minorities’. States, public order, decency or morality or in relation to
Reason(R) : The Constitution recognises only contempt of court, defamation or incitement to an
linguistic and religious minorities. offence.
Indian Polity & Constitution 24 YCT
20. The Constitution of India does not frames the (4) Right to timely medical treatment & medical aid to
‘Freedom of Press’ clearly but freedom of press injured person be provided by doctor.
has been inherited in the Article- (5) Right of women to be treated with decency and
(a) 19 (i) (a) (b) 19 (i) (b) dignity. This right states that sexual harassement of a
(c) 19 (i) (c) (d) 19 (i) (d) working Women at her work place as amounting to
violation of Article 21.
Uttarakhand UDA (Pre) 2003
(6) Right to free education up to 14 years of age.
Ans. (a) Article 19 (1) (a) asserts that all citizens shall
23. Which one of the following articles directs
have the right to freedom of speech and expression
State Government to organize Village
which includes freedom of press.
Panchayats?
21. The Supreme Court has held that hoisting the (a) Article 32 (b) Article 40
National Flag atop the private buildings is a (c) Article 48 (d) Article 51
Fundamental Right of every citizen under:
UPPCS (Pre) G.S., 2014
(a) Article 14 of the Constitution
Ans. (b) Under Directive Principle of State Policy,
(b) Article 19 (1) (a) of the Constitution Article 40 asserts that the State shall take steps to
(c) Article 21 of the Constitution organize village panchayats and endow them with such
(d) Article 25 of the Constitution powers and authority as may be necessary to enable
(e) None of the above/More than one of the them to function as units of self government.
above 24. Which Article of the Constitution provides for
BPSC (Pre) G.S. 2016 the participation of workers in the
Ans. (b) In Navin Jindal Vs. Union of India case, 2004, management of the industries?
Supreme Court gave ruling that under Article 19(1) (a) (a) Article 43 (b) Article 43A
of the Constitution, citizens had the fundamental right (c) Article 45 (d) Article 47
to fly the national flag on their premises around the U.P.Lower (Pre.) G.S. 2009
year, provided the premises do not undermine the Ans. (b) Article 43A (Directive Principle of State
dignity of the tricolour. Supreme Court also ruled that Policy) asserts that the State shall take steps by suitable
the National Honours Act and the Emblem and Names legislation or in any other way to secure the
(Prevention of Improper use) Act, 1950 will regulate the participation of workers in the management of
freedom. undertakings, establishments or other organizations
engaged in any industry. This Article was inserted by
22. Which one of the following does not fall within
the Constitution (Forty-Second Amendment) Act, 1976.
the purview of Article 21 of the Constitution?
Article 43 is related to promote cottate industries on an
(a) Medical aid to injured by a doctor
individual or co-operation basis in rural areas.
(b) Sexual Harassment of Women at work place Article 45 is related to the Provision for early
(c) Pollution of the quality of water Childhood care and education to children below the age
(d) Capital punishment of 6 year.
UPPCS (Pre)G.S, 2017 Article 47 states that it is a duty of state to raise the
Ans. (d) Capital Punishment is also known as Death level of nutrition and the standard of living and to
Penality, it voilets the Article 21 of Indian Constitution improve Public health.
and it does not fall within the purview of Article 21 of 25. Match list-I with list-II and select the correct
the Cosntitution answer using the codes given below the list:
Article 21 states that no person shall be deprived of his List – I List - II
life or personal liberty according to procedure (Constitutional (Source)
established by law. Porvision)
The following rights are the part of Article 21 are as (A) Article 40 1. Organization of village
follows : panchayat
(1) Right to decent environment including pollution free (B) Article 41 2. Right to work
water and air and protection against hazardous (C) Article 44 3. Uniform Civil Code
industries. (D) Article 48 4. Organization of
(2) Right to Privacy agriculture and animal
(3) Right to health & Shelter. husbandry

Indian Polity & Constitution 25 YCT


Code: Ans. (c) Article 51A of the Constitution is related to
A B C D Fundamental Duties. The Fundamental Duties of
(a) 1 2 3 4 citizens were added to the Constitution by the 42nd
(b) 2 3 1 4 Amendment in 1976 upon the recommendations of
(c) 1 3 4 2 the Swaran Singh Committee. India borrowed the
(d) 3 2 4 1 concept of Fundamental Duties from USSR.
UPUDA/LDA (Pre) G.S., 2001 29. Which among the following Committees was
UPUDA/LDA (Pre) G.S., 2006 responsible for the incorporation of
Ans. (a) Article 40 is related to the organization of Fundamental Duties in the Constitution?
village panchyats and Article 41 is related to right to (a) Wanchoo Committee
work, to education and to public assistance in certain (b) Sachar Committee
cases. Article 44 is related to uniform civil code for the
(c) Swaran Singh Committee
citizens while Article 48 is organization of agriculture
and animal husbandry. (d) Bhagawati Committee
26. Which one of the following is not correctly TNPSC (Pre) G.S. 2014
matched? Ans. (c) : Kindly refer the explanation of the above
(a) Article 39A-Equal Justice and Free Legal Aid question.
(b) Article 40- Organization of Village 30. Under which one of the following Articles, the
Panchayats procedure for removal of the President of India
(c) Article 44-Uniform Civil Code through the process of impeachment has been
(d) Article 48-Seperation of Judiciary from laid down?
Executive (a) Article-53 (b) Article-61
UPPCS (Pre) G.S., 2013 (c) Article-74 (d) Article-13
Ans. (d) Article 39 A under Directive Principle of State UPPCS (Pre) Re-exam. G.S., 2015
Policy is related to equal justice and free legal aid while
Ans. (b) The procedure for the impeachment of the
Article 40 is related to organization of village
President is mentioned in the Article 61 of the
panchayats, Article 44 is related to uniform civil code
Constituion.According to Article 61, when a President
and Article 48 is related to organization of agriculture
is to be impeached for ‘violation of the Constitution’,
and animal husbandry. Article 50 is related to
seperation of judiciary from executive the charge shall be preferred by either House of
Parliament with below mentioned conditions-:
27. Which Article of the Indian Constitution is
related to Indian Foreign Policy? (1) The proposal to prefer such charge is contained in
a resolution which has been moved after at least
(a) Article 380 (b) Article 312
fourteen days notice in writing signed by not less
(c) Article 60 (d) Article 51
than one-fourth of the total number of members of
UPPCS (Pre) G.S., 2016
the House that has been given of their intention to
Ans. (d) Article 51 of the Constitution is related to
move the resolution.
Indian Foreign Policy. Article 51(a) is related to
promote international peace and security, Article 51(b) (2) Such resolution has been passed by a majority of
is related to maintain just and honourable relations not less than two-thirds of the total membership of
between nations and Article 51(c) is to foster respect for that House.
international law and treaty obligations in the dealings (3) When a charge has been so preferred by either
of organized people with one another and encourage House of Parliament, the other House shall
settlement of international disputes by arbitration. investigate the charge or cause the charge to be
28. Fundamental Duties are listed in which Article investigated and the President shall have the right
of the Constitution? to appear and to be represented at such
(a) Article 50A investigation.
(b) Article 50 B (4) If as a result of the investigation, a resolution is
(c) Article 51A passed by a majority of not less than two-third of
(d) Article 51B the total membership of the House by which the
Uttarkand PCS (Pre) G.S, 2006 charge was investigated or caused to be

Indian Polity & Constitution 26 YCT


investigated, declaring that the charge preferred 34. Which one of the following Articles of the
against the President has been sustained. Such Indian Constitution provides that ‘It shall be
resolution shall have the effect of removing the the duty of the Union to protect every State
President from his office as from the date on which against external aggression and internal
the resolution is so passed. disturbance’?
31. Which one of the following Articles of the (a) Article 215 (b) Article 275
Constitution provides that the Speaker shall (c) Article 325 (d) Article 355
have and exercise a casting vote in the case of IAS (Pre) G.S., 2003
an equality of votes? Ans. (d) Article 355 of the Constitution asserts that it
(a) Article 99 (b) Article 103 shall be the duty of the Union to protect States against
(c) Article 100 (d) Article 102 external aggression and internal disturbance.
UPPCS (Pre) Re-Exam. G.S., 2015 35.
The Money Bill is defined in which article of
Ans. (c) Article 100 of the Indian Constitution provides the Constitution of India?
that the speaker shall have and exercise a asting vote in (a) Article109 (b) Article 110
case of and equality of votes. The Presiding officer of a (c) Article 111 (d) Article 112
House does not vote in first instance, but exercise a UPPCS (Pre) G.S, 2019
casting vote in case of an equality of votes. Ans. (b) Article 110 of the Constitution provides
Article 99 is related to Oath or affirmation by members
definition of Money Bill. A Bill shall be deemed to be a
of Parliament. Money Bill if it contains the imposition, abolition,
remission, alteration or regulation of any tax, the
Article 102 makes provision regarding disqualifications
for membership of Parliament. regulation of the borrowing of money or the giving of
Article 103 is related to decision on questions as toany guarantee by the Government of India, the custody
disqualifications of members. of the Consolidated Fund or the Contingency Fund of
India. If any question arises whether a Bill is a Money
32. Under which Article of the Indian Constitution
Bill or not, the decision of the Speaker of the Lok Sabha
did the President give his assent to the
shall be final.
ordinance on electoral reforms when it was
sent back to him by the Union Cabinet without 36. Under which Article of the Constitution,
making any changes (in the year 2002)? Money Bill has been defined?
(a) Article 121 (a) 109 (b) 110
(b) Article 122 (c) 111 (d) Both b and c
(c) Article 123 UPPCS (Pre) G.S., 2000
(d) Article 124 Ans. (b) Kindly refer the explanation of the above
IAS (Pre) G.S., 2003 question.
Ans. (c) Article 123 of the Constitution is power of the 37. Who decides whether a bill is a money bill or
President to promulgate Ordinances during recess of not?
Parliament. It may be withdrawn at any time by the (a) President
President. An Ordinance promulgated under this Article (b) Finance Minister
shall have the same force and effect as an Act of (c) Speaker of Lok Sabha
Parliament but every such Ordinance shall be laid (d) Chairperson of Rajya Sabha
before both House of Parliament and shall cease to Himanchal PSC (Pre) GS 2010
operate at the expiration of six weeks from the Ans. (c) Kindly refer the explanation of the above
reassemble of Parliament. question.
33. Under which Article of the Constitution of 38. Which one of the following provisions deal with
India, President of India has power to the privileges and immunities of the Members
promulgate ordinance? of Parliament under the Constitution of India?
(a) Article 360 (b) Article 123 (a) Article 104
(c) Article 200 (d) Article 356 (b) Article 105
UPPCS (Main) G.S. IInd Paper, 2016 (c) Article 82
Ans. (b) Kindly refer the explanation of the above (d) Article 117
question. Chhattisgarh PSC (Pre) G.S 2010
Indian Polity & Constitution 27 YCT
Ans. (b) Article 105 of the Constitution is related to creation of such a Council in a State having no such
powers, privileges of the Houses of Parliament and of Council, if the Legislative Assembly of the State passes
the members and committees. No Member of a resolution to that effect by a majority of the total
Parliament shall be liable to any proceedings in any membership of the Assembly and by a majority of not
court in respect of anything said or any vote given by less than two thirds of the members of the Assembly
him in Parliament. present and voting.
39. Curative Petition in India can be filed in 42. Which of the following CAN NOT be dissolved
Supreme Court under Article but abolished?
(a) 138 (b) 140 (a) House of People
(c) 142 (d) 146 (b) Council of State
UPPCS (Main) G.S. IInd 2014 (c) Legislative Assembly
Ans. (*) None of the given options are correct. The (d) Legislative Council
concept of curative petition was evolved by Supreme UPPCS (Pre) G.S, 2018
Court during Rupa Ashok Hura Vs.Ashok Hura case, Ans: (d) Kindly refer the explanation of the above
where the issue raised pertained to whether an question.
aggrieved person is entitled to any relief against the
43. Which Article of the Indian Constitution
final judgment/order of the Supreme Court after the
provided the provision of Vidhan Parishad in
dismissal of a review petition. The provision has been
the State?
made under Article 137 of the Constitution, which
(a) Article 170 (b) Article 171
allows a review of order passed by Supreme Court.
Article 138 is related with Enlargement of the (c) Article 172 (d) Article 173
Jurisdiction of the Supreme Court. MPPSC (Pre) GS 1st Paper 2014
Article 140 states the ancilliary powers of Supreme Ans. (*) None of the given options are correct as Article
Court. 169 of the Constitution is related to abolition or creation
Article 142 is related with enforcement of decrees and of Legislative Councils (Vidhan Parishad) in States.
orders of Supreme Court and orders as to discoury etc. Article 170 is related to Composition of the Legislative
Article 146 states the officers, servants and expenses of Assemblies. Article 171 of the Constitution provides
the Supreme Court. provision for composition of the Legislative Councils
provided the condition that the total number of members
40. Which Article of the Constitution permits the
in the Legislative Council of a State shall not exceed
Supreme Court to review its own judgment or
one third of the total number of members in the
order?
Legislative Assembly of that State. One third of the
(a) Article 137 (b) Article 130
members of the Legislative Council shall be elected by
(c) Article 139 (d) Article 138
electorates consisting of members of municipalities,
UPPCS (Main) G.S. IInd Paper 2009 district boards and such other local authorities in the
R.A.S (Pre) G.S, 2003 State, one twelfth shall be elected by graduates residing
U.P.P.C.S. Spl (Pre) G.S, 2005 in State ,one twelfth by teachers, one third shall be
U.P.P.P.C.S (Pre) G.S, 1998, 2003 elected by the members of the Legislative Assembly of
Ans. (a) Kindly refer the explanation of the above the State from amongst persons who are not members of
question. the Assembly and the remainder shall be nominated by
41. Procedure for creation of Legislative Council the Governor, consist of persons in the field of
in States has been described in which Article of literature, science, art, cooperative movement and social
the Constitution? service. Article 172 is regarding duration of State
(a) Article 368 (b) Article 69 Legislature while Article 173 is about qualification for
(c) Article 269 (d) Article 169 membership of the State Legislature.
UPPCS (Main) G.S. IInd Paper, 2007 44. Under which Article of the Indian
Ans. (d) Article 169 of the Constitution is related to Constitution, there is provision for the Election
abolition or creation of Legislative Councils in States. Commission?
According to Article 169 of the Constitution, Parliament (a) Article 320 (b) Article 322
may by law provide for the abolition of the Legislative (c) Article 324 (d) Article 326
Council of a State having such a Council or for the UPPCS (Pre) G.S., 2016
Indian Polity & Constitution 28 YCT
Ans. (c) According to Article 324 of the Constitution, Ans. (d) Kindly refer the explanation of the above
the superintendence, direction and control of the question.
preparation of the electoral rolls and the conduct of all 48. Under which Article of the Indian Constitution,
elections to Parliament and to the Legislature of every the Parliament can make laws on State list
State and of elections to the offices of President and
subject for giving effect to international
Vice President shall be vested in the Election
agreements?
Commission.
(a) Article 249 (b) Article 250
Article 138 states the functions of Public service
commissions. (c) Article 252 (d) Article 253
Article 322 is related with expenses of Public service UPPCS (Main) G.S. IInd 2012
commissions. UPPCS (Main) G.S. IInd 2017
45. The Constitution of India provides for an Ans. (d) Article 253 is related to legislation for giving
Election Commission under Article effect to international agreements which asserts that
(a) 321 (b) 322 Parliament has power to make any law for the whole or
(c) 323 (d) 324 any part of the territory of India for implementing any
UPPCS (Main) G.S. IInd Paper, 2010 treaty, agreement or convention with any other country
UPPCS (Main) G.S. IInd 2017 or countries or any decision made at any international
MPPSC (Pre) GS 1st Paper 2014 conference, association or other body.
Ans. (d) Kindly refer the explanation of the above 49. Which Article of the Constitution deals to
question. provide necessary facilities by the States for the
46. A resolution passed by the Rajya Sabha under use of mother tongue in their primary
Article 249 of the Constitution empowering education?
Parliament to legislate on a State subject in (a) Article 349 (b) Article 350
national interest remains in force for a period, (c) Article 350(A) (d) Article 351
(a) Not exceeding six months IAS (Pre) G.S., 2001
(b) Not exceeding two years UPPCS (Pre) G.S., 2002
(c) Not exceeding one year U.P. Lower (Pre) G.S., 2002
(d) Of unlimited time Chhattisgarh PSC (Pre) G.S., 2003
UPPCS (Pre) G.S, 2017 Ans. (c) Article 350(A) is regarding facilities for
Ans. (c) Article 249 deals with power of Parliament to instruction in mother-tongue at primary stage. It asserts
legislate with respect to a matter in the State List in the that it shall be the endeavour of every State and of every
national interest. If the Council of State (Rajya Sabha) local authority within the State to provide adequate
has declared by resolution supported by not less than facilities for instruction in the mother-tongue at the
two third of the members present and voting that it is primary stage of education to children belonging to
necessary or expedient in national interest that linguistic minority groups.
Parliament should make laws with respect to any matter
50. Match List - I with List - II and select the
enumerated in the State List specified in the resolution,
correct answer using the codes given below the
it shall be lawful for Parliament to make laws for the
lists
whole or any part of the territory of India. Resolution
passed shall remain in force for such period not List – I List – II
exceeding one year. (Institution) (Articles)
47. Article 249 of the Indian Constitution deals (A) Comptroller and 1. Article 315
with AuditorGeneralof
(a) Emergency Power of the President India
(b) Dissolution of the Lok Sabha (B) Finance 2. Article 280
(c) Administrative Power of the Parliament Commission
(d) Legislative Powers of the Parliament with (C) Administrative 3. Article 148
respect to a matter in the State list Tribunal
UPPCS (Main) G.S. IInd Paper, 2009 (D) Union Public 4. Article
UPPCS (Main) G.S. IInd 2012 Service 323(A)
UPPCS (Main) G.S. IInd 2017 Commission
Indian Polity & Constitution 29 YCT
Codes: Ans. (c) Article 312 is related to All India Service
A B C D which asserts that if the Council of States (Rajya
(a) 3 2 4 1 Sabha) has declared by resolution supported by not
(b) 3 4 2 1 less than two third of the members present and voting
that it is necessary or expedient in the national interest
(c) 1 2 4 3
so to do, Parliament may by law provide for the
(d) 4 1 3 2 creation of one or more all India services common to
UPPCS (Pre.) G.S., 2016 the Union and the States.
Ans. (a) Article 148 asserts that there shall be a 54. It has been mentioned under which Article of
Comptroller and Auditor General of India who shall be the Constitution that without the proper
appointed by the President. Article 280 is related to litigation neither tax could be imposed, nor
Finance Commission. Article 323(A) deals with collected?
Administrative Tribunal and Article 315 asserts that (a) Article 265 (b) Article 266
there shall be one Public Service Commission for the (c) Article 300 (d) Article 368
Union and a Public Service Commission for each State. UPPCS (Main) G.S. IInd Paper, 2007
51. The provision for the representation of Anglo- Ans. (a) Article 265 asserts that no tax shall be levied or
Indian community in the Lok Sabha has been collected except by authority of law, Article 266 is
made in the Constitution under: related to Consolidated Funds and Public Accounts of
(a) Article 331 (b) Article 221 India and of the States and Article 300 is related to suits
(c) Article 121 (d) Article 139 and proceedings while Article 368 is power of
Parliament to amend the Constitution.
UPPCS (Pre) Re-exam. G.S., 2015
55. Constitution can be amended under which
UPPCS (Main) G.S. IInd, 2015
Article of the Constitution?
UPPCS (Pre) G.S. 2013
(a) Article 368 (b) Article 360
UPPCS (Main) G.S. IInd 2017 (c) Article 356 (d) Article 352
Ans. (a) Article 331 of the Constitution asserts that the UPPCS (Main) G.S. IInd Paper, 2008
President may, if he is of opinion that the Anglo Indian Ans. (a) Article 368 of the Constitution is power of
community is not adequately represented in the House Parliament to amend the Constitution. It asserts that
of the people (Lok Sabha), nominate not more than two Parliament may in exercise of its constituent power
members of that community to the Lok Sabha. Article amend by way of addition, variation or repeal any
333 is related to the representation of one member of provision of the Constitution in accordance with the
Anglo Indian community in the Legislative Assemblies procedure laid down in the Constitution.
of the States by the Governor. 56. President rule can be imposed on the State
52. Special provision under Article 371 of the under which Article of the Constitution?
Constitution has been made for which of the (a) Article 370 (b) Article 368
following State? (c) Article 356 (d) Article 352
(a) Jammu & Kashmir UPPCS (Main) G.S. IInd Paper, 2012
(b) Maharashtra and Gujarat Ans. (c) Article 356 of the Constitution asserts that if
(c) Nagaland the President, on report from the Governor of the State
(d) Andhra Pradesh or otherwise, is satisfied that a situation has arisen in
which the Government of the State cannot be carried on
UPPCS (Main) G.S. IInd Paper, 2011
in accordance with the provisions of the Constitution,
Ans. (b) Article 371 is related to special provision with the President may impose President Rule in that State.
respect to the States of Maharashtra and Gujarat During President Rule, the President assumes all the
regarding the establishment of separate development powers of the State Government, exercisable by the
boards. State Governor or shall be exercisable by or under the
53. Under which Article of the Indian Constitution authority of Parliament.
provisions has been made for All India 57. Under which Article, President of India could
Service? declare ‘National Emergency’?
(a) Article 310 (b) Article 311 (a) Article 352 (b) Article 370
(c) Article 312 (d) Article 313 (c) Article 371 (d) Article 395
UPPCS (Main) G.S. IInd Paper, 2012 BPSC (Pre) G.S, 2011

Indian Polity & Constitution 30 YCT


Ans. (a) : Article 352 is regarding the proclamation of Ans. (c) According to Article 345 of the Constitution,
the national emergency which asserts that if the President Legislature of a State may by law adopt any one or
of India is satisfied with a grave emergency exists more of the languages in use in the State. Article 343
whereby the security of India or any part is threatened, asserts that the official language of the Union shall be
whether by war or external aggression or an armed Hindi in Devanagari script. Article 344 is Commission
rebellion, then he may proclaim a state of national and Committee of Parliament on official language while
emergency for the whole of India or a part of India. The Article 346 is related to official language of
proclamation of the emergency can be questioned in communication between the States and to the Union.
court and must be approved by both Houses of
62. The mandate of the official language
Parliament within one month of proclamation. The effect
commission under Article 344 is/are
of the proclamation of emergency is the emergence of the
full-fledged Unitary Government. 1. The progressive use of the Hindi language
for the official purposes of the Union
58. Under which of the following circumstances
can an emergency be declared in India? 2. Restrictions on the use of the English
language for all or any of the official
(1) External aggression
purposes of the Union
(2) Armed rebellion
Choose the correct statement(s):
(3) Internal unrest
(a) 1 Only (b) 2 Only
(4) Financial crisis
(c) Both 1 and 2 (d) Neither 1 nor 2
Codes :
(a) (1), (2) and (3) (b) (1), (3) and (4) TNPSC (Pre) G.S. 2019
(c) (2), (3) and (4) (d) (1), (2) and (4) Ans. (c) According to Article 344(2) of the
Himanchal PSC (Pre) GS, 2010 Constitution, It shall be the duty of the Commission to
make recommendations to the President as to (a) The
Ans. (a) : Kindly refer the explanation of the above
question. progressive use of the Hindi language for the official
purposes of the Union (b) Restrictions on the use of the
59. Who amongst the following is empowered to
English language for all or any of the official purposes
declare emergency under the Indian
of the Union. Hence both the statements are correct.
Constitution?
(a) The Vice-President 63. By which of the following articles, the
(b) The President procedures for the Amendment of the Indian
Constitution is given?
(c) The Prime Minister
(a) Articles - 230 (b) Articles - 320
(d) The Home Minsiter
(c) Articles - 358 (d) Articles – 368
Himanchal PSC (Pre) GS, 2010
TNPSC (Pre) G.S. 2014
Ans. (b) Kindly refer the explanation of the above
question. Ans. (d) : Articles 230- Extension of jurisdiction of
60. Under which Article of the Indian High Courts to Union territories
Constitution, it is the duty of the Union govt. to Articles - 320 - Functions of Public Service
protect states against external aggression and Commissions
internal disturbance : Articles - 358 - Suspension of provisions of Article 19
(a) 355 (b) 356 during emergencies
(c) 352 (d) 360 Articles - 368 - Power of Parliament to amend the
RAS/RTS (Pre) G.S., 2008 Constitution and procedure
Ans. (a) According to Article 355, it is the duty of the 64. Match List I with List II and select the correct
Union to protect every State against external aggression answer using the codes given below the Lists:
and internal disturbance and to ensure that the List-I (Article of the Constitution) List-II
Government of every State is carried on in accordance (Content)
with the provisions of Constitution. List – I List - II
61. In which Article of Indian Constitution the (A) Article 54 1. Election of the
procedure for adoption of official language or President of India
languages of State is given? (B) Article 75 2. Appiontment of the
(a) 343 (b) 344 Prime Min-ister and
(c) 345 (d) 346 Council of the
RAS/RTS (Pre) G.S., 2002 Minister
Indian Polity & Constitution 31 YCT
(C) Article 155 3. Appiontment of 66. Match List-I with List-II and select the correct
Governor of state answer using the codes given below the list:
(D) Article 164 4. Appointment of the List – I List - II
Chief Minister and
(A) Article 14 1. Amendment
Council of Ministers
Of a state procedure
5. Composition of (B) Article 36 2. Council of Ministers
Legislative (C) Article 74 3. Right of Equality
Assemblies (D) Article 368 4. Directive Principles
Codes: Code:
A B C D
A B C D
(a) 1 2 3 4
(a) 2 1 4 3
(b) 1 2 4 5
(c) 2 1 3 5 (b) 4 1 3 2
(d) 2 1 4 3 (c) 1 2 3 4
IAS (Pre) G.S., 2001 (d) 3 4 2 1
Ans. (a) Correctly matched list is given below- UPPCS (Pre) GS., 2010
List I List Ans. (d) Correct match is given below-
A. Article 54 1. Election of President List- I List- II
B. Article 75 2. Appointment of the A. Article 14 1. Right of Equality
Prime Minister and
B. Article 36 2. Directive Principles
Council of the Minister
C. Article 3. Appointment of C. Article 74 3. Council of Ministers
155 Governor of State D. Article 4. Amendment procedure
D. Artice 4. Appointment of the 368
164 Chief Minister and 67. Match List-I with List-II and select the correct
Council of Ministers of
answer from the codes given below the lists :
a state
List – I List – II
65. Match List-I and List-II and select the correct
answer from the codes given below: (A) Article 76 1. Comptroller and
List–I List–II Auditor General
(A) Constitutional 1. Article 360 (B) Article 148 2. Jurisdiction of
Amendments Supreme Court
(B) Finance 2. Article 312 (C) Article 75 (1) 3. Attorney General
Commission of India
(C) Financial 3. Article 280 (D) Article 131 4. Appointent of
Emergency Union Minsters
(D) All India Services 4. Article 368 Codes:
Codes: A B C D
A B C D (a) 1 2 3 4
(a) 1 3 4 2 (b) 2 1 4 2
(b) 4 3 1 2 (c) 4 1 2 3
(c) 4 1 3 2 (d) 2 3 4 1
(d) 3 1 4 2 UPPCS (Main) G.S. IInd 2015
UPPCS (Pre) G.S., 2016 Ans. (*) Correct match is given below-
Ans. (b) The correct match is given below- List –I List -II
A. Constitutional Amendments Article 368 A. Article 76 1. Attorney General
B. Finance Commission Article 280 of India
C. Financial Emergency Article 360 B. Article 148 2. Comptroller and
D. All India Services Article 312 Auditor General

Indian Polity & Constitution 32 YCT


C. Article 75 (1) 3. Appointment of Codes:
Union Minister A B C D
D. Article 131 4. Jurisdiction of (a) 1 2 3 4
Supreme Court (b) 2 1 4 3
68. Match List-I with List-II and select the correct (c) 2 3 1 4
answer from the codes given below the lists: (d) 2 1 3 4
List – I List – II UPPCS (Main) G.S. IInd Paper, 2006
UP Lower (Pre) Spl. G.S, 2002
(A) Article 164 1. Appointment of
Chief Minister and Ans. (b) Correctly match is given below-
Council of Ministers List –I List-II
(B) Article 155 2. Appointment of A. Article 14 1. Right of Equality
Governor B. Article 2 2. By law admit new
(C) Article 75 3. Appointment of States
Prime Minister and C. Article 111 3. Power of President to
Council of Ministers give assent to the Bill
(D) Article 54 4. Election of President D. Artice 22 4. Protection against arrest
of India and detention
5. Structure of Legisl- 70.Which one of the following is correctly
ative Assemblies. matched?
Codes: (a) Article 17-Equality before law
A B C D (b) Article 78-Formation of Parliament
(a) 1 2 3 4 (c) Article 193-Salary and perks of Members of
(b) 1 2 4 5 Parliament
(c) 2 1 3 5 (d) Article 352-Declaration of emergency
(d) 2 1 4 5 UP Lower (Pre) G.S., 2002
UPPCS (Main) G.S. IInd Paper, 2005, 2011 Ans. (d) : Article 17 is related to abolition of
UP Lower (Pre) G.S., 2002untouchability, Article 78 is duties of Prime Minister as
respects the furnishing of information to the President,
Ans. (a) Correct match is given below-
Article 22 is related to protection against arrest and
List- I List- II detention in certain cases. Article 193 is related to
A. Article164 1. Appointment of Chief Penality for sitting and voting before making Oath or
Minister and Council of affirmation under Article 188 or when not qualified or
Ministers when disqualified. Article 352 is declaration of national
B. Article155 2. Appointment of Governor emergency on the basis of external aggression and
armed rebellion. Hence d is the correct answer.
C. Article 75 3. Appointment of Prime
Minister and Council of 71. Which one of the following is not correctly
Ministers matched?
Article of the Indian Related State
D. Artice 54 4. Election of President of
Constitution
India
(a) 371A Nagaland
69. Match List-I with List-II and select the correct
(b) 371B Assam
answer from the codes given below the lists:
(c) 371C Meghalaya
List–I List–II
(d) 371D Andhra Pradesh
A. Article 14 1. By law admit new MPPCS (Pre) G.S, 2018
states
Ans. (c)
B. Article 2 2. Right of Equlity Article 371 Special provisions for Maharashtra
C. Article 111 3. Portection against and Gujarat
arrest and detention Article 371A Special provisions for Nagaland
D. Article 22 4. Power of President to Article 371B Special provisions for Assam
give assent to the Bill Article 371C Special provisions for Manipur
Indian Polity & Constitution 33 YCT
Article 371D Special provisions for Andhra Pradesh Schedule 5- Provisions as to the Administration and
and Telangana Control of Scheduled Areas and Scheduled Tribes
Article 371F Special provisions for Sikkim Schedule 6- Provisions as to the Administration of
Article 371G Special provisions for Mizoram Tribal Areas in the States of Assam, Meghalaya,
Tripura and Mizoram
Article 371H Special provisions for Arunanchal
Schedule 7- Union List (97), State List (66), Concurrent
Pradesh
List (47)
72. Under which Article of the Constitution of India, Schedule 8- Languages
the Governor has the power to promulgate Schedule 9- Validation of certain Acts and Regulations
ordinances during recess of Legislature? (Land Reforms) (1st Amendment, 1951)
(a) 155 (b) 156 Schedule 10- Provisions as to disqualification on
(c) 212 (d) 213 ground of defection (52nd Amendment, 1985)
MPPCS (Pre) G.S, 2018 Schedule 11- Powers, authority and responsibilities of
Ans. (d) Article 213 of the Constitution is power of Panchayats (73rd Amendment, 1993)
Governor to promulgate Ordinances during recess of Schedule 12- Powers, authority and responsibilities of
Legislature. If at any time except when the Legislative Municipalities (74th Amendment, 1993)
Assembly of a State is in session or when both Houses of 74. Match the following:
the Legislature are in session (where there is a Legislative Schedule Subject
Council in a State), the Governor is satisfied that (a) First Schedule (1) Division of powers
circumstances exist which render it necessary for him to between Union and
take immediate action, he may promulgate such States
Ordinance. An Ordinance promulgated under this Article (b) Eleventh Schedule (2) Languages
shall have the same force and effect as an Act of (c) Seventh Schedule (3) Names of States and
Legislature of the State but every such Ordinance shall be Union Territories
laid before the legislative Assembly of the State or before (d) Eighth Schedule (4) Panchayats
both the House (where there is a Legislative Council in Codes :
the State) and shall cease to operate at the expiration of (a) (b) (c) (d)
six weeks from the reassembly of the Legislature. (a) 3 4 1 2
73. Which one of the following Schedules of the (b) 1 3 4 2
Constitution of India contains provisions (c) 2 1 3 4
regarding anti-defection? (d) 3 1 4 2
(a) Second Schedule (b) Fifth Schedule TNPSC (Pre) G.S. 2019
(c) Eighth Schedule (d) Tenth Schedule Ans. (a) Correct match is-
IAS (Pre) G.S., 2014 First Schedule - Names of States and Union
Himanchal PSC (Pre) G.S. 2010 Territories
Ans. (d) Basically, Schedules are list which contains Eleventh Schedule - Panchayats
additional information which is not mentioned in Seventh Schedule - Division of powers between
Articles. Originally there were eight Schedules in the Union and States
Indian Constitution. Four more Schedules were added Eighth Schedule - Languages
through Amendment. Twelve Schedules in the Indian 75. Match List-I with List-II and select the correct
Constitution are- answer from the codes given below the list-
Schedule 1- The States, Union Territories List-I List-II
Schedule 2- Provisions (emoluments) as to the (A) 5thSchedule (1) Union list, State list
President and the Governors of States, Speaker and the and Concurrent list
Deputy Speaker of the Lok Sabha and the Chairman and (B) 6th Schedule (2) Administration and
Deputy Chairman of the Rajya Sabha, the Speaker and control of Scheduled
Deputy Speaker of the Legislative Assembly and the Areas and Scheduled
Chairman and Deputy Chairman of the Legislative Tribes
th
Council of a State, Judges of the Supreme Court and of (C) 7 Schedule (3) Languages
the High Courts, Comptroller and Auditor-General of (D) 8thSchedule (4) Adminstration of
India Tribal Areas in the
Schedule 3- Forms of Oaths and Affirmations state of Assam,
Schedule 4- Allocation of seats in Council of States Meghalaya, Tripura
(Rajya Sabha) and Mizoram

Indian Polity & Constitution 34 YCT


Codes : (c) The Ninth Schedule was added to the
(A) (B) (C) (D) Constitution by the First Amendment.
(a) (2) (4) (1) (3) (d) Anti-defection law is in the 10th Schedule of
(b) (3) (1) (4) (2) the Constitution and it was added by 42nd
(c) (2) (3) (4) (1) Constitutional Amendment.
(d) (3) (1) (2) (1) Punjab PSC (Pre) G.S 2018
UPPSC ACF-RFO Mains (IInd Paper), 2019 Ans. (d) : Anti-defection law is in the 10th Schedule of
Ans. (a) : Correct match is- the Constitution and it was added by 52nd Constitutional
List-I List-II Amendment in 1985.
(A) 5thSchedule (2) Administration and control 78. The second Schedule of the Indian Constitution
of Scheduled Areas and mentions salary, allowances and other
Scheduled Tribes privileges of certain officials. Which one of the
(B) 6th Schedule (4) Adminstration of Tribal following is not amongst those officials?
Areas in the State of (a) The Deputy Speaker of Legislative Assembly
Assam, Maghalaya, (b) The Deputy Chairman of Legislative Council
Tripura and Mizoram (c) The Attorney General of India
(C) 7thSchedule (1) Union list, State list (d) The Comptroller and Auditor General of
and Concurrent list India
th
(D)8 Schedule (3) Languages Maharashtra PSC (Pre) G.S 2016
76. Match the List–I with List–II and select the Ans. (c) The salary, allowances and other privileges of
correct answer from the codes given below: the Attorney General of India is not mentioned in the
List–I List–II second Schedule of the Constitution.
th
A. 7 Schedule 1. Languages 79. The provisions in Fifth Schedule and Sixth
B. 8th Schedule 2. Disqualification on Schedule in the Constitution of India are made
the ground of in order to
defection (a) Protect the interests of Scheduled Tribes
C. 9th Schedule 3. Union, State and (b) Determine the boundaries between States
Concurrent lists (c) Determine the powers, authority and
D. 10 th
4. Validation of responsibilities of Panchayats
Schedule certain Acts (d) Protect the interests of all the border States
Codes: IAS (Pre) G.S., 2008, 2015
A B C D Ans. (a) The provisions in Fifth Schedule and Sixth
(a) 4 2 1 3 Schedule in the Constitution of India are made in order
(b) 3 1 2 4 to protect the interests of Scheduled Tribes. 6th Schedule
(c) 2 3 4 1 is related to Administration of triable areas inm the state
(d) 3 1 4 2 of Assam, Meghalaya, Tripura and Mizoram.
UPPSC RO/ARO (Pre), 2016 80. Under which Schedule of the Constitution of
(Held on 20/09/2020) India can the transfer of tribal land to private
Ans- (d) Correct match is- parties for mining be declared null and void?
7th Schedule- Union, State and Concurrent lists (a) Third Schedule (b) Fifth Schedule
8th Schedule- Languages (c) Ninth Schedule (d) Twelfth Schedule
th
9 Schedule- Validation of certain Acts IAS (Pre) G.S, 2019
10th Schedule- Disqualification on the ground of Ans : (b) There are provisions regarding administration
defection and control of Scheduled areas and Scheduled Tribes in
77. Which of the following is incorrect with regard the Fifth Schedule of the Constituion. The Constitution
to the Schedules listed in the Constitution of provides autonomy to tribal areas in matters of
India ? governance under the Fifth and Sixth Schedules. In
(a) There are total twelve Schedules. Samatha Vs. State of Andhra Pradesh and Others (1997)
(b) Schedules XI and XII were incorporated case, Supreme Court declared that the transfer of tribal
respectively by 73rd and 74th Constitutional land to private parties for mining was null and void
Amendments. under the Fifth Schedule of the Constituion.
Indian Polity & Constitution 35 YCT
81. Under which schedule of the Constitution of Ans. (b) State land reforms are added through 9th
India can be transfer of tribal land of privateSchedule of the Constituion.
parties for mining be declared null and void? This schedule was added by the 1st constitutional
(a) Third Schedule (b) Five Schedule Amendment in 1951 to protect the law included in it
(c) Ninth Schedule (d) Twelth Schedulefrom Judicial Scruting on the ground of Violation of
Himanchal PSC (Pre) G.S. 2019Fundamental Right.
Ans : (b) Kindly refer the explanation of the above 87. Which one of the following Schedules of the
question. Constitution of India contains provisions
82. How many Schedules are there in Indian regarding Anti-Defection Act?
Constitution? (a) Second Schedule (b) Fifth Schedule
(a) 8 Schedules (b) 10 Schedules (c) Eighth Schedule (d) Tenth Schedule
(c) 12 Schedules (d) 15 Schedules IAS (Pre) G.S., 1998
MPPSC (Pre) G.S, 2010 UPPCS (Main) G.S. IInd Paper 2005
Ans : (c) There are 12 Schedules in the Indian Chattisgarh PSC (Pre) 1st, 2012
Constitution. BPSC (Pre) 1996
83. If a new state of the Indian Union is to be Ans. (d) Tenth Schedule contains provisions regarding
created, which one of the following Schedules Anti- Defection Law.
of the Constitution must be amended? 88. Eleventh Schedule of the Indian Constitution is
(a) First (b) Second related to –
(c) Third (d) Fourth (a) Panchayati Raj (b) Municipality
UPPCS (Main) G.S. IInd Paper 2011 (c) Union-State relations (d)None of the above
Ans. (a) If a new state is to be created, first Schedule of UPPCS (Main) G.S.-IInd Paper, 2006
the Constitution is to be amended. Chattisgarh PSC (Pre) G.S. 2010-11
84. Which one of the following Schedules of the BPSC (Pre) 1994
Indian Constitution lists the names of states Ans. (a) Eleventh Schedule of the Indian Constitution is
and specifies their territories? related to Panchayati Raj. This Amendment was added
(a) First (b) Second in Indian Constitution in 1992 by 73rd Amendment.
(c) Third (d) Fourth 89. How many items are there in the 11th Schedule
UPPCS (Main) G.S. IInd 2005 of the Constitution of India?
Ans. (a) First Schedule of the Constituion lists the (a) 22 (b) 24
names of States and specifies their territories. (c) 29 (d) 32
UPPCS (Pre) G.S, 2019
85. Which one of the following statement correctly
describes the Fourth Schedule of the Ans: (c) There is 29 items related to Panchayati Raj in
Constitution of India? the 11th Schedule of the Constituion of India.
90. Consider the following statements:
(a) It lists the distribution of power between the
Union and the States 1. The Parliament of India can place a particular
(b) It contains the languages listed in the law in the Ninth Schedule of the Constitution
Constitution of India.
(c) It contains the provisions regarding the 2. The validity of a law placed in the Ninth
administration of tribal areas Schedule cannot be examined by any court
(d) It allocates seats in the Council of States and no judgement can be made on it.
IAS (Pre) G.S., 2001 Which of the statements given above is/are
correct?
UPPCS Mains G.S. IInd 2015
(a) 1 only (b) 2 only
Ans. (d) The Fourth Schedule of the Constituion is
(c) Both 1 and 2 (d) Neither 1 nor 2
regarding allocation of seats in the Council of States
IAS (Pre) G.S, 2018
(Rajya Sabha).
Ans. (a) It was the First Amendment in 1951 along with
86. In order to provide constitutional protection, Article 31B under the priministership of Jawahar Lal
State land reforms are added to Nehru that marked the addition of the Ninth Schedule to
(a) 7th Schedule (b) 9th Schedule the Constitution. This was intended to protect land
(c) 8th Schedule (d) 10th Schedule reform laws from being challenged in Courts on the
UPPCS (Main) G.S. 2003 grounds of violation of Fundamental Rights. The
Indian Polity & Constitution 36 YCT
Parliament has the power to place a particular law in the Schedule is regarding allocation of seats in Council of
Ninth Schedule of the Constitution. Hence statement 1 States (Rajya Sabha). Hence (c) is not correctly
is correct. On I.R.Coehlo case, nine Judge Constitutio- matched.Tenth Schedule is related to Anti- Defection law.
nal bench of Supreme Court held that any law placed 94. Which one of the following subjects is under
under Ninth Schedule after April 23,1973 are subject to the Union List in the Seventh Schedule of the
scrutiny of Court if they violated Fundamental Rights Constitution of India?
and thus put the check on the misuse of the provision of (a) Regulation of labour and safety in mines and
the Ninth Schedule by the legislative. Hence statement oilfields
2 is incorrect. (b) Agriculture
91. The Ninth Schedule was introduced in the (c) Fisheries
Constitution of India during the prime (d) Public health
ministership of IAS (Pre) G.S., 2006
(a) Jawaharlal Nehru (b) Lal Bahadur Shastri th
Ans. (a) Union List under 7 Schedule of the
(c) Indira Gandhi (d) Morarji Desai Constituion has 99 entries which has verity of subjects
UPPCS (Pre) G.S, 2019 like preventive detention for reasons connected with
Ans : (a) Kindly refer the explanation of the above defence, foreign affairs, security of India, atomic
question. energy, interstate trade and commerce, cultivation,
92. Match List-I with List-II and select the correct manufacture and sale for export of opium, estate duty in
answer from the codes given below the lists : respect of property other than agricultural land, duties in
List – I List -II respect of succession to property other than agricultural
th land, regulation of labour and safety in mines and
A. 7 Schedule 1. Language
oilfields etc.
B. 8th Schedule 2. Disqualification on
95. Which of the following subjects are a part of
ground of defection
‘State List’ as specified in the Seventh Schedule
C. 9th Schedule 3. Distribution of of the Constitution of India?
legislative power 1. Markets and fairs
th
D. 10 Schedule 4. Validation of certain 2. Public health and sanitation; hospitals and
Articles dispanceries
Codes: 3. Cultivation, manufacture, and sale for export
A B C D of opium
(a) 3 1 2 4 4. Insurance
(b) 2 3 4 1 5. Relief of disabled and unemployable
(c) 3 1 4 2 Choose the corrct answer from the following:
(d) 4 2 1 3 (a) 1. 2 and 3 (b) 2, 3 and 5
UP RO/ARO (Pre) G.S, 2016 (c) 3, 4 and 5 (d) 1, 2 and 5
Ans. (c) Punjab PSC (Pre) G.S 2013
List- I List –II Ans. (d) Cultivation, manufacturing and sale of export
7th Schedule Distribution of legislative power of opium and insurance are the subjects mentioned in
Union List specified in Seventh Schedule of the
8th Schedule Language Constitution of India. Rest of the given subjects belong
9th Schedule Validation of certain Articles to State List.
th
10 Schedule Disqualification on ground of 96. Which of the following Entry is subject of
defection Concurrent List?
93. Which of the following is NOT correctly (a) Population control and family planning
matched? (b) Public health and sanitation
(a) Public Health and Sanitation State List (c) Per Person Tax
(b) Census Union List (d) Treasure Travel
(c) Allocation of seats Second IAS (Pre) G.S., 1993
in the Council of states Schedule Ans. (a) Concurrent List is a list of subjects appended
(d) Anti-Defection Tenth to a federal Constitution in respect of which the federal
Schedule legislature and the State legislatures have power to
UPPCS (Pre) G.S, 2019 make laws. Federal law prevails in case of conflicts.
Ans: (c) Public health and sanitation comes under State Concurrent List has 47 subjects. Population control and
List while cencus comes under Union List. Fourth family planning is subject of Concurrent List.

Indian Polity & Constitution 37 YCT


97. Under the Constitution of India, which pair of 102. Marriage, Divorce, Adaptation are mentioned
subjects and related list is not correctly matched? in which list of the seventh Schedule of
Subject List Constitution?
(a) Forest Concurrent List (a) List-I Union List
(b) Share Market Concurrent List (b) List-II State List
(c) Post office savings bank Union List (c) List-III Concurrent List
(d) Public Health State List (d) Not in any one of the list
UPPCS (Pre) G.S., 2009 UPPCS (Main) G.S. IInd Paper, 2006
UPPCS (Main) G.S. IInd 2007 UP Lower (Pre) G.S. (SPL. 2008
Ans. (b) Share Market has been mentioned in Union Ans. (c) Marriage, divorce and adaptation are listed in
List. Rest three are correct. Concurrent list of Seventh Schedule of the Constitution.
98. Which one of the following subject comes 103. Which one of the following is not mentioned
under Concurrent List? under the Union List of seventh schedule of
Indian Constitution?
(a) Police
(a) Banking (b) Insurance
(b) Criminal Procedure
(c) Census (d) Gas
(c) Radio and Doordarshan
UPPCS (Pre.) G.S., 1997
(d) Foreign Affairs
Ans. (d) Gas is listed under State List of the Seventh
UPPCS (Main) G.S. IInd Paper 2014
Schedule of the Constitution. Rest are mentioned in
Ans. (b) Criminal procedure comes under Concurrent Union List.
List while Police is State subject.Radio and
104. Which of the following are enshrined in the
Doordarshan, Foreign Affairs comes under Union List.
third schedule of the Indian Constitution?
99. Who of the follwoing is not included in the 1. Form of oath of office for a Minister for the
State-list given in the Constitution? Union.
(a) Criminal Procedure Code 2. Form of oath to be made by the Chief Justice
(b) Prisons of Supreme Court.
(c) Law of order 3. Form of oath of office for the President of
(d) Police India.
UPPSC ACF-RFO Mains (IInd Paper), 2019 4. Form of oath to be made by a Member of
Ans. (a) : Kindly refer the explanation of the above Parliament.
question. Select the correct answer using the code given
100. Which one of the following subject is not below :
included in the Union List of Indian Codes:
Constitution? (a) 1, 2, 3 (b) 2, 3, 4
(a) Defence (b) Foreign Affairs (c) 1, 2, 4 (d) 1, 2, 3, 4
(c) Railway (d) Agriculture UPPCS (Pre) G.S, 2017
Chattisgarh PSC (Pre) G.S. 2010-11 Ans. (c) Third Schedule provides forms of Oaths and
UPPCS (Pre) Opt. Pol. Science 2008 Affirmations of Minister for the Union Government,
Member of Parliament, Judges of the Supreme Court and
Ans. (d) Agriculture subject is mentioned in State list
High Courts, the Comptroller and Auditor-General of
under Schedule 7 of the Indian Constitution. Rest are
India, Minister for a State Government and member of
mentioned in Union List. the Legislature of the State. It is to be noted that oaths
101. Which one of the following subject is not and affirmations of President of India are not mentioned
included in State List of Indian Constitution? in the third Schedule. Hence c is the correct answer.
(a) Criminal Procedure 105. Coelho case is related to which Schedule of the
(b) Police Constitution of India?
(c) Law & Order (a) 7th (b) 8th
(d) Jail (c) 9th (d) 10th
UPPCS (Main) G.S. - IInd Paper, 2006 UPPCS (Main) 2017 G.S. IInd Paper
Ans. (a) Criminal Procedure subject is mentioned in Ans. (c) In Coelho case (popularly known as 9th
Concurrent List while rest three are mentioned in State Schedule case), the nine Judges bench of Supreme
List. Court gave a unanimous verdict upholding the authority
Indian Polity & Constitution 38 YCT
of the judiciary to review any law which destroy or 109. Which of the following is NOT correctly
damage the basic structure as indicated in fundamental matched as per Constitution of India?
rights, even if they have been put in 9th Schedule.
(a) The Panchayats – Part IX
106. Indian Constitution is divided in to how many
parts? (b) The Muncipalities – Part IX A
(a) Sixteen parts (b) Twenty two parts (c) The Cooperative – Part IX B
(c) Twenty four parts (d) Twenty five parts Societies
UPPCS (Main) G.S. IInd Paper 2012 (d) Tribunals – Part X
U.P. Lower Pre. 2004-05
UPPSC RO/ARO (Pre), 2016
UPPCS (Main) SPl. G.S. IInd 2008
(Held on 20/09/2020)
Ans. (b) The Constitution of India is divided in to
twenty two parts. Further three sub parts have been Ans. (d) The Constituion (Fourty-Second Amendment)
added namely IVA (Fundamental Duties), IX A Act, 1976 inserted Part XIV A in the Constituion
(Municipalities) and XIVA (Tribunals). consisting of Article 323A and 323B. Article 323A
107. Which one of the following is NOT correctly provides for the establishment of Administrative
matched? Tribunals for adjudication or trial of disputes and
(a) Fundamental Duties – Part IVA complaints with respect to recruitment and condition of
(b) The States – Part V service of persons appointed to public services. Article
(c) Attorney General of India – Part XIII
323B makes provision for the creation of tribunals for
(d) Services under the Union – Part XIV
adjudication or trial of disputes, complaints and
and States
offences connected with tax, foreign exchange,
UPPCS (Pre) G.S 2020
industrial and labour disputes, land reforms, ceiling on
Ans. (c) Article 76 under Part V of the Constituion
asserts that the President shall appoint a person who is urban property, elections to Parliament and State
qualified to be appointed a Judge of the Supreme Court Legislature. Where as part X is related to the scheduled
to be Attorney General for India. Rest is correctly and Tribal Areas.
matched. 110. Match List-I with List-II and select the correct
108. Match list-I with list-II and select the correct answer by using the codes given below:
answer from the codes given below-
List – I List –II
List-I List-II
(Part of the (Deals with subject) A. Part IX of the 1. TheUnion
Constitution) Indian Constituton Territories
(A) Part I (1) Citizenship B. Part VIII of the 2. The
(B) Part II (2) Fundamental Rights Indian Constitution Municipalities
(C) Part III (3) Directive C. Part IV A of the 3. The
(D) Part IV (4) The Union and its Indian Constitution Panchayats
Territory
D. Part IX A of the 4. Fundamental
Code :
Indian Constitution Duties
( a) ( b ) ( c) ( d )
(a) (2) (3) (1) (4) Codes:
(b) (4) (3) (2) (1) A B C D
(c) (4) (1) (3) (2) (a) 3 1 4 2
(d) (4) (1) (2) (3) (b) 1 2 3 4
UPPSC ACF-RFO Mains (IInd Paper), 2019 (c) 2 4 1 3
Ans. (d) List-I List-II (d) 4 3 2 1
(Part of the (Deals with subject)
UPPCS (Main) G.S. IInd Paper, 2009
Constitution)
(A) Part I (4) The Union and its Ans. (a) Part IX of the Constitution is related to
Territory Panchayats, Part VIII of the Constitution is related to
(B) Part II (1) Citizenship the Union Territories, Part IV A of the Constitution is
(C) Part III (2) Fundamental Rights related to Fundamental Duties and Part IX A of
(D) Part IV (3) Directive principal the Constitution is related to Municipalities.

Indian Polity & Constitution 39 YCT


111. Match List-I with List-II and select the correct 113. Which one of the following is correctly
answer by using the codes given below : matched?
(a) IInd part of the Constitution: Fundamental
List – I List -II Right
A. Fundamental 1. Part IVA, (b) IIIrd part of the Constitution: Citizenship
Rights Constitution of (c) IVA part of the Constitution: Fundamental
India Duties
th
B. Fundamental 2. Part III, (d) V part of the Constitution: Directive
Principle of State Policy
Duties Constitution of
UPPCS (Main) G.S. IInd Paper, 2007
India
Ans. (c) Fundamental Rights are in part III, Citizenship
C. Union 3. Part XVIII, is in part II and Directive Principle of State Policy is in
Constitu-tion of Part IV. Fundamental Duties are in part IVA. Hence c is
India the correct answer. The Union Government are in Part
V of the Constitution.
D. Emergency 4. Part V, Constitution
114. Match List-I with List-II and select the correct
Provisions of India answer by using the codes given below :
Codes: List – I List –II
A B C D A. Part XV of 1. Emergency
(a) 2 1 4 3 the Constitut Provisions
(b) 3 4 1 2 ion
B. Part XVI of 2. Official Language
(c) 2 1 3 4
the
(d) 4 3 2 1 Constitution
UPPCS (Main) G.S. IInd Paper, 2013 C. Part XVII of 3. Special provisions
Ans. (a) Fundamental Rights are mentioned in part III, the Constitut relating to certain
Fundamental Duties are in part IVA, Union is in part V ion cases
and Emergency Provisions are mentioned in part XVIII D. Part XVIII 4. Elections
of the
of the Indian Constitution. Indian Const
112. Part XVI (Article 330 to 342) is related to- itution
(a) Administrative Tribunals Codes :
(b) All India Services A B C D
(c) Finance Commission (a) 3 4 1 2
(b) 1 2 3 4
(d) Reservation of Schedule Caste, Schedule
(c) 4 3 2 1
Tribes in House of People and representation (d) 2 1 4 3
of Anglo-Indian community UP RO/ ARO (Pre) G.S., 2013
UPPCS (Main) Spl. G.S. IInd 2004 Ans. (c)
Ans. (d) Part XVI (Article 330 to 342) of the List I List
Constitution is related to special provisions relating to Part XV of Elections
certain classes. Article 330 is about reservation of seats the Constitution
for Scheduled Castes and Scheduled Tribes in the Lok Part XVI of the Special provisions
Sabha, Article 331 is representation of the Anglo-Indian Constitution relating to certain cases
community in the Lok Sabha while Article 332 is Part XVII of Official Language
the Constitution
related to reservation of seats for Scheduled Castes and
Part XVIII of the Emergency Provisions
Scheduled Tribes in the Legislative Assemblies of the
Indian Constitution
States. Article 333 is representation of Anglo-Indian
115. Union-State relationship has been mentioned in
community in Legislative Assemblies of the States. which part of the Constitution?
Article 338 is related to National Commission for SC (a) Part X (b) Part XI
and ST, Article 339 is control of the Union over the (c) Part XIV (d) Part XV
administration of Scheduled areas and welfare of the UP RO/ ARO (Pre) G.S., 2014
Scheduled Tribes. Article 340 is appointment of a UPPCS (Main) Spl. G.S. IInd Paper, 2004
commission to investigate the conditions of the Ans. (b) Union-State relationship has been mentioned
backward class. in Part XI of the Constitution.
Indian Polity & Constitution 40 YCT
116. In which Part of the Constiution of India we 121. Match List I (Article of Indian Constitution)
find the provisions relating to citizenship? with List II (Provisions) and select the correct
answer using the codes given below
(a) Part I (b) Part II
List – I List -II
(c) Part VII (d) Part IX
(Article of Indian (Provisions)
UPPCS (Pre) G.S, 2018 Constitution)
Ans: (b) Citizenship is mentioned in part II of the A. Article 1. No person shall be
Constitution. 16(2) deprived of his property
117. Which part of the Constitution is related to save by the authority of
law
citizenship?
B. Article 29 2. No person can be
(a) II (b) III discriminated against in
(c) IV (d) V the matter of public
Uttarakhand RO/ARO, 2016 appointment on the
Ans. (a) Kindly refer the explanation of the above ground of race, religion
or caste
question.
C. Article 30 3. All minorities whether
118. The concept of three layered Panchayat is based on religion or
mentioned in which part of our Constitution? language shall have the
(a) Part 9 (b) Part 10 fundamental right to
(c) Part 11 (d) Part 12 establish and administer
educational institutions
U.P. Lower (Pre.) G.S. 2013 of their choice
RAS/RTS (Pre) G.S., 2015 D. Article 31 4. No citizen shall be
Ans. (a) PART IX of the Constitution envisages a three- denied admission into
tier system of Panchayats, namely (a) The village level any educational insti-
(b) The District Panchayat at the district level (c) The tution maintained by the
Intermediate Panchayat which stands between the State, or receiving State
aid, on grounds of
village and district Panchayats in the States where the religion, race, caste,
population is above 20 lakhs. language or any of them.
119. Consider the following statements: Codes :
1. The Constitution of India has 20 parts. A B C D
2. There are 390 Articles in the Constitution of (a) 2 4 3 1
India in all. (b) 3 1 2 4
3. Ninth, Tenth, Eleventh and twelth schedules (c) 2 1 3 4
were added to Constitution of India by the (d) 3 4 2 1
IAS (Pre) G.S., 2002
Constitution (Amendment) Acts.
Ans. (a) Correctly match is-
Which of the statements given above is/are
correct? List-I List-II
(Article of (Provisions)
(a) 1 and 2 (b) 2 only Indian
(c) 3 only (d) 1,2 and 3 Constitution)
IAS (Pre) G.S., 2005 Article 16(2) No person can be discriminated
Ans: (c) Constitution of India has 22 parts hence against in the matter of public
statement 1 is incorrect. There are 395 Articles in the appointment on the ground of race,
religion or caste
Constitution of India. Therefore statement 2 is also
th Article 29(2) No citizen shall be denied
incorrect. 9 Schedule was added by first Amendment,
admission into any educational
10th Schedule was added by 52nd Amendment, 11th institution maintained by the State,
rd th
Schedule was added by 73 Amendment and 12 or receiving State aid, on grounds
Schedule was added by 74th Amendment. Hence of religion, race, caste, language or
statement 3 is correct. any of them.
120. Which of the following Schedule was inserted Article 30(1) All minorities whether based on
th
by the 74 Amendment Act? religion or language shall have the
th th fundamental right to establish and
(a) 12 Schedule (b) 11 Schedule administer educational institutions
(c) 13th Schedule (d) 10th Schedule of their choice
TNPSC (Pre) G.S. 2016 Article 31(1) No person shall be deprived of his
Ans. (a) : Kindly refer the explanation of the above property save by the authority of
question. law

Indian Polity & Constitution 41 YCT


122. Which one of the following pair is not correctly Ans: (c) Correct match is:
matched? List-I List-II
(a) Prohibition of traffic in human beings and Abolition of Titles Article 18
forced labour Article 23
Freedom to manage Article 26
(b) Protection of interests of minorities
religious affairs
Article 29
Protection of language Article 29
(c) Remedies for enforcement of rights
of minorities
Article 32
(d) Right of minorities to establish and Right to education Article 21A
administer educational institutions Article 31 126. Under which Article special provision has been
UPPCS (Main) G.S. IInd 2015 made for Finance Bill?
Ans. (d) Right of minorities to establish and administer (a) Article 117 (b) Article 119
educational institutions comes under Article 30. Article (c) Article 121 (d) Article 123
31 states the compulsory acquisition of property but this UPUDA/LDA Special (Main) G.S., 2010
article was repeated by 44th Constitutional Amendmet in Ans. (a) Special provision for Finance Bill has been
1978. Rest three are correctly matched. made under Article 117 of the Constitution.
123. Under which Article of the Indian Constitution 127. Match List I with List II and select the correct
procedure established by law has been added? answer using the codes given below
(a) 11 (b) 16 List - I List –II
(c) 21 (d) 26 (Article) (Provisions)
UPPCS (Main) G.S. IInd Paper, 2014 A. 215 1. Transfer of a Judge from one
Ans: (c) Article 21 is protection of life and personal High Court to another
liberty which asserts that no person shall be deprived of B. 222 2. Power of superintendence over
his life or personal liberty except according to all courts by the High Court
procedure established by law. C. 226 3. Power of High Courts to issue
124. The word ‘Hindu' in Article 25 of the certain writs
Constitution of India does not include D. 227 4. High Courts to be courts of
(a) Buddhist (b) Jains record
(c) Sikhs (d) Parsees Codes:
UPPCS (Main) G.S. IInd Paper, 2014 A B C D
Ans. (d) According to Article 25(2) (b) of the (a) 4 1 3 2
Constitution, reference to Hindus shall be construed as (b) 2 1 3 4
including a reference to persons professing the Sikh, (c) 1 4 3 2
Jaina or Buddhist religion. Parsees are not mentioned
(d) 4 2 3 1
under this Article.
RAS/RTS (Pre) G.S., 2013
125. Match List I with List II and select the correct
Ans: (a) Correct match is-
answer using the codes given below
List-I List-II
List – I List –II
Article 215 High Courts to be courts of record
A. Abolition of Titles 1. Article 29
Article 222 Transfer of a Judge from one High
B. Freedom to 2. Article 21A
Court to another
manage religious
affairs Article 226 Power of High Courts to issue
certain writs
C. Protection of 3. Article 18
language of Article 227 Power of superintendence over all
minorities courts by the High Court
D. Right to education 4. Article 26 128. Annual budget is placed under which Article of
Codes: the Constitution?
A B C D (a) Article 110 (b) Article 115
(a) 1 3 4 1 (c) Article 112 (d) Article 118
(b) 3 2 1 4 U.P.P.C.S (Pre) G.S, 2003, 2007
(c) 3 4 1 2 Ans: (c) Under the Article 112 of the Constitution,
(d) 4 3 2 1 word ‘Annual Financial Statement’ (Budget) has been
used for Budget.
RAS/RTS (Pre) G.S., 2016
Indian Polity & Constitution 42 YCT
06.
The Union and its Territories
1. Which one of the following is not a feature 4. Reorganization of States on linguistic basis was
of Indian federalism? done in?
(a) There is an independent judiciary in India. (a) 1956 (b) 1960
(b) Powers have been clearly divided between the (c) 1962 (d) 1978
Centre and the States. MPPSC (Pre) GS 1990
(c) The federating units have been given unequal Ans. (a) Reorganization of States on linguistic basis
representation in the Rajya Sabha. was done on November 1, 1956. In 1953, Andhra
(d) It is the result of an agreement among the Pradesh was the first State in India to be created on
federating units. linguistic basis. This led to the demand for creation of
IAS (Pre) G.S, 2017 States on linguistic basis from other parts of country
Ans. (d) The Constitution of India has described India and on December 22, 1953, Jawaharlal Nehru
as a ‘Union of States’, not a federation. It was not the announced the appointment of a commission under Fazl
result of an agreement by the States to join a federation. Ali to consider this demand. The other two members of
No State had the right to secede from it. The federation the commission were K M Panikkar and HN Kunzru.
is a Union as it is indestructible. The States are Commission recommended States reorganization bill
sovereign in the field which is left to them by the which was implemented in November 1956.
Constitution. Rests of the options given are features of
the Indian federalism. 5. Which of the following is the first State in India
formed on the basis of languages?
2. Which among the following is not a
(a) Kerala (b) Madhya Pradesh
constitutional provision for the formation of
new states? (c) Andhra Pradesh (d) Uttar Pradesh
(a) Increase the area of any state. UPPCS (Pre) G.S, 2018
(b) Diminish the area of any state. Ans: (c) Kindly refer the explanation of the above
(c) Alter the name of any state. question.
(d) A state may include a Union territory. 6. Which one of the following is the correct
UPPCS (Main) G.S. IInd Paper 2013 chronological order of the formation as full
Ans. (d) Article 3 of the Constitution is regarding States of the Indian Union?
formation of new States and alteration of areas, (a) Sikkim-ArunachalPradesh-Nagaland-Haryana
boundaries or names of existing States which asserts (b) Nagaland-Haryana-Sikkim-Arunachal
that Parliament may by law increase the area of any Pradesh
State, diminish the area of any State, alter the (c) Sikkim-Haryana-Nagaland-Arunachal
boundaries of any State, alter the name of any State but Pradesh
State does not include a Union territory. (d) Nagaland-Arunachal Pradesh-Sikkim Har-
3. A Bill for the purpose of creating a new State yana
in India must be passed by IAS (Pre) G.S., 2007
(a) A simple majority in Parliament and Ans. (b) Nagaland was carved out of Assam in 1963,
ratification by not less than two- third of the Haryana was carved out of Punjab in 1966, Sikkim
States.
became part of the Indian Union in 1975 while in 1987,
(b) A simple majority in Parliament. Arunanchal Pradesh and Goa became States which were
(c) A two-third majority in Parliament and earlier Union Territories.
ratification by not less than two-third of the
7. When was the state of Nagaland Formed?
States.
(d) None of the above (a) 1961 (b) 1963
UPPCS (Main) G.S. IInd Paper 2013 (c) 1965 (d) 1967
TNPSC (Pre) G.S. 2016
Ans. (b) According to Article 3, the simple majority in
Parliament is the sole authority on making a decision on Ans. (b) : Kindly refer the explanation of the above
a new State. question.
Indian Polity & Constitution 43 YCT
8. Identify the correct chronological order in 12. Which one of the following pairs is not
which the following States were created in correctly matched?
India from the codes given below: (a) States Reorganization Act: Andhra Pradesh
1. Andhra Pradesh 2. Himachal Pradesh (b) Treaty of Yandabu : Assam
3. Haryana 4. Sikkim (c) State of Bilaspur : Himachal Pradesh
Codes: (d) Year 1966 : Gujarat becomes a State
(a) 1, 2, 3, 4 (b) 1, 3, 2, 4 IAS (Pre) G.S., 2006
(c) 4, 3, 1, 2 (d) 3, 4, 1, 2 Ans. (d) Andhra Pradesh was the first State created
UPPCS (Pre) G.S., 2014 under States Reorganization Act, 1956. The Treaty of
Ans. (b) Yandaboo was signed in the year 1826. It was a peace
State Year of creation treaty between East India Company and the King of
Andhra Pradesh 1953 Ava that ended the First Anglo-Burmese War. The
Yandabo treaty marks the start of British rule in Assam.
Haryana 1966
The Bilaspur district is situated in Satluj valley in
Himachal Pradesh 1971
Himachal Pradesh. The new township Bilaspur should
Sikkim 1975 be regarded as the first planned hill town of the country.
9. Which of the following States joined the Indian Gujarat and Maharashtra were formed as independent
Union after the conduct of special opinion poll States in 1960. Hence (d) is not correctly matched.
in 1975? 13. Consider the following events:
(a) Manipur (b) Meghalaya 1. Fourth general elections in India
(c) Goa (d) Sikkim 2. Formation of Haryana State
Punjab PSC (Pre) G.S 2013 3. Mysore named as Karnataka State
Ans. (d) Sikkim joined the Indian Union in 1975 after 4. Meghalaya and Tripura become full States
the conduct of special opinion poll. Which one of the following is the correct
10. Which of the following pair is correctly chronological order of the above?
matched? (a) 2 - 1 - 4 - 3 (b) 4 - 3 - 2 - 1
(a) Haryana-November 1st, 1966 (c) 2 - 3 - 4 - 1 (d) 4 - 1 - 2 – 3
(b) Mizoram-25th June, 1986 IAS (Pre) G.S., 2004
(c) Telangana-15th August, 2014 Ans. (a) Haryana was formed as State in 1966, fourth
(d) Chhattisgarh-November 20, 2000 general elections in India were held in 1967, under the
Uttarakhand RO/ARO, 2016 North Eastern Region (reorganization) Act, 1971,
Ans. (a) Haryana was granted Statehood on November Tripura, Manipur and Meghalaya became full-fledged
1st, 1966 after the reorganization of Punjab.Mizoram States on Jan 21, 1972 while Mysore named as
was granted Statehood on February 20th, 1987, Karnataka State in 1973.
Chhattisgarh was granted Statehood on November 1st, 14. Union territories in India is governed by-
2000 while Telangana was granted Statehood on June (a) President (b) Lieutenant Governor
2nd, 2014. Hence (a) is correctly matched. (c) Home Minister (d) Administrator
11. The following States were created after UPPCS (Pre) G.S., 1995
1960.Arrange them in ascending chronological Ans. (a) Union Territories in India are administrated by
order of their formation : the President of India through an Administrator
1. Haryana 2. Sikkim appointed by him. Administrators of Andaman and
3. Nagaland 4. Meghalaya Nicobar Islands, Delhi and Puducherry are designated
Choose your answer from the given codes: as Lieutenant Governors. The National Capital Territory
(a) 1,2,3,4 (b) 2,3,4,1 of Delhi and Union Territory of Puducherry have a
(c) 3,1,4,2 (d) 2,4,1,3 Legislative Assembly and Council of Ministers.
UPPCS (Pre) G.S., 2010 15. Who said ‘India is a ‘Quasi-Federal State’?
UPPCS (Main) Spl. G.S. IInd 2008 (a) Lord Bryce (b) Ivor Jannigs
Ans: (c) (c) H.J. Laski (d) K.C. Wheare
Nagaland 1963 UPPCS (Pre) G.S, 2019
Haryana 1966 Ans: (d) Prof. K.C. Wheare described India as Quasi-
Meghalaya 1972 Federal State whereas D.D Basu described Constituion
Sikkim 1975 of India as Unitary and Federal.

Indian Polity & Constitution 44 YCT


07.
Citizenship
1. Which Articles of the Indian Constitution are Select the correct answer using the codes given
related to citizenship? below:
(a) Article 3 to 10 (a) 2, 3, 4, 5 (b) 1, 2, 3, 4
(b) Article 4 to 11 (c) 1. 3, 4, 5 (d) 1, 2, 3, 5
(c) Article 5 to 11 Maharashtra PSC (Pre) G.S 2014
(d) Article 6 to 11 Ans. (a) Kindly refer the explanation of the above
UPPCS (Pre) G.S 2020 question.
Ans. (c) Part II, Article 5 to 11 of the Indian 4. Who/which of the following is competent of
Constituion is related to citizenship. prescribe conditions for acquisition of
2. Which of the following are the ways of citizenship?
acquiring citizenship of India? (a) Election Commission
(1) Birth (b) President
(2) Descent (c) Parliament and State Legislatures Jointly
(3) Incorporational of Territory (d) Parliament
(4) Naturalization UPPCS (Main) G.S. IInd Paper 2013
Select the correct answer from the codes given Ans. (d) Article 11 of Part II of the Constitution is
below: regarding power of Parliament to make any provision
Codes: with respect to the acquisition and termination of
(a) (1) and only citizenship and all other matters relating to citizenship.
(b) (1) and (2) 5. Provisions regarding citizenship have been
(c) (1), (2) and (4) provided in which part of the Constitution?
(d) (1), (2), (3) and (4) (a) Part I (b) Part II
Himanchal PSC (Pre),G.S, 2013 (c) Part III (d) Part IV
Ans. (c) There are four ways in which Indian UPUDA/LDA (Main) G.S., 2010
citizenship can be acquired. They are; by birth, descent, Ans. (b) Kindly refer the explanation of the above
registration and naturalisation. The provisions are listed question.
under sections 3, 4, 5(1) and 5(4) of the Citizenship Act, 6. Who among the following has the exclusive
1955. power of determining the issue of citizenship in
India?
3. Under the Citizenship Act, 1955, by which of (a) The Court
the following ways can a person be citizen of (b) The President
India? (c) Lok Sabha
1. By incorporation of territory (d) Central Government
2. By descent (e) State Government
3. By Birth CG PSC (Pre) G.S., 2018
4. By naturalization Ans. (d) Exclusive power of determining the issue of
5. By registration citizenship in India lies with Central Government.

Indian Polity & Constitution 45 YCT


7. Which one of the following features of 10. How many years a person of Indian origin has
citizenship in India is correct? to spend to become citizen of India under the
(a) Dual Citizenship of the States and Nation Citizenship Act, 1955?
(b) Single citizenship of a State (a) 5 years
(c) Single citizenship of whole of India (b) 3 years
(d) Dual citizenship of India and another country (c) 7 years
UPPCS (Pre) G.S., 2015 (d) 9 years
Ans. (c) The Constitution of India provides single (e) 10 years
citizenship of whole of India. Chhatisgarh PSC (Pre.) 1st, 2013

8. A citizen of India loses his/her citizenship, if Ans. (c) Under the Citizenship Act 1955, persons of
he/she- Indian origin, who are ordinarily resident in India for
(a) Gives up Indian citizenship seven years could become the citizen of India.
Citizenship of India by naturalization can be acquired
(b) Willfully acquires the citizenship of another
by a foreigner (not illegal migrant) who is ordinarily
country
resident in India for twelve years.
(c) Get married to the citizen of another country
11. Under Citizen Amendment Act, 2015 which of
(d) Criticizes the Government
the following Indian cardholder is not
Select the correct answer by using the codes
authorized to register as emigrant citizen?
given below-
(a) A minor child whose parents are Indian
(a) 1, 2 and 3
citizen
(b) 2, 3 and 4
(b) Foreigner wife of Indian origin
(c) Only 1 and 2
(c) Those Indian who migrated to Pakistan after
(d) Only 1 and 4
partition
UPPCS (Main) 2017 G.S. IInd Paper
(d) A child or a grand: (c)hild or a great grand-
Ans. (c) Section 9 of Citizenship Act, 1955 deals with child of a citizen of another country
termination of the citizenship. According to the
UPPCS (Main) G.S. IInd Paper, 2016
provision, any citizen of India who by naturalization,
Ans.(c) Citizenship (Amendment) Act, 2015 introduced
registration otherwise voluntarily acquires the
the concept of an 'Overseas Citizen of India Cardholder'
citizenship of another country shall cease to be a citizen
(OCC) in India. Pursuant to section 7A of the
of India. Central Government may by order deprive any
Amendment Act, a person shall be eligible to register as
citizen of Indian citizenship under special circumstances
an OCC if any of the following conditions are satisfied-
provided by law.
1. A person who is a citizen of another country, but
9. A person will lose his citizenship if was a citizen of India at the time of
(a) He voluntarily surrenders his citizenship commencement of the Constitution.
(b) The Government withdraws his citizenship 2. A child or a grand-child or a great grand child of
(c) He voluntarily accepts citizenship of another a citizen of another country.
country 3. A minor child whose both parents or one of the
(d) All of the above parents is a citizen of India.
(e) None of these 4. Spouse of a citizen of India being of foreign
Chhattisgarh PSC (Pre.) 1st Paper, 2012 origin.
Ans. (d) Kindly refer the explanation of the above 5. Citizens of Pakistan or Bangladesh are not
question. permitted to register as OCC.
Indian Polity & Constitution 46 YCT
08.
Fundamental Rights
1. Other than the fundamental rights, which of the Ans. (b) Law is set of rules and regulations that govern
following parts of the Constitution of the conduct of people for the well being of society. On
India reflect/reflects the principles and provisions the other hand liberty comes from the Latin root word
of the Universal declaration of human rights ‘liber’ meaning freedom. It is the absence of constraints
(1948)? and not merely the absence of restraints. Law and
liberty complement each other and true liberty comes as
1. Preamble
a result of established law. In reality, it is law that
2. Directive principles of state policy guarantees freedom and liberty to individuals. The
3. Fundamental duties Indian Constitution provides liberty to its citizen
through Article 21 and varied types of freedoms
Select the correct answer using the given code including speech and expression, movement, assembly,
below- profession through Article 19. The judiciary is the
(a) 1 and 2 only (b) 2 only watchdog of the Constitution and applies the law to
protect the liberty of the people. Hence if there are no
(c) 1 and 3 only (d) 1, 2 and 3
laws, there is no liberty.
IAS (Pre) G.S 2020 4. In the context of polity, which one of the
Ans. (d) The Constitution of India provides special following would you accept as the most
provisions for human rights. The Preamble, the appropriate definition of liberty?
Fundamental Rights and the Directive Principles of (a) Protection against the tyranny of political rulers
State Policy reflect the principles and provisions of the (b) Absence of restraint
Universal Declaration of Human Rights (1948). (c) Opportunity to do whatever one likes
2. Which of the following categories of (d) Opportunity to develop oneself fully
Fundamental Rights Incorporates protection IAS (Pre) G.S, 2019
against untouchability as a form of discrimina- Ans: (d) Kindly refer the explanation of the above
tion? question.
(a) Right against exploitation 5. Which of the following are regarded as the
(b) Right to freedom main features of the “Rule of Law”?
1. Limitation of powers
(c) Right to Constitutional remedies 2. Equality before law
(d) Right to equality 3. People’s responsibility to the Government
IAS (Pre) G.S. 2020 4. Liberty and civil rights
Ans. (d) Right to Equality (Article 14 to 18) is Select the correct answer using the code given
guaranteed in Constitution of India. Article 14 is below:
equality before law, Article 15 is prohibition of (a) 1 and 3 only (b) 2 and 4 only
discrimination on grounds of religion, race, caste, sex or (c) 1, 2 and 4 only (d) 1, 2, 3 and 4
place of birth, Article 16 is equality of opportunity in IAS (Pre.) G.S, 2018
matters of public employment, Article 17 is abolition of Ans. (c) “Rule of Law" refers to a Government, based
untouchability and Article 18 is abolition of titles. on principles of law and not of men. In a broader sense,
3. Which one of the following reflects the most rule of law means that law is supreme and is above
appropriate relationship between law and every individual. Dicey’s theory of rule of law consists
liberty? of three main principles: absence of arbitrary power or
supremacy of law which means no man can be punished
(a) If there are more laws, there is less liberty. for anything else than a breach of law which is already
(b) If there are no laws, there is no liberty. established, equality before law is the second principle
(c) If there is liberty laws have to be made by the which means equality of law or equal subjection of all
people. classes of people to the law of the land and third
(d) If laws are changed too often, liberty is in principle states that right to personal liberty, freedom,
danger. arrest etc are provided by the written Constitution of a
IAS (Pre) G.S, 2018 Country. Hence (c) is the correct answer.
Indian Polity & Constitution 47 YCT
6. Why the rights are called Fundamental Rights Ans. (c) Rights are claims (not the privileges) of the
because citizens against the State. State originated to protect the
(i) It is written in the Constitution rights of its people and it is the primary responsibility of
(ii) It is democratic the State to protect the rights and freedoms of its subjects.
(iii) It is public welfare 9. Which of the following are envisaged by the
(iv) It is essential for personal development Right against Exploitation in the Constitution
(v) Parliament can’t make law against it of India?
(a) (i) (ii) (iii) 1. Prohibition of traffic in human beings and
forced labour
(b) (i) (iii) (iv)
2. Abolition of untouchability
(c) (i) (iv) (v)
3. Protection of the interests of minorities
(d) (ii) (iii) (v)
4. Prohibition of employment of children in
(e) None of the above factories and mines
Chhattisgarh PSC (Pre) G.S. 2017 Select the correct answer using the code given
Ans. (c) The rights, which are enshrined in the below :
Constitution, are called ‘Fundamental Rights’. These (a) 1, 2 and 4 only (b) 2, 3 and 4 only
rights ensure the fullest physical, mental and moral (c) 1 and 4 only (d) 1, 2, 3 and 4
development of every citizen. Fundamental Rights IAS (Pre) G.S, 2017
generate a feeling of security amongst the minorities in
Ans. (c) Right against exploitation is provided under
the country. Fundamental Rights provide standards of Article 23 and 24 of the Constitution. Article 23 is
conduct, citizenship, justice and fair play. They serve as about prohibition of traffic in human beings and forced
a check on the Government. These rights are Justiciable. labour and Article 24 is no child below the age of
Constitution of India does not permit the legislature and fourteen years shall be employed to work in any factory
the executive to curb these rights either by law or by an or mine or engaged in any other hazardous employment.
executive order.
10. Consider the following statements about the
7. One of the implications of equality in society is Fundamental Rights in India :
the absence of 1. They are a guarantee against state action
(a) Privileges (b) Restraints 2. They are enumerated in Part III of the
(c) Competition (d) Ideology Constitution
IAS (Pre) G.S, 2017 3. They ensure social, economic and political
Ans. (a) Article 14 to 18 of the Constitution provides justice
‘Right of Equality’ to every citizen which in turn are 4. They are unlike Bill of Rights in the U.S.A.
Fundamental Rights. Article 14 embodies equality Now, select the correct answer from the codes
expressed in the Preamble of the Constitution. The given below :
succeeding Articles 15, 16, 17 and 18 lays down (a) 1 and 2 are correct
specific applications of the general rules laid down in (b) 2 and 3 are correct
Article 14. Article 15 relates to prohibition of (c) 1,3 and 4 are correct
discrimination on grounds of religion, race, caste, sex or (d) 2,3 and 4 are correct
place of birth. Article 16 guarantees equality of UPPCS (Pre) G.S. 1998
opportunity in matters of public employment. Article 17 Ans. (a) Part III of the Constitution of India is related to
abolishes untouchability and Article 18 abolishes title, Fundamental Rights which are guaranteed by
which provides privileges in society. Constitution of India against State action. Social,
8. Which one of the following statements is economic and political justice has been ensured by
correct? Directive Principle of the State Policy. Fundamental
(a) Rights are claims of the State against the Rights are like the Bill of Rights in U.S.A. Hence
citizens. option (a) is the correct answer.
(b) Rights are privileges which are incorporated 11. Fundamental Rights-
in the Constitution of a State. (a) Cannot be suspended
(c) Rights are claims of the citizens against the (b) Can be suspended by the order of the Prime
State. Minister
(d) Rights are privileges of a few citizens against (c) May be suspended on the will of President
the many. (d) May be suspended during Emergency
IAS (Pre) G.S, 2017 UP Lower (Pre) G.S.,2002
Indian Polity & Constitution 48 YCT
Ans. (d) During normal circumstances, Fundamental Ans. (a) Kindly refer the explanation of the above
Rights cannot be suspended. Fundamental Rights are question.
Justiciable which means that the citizens can seek the 16. Consider the following provisions :
assistance of the courts for the enforcement of their 1. According to Article 21 of the Indian
Fundamental Rights under Article 32 (Supreme Court) Constitution, no person shall be deprived of
and Article 226 (High Courts).Fundamental Rights his life or personal liberty except according to
could only be suspended during Emergency. Article 352 the 'due process of Law'.
of the Constitution provides for declaration of 2. According to Article 20(2) of the Indian
Emergency. Under Article 358, on a declaration of Constitution, no person shall be prosecuted
emergency, the Fundamental Rights guaranteed by and punished for the same offence more than
Article 19 (Protection of certain Rights regarding once.
freedom of speech etc.) stand suspended. Furthermore
3. According to Article 14 of the Indian
Article 359 of the Constitution provided that when a
Constitution, the State shall not deny to any
proclamation of emergency is in operation, the
person equality before the law or the equal
enforcement of Fundamental Right may be suspended
protection of the laws within the territory of
by the issue of a Presidential Order except Article 20
India.
(Protection in respect of conviction for offences) and
Which of the provisions given above is not
Article 21(Protection of life and personal liberty).
correct?
12. The Right to Freedom under Article 19 of the (a) 1 (b) 2
Constitution of India may be suspended by the
(c) 3 (d) None of the above
President of India?
Maharashtra PSC (Pre) G.S, 2019
(a) Under Article 360
Ans. (a) : According to Article 21 of the Constituion of
(b) Under Article 356
India is regarding protection of life and personal liberty.
(c) Under Aricle 352
It asserts that no person shall be deprived of his life or
(d) At any time he so desires
personal liberty except according to ‘procedure
Himanchal PSC (Pre) G.S, 2013 established by law’. Hence statement 1 is not correct.
Ans. (c) Kindly refer the explanation of the above Article 20(2) of the Constituion asserts that no person
question. shall be prosecuted and punished for the same offence
13. Under the Constitution of India who is the more than once. Hence statement 2 is correct. Further
guardian of fundamental rights? Article 14 provides Right to Equality which asserts that
(a) Parliament (b) President the State shall not deny to any person equality before
(c) Judiciary (d) Cabinet the law or the equal protection of the laws within the
Himanchal PSC (Pre) GS, 2010 territory of India. Hence statement 3 is also correct.
Ans: (c) Kindly refer the explanation of the above 17. Which one of the following is not among the six
question. Fundamental Rights provided by the
14. Which one among the following pairs of Constitution of India?
Articles relating to the Fundamental Rights (a) Right to equality
cannot be suspended during the enforcement of (b) Right to protest,
emergency under Article 359 of the Indian (c) Right against exploitation
Constitution? (d) Right to freedom of religion
(a) Articles 20 and 21 (b) Articles 14 and 15 UPPCS (Main G.S. IInd Paper 2015
(c) Articles 16 and 17 (d) Articles 24 and 25 Ans. (b) The six Fundamental Rights provided by
UPPCS (Pre) G.S.,2015 Constitution of India are-
UPPCS (Mains) G.S. IInd Paper 2009 1. Right to Equality (Article 14 to 18)
Ans. (a) Kindly refer the explanation of the above 2. Right to Freedom (Article 19 to 22)
question. 3. Right against Exploitation (Article 23 and 24)
15. Fundamental Rights are- 4. Right to Freedom of Religion (Article 25 to 28)
(a) Justiciable (b) Non-Justiciable 5. Cultural and Educational Rights (Article 29 and
(c) Flexible (d) Rigid 30)
UP Lower (Pre) G.S., 2002 6. Right to Constitutional Remedies (Article 32)

Indian Polity & Constitution 49 YCT


18. In the Indian Constitution, the Right of (a) I and III (b) I and IV
Equality is granted by five Articles. They are (c) II and IV (d) II and III
(a) Article 16 to Article 20 BPSC (Pre.) G.S. 2016
(b) Article 15 to Article 19 Ans. (a) Kindly refer the explanation of the above
(c) Article 14 to Article 18 question.
(d) Article 13 to Article 17 23. A British citizen staying in India cannot claim
IAS (Pre.) G.S., 2002 Right to
UPPCS (Pre.) G.S. 2006, 2009 (a) Freedom of trade and profession
Ans. (c) Kindly refer the explanation of the above (b) Equality before the law
question. (c) Protection of life and personal liberty
19. Which one of the following Articles of the (d) Freedom of religion
Constitution of India is related to equality IAS (Pre.) G.S., 1999
before the law? Ans. (a) Kindly refer the explanation of the above
(a) Article 16 (b) Article 15 question.
(c) Article 14 (d) Article 13 24. Which of the following Fundamental Rights
UPPCS (Main) G.S. IInd Paper 2015 under the Constitution is guaranteed only to
Ans. (c) Kindly refer the explanation of the above the citizens of India and not to foreigners living
question. in India?
20. Which gurantees right to freedom of religion of (a) Equality before law
all persons in all its aspects? (b) Freedom of speech and expression
(a) Article 25 to 28 (b) Article 29 (c) Protection of life and personal liberty
(c) Article 30 (d) Article 34 (d) Free practice of religion
Tamil Nadu PSC (Pre) G.S 2011 UPPCS (Pre.) Re-exam. G.S., 2015
Ans. (a) Kindly refer the explanation of the above Ans. (b) Kindly refer the explanation of the above
question.
question.
25. Which one of the following right provided by
21. Which of the following Fundamental Rights
Constitution of India is available to the
have been not provided to foreigners?
foreigners also?
(a) Equality before law
(a) Freedom of expression
(b) Freedom of expression
(b) To move freely and to reside and settle in any
(c) Protection of life and personal liberty part of the territory of India
(d) Right against Exploitation (c) Right to property
UPPCS (Pre.) G.S., 2007 (d) Right to Constitutional remedies
Ans. (b) As per the Constitution of India, Fundamental Jharkhand PSC (Pre.) G.S. 2010
Rights, available to Indian citizen only and not to BPSC (Pre.) 2011
foreigners are-
Ans. (d) Kindly refer the explanation of the above
Article 15- Prohibition of discrimination on grounds of question.
religion, race, caste, sex or place of birth.
26. Which of the following Rights are not available
Article 16- Equality of opportunity in matters of
to all persons in India?
public employment.
Select your answer by using the codes given
Article 19- Protection of certain rights regarding
below :
freedom of speech, etc.
1. Equality before the Law
Article 29- Protection of interests of minorities.
2. Right against Discrimination
Article 30- Right of minorities to establish and
3. Freedom to move freely throughout the
administer educational institutions.
country
Rest of the Fundamental Rights is provided 4. Right to contest election
to both Indian citizens and foreigners.
Code:
22. Choose those Fundamental Rights which are (a) 1, 3, 4 (b) 1, 2, 4
provided to Indian citizens but not to non- (c) 1, 2, 3 (d) 2, 3, 4
citizens?
UPPCS (Main) G.S. 2002
I. Freedom of expression and speech
Ans. (d) Article 14 of the Constitution provides equality
II. Equality before law
before the law or equal protection to all persons within
III. Interests of minorities the territory of India, but right against discrimination
IV. Protection of life and personal liberty (Article 14, 15, 16), Freedom to move freely throughout
Indian Polity & Constitution 50 YCT
the country [Article 19(1)(d)] and right to contest Ans. (b) In the Randhir Singh Vs Union of India case,
election[Article 84(a) for Parliament and Article 173(a) Supreme Court ruled that the principle of 'equal pay for
for State Legislative Assemblies] are provided only to equal work' is not expressly declared by our
Indian citizens. Constitution to be a Fundamental Right but it certainly
27. Which of the following is considered as is a ‘Constitutional goal’.
Fundamental Rights of second generation? 31. In Indian Constitution, Right to Freedom has
(a) Right to work (b) Right to education been provided by four Articles which are-
(c) Right to freedom (d) Right to equality (a) Article 19 to Article 22
UPUDA/LDA Special (Pre.) G.S., 2010 (b) Article 16 to Article 19
Ans. (d) Right to equality is considered as Fundamental (c) Article 17 to Article 20
Rights of second generation. (d) Article 18 to Article 21
28. Which of the following is correct? UPPCS (Main) G.S. IInd Paper, 2016
(a) Social equality is not guaranteed in our Ans. (a) Right to Freedom in Indian Constitution has
Constitution. been provided under Article 19 to Article 22. Article 19
(b) Social equality already existed in our country. is protection of certain rights regarding freedom of
(c) Social equality is guaranteed in our speech, Article 20 is protection in respect of conviction
Constitution. for offences, Article 21 is protection of life and personal
(d) None of the above. liberty and Article 22 is protection against arrest and
UPPCS (Main) G.S. IInd Paper 2008 detention in certain cases.
Ans. (c) Right to Equality (Article 14 to 18) is 32. Which Article of the Constitution of India is
guaranteed in Constitution of India. Article 14 is related to protection of life and personal
equality before law, Article 15 is prohibition of liberty?
discrimination on grounds of religion, race, caste, sex or (a) Article 19 (b) Article 21
place of birth, Article 16 is equality of opportunity in (c) Article 20 (d) Article 22
matters of public employment, Article 17 is abolition of Uttarakhand PSC (Pre)-2016
untouchability and Article 18 is abolition of titles.
Ans. (b) Kindly refer the explanation of the above
29. Which of the following Articles of the Indian question.
Constitution guarantees equality of
opportunity to all citizens of India in matters 33. Which Article gives safeguard to the
relating to public employment? Fundamental Rights of arrested person?
(a) Article 15 (a) Article 15 (b) Article 17
(b) Article 16(1) and 16(2) (c) Article 21 (d) Article 22
(c) Article 16(3) UPPCS (Main) G.S. IInd Paper 2013
(d) Article 16(3), (4) and (5) Ans. (d) Kindly refer the explanation of the above
UPPCS (Main) G.S. IInd Paper 2009 question.
Ans. (b) Article 16 of the Constitution is equality of 34. Right to Privacy is protected as an intrinsic
opportunity in matters of public employment. Article part of Right to Life and Personal Liberty.
16(1) asserts that there shall be equality of opportunity Which of the following in the Constitution of
for all citizens in matters relating to employment or India correctly and appropriately imply the
appointment to any office under the State whereas above statement?
Article 16(2) asserts that no citizen shall, on grounds of (a) Article 14 and the provisions under the 42nd
religion, race, caste, sex, descent, place of birth, Amendment to the Constitution
residence or any of them be discriminated against in (b) Article 17 and the Directive Principles of
respect of any employment or office under the State. State Policy in Part IV
Additionally Article 16(3), 16(4) and 16(5) is related to (c) Article 21 and the freedoms guaranteed in
the power of the Parliament to make law for reservation Part III
of appointment for schedule caste, schedule tribe and (d) Article 24 and the provisions under the 44th
backward classes in public employment. Amendment to the Constitution
30. Which of the following is not a Fundamental IAS (Pre.)G.S, 2018
Right? Ans. (c) A nine-Judge Constitutional Bench of Supreme
(a) Right against Exploitation Court held that the right to privacy is protected as
(b) Equal pay for equal work intrinsic part of the right to life and personal liberty
(c) Equality before law under Article 21 of the Constitution and as a part of the
(d) Right to freedom of Religion freedoms guaranteed by Part III (Fundamental Rights)
UPPCS (Main) G.S. IInd Paper 2014 of the Constitution.
Indian Polity & Constitution 51 YCT
35. In India, if a religious sect/community is given (c) (A) is true, but (R) is false
the status of a national minority, what special (d) (A) is false, but (R) is true
advantage it is entitled to? UPPCS (Pre) G.S, 2019
1. It can establish and administer exclusive Ans: (b) Article 30 is right of minorities to establish
educational institutions. and administer educational institutions but the
2. The President of India automatically Constitution of India does not define the word
nominates a representative of a community to ‘Minority’ and only refers to ‘Minorities’ and speaks of
Lok Sabha. those ‘based on religion or language’. Therefore both
3. It can drive benefits from the prime minister’s (A) and (R) are true but (R) is not the correct
15-point program. explanation of (A).
Which of the statements given above is/are 38. Right to Education to all children between the
correct? age group of 6 to 14 years is-
(a) 1 only (b) 2 and 3 only (a) Included in the Directive Principle of State
(c) 1 and 3 only (d) 1, 2 and 3 Policy
IAS (Pre.) G.S., 2011 (b) A Fundamental Right
Ans. (c) According to Article 30 (1) of the Constitution, (c) A statutory Right
all minorities, whether based on religion or language, (d) None of the above
shall have the right to establish and administer UPPCS (Pre.) G.S., 2006
educational institutions of their choice. Hence statement
Ans. (b) The Constitution (Eighty-Sixth Amendment)
1 is correct. As per the Constitution, President of India Act, 2002 inserted Article 21A in the Constitution to
may nominate two members from the Anglo-Indian
provide free and compulsory education to all children in
community in Lok Sabha (Article 331) and twelve
the age group of 6 to 14 years as a Fundamental Right.
members in Rajya Sabha from amongst persons who The Right of Children to Free and Compulsory
have special knowledge or practical experience in
Education (RTE) Act, 2009 means that every child has a
respect of such matters as literature, science, art and
right to full time elementary education of satisfactory
social service (Article 80). Hence statement 2 is and equitable quality in a formal school which satisfies
incorrect. The Prime Minister’s 15 Point Program
certain essential norms and standards. Article 21A and
launched in 2005 designed to ensure the welfare of
RTE Act came into effect on 1 April, 2010. The title of
religious minorities. Currently, Muslims, Christians, the RTE Act incorporates the words ‘free and
Sikhs, Buddhists, Parsis and Jains are identified as
compulsory’.
religious minorities. Hence statement 3 is correct.
39. The Constitution 86thAmendment Act, 2002
36. The Constitution of India recognizes deals with:
(a) Only religious minorities (a) Right to Education a Fundamental Right for
(b) Only linguistic minorities children in the range of 6-14 years
(c) Religious and linguistic minorities (b) Right to employment a Fundamental Right
(d) Religious, linguistic and ethnic minority (c) Right to employment a Fundamental Right
IAS (Pre.) G.S., 1999 for rural people
Ans. (c) The Constitution of India does not define the (d) Right to Education a Fundamental Right for
word ‘Minority’ and only refers to ‘Minorities’ and children in the range of 5-13 years
speaks of those ‘based on religion or language’. Hence Himanchal PSC (Pre) G.S. 2019
Constituion of India recognizes religious and linguistic
Ans. (a) : Kindly refer the explanation of the above
minorities.
question.
37. Given below are two statements, one is labelled
40. Article 21A provides free and compulsory
as Assertion (A) and the other as Reason (R)
education to children in the age group of-
Assertion (A) : Article 30 of the Constitution of
(a) 3 to 12 years (b) 3 to 14 years
India does not define the term ‘minorities’.
(c) 5 to 18 years (d) 6 to 14 years
Reason(R) : The Constitution recognises only
linguistic and religious minorities. UPPSC ACF-RFO Mains (IInd Paper), 2019
Select the correct answer by using the codes Ans. (d) : Kindly refer the explanation of the above
given below: question.
Codes: 41. Which Article of India Constitution deals with
(a) Both (A) and (R) are true and (R) is the rights to elementary education?
correct explanation of (A) (a)Article 20 (A) (b) Article 21 (A)
(b) Both (A) and (R) are true but (R) is not the (c)Article 22 (A) (d) Article 21
correct explanation of (A) TNPSC (Pre) G.S. 2016

Indian Polity & Constitution 52 YCT


Ans. (b) : Kindly refer the explanation of the above 46. Consider the following statements:
question. Assertion (A) Dr. Ambedkar had described Article
42. Which one of the following human right is also 32 of the Constitution as the very soul of it.
a Fundamental Right under the Constitution of Reason (R) Article 32 provides effective remedy
India? against the violation of Fundamental Rights.
(a) Right to Information Select the correct answer using the codes given
(b) Right to Work below:
(c) Right to Education Codes:
(d) Right to Housing (a) (A) and (R) both are true and (R) is the
BPSC (Pre.) G.S, 2011 correct explanation of (A).
Ans. (c) Kindly refer the explanation of the above (b) (A) and (R) both are true, but (R) is not the
question. correct explanation of (A).
43. The aim of Right to Education Act, 2009 is to (c) (A) is true, but (R) is false.
make education free and compulsory. This is (d) (A) is false, but (R) is true.
for the children who are-
UPPCS (Pre.) G.S., 2016
(a) Up to the primary level
(b) Up to the middle level Ans. (a) Article 32 of the Constitution of India is
remedies for enforcement of Fundamental Rights which
(c) Up to the higher secondary level
asserts that the citizen has right to move the Supreme
(d) Graduation level
Court by appropriate proceedings for the enforcement
UPPCS (Pre.) G.S., 2015
of the rights. The framers of the Constitution provided
Ans. (a) Kindly refer the explanation of the above
guarantee for the enforcement of Fundamental right
question.
through legal remedy mentioned in Article 32. Dr. B. R.
44. When was Right to Education was added after Ambedkar, architect of the Indian Constitution said,
amending the Constitution of India? Article 32 is heart and soul of the Indian Constitution as
(a) 1st April, 2010 it provides remedies in case of the violation of other
(b) 1st August, 2010 fundamental rights provided by Indian Constitution.
(c) 1st October, 2010 Hence both (A) and (R) are true and (R) is the correct
(d) 1st December,2010 explanation of (A).
BPSC (Pre.) G.S, 2011
46. Which one of the following rights was
Ans. (a) Kindly refer the explanation of the above described by Dr B.R.Ambedkar as the heart
question.
and soul of the Constitution?
45. Assertion (A) The state shall provide free and (a) Right to freedom of religion
compulsory education to all children of the age
(b) Right to property
group of six to fourteen years.
Reason (R) : In a democratic society, right to (c) Right to equality
education is indispensible in the interpretation (d) Right to Constitutional remedies
of the right to development as a human right. UPCDA/LDA Special (Pre.) G.S., 2010
In the context of the above statements select the IAS (Pre.) G.S., 2002
correct answer. UPPCS (Main) Spl. G.S. IInd Paper, 2004
Code: Ans. (d) Kindly refer the explanation of the above
(a) Both (A) and (R) are true, and (R) is the question.
correct explanation of (A).
47. Who is the custodian of the Fundamental
(b) Both (A) and (R) are true, but (R) is not the
Right?
correct explanation of (A).
(a) Supreme Court (b) Parliament
(c) (A) is true, but (R) is false.
(d) (R) is true, but (A) is false. (c) President (d) Prime Minister
UPPCS (Main) G.S. IInd Paper, 2016 UPPCS (Pre.) G.S., 1992
Ans. (a) According to Article 21A, the State shall U.P. Lower (Pre.) 2004-05
provide free and compulsory education to all children of Uttarakhand UDA/LDA (Pre.) 2006
the age group of six to fourteen years. Hence assertion Chhattisgarh PSC (Pre.) 1st, 2012
A is correct. Education is fundamental right and it is Ans. (a) Article 32 of the Constitution is regarding
indispensible in the interpretation of the right to remedies for the enforcement of Fundamental Rights,
development as a human right. Hence reason R is also which asserts that the citizen has right to move the
correct and R is the correct explanation of A. Supreme Court by appropriate proceedings for the
Indian Polity & Constitution 53 YCT
enforcement of the rights and Supreme Court shall have 51. Under Article 22 of the Constituion of India, with
power to issue directions or orders or writs, including the exceptions of certain provisions stated there
writs in the nature of habeas corpus, mandamus, in, what is the maximum period for detention of a
prohibition, quo warranto and certiorari, whichever may person under preventive detention?
be appropriate for the enforcement of any of the (a) 2 months (b) 3 months
Fundamental Rights.
(c) 4 months (d) 6 months
48. Which of the following cases comes under the
Maharashtra PSC (Pre) G.S 2014
jurisdiction of High Court and Supreme
Court? Ans. (b) Kindly refer the explanation of the above
(a) Dispute between center and state question.
(b) Dispute between states 52. At present in the Constitution of India Right of
(c) Enforcement of Fundamental Rights Property is a-
(d) Remedy for the violation of the Constitution (a) Fundamental Right (b) Statuary Right
IAS (Pre.) G.S., 1993 (c) Moral Right (d) None of the above
Ans. (c) Enforeement of Fundamental Rights comes UPPCS (Pre.) G.S., 2007
under the jurisdiction of Supreme Court (Article 32) and Uttarakhand PCS (Pre.) 2009
High Courts (Article 226). Ans. (b) The Constitution of India does not recognize
49. Under the Preventive Detention a person can ‘Right of Property’ as Fundamental Right. Constitution
be detained without trial for 44th Amendment, 1978, eliminated the right to acquire,
(a) One month (b) Three months hold and dispose of property as Fundamental Right.
(c) Six months (d) Nine months However in another part of the Constitution, Article
UPPCS (Main) G.S. IInd Paper 2009 300(A) was inserted to affirm that no person shall be
Ans. (b) According to Article 21 of the Constitution, no deprived of his property by authority of law. Due to
person can be deprived of his life and personal liberty this, Right of Property as a Fundamental Right is now
except according to procedure established by law. substituted as ‘Statuary Right’.
Further, Article 22 of the Constitution provides those 53. Which Constitutional amendment deleted the
procedural requirements which must be included and right to property from Chapter III and made it
adopted in any procedure enacted by the Legislature. legal right only?
Article 22 deals with two separate matters; (1) Personal (a) 24th Amendment
arrest under the ordinary law of crime (2) Person
(b) 29th Amendment
detained under the law of Preventive Detention. There
is no authentic definition of Preventive Detention in (c) 42nd Amendment
Indian law. The objective of Preventive Detention is not (d) 44th Amendment
to punish a person for having done something but to Punjab PSC (Pre) G.S 2015
intercept him before he does it and to prevent him from Ans. (d) Kindly refer the explanation of the above
doing it. Preventive Detention laws are not found in any question.
democratic country, except India. The Preventive 54. Which of the following is a statuary right but
Detention Act was enacted by the Parliament on 26th
not the Fundamental Right?
February, 1950. A person may be taken to Preventive
(a) Protection of life and personal liberty
Custody only for 3 months at first instance. If the period
of detention is extended beyond 3 months, the case must (b) To move freely throughout the territory of
be reffered to an Advisory Board which means the India
period of detention may be extended beyond 3 months (c) To assemble peaceably
only on approval by the Advisory Board. (d) Right to property
50. Under the Preventive Detention Act, a person U.P. Lower (Pre.) G.S. 2013
can be arrested without trial for MPPSC (Pre) G.S. 1993
(a) 1 month (b) 3 months BPSC (Pre) 1997
(c) 6 months (d) 9 months U.P.P.C.S (Pre.) G.S. 1992, 1994, 2000
U.P. Lower (Pre.) G.S. 2013 Uttarakhand PCS (Pre) 2002-03
Ans. (b) Kindly refer the explanation of the above Ans. (d) Kindly refer the explanation of the above
question. question.
Indian Polity & Constitution 54 YCT
55. Which of the Fundamental Right has not been anyone to sing the National Anthem as it violates the
provided to citizens in the Constitution of Article 19(1) (a) [freedom of speech] and 25(1)[ freedom
India? to practice religion] of the Constitution of India.
(a) To reside and settle in any part of the territory 58. In India, who is competent to amend
of India Fundamental Rights of the citizens?
(b) Right to gender equality (a) LokSabha (b) RajyaSabha
(c) Right to information (c) Parliament (d) Supreme Court
(d) Right against exploitation Himanchal PSC (Pre) G.S 2014
RAS/RTS (Pre.) G.S., 2003 Ans. (c) Under Article 368 of the Constitution,
Parliament has power to amend the Constitution
Ans. (c) Right to information is not a Fundamental
including Fundamental Rights.
Right as per the Constitution of India but rest all three
59. In which of the following cases, Supreme Court
options given are Fundamental Right. Right to
held that:
information Act 2005 provide practical regime of right
“Fundamental Rights enable a man to chalk
of information for citizens of India to secure access to
out his own life in the manner he likes best”?
information under the control of public authority.
(a) Indira Gandhi vs Raj Narain
56. Consider the following : (b) Golak Nath vs State of Punjab
1. Right to education. (c) Bank Nationalization
2. Right to equal access to public service. (d) Azhar vs Municipal Corporation
3. Right to food. UPPCS (Pre.) G.S., 2012
Which of the above is/are human rights under Ans. (b) In Golak Nath Vs State of Punjab case (1969),
“universal declaration of human rights”? Supreme Court said Fundamental Rights enable a man
(a) 1 only (b) 1 and 2 only to chalk out his own life in the manner he likes best.
(c) 3 only (d) 1,2 and 3 60. An offence arising out of “Untouchability”
IAS (Pre.) G.S., 2011 would not make out where?
Ans. (d) Universal declaration of human rights was (a) The act of accused with regard to the
proclaimed by the United Nations General Assembly in enjoyment of benefit under a charitable trust
Paris on 10th December, 1948 as a common standard of created for the benefit of any section of the
achievements for all peoples and all nations. It sets out, for general public
the first time, fundamental human rights to be universally (b) The act of accused is with regard to the use of
protected. Universal declaration of human rights has 30 finery
Articles which contains right to life, right to education, (c) As accused is not being competent to commit
right to equal access to public service and right to food etc. an offence
(d) The accused and complainant victim belong
57. Consider the following statements:
to the same social group.
No one can be compelled to sing the National
UPPCS (Main) G.S. IInd Paper 2014
Anthem since-
Ans. (d) An offence arising out of “Untouchability”
1. It will be violation of the Right to Freedom of
would not make out if the accused and complainant
Speech and Expression victim belong to the same social group.
2. It will be violation of the Right to Freedom of
61. On the basis of which of the following grounds,
Conscience and Practice and Propagation of a citizen’s freedom of expression may not be
Religion subjected to restriction?
3. There is no legal provision obliging anyone to (a) Sovereignty of India
sing the National Anthem (b) Public order
In these statements- (c) Contempt of Court
(a) 1 and 2 are correct (d) Unbecoming criticism
(b) 2 and 3 are correct (e) All of the above
(c) 1,2 and 3 are correct Chattisgarh PSC (Pre.) 1st G.S., 2013
(d) None is correct Ans. (d) Constitution of India provides freedom of
IAS (Pre.) G.S., 1996 speech and expression through Article 19(1) (a) but
Ans. (c) In Bijoe Emmanuel Vs State of Kerala case, Article 19(2) allows for reasonable restrictions to be
Supreme Court ruled that no provision of law obliges imposed on freedom to speech and expression. Grounds
Indian Polity & Constitution 55 YCT
of restrictions on freedom to speech and expression are 65. Reservation of schedule caste and schedule
security of State, friendly relations with foreign States, tribes in educational institutions is governed
public order, decency or morality, contempt of court, by:
defamation, Incitement to an offence, sovereignty and (a) Article 15(4) of the Constitution
integrity of India. (b) Article 16(4) of the Constitution
62. For the enforcement of Fundamental Rights, (c) Article 29(2) of the Constitution
which of the following is included in the (d) Article 335 of the Constitution
definition of ‘State’ under the Article 12? UPPCS (Pre.) G.S. 2003
U.P. Lower (Spl.) 2002
(a) Life Insurance Corporation
UPPCS (Spl.) 2004
(b) Partnership firm
Ans. (a) Article 15(4) of the Constitution provides
(c) Company registered under company Act,
reservation of Schedule Caste and Schedule Tribes in
1956
educational institutions.
(d) A cooperative society registered under
66. Under which Article of the Constitution
cooperative society Act, 1912
provision has been made for the reservation of
UPPCS (Pre.) G.S. 2000 O.B.C (other backward class)?
Ans. (a) Article 12 of the Constitution is related to (a)
Article 13(2) and 14
definition of the ‘State’ which asserts that the State (b)
Article 14 and 15
includes the Government and Parliament of India and (c)
Article 15(4) and 16(4)
the Government and the Legislature of each of the (d)
Article 17 and 18
States and all local or other authorities within the Uttarakhand-PSC (Pre.) 2016
territory of India or under the control of the Ans. (c) Under Article 15(4), special provisions may be
Government of India. In Sukhdev Vs Bhagatram case, made for the advancement of any socially and
Supreme Court ruled that Life Insurance Corporation, educationally backward classes of citizens and for the
ONGC and FCI (Food Corporation of India) comes Scheduled Castes and the Scheduled Tribes whereas
under the definition of State. Article 16(4) permits the State to make provisions for
63. Which of the following is State under the the reservation of appointments or posts in favor of any
Article 12 of the Constitution? backward class of citizens which in the opinion of the
State is not adequately represented in the services under
(a) Sri Gandhi Bhandar Ashram
the State.
(b) Constitutional and Parliamentary educational
67. Word ‘Hindu’ used in Article 25 of the
institute
Constitution of India does not include?
(c) Educational research and training academy (a) Buddhist (b) Jains
(d) Delhi Stock Exchange (c) Parsi (d) Sikhs
U.P. Lower (Pre) G.S, 2004 Uttarakhand PCS (M) GS Ind 2006
Ans. (d) Delhi Stock Exchange was estabished in 1947, Ans. (c) Article 25 of the Constitution is regarding
since it is regulated by SEBI (Securities and Exchange freedom of conscience and free profession, practice and
Board of India) hence it comes under domain of the propagation of religion. Article 25(2) (b) asserts that
state. Hindus shall be construed as including a reference to
64. For the enforcement of Fundamental Right, not persons professing the Sikh, Jaina or Buddhist religion.
Parsi has not been mentioned there. Hence c is the
included in the word State-
correct answer.
(a) Allahabad University
68. By which of the following Articles Constitution
(b) Life Insurance Corporation of India
of India provides certainty of Fundamental
(c) Uttar Pradesh Electricity Board Rights?
(d) Institute of Constitutional and Parliamentary (a) Article 12 to 35 (b) Article 12 to 30
studies (c) Article 15 to 35 (d) Article 14 to 32
UPPCS (Pre.) G.S. 1995, 2007 UP Lower (M) G.S. 2015-16
Ans. (d) Kindly refer the explanation of the above Ans. (a) Fundamental Rights in Constitution of India
question. has been mentioned under Part III (Article 12- 35).

Indian Polity & Constitution 56 YCT


09.
Fundamental Duties
1. Which one of the following is incorrect? Ans. (c) Part IV-A of the Constitution specifies eleven
(a) Fundamental Duties are the part of the Fundamental Duties in Article 51A added after 42nd
Fundamental Rights. Constitution Amendment, 1976. These Fundamental
(b) Fundamental Duties are not a part of Duties are legally not enforceable. According to Article
Fundamental Rights. 51A, it shall be the duty of every citizens of India
(c) Fundamental Duties are enumerated in the (a) To abide by the Constitution and respect its ideals
Part –IV A of Indian Constitution. and institutions, the National Flag and the
(d) Article 51 A explains 10 duties of every National Anthem
citizen of India. (b) To cherish and follow the noble ideals which
nd
UPPCS (Main) G.S. II Paper 2009 inspired our national struggle for freedom
Ans. (a) The Constituion 42nd Amendment Act, 1976 (c) To uphold and protect the sovereignty, unity and
added Chapter IVA, consists of only one Article 51A, integrity of India
which deals with a code of eleven Fundamental Duties for (d) To defend the country and render national service
citizens. Fundamental Duties are intended to serve as a when called upon to do so
constant reminder to every citizen that while the (e) To promote harmony and the spirit of common
Constitution specifically conferred on them certain brotherhood amongst all the people of India
Fundamental Rights, it also requires citizens to observe transcending religious, linguistic and regional or
certain basic norms of democratic conduct and democratic sectional diversities; to renounce practices
behavior as rights and duties are co-relative. Fundamental derogatory to the dignity of women
Duties are borrowed from the Constitution of U.S.S.R. (f) To value and preserve the rich heritage of our
2. Which of the following is/are among the composite culture
Fundamental Duties, laid down in the Indian (g) To protect and improve the natural environment
Constitution? including forests, lakes, rivers and wild life and to
1. To preserve the rich heritage of our have compassion for living creatures
composite culture (h) To develop the scientific temper, humanism and
2. To protect the weaker sections from social the spirit of inquiry and reform
injustice (i) To safeguard public property and to abjure
3. To develop the scientific temper and spirit of violence
inquiry (j) To strive towards excellence in all spheres of
4. To strive towards excellence in all spheres of individual and collective activity so that the
individual and collective activity nation constantly rises to higher levels of
Select the correct answer, using the codes endeavour and achievement
below: (k) To provide opportunities for education to his
(a) 1 & 2 Only (b) 2 Only child or ward between the age of 6 to 14 year.
(c) 1, 3, & 4 only (d) 1, 2, 3 & 4 This duty was added by 86th Constitutional
IAS (Pre) G.S., 2012 Amendment Act, 2002.
Indian Polity & Constitution 57 YCT
3. How many Fundamental duties are there in the (c) To help in organizing Village Panchayats.
Constitution of India? (d) To safeguard public property and to abjure
(a) 9 (b) 11 violence.
(c) 12 (d) 20 UPPCS (Pre) Re-exam G.S., 2015
UPPCS (Main) G.S. IInd Paper 2014 Ans. (c) Kindly refer the explanation of the above
UP RO/RAO (Main) G.S., 2014 question.
Ans. (b) Kindly refer the explanation of the above 8. Which one of the following is NOT provided in
question. part IV A (Fundamental Duties) of the Indian
4. "To uphold and protect the Sovereignty, Unity Constitution?
and Integrity of India" is a provision made in the (a) To respect the National Flag.
(a) Preamble of the Constitution (b) To promote the spirit of brotherhood amongst
(b) Directive Principles of State Policy all people of India.
(c) Fundamental Rights (c) To respect our parents and teachers.
(d) Fundamental Duties (d) To preserve the rich heritage of our
IAS (Pre) G.S., 2015 composite culture.
RAS/RTS (Pre) G.S., 2015 UPPCS (Main) G.S. IInd Paper 2014
Ans. (d) Kindly refer the explanation of the above Ans. (c) Kindly refer the explanation of the above
question. question.
5. Under the constitution of India, which one of 9. Which one of the following is not a
the following is not a fundamental duty? Fundamental Duty?
(a) To vote in public elections. (a) To respect the National Anthem
(b) To develop the scientific temper. (b) To safeguard public property
(c) To safeguard public property. (c) To protect monuments and places of public
(d) To abide by the constitution and respect its importance
ideals. (d) To protect and improve the natural
IAS (Pre) G.S., 2011 environment
Ans. (a) Kindly refer the explanation of the above UPPCS (Main) G.S. IInd 2007
question. Ans. (c) Kindly refer the explanation of the above
question.
6. Which one of the following duties have been
prescribed for the citizens? 10. Protection of which one of the following is
(1) To organize village panchayats fundamental duty of Indian citizen?
(2) To minimize inequalities in income (a) Village panchayats
(3) To defend the country (b) National flag
(4) To render military service (c) Schedule caste/Schedule tribe
(5) To secure uniform civil code (d) Wild animal
(6) To vote in public elections UP. Lower (Pre.) G.S. SPL 2008
(a) Only 1, 2 and 3 (b) Only 2, 3 and 5 Ans. (b) Kindly refer the explanation of the above
(c) Only 4, 5 and 6 (d) Only 3 question.
Maharashtra PSC (Pre) G.S. 2017 11. On the recommendation of which one of the
Ans. (d) Kindly refer the explanation of the above following Fundamental Duties have been
question. included in the Constitution of India?
(a) Balwant Rai Mehta Committee
7. Which one of the following is not a part of
(b) Aayangar Committee
Fundamental Duties under Indian
Constitution? (c) Swaran Singh
(a) To defend the country and render national (d) Thakkar Committee
service. MPPCS (Pre) G.S, 2017
(b) To value and preserve the rich heritage of our UPPCS (Pre.) G.S., 2012
composite culture. Uttarakhand PCs (Pre) 2002-03
Indian Polity & Constitution 58 YCT
Ans. (c) Fundamental Duties were added by 42nd Ans. (b) : In 1998, Atal Bihari Vajpayee's Government
Constitutional Amendment Act, 1976 on the had appointed the Justice J.S. Verma Committee to
recommendation of Swaran Singh committee. operationalize the suggestions to teach Fundamental
12. Fundamental duties were incorporated in the Duties to the citizens of India.
Constitution of India-
16. Fundamental duties of Indian citizens are
(a) 40th constitutional amendment
enumerated in:
(b) 42nd constitutional amendment
(a) Part I of the Constitution
(c) 43rd constitutional amendment
(d) 44th constitutional amendment (b) Part IVA of the Constitution
UPPCS (Pre) G.S. 1993, 1995 (c) Part II of the Constitution
BPSC (Pre) G.S, 1996 (d) Part IV of the Constitution
Ans. (b) Kindly refer the explanation of the above UPPCS (Main) G.S. IInd Paper 2006, 2011, 2012
question. Ans. (b) Article 51 A contained in Part IV A of the
13. When Fundamental Duties were included in Constitution deals with Fundamental Duties.
the Constitution? 17. Which of the following statements is/are true of
(a) 1975 (b) 1976 the Fundamental Duties of an Indian citizen?
(c) 1978 (d) 1979
1. A legislative process has been provided to
RAS/RTS (Pre) G.S., 2016
enforce these duties.
Ans. (b) Kindly refer the explanation of the above
2. They are correlative to legal duties. Select the
question.
correct answer using the code given below:
14. Which of the following statements regarding
(a) 1 only (b) 2 only
the Fundamental Duties contained in the
Constitution of India are correct? (c) Both 1 and 2 (d) Neither 1 nor 2
I. Fundamental Duties have formed a part of the IAS (Pre) G.S, 2017
Constitution of India since its adaptation Ans. (d) Fundamental duties were incorporated in the
II. Fundamental Duties are applicable only to Constitution of India to remind every citizen that they
Citizen of India should not only be conscious of their rights but also for
III. Fundamental Duties have become a part of their duties. There is neither specific provision for
the Constitution of India in accordance with enforceability nor any specific prohibition. Hence both
the recommendation of the Swaran Singh
1and 2 are incorrect.
Committee.
IV. Fundamental Duties can be enforced through 18. Which of the following statements regarding
writ jurisdiction Fundamental Duties is not true?
(a) I, II and III correct (a) They can be enforced by the writs
(b) I, II and IV correct (b) They can only be promoted by the
(c) II and III correct constitutional method
(d) II and IV correct (c) They can be used for interpreting ambiguous
TNPSC (Pre) G.S. 2019 statutes
Ans. (c) : Fundamental Duties were added by 42nd (d) The performance of any particular duty
Constitutional Amendment Act, 1976 on the comes within the sphere of constitutional law
recommendation of Swaran Singh committee. Hence which court has to decide
statement I is not correct but statement III is correct.
UPUDA/LDA (Pre.) G.S., 2000
Fundamental Duties are applicable only to citizens of
India. Hence statement II is correct. Fundamental Ans. (a) Fundamental Duties are not legally enforceable
Duties are legally not enforceable. Hence statement IV hence cannot be enforced by writs. Fundamental duties
is not correct. are intended to serve as a constant reminder to every
15. The Verma Committee on Fundamental duties citizen that while the constitution specifically conferred
was Setup in the year. on them certain Fundamental Rights, it also requires
(a) 1999 (b) 1998 citizens to observe certain basic norms of democratic
(c) 1996 (d) 1994 conduct and democratic behaviour because rights and
TNPSC (Pre) G.S. 2015 duties are co-relative.
Indian Polity & Constitution 59 YCT
10.
Directive Principles of
State Policy
1. Which part of the Constitution of India 3. In India, separation of judiciary from executive
declares the ideal of Welfare state? is enjoined by
(a) Directive Principles of State Policy (a) The preamble of the constitution
(b) Fundamental Rights (b) The directive principles of state policy
(c) The seventh schedule
(c) Preamble
(d) The conventional practice
(d) Seventh Schedule
IAS (Pre) G.S. 2020
IAS (Pre) G.S 2020
Ans. (b) Directive Principles of the State Policy are
Ans. (a) Welfare State is a concept of governance in mentioned in Part IV (Article 36 to 51) of the
which the State plays a key role in the protection and Constitution. Article 50 of the Constituion is related to
promotion of economic and social well-being of its the separation of the judiciary from executive.
citizens. It is based on the principles of equality of 4. Which part of the Indian Constitution provides
opportunity and equitable distribution of wealth. It also for establishing India as a Welfare State?
focuses on the governmental responsibility for those (a) The preamble of the Constitution
who are unable to avail themselves of the minimal (b) Fundamental Rights (Part III of the
provisions of a good life. Under this system, the welfare Constitution)
of its citizens is the responsibility of the State. The (c) Directive Principles of State Policy (Part IV
Constitution of India lays down certain Directive of the Constitution)
Principles of State Policy [Part IV (Article 36 to 51)] (d) Schedule IV of the Indian Constitution
which, though not justiciable but are 'fundamental in Haryana PSC (Pre) G.S., 2014
governance of the country' and it is the duty of the State Ans. (c) Kindly refer the explanation of the above
to apply these principles in making laws. Directive question.
Principles lay down that the State shall strive to 5. The ideal of ‘Welfare State' in the Indian
promote the welfare of people by securing and Constitution is enshrined in its
protecting as effectively as it may a social order in (a) Preamble
which justice; social, economic and political shall form (b) Directive Principles of State Policy
in all institutions of national life. (c) Fundamental Rights
2. With reference to the provisions contained in (d) Seventh Schedule
Part IV of the Constitution of India, which of IAS (Pre) G.S., 2015
the following statements is/are correct? U.P. Lower (M) 2013
1. They shall be enforceable by courts BPSC (Pre) 1994
2. They shall not be enforceable by any court Uttarakhand PCs (Pre) 2002-03
3. The principles laid down in this part are to Ans. (b) Kindly refer the explanation of the above
question.
influence the making of laws by the State.
6. The purpose of the inclusion of Directive
Select the correct answer using the code given
Principles of State Policy in the Indian
below:
Constitution is to establish:
(a) 1 only (b) 2 only
(a) Political democracy
(c) 1 and 3 only (d) 2 and 3 only (b) Social democracy
IAS (Pre) G.S 2020 (c) Gandhian democracy
Ans. (d) Kindly refer the explanation of the above (d) Social and economic democracy
question. IAS (Pre) G.S., 2002
Indian Polity & Constitution 60 YCT
Ans. (d) Kindly refer the explanation of the above country and it shall be the duty of the State to apply
question. these principles in making laws. Hence Directive
7. Which one of the following is not the aim of the Principle of State Policy does not constitute limitations
concept of Directive Principle of the State on either legislative function or executive function. On
Policy? the other hand, Constitution of India constitutes
(a) To establish a social welfare state limitation on the functioning of both legislative and
executive through Fundamental Rights mentioned in
(b) To ensure social economic justice
Part III of the Constitution from Article 12 to 35.
(c) To establish a religious State
11. The Judiciary was separated from the
(d) To establish a secular State
Executive under which Article:
UP RO/ARO (Pre) G.S., 2013
(a) Article 50 (b) Article 64
UPPCS (Main) G.S IInd 2017
(c) Article 60 (d) Article 51
Ans. (c) Directive Principles of the State Policy
TNPSC (Pre) G.S. 2019
mentioned in Part IV (Article 36 to 51) of the
Constitution does not aim to establish a religious State. Ans. (a) Kindly refer the explanation of the above
question.
8. In Indian Constitution the Directive Principals
of State policy were incorporated in Article 12. Which principle among the following was
from. added to the Directive Principles of State Policy
(a) Art. 40 to Art. 51 by the 42nd Amendment to the Constitution?
(a) Equal pay for equal work for both men and
(b) Art. 36 to Art. 51
women
(c) Art. 39 to Art. 51
(b) Participation of workers in the management
(d) Art. 25 to Art. 51
of industries
TNPSC (Pre) G.S. 2014
(c) Right to work, education and public
Ans. (b) : Kindly refer the explanation of the above assistance
question. (d) Securing living wage and human conditions
9. The Directive Principles of State Policy may be of work to workers
classified in to which of the following parts for IAS (Pre) G.S, 2017
the sake of convenient study?
Ans. (b) The Constitution (Forty-Second Amend-ment)
(a) Socialist Act, 1976 added three more Directive Principles namely
(b) Liberal intellectualistic 39A, 43A and 48A. Article 39A deals with equal justice
(c) Gandhian and free legal aid, Article 43A deals with participation
(d) All of the above of workers in the management of industries and Article
UPPCS (Pre) G.S, 2018 48A is protection and improvement of environment and
Ans: (d) Directive Principles of the State Policy are safeguarding of forests and wildlife.
classified under the following three categories: 13. In which of the following ‘the provisions for the
Gandhian Principles, Socialist Principles and Liberal- protection and improvement of environment
Intellectual Principles. and safeguarding of forests and wildlife’ are
10. Consider the following statements: found in the Constitution of India?
With reference to the Constitution of India, the (a) Only in the Directive Principles of State
Directive Principles of State Policy constitute Policy
limitations upon (b) Only in the Fundamental Duties
1. Legislative function. (c) Both (a) and (b)
2. Executive function. (d) None of the above
Which of the above statements is/are correct? UPPCS (Pre) G.S, 2019
(a) 1 only (b) 2 only Ans: (c) Article 51 A (g) under Fundamental Duties
(c) Both 1 and 2 (d) Neither 1 nor 2 asserts that it shall be the fundamental duty of every
IAS (Pre) G.S, 2017 citizen to protect and improve the natural environment
Ans. (d) Directive Principles of the State Policy are including forests and wildlife whereas Article 48 A
mentioned in Part IV (Article 36 to 51) of the under the Directive Principle of State Policy is
Constitution. Directive Principles are not enforceable by regarding protection and improvement of environment
any court but the principles therein laid down are and safeguarding of forests and wildlife. Hence (c) is
nevertheless fundamental in the governance of the the correct answer.

Indian Polity & Constitution 61 YCT


14. Which of the following Directive Principles did Rights. It proposed that the scope of the Article 31C
not form part of the original constitution and (saving of laws giving effect to certain Directive
were added through constitutional Principles) should be widened so as to cover legislation
amendments? for implementation of all or any of the Directive
(1) To minimize inequality in income. Principles enumerated in Part IV of the Constitution and
(2) To protect and improve the environment and such legislation should not be called in question on the
to safeguard forests and wildlife. ground of infringement of any of the Fundamental Rights
(3) Equal pay for equal work for both men and mentioned in Part III.
women 17. Which amendment of the Constitution
(4) To ensure equal justice and free legal aid to provided that no law passed to give effect to
the poor. Directive Principles of State Policy contained in
(5) To secure adequate means of livelihood for Articles 39 (b) & (c) shall be deemed to be void
all citizens. on the ground that it abridges the rights
(a) 1, 2 and 3 (b) 2, 3 and 4 conferred by Articles 14 and 19?
(c) 3, 4 and 5 (d) 1, 2 and 4 (a) 25th Amendment (b) 28th Amendment
Maharashtra PSC (Pre) G.S 2018 (c) 42ndAmendment (d) 44th Amendment
Ans. (d) To minimize inequality in income was added UPPCS (Main) G.S., IInd Paper 2009
by the Constitution (Forty-fourth Amendment) Act,
Ans. (c) Kindly refer the explanation of the above
1978 under Article 38 of the Directive Principle of the
question.
State Policy. Protection and improvement of
environment and safeguarding of forests and wildlife 18. Which one of the following is included in the
(Article 48A), equal justice and free legal aid (Article Directive Principles of State Policy?
39A) were added by the Constitution (Forty-second (a) Protection in respect of Conviction for
Amendment) Act, 1976. Equal pay for equal work for offences.
both men and women has been mentioned in Article 39 (b) Protection of life and personal liberty.
(d) and to secure adequate means of livelihood for all (c) Protection of interest of minorities.
citizens has been provided under Article 39 (a), which (d) Equal pay for equal work for both men and
are part of the original Constituion. women
15. Which one of the following Directive Principle UPPCS (Main) G.S.IInd Paper 2012
was added after the implementation of the
Ans. (d) Equal pay for equal work for both men and
Constitution?
women is Directive Principle of the State Policy
(a) Organization of village panchayats
mentioned in Article 39(d) of the Constitution. Rest
(b) Ban on cow slaughter
three is Fundamental Rights.
(c) Free legal aid
19. Which of the following is/are included in the
(d) Uniform civil code
Directive Principles of State policy?
UP RO/ARO (Pre) G.S., 2014
1. Prohibition of traffic in human beings and
Ans. (c) Kindly refer the explanation of the above
forced labour
question.
2. Prohibition of Consumption except for
16. The Swaran Singh committee considered the
medicinal purposes of intoxicating drinks and
question of
of other drugs which are injurious to health
(a) More authority to Punjab on the model of
Select the correct answer using the code given
Jammu & Kashmir
below:
(b) The suitability of the Presidential form of
Government in India (a) 1 Only (b) 2 Only
(c) The precedence of Directive Principles over (c) Both 1 and 2 Only (d) Neither 1 nor 2
Fundamental Rights IAS (Pre) G.S., 2008
(d) Administrative reforms Ans. (b) Fundamental Right mentioned in Article 23 of
IAS (Pre) G.S., 1993 Part III of the Constitution of India is prohibition of
nd traffic in human beings and forced labour. Hence
Ans. (c) 42 Constitution Amendment was made on the
recommendation of Swaran Singh Committee suggested statement 1 is not Directive Priciple of State Policy.
the precedence of Directive Principles over Fundamental Article 47 of the Directive Principle of the State Policy is
Indian Polity & Constitution 62 YCT
regarding duty of the State to raise the level of nutrition (c) Equal pay for equal work
and the standard of living and to improve public health. It (d) Right to Information
asserts that the State shall regard the raising of the level UPPCS (Pre) G.S., 2006
of nutrition and the standard of living of its people and UPPCS (Main) G.S., IInd Paper 2010
the improvement of public health as among its primary Ans. (d) Right to Information is statuary right while rest
duties and the State shall endeavour to bring about are Directive Principle of the State Policy.
prohibition of the consumption except for medicinal 23. Which of the following is/are mentioned in the
purposes of intoxicating drinks and of drugs which are Directive Principles of State Policy?
injurious to health. (a) Securing a social order for the promotion of
20. Which one of the following is a Directive welfare of the people.
Principle of the State Policy? (b) Organization of Agriculture and Animal
(a) Uniform Civil Code Husbandary.
(b) Freedom of the Press (c) Provision of giving opportunities and
facilities to the children for their development
(c) Freedom of the Religion
in a healthy manner.
(d) Equality before Law
(d) Provision of safeguarding public property.
UPPCS (Main) G.S. IInd Paper 2010 Choose the correct answer from the following:
Ans. (a) Article 44 of the Directive Principle of the (a) 2 and 3 (b) 1, 3 and 4
State Policy asserts that the State shall endeavour to (c) 1, 2 and 3 (d) 1, 2, 3 and 4
secure uniform civil code for the citizens throughout the Punjab PSC (Pre) G.S 2013
territory of India. Freedom of press in India comes Ans. (c) Provision of safeguarding public property is a
under Article 19(1) (a) which asserts all the citizens Fundamental Duty under Article 51A of the
shall have right to freedom of speech and expression. Constitution. Rest are Directive Principles of the State
Freedom of religion and equality before law comes Policy.
under Fundamental Rights mentioned in Article 25 and 24. Constitution is silent about which of the
14 respectively. following Directive Principle of the State
21. Which one of the following is a Directive Policy?
(a) Adult education
principle of State Policy?
(b) Living wages for workers
(a) The State shall endeavour to protect and
(c) Free legal aid to poor
improve the environment.
(d) Free and compulsory education
(b) The State shall not deny to any person
U.P. Lower (Pre) G.S. 2008
equality before law.
Ans. (a) Under the Directive Principle of the State
(c) The State shall not discriminate against any
Policy, living wages for workers is mentioned in Article
person on grounds of religion, race, caste, sex 43, free legal aid to poor in Article 39A and free and
or place or birth. compulsory education in Article 45. The Constitution of
(d) Untouchability enforcement. India is silent about the adult education yet.
UPPCS (Main) G.S. IInd Paper 2009 25. Which one of the following is not Directive
Ans. (a) By the 42nd Constitution Amendment Act, Principle?
1976, Article 48 A was inserted which asserts that the (a) Prohibition of liquor
State shall endeavour to protect and improve the (b) Prohibiting the slaughter
environment and to safeguard the forests and wild life (c) Environment protection
of the country. Rest three given options are related to (d) Free education for the children up to fourteen
Fundamental Rights. years
UPUDA/LDA (Pre.) G.S., 2010
22. Which one of the following is not a Directive
Ans. (d) By the Constitution 86th Amendment Act,
Principle of State Policy?
2002, Article 21A was added to the Constituion which
(a) Prohibition of Liquor
asserts that the State shall provide free and compulsory
(b) Right to Work
education to all children of the age of six to fourteen
Indian Polity & Constitution 63 YCT
years and substituted a new heading for Article 45 as 29. Consider the following provisions under the
‘provision for early childhood care and education to Directive Principles of State Policy as
children below the age of six years’. Hence free enshrined in the Constitution of India:
1. Securing for citizens of India a uniform civil
education for the children up to fourteen years is
code
Fundamental Right instead of Directive Principle of the
2. Organizing village Panchayats
State Policy.
3. Promoting cottage industries in rural areas
26. Which of the following is not the Directive 4. Securing for all the workers reasonable
Principle of the State Policy? leisure and cultural opportunities
(a) Prohibition on Liquor Which of the above are the Gandhian
(b) Prohibition on cow slaughter Principles that are reflected in the Directive
(c) Protection of environment Principles of State Policy?
(a) 1, 2 and 4 only (b) 2 and 3 only
(d) Free education for the children up to fourteen
(c) 1, 3 and 4 only (d) 1, 2, 3 and 4
years
IAS (Pre) G.S., 2012
U.P. U.P. Lower (Pre) G.S. 2013
Ans. (b) Directive Principle of the State Policy
Ans. (d) Kindly refer the explanation of the above contained in Part IV from Article 36 to 51 of the
question. Constitution of India are not enforceable by any court
27. In the Constitution of India, promotion of but the principles laid down therein are considered
fundamental in the governance of the country, making it
international peace and security is included in
the duty of the State to apply these principles in making
the
laws to establish socio-economic democracy in the
(a) Preamble to the Constitution country. Article 40 regarding village panchayats, Article
(b) Directive Principles of State Policy 43 regarding the promotion of cotton industry, Article
(c) Fundamental Duties 43 B regarding management of co-operative society,
(d) Ninth Schedule Article 46 regarding social and economic development
of schedule caste and scheduled tribes, Article 47
IAS (Pre) G.S., 2002 regarding prohibit the consumption of intoxicating
IAS (Pre) G.S., 2014 drink, drugs and Article 48 regarding prohibit the
UPPCS (Main) G.S. 2017 slaughter of cows. These all Directive Principles are
Ans. (b) Article 51 of the Directive Principle of the based on Gandhian Ideology.
State Policy asserts that the State shall endeavour to 30. Consider the following statements regarding
the Directive Principles of State Policy:
promote international peace and security, maintain just
1. The Principles spell out the socio-economic
and honourable relations between nations, foster respect
democracy in the country.
for international law and treaty obligations in the 2. The provisions contained in these Principles
dealings of organized peoples with one another and are not enforceable by any court.
encourage settlement of international disputes by Which of the statements given above is / are
arbitration. correct?
28. Provision for Panchayati raj system in India (a) 1 only (b) 2 only
has been made under which of the following? (c) Both 1 and 2 (d) Neither 1 nor 2
IAS (Pre) G.S., 2015
(a) Fundamental Rights
Ans. (c) Kindly refer the explanation of the above
(b) Fundamental Duties
question.
(c) Directive Principle of the State Policy
31. The ''Instrument of Instructions" contained in
(d) Election commission Act the Government of India Act 1935 have been
BPSC (Pre) G.S., 2002 incorporated in the Constitution of India in the
Ans: (c) Article 40 of the Directive Principle of the year 1950 as
(a) Fundamental Rights
State Policy is regarding the organization of village
(b) Directive Principles of State Policy
panchayats which asserts that the State shall take steps
(c) Extent of executive power of State
to organize village panchayats and endow them with
(d) Conduct of business of the Government of
such powers and authority as may be necessary to India
enable them to function as units of self government. IAS (Pre) G.S., 2010
Indian Polity & Constitution 64 YCT
Ans. (b) Instruments of instructions were issued to the Ans. (a) Mahatma Gandhi National Rural Employ-ment
Governor General of India by the British Government Guarantee Act (MGNREGA) scheme is the largest
under the 1935 Act under the Draft Constitution. It was social security scheme in the world, providing legal
proposed to issue such instructions to the Viceroy and guarantee for atleast 100 days of unskilled manual work
Governors. The ''Instrument of Instructions" contained to all rural households in India. MGNREGA was
in the Government of India Act,1935 have been launched to implement Article 43 of the Constitution
incorporated in the Constitution of India in the year under the Directive Principle of State Policy related to
1950 as Directive Principle of the State Policy. living wages for workers.
32. With reference to the Constitution of India, 35. Which of the following is/are listed among the
consider the following: Directive Principles in Part-IV of the
Constitution of India?
1. Fundamental Rights
I. Equal Pay for Equal Work
2. Fundamental Duties
II. Uniform Civil Code
3. Directive Principles of State Policy
III. Small family norm
Which of the above provisions of the
IV. Education through mother tongue at primary
Constitution of India is/are fulfilled by the
level
National Social Assistance Programme
Code :
launched by the Government of India?
(a) I, II and III
(a) 1 only (b) 3 only
(b) I and II
(c) 1 and 3 only (d) 1, 2 and 3
(c) II and III
IAS (Pre) G.S., 2010
(d) I, II and IV
Ans. (b) The National Social Assistance Programme
(e) None of the above/More than one of the
(NSAP) came into effect from 15th August,1995,
above
represents a significant step towards the fulfillment of
BPSC (Pre) G.S. 2016
the Directive Principles in Article 41( right to work, to
Ans. (b) Directive Principles of the State Policy are
education and to public assistance in certain cases) of
contained in Part IV from Article 36 to 51 of the
the Constitution. The program introduced a national
Constitution. Article 39(d) of the Constitution is about
policy for social assistance for the poor and aims at
equal pay for equal work for both men and women and
ensuring minimum national standard for social
Article 44 is about uniform civil code. Small family
assistance.
norm has not been mentioned in the Constitution of
33. 'The Directive Principles of State Policy is a India while education through mother tongue at primary
cheque which is paid on Bank's Convenience. level is mentioned in Article 350 (A), which is not
Who told it? Directive Principle of the State Policy.
(a) B.R. Ambedkar 36. According to the Constitution of India, which
(b) K.M. Munshi of the following are fundamental for the
(c) Dr. Rajendra Prasad governance of the country?
(d) K.T. Seth (a) Fundamental Rights
(UPPCS (Main) G.S. IInd Paper 2007) (b) Fundamental Duties
Ans. (d) Above statement was quoted by Prof (c) Directive Principles of State Policy
K.T.Shah. (d) Fundamental Rights and Fundamental Duties
34. MGNREGA scheme was launched to IAS (Pre) G.S., 2013
implement which Article of the Constitution of Ans. (c) The Constitution of India lays down certain
India? Directive Principles of State Policy in Part IV which
(a) Article 43 (b) Article 45 though not justifiable but are 'fundamental in
(c) Article 47 (d) Article 50 governance of the country' and it is the duty of the State
to apply these principles in making laws.
(UPUDA/LDA (Pre.) G.S., 2010)
Indian Polity & Constitution 65 YCT
11.
Parliamentary System
1. A Parliamentary System of Government is one 4. Which of the following statement is correct?
in which (a) Constitution of India is Presidential.
(a) All political parties in the Parliament are (b) India is a virtual democracy
represented in the Government (c) India is a noble democracy
(b) The Government is responsible to the (d) India is a Parliamentary democracy
Parliament and can be removed by it UPPCS (Pre) G.S., 2005
(c) The Government is elected by the people and Ans. (d) Kindly refer the explanation of the above
can be removed by them question.
(d) The Government is chosen by the Parliament 5. The Indian parliamentary system is different
but cannot be removed by it before from British parliamentary system in that
completion of a fixed term India has
IAS (Pre) G.S 2020 (a) Both a real and a nominal executive
Ans. (b) Democratic India has Parliamentary form of (b) A system of collective responsibility
government where head of the State is President. The (c) Bicameral legislature
executive authority of the President is exercised by the (d) The system of judicial review
Prime Minister and his council of ministers. The IAS (Pre) G.S., 1998
council of ministers (government) is collectively Ans. (d) The system of judicial review in Indian
responsible to the Lok Sabha. parliamentary system provides power to judiciary to
2. Which of the following is not correct regarding review and determine the validity of a law. It has been
taken from the Constitution of America. This special
Indian political system?
feature is not available in British parliamentary system.
(a) Secular State
6. Which one of the following points differentiates
(b) Parliamentary form of Government
the Indian Parliamentary system from the
(c) Union policy
British Parliamentary system?
(d) Presidential form of Government
(a) Collective Responsibility
UPPCS (Main) SPl. GS. IInd Paper 2008
(b) Judicial Review
Ans. (d) Kindly refer the explanation of the above (c) Bicameral Legislature
question. (d) Real and nominal Executive
3. The Parliamentary form of Government was UPPCS (Main) GS. IInd Paper 2010
first introduced in which country? Ans. (b) Kindly refer the explanation of the above
(a) Great Britain (UK) (b) Belgium question.
(c) France (d) Switzerland 7. In the context of India, which of the following
UPPCS (Pre) G.S 2012 principles is/are implied institutionally in the
Ans: (a) The Parliamentary form of Government was parliamentary government?
first introduced in Great Britain (UK). The first British 1. Members of the Cabinet are Members of the
Parliament was convened in 1215 with the creation and Parliament.
signing of the ‘Magna Carta’. The concept of Indian 2. Ministers hold the office till they enjoy
Parliamentary form of government has been borrowed confidence in the Parliament.
from Britain. 3. Cabinet is headed by the Head of the State.
Indian Polity & Constitution 66 YCT
Select the correct answer using the codes given Ans. (c) Judiciary, legislature and executive are the
below. three pillars of parliamentary form of democracy.
(a) 1 and 2 only (b) 3 only Together, the executive and legislature constitutes the
(c) 2 and 3 only (d) 1, 2 and 3 Government. The role of a legislative branch within a
IAS (Pre) G.S., 2013 Government is to make laws. On the other hand
executive branch deals with initiating policies and puts
Ans. (a) Head of the State in Parliamentary form of
the laws that are passed by the legislature into practice.
Government in India is President but Cabinet is headed
The role of judiciary is to adjudicate disputes between
by the Prime Minister. Hence statement 3 is not correct.
parties in a legal setting by relaying on the laws that are
All cabinet ministers are mandated by the Constitution
passed by the legislative branch.
of India to be member of either house of the Parliament.
Hence statement 1 is correct. Ministers can be removed 11. In a Parliamentary system of the Government
from his office by issuing no confidence motion in the (a) Judiciary controls Executive
Parliament. Hence statement 2 is also correct. (b) Executive controls Judiciary
(c) Executive controls Legislature
8. Under the Indian political system executive
works under______? (d) Legislature controls Executive

(a) Judiciary UPPCs (Main) G.S. IInd Paper 2010

(b) Legislature Ans. (d) The essence of parliamentary democracy is


that the executive branch is accountable and subordinate
(c) Election Commission
to the legislative branch.
(d) Union Public Service Commission
12. Which one of the following statements is
BPSC (Pre.) G.S. 2002
correct regarding parliamentary form of
Ans. (b) India adopted parliamentary form of
Government?
democracy in which executive works under legislature.
(a) The legislature is responsible to Judiciary.
9. There is a Parliamentary System of
(b) The Legislature is responsible to Executive.
Government in India because the
(c) The Legislature and the Executive are
(a) Lok Sabha is elected directly by the people
independent.
(b) Parliament can amend the constitution (d) The President is responsible to Judiciary
(c) Rajya Sabha cannot be dissolved (e) None of the above/ More than one of the
(d) Council of Ministers is responsible to the Lok above
Sabha BPSC (Pre) G.S 2019
IAS (Pre) G.S., 1997 Ans. (e) None of the option is correct in the above
Ans. (d) Collective responsibility of the Council of question as in parliamentary form of Government,
Ministers to the Lok Sabha (Article 75) lies at the heart Legislature is responsible to the Constituion of the
of a Parliamentary democracy. Rest are characteristics country. Both Legislature and Executive are not
of democracy but these are not necessarily for independent as Legislature is bound by Constituion and
parliamentary forms of democracy. Executive is accountable to Legislature. It is the duty of
10. Parliamentary Government works on the the President to preserve, protect and defend the
principle of- Constituion. Therefore President is responsible to the
(a) Division of power Constituion of the country. Hence none of the given
(b) Control and balance options is correct.

(c) Close relationship between judiciary and 13. Which of the following is not characteristic of
executive the Indian Constitution?
(d) Control of judiciary over executive (a) Parliamentary Government

Uttarakhand RO/ARO, 2016 (b) Presidential Government

Indian Polity & Constitution 67 YCT


(c) Independent Judiciary system of governance. The relationship between Union
(d) Federal Government and State are well defined in written Constitution. In
UPPCS (Main) G.S. II Paper 2015 case of a conflict between the Union and the State on
nd

Ans. (b) India adopted parliamentary form of any issue, the judiciary has the powers to resolve the
Government where Parliament is supreme. In India, disputes.
Judiciary is independent and in place of federal 16. With reference to the federal system in India,
Government, Constitution of India describes India as which of the statements is/are correct?
‘Union of States’. 1. States have no right to secede from the
14. Consider the following statements and choose Union under the constitution of India
the correct answer by using the codes given 2. Just advocacy of secession will have the
below: protection of freedom of expression
Assertion [A] : India is a democratic country. Select the correct answer from the codes given
Reason [R] : India has a constitution of its own below
Codes : (a) 1 only
(a) Both [A] and [R] are true and [R] is the (b) 2 only
correct explanation of [A] (c) Both 1 and 2
(b) Both [A] and [R] are true but [R] is not the (d) Neither 1 nor 2
correct explanation of [A] UPPCS (Pre) G.S, 2019
(c) [A] is true but [R] is false Ans: (a) Article 1 of the Constitution of India asserts
(d) Both [A] and [R] are false that India is ‘Union of States’. Here States have no right
UPPCS (Pre) G.S, 2017 to secede from the Union under the Constitution of
Ans. (b) India is a democratic country as Government is India. Hence statement 1 is correct. Just advocacy of
elected by people hence assertion (A) is correct. India secession will not have the protection of freedom of
has longest written Constitution in world hence reason expression under Article 19 (1) (a) of the Constituion of
is also correct but reason is not the correct explanation India. Hence statement 2 is incorrect.
of assertion. 17. The Constitution of India declares India as
15. A Federal polity involves (a) A voluntary Federation
1. Relations between Union & States. (b) A Confederation
2. Relations among States (c) A Union of States
3. Mechanism for coordination (d) A Federation
3. Mechanism for resolving disputes. Uttarakhand RO/ARO, 2016
Select your correct answer from the codes Ans: (c) Kindly refer the explanation of the above
given below: question.
Codes : 18. Which of the following determines that the
(a) 1, 2, 3 and 4 (b) only 1, 2 and 3 Indian Constitution is Federal?
(c) only 1, 2 and 4 (d) only 2, 3 and 4 (a) A written and rigid Constitution
UPPCS (Mains) G.S.IInd Paper 2010 (b) An independent judiciary
Ans. (a) Federalism is classically defined as a system of (c) Vesting of residuary powers with centre
governance in which the distribution of power is (d) Distribution of powers between Centre and
governed by a Constitution and the two levels namely the States
Union and the States. Each level of the Government has Jharkhand PSC (Pre) G.S. 2006
distinct powers and responsibilities and has a separate IAS (Pre) G.S., 1994
Indian Polity & Constitution 68 YCT
Ans. (d) In the federal system, there is two tier of 22. Which of the following is correct?
Government (Center and State) with well assigned The main feature of a Presidential form of
power and functions. The Center and State Government Government is/are
work in coordination with each other. (a) The Head of the Executive is the President.
19. India is a federal State, how? (b) The President appoints his council of
(a) Independent Judiciary Ministers.
(b) Division of power between Center and State (c) The President cannot dissolve the Legislature.
(c) Residual power of Center (d) All of the above.
(d) Formation of Finance Commission UPPCS (Main) G.S. IInd Paper 2014
Jharkhand PSC (Pre) G.S. 2008
Ans. (d) In Presidential form of Government, President
Ans. (b) Kindly refer the explanation of the above is the head of Government as well as the State. United
question.
States of America is an example of Presidential form of
20. Which of the following is a feature common to Government. The President can appoint the members of
both the Indian Federation and the American Cabinet and can be removed by him only.
Federation? The Legislature cannot be dissolved by the President.
(a) A single citizenship
23. Which of the following institutes are
(b) Three lists in the Constitution
considered necessary to promote ‘Unity among
(c) Dual judiciary
Diversity’ in the Indian Federalism?
(d) A federal Supreme Court to interpret the
(a) Inter-State Councils & National Develope
Constitution
Council
IAS (Pre) G.S., 1993
(b) Finance Commission & Regional Council
Ans. (d) The Federation of India has taken the concept
(c) Unitary Judicial System & All India Services
of a federal Supreme Court to interpret the Constitution
(d) All of above
from the Constitution of America.
Uttarakhand PSC (Pre.) 2016
21. The Union Territories get representation in
(a) Lok Sabha Ans. (d) Indian Federalism is a unique example of

(b) Rajya Sabha ‘Unity among Diversity’. Constitution of India is

(c) Both houses of Parliament supreme and power is divided between Union and
States. In case of any dispute between Centre and State,
(d) None of the above
several provisions have been made by the Constitution.
UPPCS (Main) G.S. IInd Paper 2015
Inter-State Council is a body set up for strengthening
Ans: (c) The Constitution of India (Article 80) has
Center-State and inter State relationship. National
made provision that the Rajya Sabha shall consist of
Development Council is established for the proper
250 members, of which 12 members shall be nominated
distribution of resources among States. Finance
by the President and not more than 238 representatives
of the States and of the two Union Territories (Delhi Commission was created to define the financial
and Puducherry). The present strength of Rajya Sabha is relations between Centre and State Governments and
245, out of which 233 are representative of the States individual State Governments. Zonal Councils are
and Union territories of Delhi and Puducherry and 12 advisory boards, meant to resolve irritants between
are nominated by the President. The maximum strength Centre and States and amongst States through free and
of the Lok Sabha envisaged by the Constitution is now frank discussions and consultations. Judicial system in
552 (530 members to represent States, 20 to represent India is independent and unitary in nature. All India
Union Territories and not more than two members of Services (IAS, IPS, IFS) also represent unity among
the Anglo-Indian community). diversity in Indian Federism.
Indian Polity & Constitution 69 YCT
12.
Centre-State and Inter
State Relations
1. Center and State Governments gets executive power of the Union and the executive power
authorization- of the Union shall extend to the giving of such
(a) From Constitution of India directions to a State as may appear to the Government
(b) From President of India of India to be necessary for that purpose.
(c) From Prime Minister of India 4. Division of constitutional power between
(d) From the Parliament of India Center and State is provided in-
UPPCS (Main) 2017 G.S. IInd Paper (a) Fifth Schedule (b) Sixth Schedule
Ans. (a) The Constitution of India provides a dual (c) Seventh Schedule (d) Eight Schedule
polity with a clear division of power between Union and UPPCS (Pre.) G.S., 1993
States, each being supreme within sphere allotted to it. Ans. (c) Division of constitutional power between
Article 245 to 255 in part XI of the Constitution of India Center and State is provided in the Seventh Schedule of
deals with the legislative relationship between Center the Constitution. It has been divided in three lists
and State. namely Union, State and Concurrent list.

2. Which part and chapter of the Indian 5. Constitution of India provides three Articles
Constitution deal with the legislative relations for the division of power between Center and
between the Union and the States? States. Which of the two Articles among given
Articles regulates the distribution of power?
(a) Part XI and Chapter I
(a) Article 245 and 246
(b) Part XI and Chapter II
(b) Article 4 and 5
(c) Part XII and Chapter I
(c) Article 141 and 142
(d) Part XII and Chapter II
(d) Article 56 and 57
UPPCS (Main) G.S. IInd Paper 2012
UPPCS (Pre.) G.S., 2003
Ans. (a) Part XI and Chapter I of the Indian Constituion
Ans. (a) The relationship between Center and State are
deals with the legislative relations between the Union
divided as legislative relations, administrative relations
and the States while chapter II deals with administrative
and financial relations. Articles 245 to 255 in Part XI of
relations between the Union and the States.
the Constitution deal with the legislative relations
3. Which one of the following Articles of the between the Center and the State. Article 256 to 263
Constitution of India says that the executive deals with administrative relationship while Article 268
power of every State shall be so exercised as to 293 deals with the provisions of financial relations
not to impede or prejudice the exercise of the between Center and States.
execution power of the Union? 6. Which of the following is not related to Center-
(a) Article 257 (b) Article 258 State relationship?
(c) Article 355 (d) Article 356 (a) Sarkaria Commission
IAS (Pre) G.S., 2014 (b) Rajamannar Committee
Ans. (a) Article 257(1) of the Constitution asserts that (c) Indrajit Gupta Commission
the executive power of every State shall be so exercised (d) Punchhi Commission
as not to impede or prejudice the exercise of the UPPCS (Pre) G.S., 2015
Indian Polity & Constitution 70 YCT
Ans: (c) The Indrajit Gupta Committee was related to Government of India on June 9, 1983 constituted a
State funding of election which backed the idea of State commission under the Chairmanship of Justice R.S.
funding of elections on principle. Sarkaria Commission Sarkaria with Shri B. Sivaraman and Dr. S.R. Sen as its
was set up in June 1983 to examine the relationship and members. Sarkaria commission has submitted his final
balance of power between State and Central report in October 1987.
government and suggest the setting of an Inter-State 10. The Provisions regarding division of taxes
Council. Rajamannar Committee was appointed by between the Union and the States
Tamil Nadu Government in 1969 to examine Center-
(a) Can be suspended during financial
State relationship. It recommended abolition of All
emergency.
India Services. The Government of India constituted
(b) Can be suspended during National
Punchhi Commission on Centre-State Relations on 27th
emergency.
April 2007 to look into the new issues of Centre-State
relations keeping in view the changes that have taken (c) Cannot be suspended under any
place in the polity and economy of India. circumstances.

7. The recommendations of Sarkaria Commission (d) Can be suspended only with the consent of
relate to which of the following? majority of the state Legislatures.
(a) Distribution of revenue Chattisgarh PSC (Pre) G.S. 2010-11
(b) Power and functions of the president Ans. (b) During national emergency, the President by
(c) Membership of Parliament order can direct that all provisions regarding division of
(d) Centre-State relations taxes and grants-in-aid remain suspended. However
UPPCS (Pre) G.S, 2018 such suspension shall not go beyond the expiration of
the financial year.
Ans: (d) Kindly refer the explanation of the above
question. 11. Which one of the following is not a feature of a
8. Which Commission advocated the zonal council?
establishment of a permanent Inter-state (a) It is a constitutional body
Council, called 'Inter-governmental Council? (b) Five Zonal Councils have set up under the
(a) Punchhi Commission States Reorganization Act, 1956
(b) Sarkaria Commission (c) Chandigarh although not a state is included in
(c) Radhakrishnan Commission Zonal Council
(d) Moily Commission (d) It is an Advisory body
st
Jharkhand PSC (Pre.) G.S. I Paper, 2013 UPPCS (Pre) G.S, 2017
Ans. (b) Kindly refer the explanation of the above Ans. (a) In the light of the vision of Pandit Nehru, five
question. Zonal Councils (Northern, Central, Eastren, Westren
9. Which of the following was the member of and Southern) were set up by Part-III of the States Re-
Sarkaria Commission? organisation Act, 1956 to provide a common meeting
(a) V.Shankar ground to the States and Union Territories in each zone
(b) K. Hanumanthhaiya It is an advisory body not a constitutional body. The
(c) Dr.S.R.Sen Zonal Council meetings are chaired by the Union Home
(d) O.V.Algesan Minister. Chandigarh falls under the Northern Zonal
UPPCS (Pre) G.S,2017 Council.
Ans. (c) With a view to reviewing the working of the 12. Which of the following statements about Zonal
existing arrangements between the Union and the States Councils are not correct?
in the changed socio-economic scenario, the (1) The Zonal Councils are the constitutional
bodies.
Indian Polity & Constitution 71 YCT
(2) They are established by the States up under Article 263 of the Constitution by the
Reorganisation Act of 1956. Presidential order. Prime Minister being chairman of the
(3) Home Minister of the Central Government is Inter-State Council and the mebers of Inter State
the Chairman of Zonal Councils. Council are; Chief Ministers of all the States, Chief
(4) India is divided into seven zones and Ministers of Union Territories having a Legislative
provided a Zonal Council for each zone. Assembly and Administrators of Union Territories not
(a) Only 1 and 3 (b) Only 1 and 4 having a Legislative Assembly, Six Ministers of
(c) Only 1, 3 and 4 (d) All of the above Cabinet rank in the Union Council of Ministers (to be
Maharashtra PSC (Pre) G.S 2017 nominated by the Prime Minister) and four Ministers of
Ans. (b) Kindly refer the explanation of the above Cabinet rank are as permanent invitee’s members.
question. 15. Who is the Chairman of the Inter-State
13. Consider the following statements: Council?
(1) Zonal Councils are the constitutional bodies. (a) The President of India
(2) The Prime Minister acts as the Chairman of (b) The Union Cabinet
all Zonal Councils.
(c) The Prime Minister
(3) Each Chief Minister acts as the Vice-
(d) The Union Home Minister
Chairman of the Zonal Council by rotation.
Punjab PSC (Pre) G.S 2018
(4) Joint meetings of two or more Zonal Councils
are presided over by the Union Home Ans. (c) Kindly refer the explanation of the above
Minister. question.
(a) Statements 1, 2 and 3 are correct 16. The source of the formation of Inter State
(b) Statements 2, 3 and 4 are correct Council is-
(c) Statements 1, 3 and 4 are correct (a) Constitutional provision
(d) Statements 3 and 4 are correct (b) Parliamentary law
Maharashtra PSC (Pre) G.S 2018 (c) Recommendation of Planning Commission
Ans. (d) The Zonal Council meetings are chaired by the (d) Ethics approved after Chief Ministers
Union Home Minister. The Chief Ministers and two meeting
Ministers from the member States are the members in UPPCS (Pre) G.S., 2000
each Zonal Council. Each Chief Minister acts as the
Ans. (a) Kindly refer the explanation of the above
Vice-Chairman of the Zonal Council by rotation,
question.
each holding office for a period of one year at a time. In
17. Under which Article of the Constitution Inter-
case of Union Territories, there are two members from
each Union Territories. State Council is being constituted?
(a) Article 254 (b) Article 260
14. Who among the following is empowered to
establish the inter-state council? (c) Article 263 (d) Article 267
(a) Parliament U.P. Lower (Pre.) G.S. 2009
(b) President Ans. (c) Kindly refer the explanation of the above
(c) Speaker of Loksabha question.
(d) Prime Minister 18. Consider the following statements about the
UPPCS (Pre) G.S, 2017 States Reorganization Commission and Act?
Ans. (b) By the recommendation of Sarkaria 1. The commission was appointed in December,
Commission the permanent Inter-State-Council was set 1953.

Indian Polity & Constitution 72 YCT


2. The commission was headed by Fazal Ali and Ans. (b) : Kindly refer the explanation of the above
two members of the commission were H.V. question.
Kamath and Govind Ballabh Pant. 20. Which of the following is/are extra-
3. The commission submitted its report on constitutional and extra legal device(s) for
September 30, 1954. securing cooperation and coordination between
4. The States Re-organisation Act was enacted the States in India?
on August 31, 1956. I. National Development Council
5. The States Re-organisation Act came into II. The Governors Conference
effect on January 1, 1957. III. Zonal Councils
Which of the statements give above are IV. The Inter-State Council
correct? Choose the correct answer from the codes
(a) 1, 2 and 3 given below:
(b) 2, 3 and 4 Codes:
(c) 2, 4 and 5 (a) I, II and III
(d) 1 and 4 only (b) I, III and IV

Maharashtra PSC (Pre) G.S, 2019 (c) III and IV


(d) IV Only
Ans. (d) : In December 1953, Nehru Government
appointed a three man State Reorganization I.A.S. (Pre) G.S. 1995

Commission to investigate the problems, historic Ans. (b) National Development Council is neither a
background and existing situation of the Indian States. constitutional nor a statutory body but it is an apex body
Hence statement 1 is correct. The States Reorganisation for decision making on development matters in India. It
Commission was headed by Mr. Fazl Ali and its two was set up in 1951. Zonal Councils and Inter-State

other members were Pandit Hridayanath Kunzuru and Councils are also tools for securing cooperation and

Sardar K.M. Panikar. The Commission submitted its coordination between the States in India.

report to the government of India on September 30, 21. In order to impose any international treaty at
1955. Hence statement 2 and 3 are not correct. The any part of India or whole India, Parliament
States Re-organisation Act, 1956, which was an Act to could make any law?

provide reorganization of the States of India and for the (a) With the consent of all States
matters connected therewith, was enacted on August 31, (b) With the consent of majority States
1956. Hence statement 4 is correct while statement 5 is (c) With the consent of related States
incorrect. (d) Without the consent of any State

19. Which of the following group constituted State UPPCS (Pre) G.S., 2006

Reorganization Commission? Ans. (d) Article 253 of the Constitution asserts that the
(a) Pannikar, Fazl Ali, Katju Parliament has power to make law for the whole or any
part of the territory of India for implementing any
(b) Fazl Ali, Pannikar, Kunzru
treaty, agreement or convention with any other country
(c) Dhar, Fazl Ali, Pannikar
or countries or any decision made at any international
(d) Kunzru, Katju, Dhar
conference, association or other body.
TNPSC (Pre) G.S. 2019

Indian Polity & Constitution 73 YCT


13.
Emergency Provisions
1. If the President of India exercises his power as Select the correct answer using the code given
provided under Article 356 of the Constitution below:
in respect of a particular state, then (a) 1 and 2 only (b) 1 and 3 only
(a) The Assembly of the state is automatically (c) 2 and 3 only (d) 1, 2 and 3
dissolved IAS (Pre) G.S, 2017
(b) The powers of the Legislature of that State Ans. (b) Article 356 of the Constitution is regarding the
provisions, in case of failure of constitutional machinery
shall be exercisable by or under the authority
of the Parliament in State. Once Article 356 (President’s rule) is imposed,
the Government of the State (Council of Ministers) will
(c) Article 19 is suspended in that state
not be functional and the State will come under the
(d) The President can make laws relating to that
direct control of the Central Government through the
State
Governor. When the President's rule is imposed, the
IAS (Pre) G.S, 2018
assembly either is dissolved or is suspended for the
Ans. (b) Article 356 carries the marginal heading time.Dissolution of the local bodies is not related to
‘Provisions in case of failure of constitutional machinery
President Rule under Article 356.
in States’. When Article 356 is imposed, President 3. Consider the following statements in respect of
dismisses the Council of Ministers headed by Chief financial emergency under Article 360 of the
Minister and administers state through Governor. The Constitution of India:
Assembly of the State is not dissolved. President may (1) A Proclamation of financial emergency
suspend or terminate the Assembly only after the issued shall cease to operate at the expiration
proclamation is being passes by the Paliamant. Hence of two months, unless before the expiration of
statement 1 is not correct. If the President rule is that period it has been approved by the
imposed under Article 356, President may by resolution of both houses of Parliament.
proclamation assume to himself all or any of the (2) If the proclamation of financial emergency is
functions of the Government of the State and is in operation, it is competent for the president
exercisable by the Governor of the State, President of India to issue directions for the reduction
declare that the powers of the Legislature of the State of salaries and allowances of all or any class
shall be exercisable by or under the authority of of persons serving in connection with the
affairs of the Union but excluding the Judges
Parliament. Hence statement 2 is correct. According to
of the Supreme Court and the High Courts.
the Constituion 44th Amendment, freedoms listed in
Which of the statements given above is/are
Article 19 can be suspended only in case of proclamation
correct?
on the ground of war or external aggression. Hence
(a) 1 only (b) 2only
statement 3 is not correct. When President rule is (c) Both 1 and 2 (d) Neither 1 nor 2
imposed under Article 356, Parliament (Lok Sabha, IAS (Pre) G.S., 2007
Rajya Sabha and President) can make laws relating to Ans. (a) Article 360 of the Constitution of India is
that State. Hence statement 4 is not correct. regarding provisions as to financial emergency.
2. Which of the following are not necessarily the Financial emergency shall cease to operate at the
consequences of the proclamation of the expiration of two months unless before the expiration of
President’s rule in a State? that period, it has been approved by resolutions of both
(1) Dissolution of the State Legislative Assembly Houses of Parliament. Article 360 (4) (b) asserts that if
(2) Removal of the Council of Ministers in the financial emergency is in operation, President may issue
State directions for the reduction of salaries and allowances
(3) Dissolution of the local bodies of all or any class of persons serving in connection with
Indian Polity & Constitution 74 YCT
the affairs of the Union including the Judges of the
Ans. (c) Article 352 of the Constitution is regarding
Supreme Court and the High Courts. Financial proclamation of emergency national. President can
emergency may be revoked or varied by a subsequent proclaim national emergency at any part of the country
proclamation. or whole India on the ground of war or external
4. Under which Article of the Constitution, aggression or armed rebellion. The president can
Financial Emergency can be imposed? proclaim a national emergency only after receiving a
(a) 350 (b) 352 written recommendation from the cabinet. Article
(c) 356 (d) 360 352(4) asserts that the Proclamation of national
UPPSC (Pre) ACF-RFO Mains emergency must be aproved by both houses of
Parliament within one month from the date of its issue.
(IInd Paper), 2019
Ans. (d) Kindly refer the explanation of the above Article 352(6) asserts that a resolution may be passed
question. by either House of Parliament only by a majority of the
5. Article 360 of the Indian Constitution deals total membership of that House and by a majority of not
with- less than two thirds of the members of that House
(a) War emergency present and voting. Article 352(8) asserts that such
(b) Constitutional Emergency in the States. resolution could be revoked by a simple majority in the
(c) Financial Emergency Parliament.
(d) Administrative Emergency 8. Which of the following is not the ground for
TNPSC (Pre) G.S. 2015 the proclamation of National Emergency?
(a) War
Ans. (c) Kindly refer the explanation of the above
(b) External aggression
question.
(c) Threat to internal peace
6. How many times Financial Emergency has
(d) Armed rebellion
been imposed in India?
(a) 5 times (b) 4 times UPPCS (Main) G.S. IInd Paper, 2016
(c) Once (d) Never Ans. (c) Kindly refer the explanation of the above
UPPCS (Pre.) G.S., 2006 question.
Ans. (d) Financial Emergency in India has never been 9. Which one of the following is the time limit for
imposed till date. the ratification of an emergency period by the
7. What is correct about external emergency as Parliament?
per Article 352? (a) 14 days (b) 1 month
(i) It is proclaimed by the President. (c) 3 month (d) 6 months
(ii) It should be approved by the Parliament UPPCS (Pre.) G.S. 2006
within two months. Ans. (b) Kindly refer the explanation of the above
(iii) It should be approved by the Parliament question.
within one month. 10. A proclamation of emergency must be placed
(iv) It should be approved by the Parliament by before the Parliament for its approval-
simple majority. (a) Within one month (b) Within two month
(v) It should be approved by the Parliament by (c) Within six month (d) Within one year
2/3 majority. UPPCS (Main) G.S. 2003
(vi) It can be revoked by simple majority in
Ans. (a) Kindly refer the explanation of the above
Parliament.
question.
(vii) It can be revoked by 2/3 majority in
Parliament. 11. President’s Rule can be imposed for a
(viii) It can be approved in joint session of maximum period of-
Parliament. (a) 1 year (b) 2 years
(a) (i) (ii) (iv) (viii) (b) (i) (iii) (vi) (viii) (c) 6 months (d) 3 years
(c) (i) (iii) (v) (vi) (d) (i) (iv) (vi) (viii) UPPCS (Pre.) G.S., 1992
(e) None of the above Ans. (d) Once imposed, President Rule can continue for
Chhatisgarh PSC (Pre) G.S. 2017 6 months and can be extended to a maximum of 3 years,
Indian Polity & Constitution 75 YCT
subject to Parliament’s approval for every six months. It (c) Punjab
can be revoked any time by a simple majority in the (d) Bihar
Parliament. UPPCS (Main) G.S. IInd Paper 2010
12. The term of Legislative Assembly could be Ans. (c) Under Article 356, President's rule was
extended during emergency- imposed for the first time in June 1951 in Punjab.
(a) By the President Chhattisgarh and Telangana are the only States in India
(b) By the Governor of the State where President's rule hasn't been imposed so far.
(c) By the State Legislative Assembly 16. How many emergency provisions are there
(d) By the Parliament under the Constitution of India?
UPPCS (Pre.) G.S. Spl., 2004 (a) 2 (b) 4
Ans. (d) While a Proclamation of National Emergency (c) 3 (d) 1
is in operation, the parliament may extend the normal UPPCS (Main) G.S. IInd Paper 2014
tenure of a legislative assembly (5 years) by one year
Ans. (c) Part XVIII of the Constitution has made
each time. It is subjet to a maximum period of 6 months
provision for three types of emergencies namely
after the emergency has ceased to operate.
National Emergency (Article 352), emergency due to
13. Which one of the following cannot be
failure of constitutional machinery in States (Article
suspended or restricted even during national
356) and Financial Emergency (Article 360).
emergency?
17. Which one of the following is the correct
(a) Right to reside and settle in any part of the
statement with regard to the Emergency
country.
Provisions of Indian Constitution?
(b) Right to life and personal liberty.
(a) The constitution mentions four types of
(c) Right to move freely throughout the territory
emergency.
of India.
(b) Emergency provisions are described from
(d) Right to carry on any profession or business.
Articles 340 to 356 in the Constitution.
UPPCS (Main) G.S. IInd Paper 2008
(c) Financial emergency has never been imposed
Ans. (b) During National Emergency, all Funda-mental
in India.
Rights are deemed to be suspended except Article
(d) Emergency under Article 356 can be imposed
20(Protection in respect of conviction for offences) and
on the failure of Constitutional Machinery at
Article 21 (Protection of life and personal liberty).
the union level.
14. Under which Article of the Indian
Himanchal PSC (Pre) G.S. 2016
Constitution, it is the duty of the Union
Government to protect States against external Ans. (c) Part XVIII of the Constitution has made
aggression and internal disturbance? provision for three types of emergencies namely
(a) 355 (b) 356 national emergency (Article 352), emergency due to
(c) 352 (d) 360 failure of constitutional machinery in States (Article
RAS/RTS (Pre.) G.S., 2008 356) and financial emergency (Article 360). Financial
Ans. (a) Article 355 of the Constitution asserts that it emergency has never been imposed in India till date.
shall be the duty of the Union to protect every State 18. According to Dr. B.R. Ambedkar, which of the
against external aggression and internal disturbance and following Articles of the Indian Constitution
to ensure that the Government of every State is carried was to remain a dead-letter?
on in accordance with the provisions of the (a) Article 360 (b) Article 356
Constitution. (c) Article 352 (d) Article 32
15. Under Article 356 of the Constitution of India, Himanchal (PSC) Pre, G.S, 2014
President's rule was imposed for the first time Ans. (b) Dr. B.R. Ambedkar, while seeking adoption of
in Articles 356 had ironically stressed that members ought
(a) Uttar Pradesh to expect that “such article will never be called into
(b) Travancore- Cochin operation and that they would remain a dead letter”.
Indian Polity & Constitution 76 YCT
14.
Parliament-Structure
and its Functions
1. Along with the budget, the Finance Minister Which of the above given statements is/are
also places other documents before the correct?
parliament which include “The Macro (a) 1 only
Economic Framework Statement” The (b) 2 only
aforesaid document is presented because this (c) 1 and 3 only
mandated by (d) 2 and 3 only
(a) Long standing parliamentary convention
(b) Article 112 and article 110(1) of the IAS (Pre) G.S. 2020
constitution of India Ans. (c) According to Article 85(1) of the Constitution,
(c) Article 114 of the Constitution of India the President shall form time to time summon each
(d) Provisions of the Fiscal Responsibility and House of Parliament to meet at such time and place as
Budget Management act, 2003 he thinks fit, but six months shall not intervene between
IAS (Pre) G.S 2020 its last sitting in one session and the date appointed for
Ans. (d) Fiscal Responsibility and Budget Management its first sitting in the next session. It means that the
(FRBM) became an Act in 2003. It is an Act to provide Parliament should meet atleast twice a year. Hence
for the responsibility of the Central Government to statement 1 is correct but statement 2 is not correct. It is
ensure intergenerational equity in fiscal management no where mentioned in the Constituion about minimum
and long-term macro-economic stability by removing number of days that the parliament is required to meet
fiscal impediments in the effective conduct of monetary in a year. Hence statement 3 is correct.
policy and prudential debt management consistent with
3. Which one of the following statement is not
fiscal sustainability through limits on the Central
Government borrowings, debt and deficits, greater regard to powers of the Parliament?
transparency in fiscal operations of the Central (a) Parliament can approve three types of
Government and conducting fiscal policy in a medium emergency provisions
term framework. The Fiscal Responsibility and Budget (b) It cannot abolish State Legislative Council
Management Act made it mandatory for the (c) It can alter boundaries of the States
Government to place the following along with the (d) It can establish a common High Court for
Union Budget documents in Parliament annually: two or more States
1. Medium Term Fiscal Policy Statement
TNPSC (Pre) G.S. 2019
2. Macroeconomic Framework Statement 3. Fiscal
Policy Strategy Statement. The Fiscal Responsibility Ans. (b) Parliament could impose three types of
and Budget Management Act proposed that revenue emergencies under the Constitution of India namely
deficit, fiscal deficit, tax revenue and the total National Emergency (Article 352), Failure of
outstanding liabilities be projected as a percentage of constitutional machinery in States (Article 356) and
gross domestic product (GDP) in the medium-term Financial Emergency (Article 360). According to
fiscal policy statement. Article 169 (1) of the Constitution, Parliamant can
2. Consider the following statements: abolish a Legislative Council or create it, if the
1. The president of India can summon a Legislative Assembly of the concerned state passes a
session of parliament at such a place as resolution to that effect. Hence statement (b) is not
he/she thinks fit. correct. Article 3 of the Constituion empowers
2. The constitution of India provides for Parliamant to the formation of new states and alteration
three sessions of the parliament in a year of areas, boundaries or names of existing states.
but it is not mandatory to conduct all the According to Article 231(1) of the Constituion,
sessions
Parliament may by law establish a common High Court
3. There is no minimum number of days that
for two or more States or for two or more States and a
the parliament is required to meet in a
year Union territory.
Indian Polity & Constitution 77 YCT
4. The Parliament of India exercises control over no right to vote as the President of India is elected by
the functions of the Council of Ministers the members of an electoral college consisting of the
through elected members of both the Houses of Parliament and
1. Adjournment motion the elected members of the Legislative Assemblies of
2. Question hour States and the Union Territories of Delhi and
3. Supplementary questions Pondicherry. In the election of Vice President,
Select the correct answer using the code given nominated members have right to vote. Hence statement
below: 2 is correct.
(a) 1 only (b) 2 and 3 only 6. Which of the following are/is stated in the
(c) 1 and 3 only (d) 1, 2 and 3 Constitution of India?
IAS (Pre.) G.S, 2017 I. The President shall not be a member of either
house of Parliament.
Ans. (d) Several provisions have been made to exercise
II. The Parliament shall consist of the President
control over the functions of the Council of Ministers
and two Houses.
by the Parliament. One of them is ‘adjournment motion’
where a motion for an adjournment of the business of Choose the correct answer from the codes
the House for the purpose of discussing a definite given below:
matter of urgent public importance may be made with Codes:
the consent of the Speaker. Another is ‘question hour (a) Neither I nor II (b) Both I and II
’which is the first hour of a sitting of Lok Sabha, (c) I alone (d) II alone
devoted to the questions. Question Hour has assumed IAS (Pre.) G.S., 1997
greater importance as the members can elicit Ans. (b) Article 79 of the Constitution asserts that there
information through questions on matters affecting the shall be a Parliament for the Union which shall consist
day-to-day life of the citizens for which Ministers are of the President and two Houses (Lok Sabha and Rajya
collectively answerable to the Legislature. Further, Sabha). Article 59 is regarding the condition of the
when a question is answered orally, supplementary President’s office which asserts that the President shall
questions can be asked thereon. not be a member of either House of Parliament or of a
5. Consider the following statements: House of the Legislature of any State. Hence both the
1. The Chairman and the Deputy Chairman of statements are correct.
the Rajya Sabha are not the members of that 7. Which of the following is included in Indian
House. Parliament?
2. While the nominated members of the two (a) Lok Sabha and Rajya Sabha
Houses of the Parliament have no voting right (b) Lok Sabha, Rajya Sabha and Prime Minister
in the Presidential election, they have the (c) Chairman of the Lok Sabha and Lok Sabha
right to vote in the election of the Vice (d) President and both the Houses
President.
UPPCS (Pre.) G.S., 2012
Which of the statements given above is/are
Ans. (d) Kindly refer the explanation of the above
correct?
question.
(a) 1 only (b) 2 only
(c) Both 1 and 2 (d) Neither 1 nor 2 8. The Parliament of India acquires the power to
legislate on any item in the State List in the
IAS (Pre) G.S., 2013
national interest if a resolution to that effect is
Ans. (b) According to Article 89 (1) of the Consti- passed by the
tution, the Vice-President of India is ex (a) Lok Sabha by a simple majority of its total
officio Chairman of Rajya Sabha. According to Article membership
89(2) of the Constitution. The Deputy Chairman is
(b) Lok Sabha by a majority of not less than two-
elected by Rajya Sabha itself from amongts its
thirds of its total membership
members. Hence statement 1 is incorrect. Article 80 of
(c) Rajya Sabha by a simple majority of its total
the Constitution provides provision to nominate 12
membership
members by the President. The members nominated by
the President are persons having special knowledge or (d) Rajya Sabha by a majority of not less than
practical experience in respect of such matters as two-thirds of its members present and voting
literature, science, art and social service. Nominated IAS (Pre.) G.S., 2016
members enjoy all powers, privileges and immunities Ans. (d) Article 249 of the Constitution is regarding the
available to an elected member of Parliament but for the power of Parliament to legislate with respect to a matter
election of President of India, nominated members have
Indian Polity & Constitution 78 YCT
in the State List in the national interest. It asserts that if 12. Which one of the following statements about
the Rajya Sabha declared that it is necessary in the the Parliament of India is NOT correct?
National interest that Parliament should make laws on a (a) The Constitution provides for a Parliamentary
matter in the state list, then Parliament becomes form of Government.
competent to make law on that matter. Such a resolution (b) The foremost function of the Parliament is to
must be supported by a majority of not less than two- provide a Cabinet.
third of its members present and voting. The resolution (c) The membership of the Cabinet is restricted
remains in force for one year. to the Lower House.
9. Parliament can legislate on a subject in the (d) The Cabinet has to enjoy the confidence of
the majority in the popular Chamber.
State list
UPPCS (Pre.) G.S., 2011
(a) By the wish of the President
Ans. (c) Among the given statements, the member-ship
(b) If the Rajya Sabha passes such a resolution
of the Cabinet is restricted to the Lower House (Lok
(c) Under any circumstances Sabha) is not correct as it is the privilege of the Prime
(d) By asking the legislature of the concerned Minister to appoint the member of his cabinet
State irrespective of Houses. Rest all the statements are correct.
UPPCS (Pre.) G.S., 2012 13. Quorum of Lok Sabha and Rajya Sabha is-
Ans. (b) Kindly refer the explanation of the above (a) 1/5th of the total number of member
question. (b) 1/6th of the total number of member
10. Union Parliament can make law for the (c) 1/10th of the total number of member
matters related to State List- (d) 1/8th of the total number of member
1. To implement the international treaties UPPCS (Main) G.S. 1st Paper 2016
2. With the consent of the concern State Ans. (c) Under Article 100 of the Constitution, the
3. If President Rule is imposed in State quorum to constitute a meeting of either House of
4. If Rajya Sabha passes resolution with two Parliament is one-tenth (1/10) of the total number of
third majorities in national interest members of the House. If at any time during a meeting
of a House, there is no quorum, it shall be the duty of
Choose the correct answer by using below
the Chairman or person acting as such, either to
given codes: adjourn the House or to suspend the meeting until
Code: there is a quorum.
(a) 1, 2 and 3 (b) 2, 3 and 4 14. Given below are two statements:
(c) 1, 2 and 4 (d) All four Assertion (A) The President is the part of the
UPPCS (Pre.) G.S., 1995 Parliament.
Ans. (d) Union Parliament can make law for the matters Reason (R) A Bill passed by the two Houses of
related to State List; to implement the international Parliament cannot become law without the assent
treaties (Article 253), with the consent of the concern of the President.
State (Article 252), if President Rule is imposed in State In the context of the above, which one of the
(Article 356), If Rajya Sabha passes resolution with two following is correct?
third majorities in national interest (Article 249). (a) Both (A) and (R) are true and (R) is the
11. A money bill passed by Lok Sabha is deemed to correct explanation of (A).
have been passed by Rajya Sabha also when no (b) Both (A) and (R) are true, but (R) is not the
correct explanation of (A).
action is taken by the Rajya Sabha within
(c) (A) is true, but (R) is false.
(a) 10days (b) 14days
(d) (A) is false, but (R) is true.
(c) 15days (d) 16 days
UPPCS (Main) G.S. IInd Paper. 2007
UPPCS (Pre.) G.S., 2016
Ans. (a) Article 79 of the Constitution asserts that there
Ans. (b) Article 109 of the Constitution is regarding shall be a Parliament for the Union which shall consist of
special procedure in respect of Money Bills. Article the President and two Houses to be known respectively
109(5) asserts that if a Money Bill passed by the Lok as the Council of States (Rajya Sabha) and the House of
Sabha and transmitted to the Rajya Sabha for its the People (Lok Sabha). Further a Bill passed by the two
recommendations, is not returned to the Lok Sabha Houses of Parliament becomes law only after the assent
within fourteen days (14 days), it shall be deemed to of the President. Hence Both (A) and (R) are true and (R)
have been passed by the Rajya Sabha. is the correct explanation of (A).
Indian Polity & Constitution 79 YCT
15. Who has right to issue money from the 19. Parliament can amend the provision on official
Consolidated Fund of India? language of India under the Constitution by:
(a) Comptroller and Auditor General (a) A simple majority of its members
(b) Finance Minister of India (b) 2/3rd majority
(c) Authorized Minister (c) 3/4th majority
(d) None of the above
(d) Support of 1/3 of its members
UPPCS (Pre.) G.S., 2000
UPPCS (Pre) G.S, 2018
Ans. (d) Consolidated Fund of India was constituted
under Article 266 (1) of the Constitution. All revenues Ans: (a) Parliament could amened the provision on
received by the Government by way of direct taxes and official language of India under the Constitution by a
indirect taxes, money borrowed and receipts from loans simple majority of its members.
given by the Government flow into the Consolidated 20. A deadlock between the Lok Sabha and the
Fund of India. All government expenditure is made Rajya Sabha calls for a joint sitting of the
from this fund, except exceptional items, which are met Parliament during the passage of
from the Contingency Fund or the Public Account. No 1. Ordinary Legislation
money can be withdrawn from this fund without
2. Money Bill
approval of the Parliament.
3. Constitution Amendment Bill
16. All revenues received by the union government
by way of taxes and other receipts for the Select the correct answer using the codes given
conduct of government business are credited to below:
the? (a) 1 only
(a) Contingency fund of India (b) 2 and 3 only
(b) Public account (c) 1 and 3 only
(c) Consolidated fund of India (d) 1, 2 and 3
(d) Deposits and advances fund IAS (Pre.) G.S., 2012
IAS (Pre) G.S., 2011 Himanchal PSC (Pre) G.S 2013
Ans. (c) Kindly refer the explanation of the above Ans. (a) Article 108 of the Constitution is regarding the
question. joint sitting of both Houses in certain cases. It asserts
17. The authorization for the withdrawal of funds that joint sitting of both Houses is possible if after a
from the consolidated fund of India must come Bill has been passed by one House and transmitted to
from? the other House and the Bill is rejected by the other
(a) The President of India House or the Houses have finally disagreed as to the
(b) The Parliament of India Amendments to be made in the Bill or more than six
(c) The Prime minister of India months elapse from the date of the reception of the Bill
(d) The Union Finance Minister by the other House without the Bill being passed by it,
IAS (Pre) G.S., 2011 the President may, unless the Bill has elapsed by reason
Ans. (b) Kindly refer the explanation of the above of a dissolution of the Lok Sabha notify his intention to
question. summon them to meet in a joint sitting for the purpose
18. The Parliament can make any law for whole or of deliberating and voting on the Bill. Article 108(1) (c)
any part of India for implementing assets that joint sitting is not applicable in case of
international treaties Money Bill. Article 368 of the Constituion asserts that
(a) With the consent of all the States the Constitution Amendment Bills have to be passed by
(b) With the consent of the majority of States both Houses, individually. Hence joint sitting is
(c) With the consent of the States concerned possible only for ordinary legislation.
(d) Without the consent of any State 21. Joint sitting of Lok Sabha and Rajya Sabha
IAS (Pre.) G.S., 2000, 2013 can take place to resolve deadlock over
Ans. (d) Article 253 of the Constitution asserts that the (a)Ordinary Legislation
Parliament has power to make any law for the whole or (b)Money Bill
any part of the territory of India for implementing any (c)Constitutional Amendment Bill
treaty, agreement or convention with any other country
(d)Appropriation Bill
or countries or any decision made at any international
conference, association or other body. Therefore UPPCS (Pre) G.S. 2020
consent of the State is not required for implementing Ans. (a) Kindly refer the explanation of the above
international treaties. question.
Indian Polity & Constitution 80 YCT
22. When a bill is referred to a joint sitting of both 25. The joint sitting of Lok Sabha and Rajya
the Houses of the Parliament, it has to be Sabha under Article 108 is summoned by
passed by (a) Chairman of Rajya Sabha
(a) A simple majority of members present and (b) Speaker of Lok Sabha
voting (c) Prime Minister
(b) Three-fourths majority of members present (d) President
and voting UPPCS (Main) G.S. IInd Paper, 2015
(c) Two-thirds majority of the Houses Himanchal PSC (Pre) G.S 2016
(d) Absolute majority of the Houses Ans. (d) Kindly refer the explanation of the above
UPPSC (Main) G.S. IInd Paper 2005 question.
Ans. (a) When a bill is referred to a joint sitting of both 26. Who presides the joint sitting of the Parliament
the Houses of the Parliament, it has to be passed by a if neither the Speaker nor the Deputy Speaker
simple majority of members present and voting. of the Lok Sabha is available?
23. Consider the following statements: (a) Chairman of the Rajya Sabha
1. The joint sitting of the two houses of the (b) Deputy Chairman of the Rajya Sabha
Parliament in India is sanctioned under (c) Senior most member of Lok Sabha
Article 108 of the Constitution (d) A member nominated by the President
2. The first joint sitting of Lok Sabha and Rajya Maharastra PSC (Pre) G.S 2016
Sabha was held in the year 1961 Ans. (b) If neither the Speaker nor the Deputy Speaker
3. The second joint sitting of the two Houses of of the Lok Sabha is available during the joint sitting of
Indian Parliament was held to pass the the Parliament, in such circumstances, Deputy
Banking Service Commission (Repeal) Bill. Chairman of the Rajya Sabha would preside the joint
Which of these statements is/are correct? sitting of the Parliament. If he is also absent from joint
(a) 1 and 2 (b) 2 and 3 sitting then such other person as may be determined by
the members present at the joint sitting, presides over
(c) 1 and 3 (d) 1, 2 and 3
the meeting.
IAS (Pre) G.S., 2003
27. Which of the following are the methods of
Ans. (d) Article 108 of the Constitution is regarding Parliamentary control over public finance in
joint sitting of both the Houses in certain cases. The India?
first joint sitting was held on 6 and 9 May, 1961 1. Placing Annual Financial Statement before
following disagreement between the two Houses over the Parliament
certain amendments to the Dowry Prohibition Bill,
2. Withdrawal of moneys from Consolidated
1959. The second joint sitting was held on 16 May,
Fund of India only after passing the
1978 following rejection by the Rajya Sabha of the Appropriation Bill
Banking Service Commission (Repeal) Bill, 1977. The
3. Provisions of supplementary grants and vote-
third joint sitting was held on 26 March 2002 when the
on-account
motion to consider the Prevention of Terrorism Bill,
4. A periodic or at least a mid-year review of
2002, seeking to replace the Prevention of Terrorism
programme of the Government against
Ordinance (POTO) as passed by the Lok Sabha was
macroeconomic forecasts and expenditure by
rejected by the Rajya Sabha. a Parliamentary Budget Office
24. Who of the following presides over the Joint 5. Introducing Finance Bill in the Parliament
Session of the Lok Sabha and the Rajya Sabha Select the correct answer using the codes given
in India? below:
(a) President of India (a) 1, 2, 3 and 5 only (b) 1, 2 and 4 only
(b) Vice President of India (c) 3, 4 and 5 only (d) 1, 2, 3, 4 and 5
(c) Speaker of the Lok Sabha IAS (Pre.) G.S., 2012
(d) Prime Minister Ans. (a) Parliament acts on the behalf of people of India
UPPCS (Main) G.S. IInd Paper, 2006 to get information regarding public finance in India.
Ans. (c) Article 108 of the Constitution is regarding Article 112 of the Constitution is regarding Annual
joint sitting of both the Houses in certain cases. For the Financial Statement (Budget), Article 114 is about
joint sitting of both the Houses, the Secretary-General withdrawal of money from the Consolidated Fund of
shall issue summons to each member specifying the India only after passing of the Appropriation Bill,
time and place for a joint sitting. The joint sitting of Article 115 is provision of supplementary, additional or
Parliament is summoned by the President and presided excess grants and Article 117 is regarding Finance Bill
over by the Speaker of the Lok Sabha. to be introduced in the Parliament with the

Indian Polity & Constitution 81 YCT


recommendation of the President of India. A periodic or Ans. (b) Article 248 of the Constitution is regarding
at least a mid-year review of programme of the residuary powers of legislation. Article 248 (1) asserts
Government is not under the Parliamentary control. that the Parliament has exclusive power to make any
28. Consider the following statements: law with respect to any matter not enumerated in the
An amendment of the Constitution of India can Concurrent List or State List. Further Article 248 (2)
be initiated by the asserts that such power shall include the power of
I. Lok Sabha II. Rajya Sabha making any law imposing a tax not mentioned in either
of those Lists.
III. State Legislatures IV. President
Which of the above statements is/are correct? 31. Which of the following statements regarding
Zero Hour are correct?
(a) I alone (b) I, II and III
(1) Zero Hour is an Indian innovation in the field
(c) I, II and IV (d) I and II of Parliamentary Procedure.
IAS (Pre.) G.S., 1999 (2) The Time gap between the question hour and
Ans. (d) Article 368 of the Constitution is regarding the agenda of house is known as zero Hour.
power of Parliament to amend the Constitution which (3) This device has been in existence since 1962.
asserts that an amendment of the Constitution may be (4) It is an informal device available to the
initiated only by the introduction of a Bill in either members of the Parliament.
House of Parliament and the Bill is to be passed in each (a) 1, 2 and 3 (b) 1, 3 and 4
House by a majority of the total membership of that (c) 2 and 3 (d) All of the above
House present and voting. Maharashtra PSC (Pre) G.S 2018
29. Which one of the following statements about a Ans. (d) The concept of ‘Zero Hour’ has been innovated
Money Bill is not correct? by India since 1962. Apart from the Question Hour,
(a) Money Bill can be tabled in either House of Parliament allocates additional time slot for the members
Parliament to discuss issues. Although it’s not recognized in our
(b) The Speaker of Lok Sabha is the final parliamentary procedure, Members wishing to raise
authority to decide whether a Bill is a Money matters during the Zero Hour need to give notice to the
Bill or not Speaker/Chairman prior to the start of the daily session.
(c) The Rajya Sabha must return a Money Bill The notice should clearly state the subjects they want to
passed by the Lok Sabha and send it for raise. The Speaker is the final authority who can either
consideration within 14 days reject or accept such request. In Lok Sabha, only 20
(d) The President cannot return Money Bill to matters are allowed to be raised during the Zero Hour.
Lok Sabha for reconsideration Again, it is the Speaker who decides which subjects are
to be discussed in the first phase (after Question Hour)
IAS (Pre.) G.S., 2000
and which other issues could be taken up during the
Ans. (a) Article 109(1) of the Constitution asserts that a second phase (after 6 pm/ at the end of the regular
Money Bill shall not be introduced in the Council of business). In Rajya Sabha, total number of requests is not
States (Rajya Sabha). Hence statement 1 is incorrect. allowed to exceed seven on a single day. The total time
Article 110 (3) asserts that if any question arises allocated for Zero Hour is 30 minutes wherein a member
whether a Bill is a Money Bill or not, the decision of the gets three minutes to raise the issue. The session should
Speaker of the Lok Sabha shall be final. Article 109(2) be completed before 1 pm. Moreover, a member is
asserts that after a Money Bill has been passed by the allowed to make only one Zero Hour request during a
Lok Sabha, it shall be transmitted to the Rajya Sabha week. It is not mandatory to have a Zero Hour every day
for its recommendations and the Rajya Sabha shall during the session. It is to be noted that the Government
within a period of fourteen days from the date of its is not obliged to answer the questions raised during the
receipt of the Bill, return the Bill to the Lok Sabha with Zero Hour.
its recommendations. Under the Article 110 (19) of the 32. ‘Zero Hour’ Parliamentary system has been
Constitution, the President may either give or withhold provided by which country?
assent to a Money Bill. Under the Constituion, a Money (a) India (b) America
Bill cannot be returned to the House by the President for (c) Britain (d) Switzerland
reconsideration. UPPCS (Main) SPl G.S. IInd Paper, 2004
30. Who can legislate on the subjects which are not Ans. (a) Kindly refer the explanation of the above
included in any of the lists of subjects given in question.
the Constitution? 33. Time of zero-hour in Parliament is:
(a) Supreme Court (b) Parliament (a) 9 am to 10 am (b) 10 am to 11 am
(c) State Legislature (d) Regional Councils (c) 11 am to 12 noon (d) 12 noon to 1.00 pm
UPPCS (Pre.) Re-exam. G.S., 2015 UPPCS (Main) G.S. IInd Paper G.S. 2015
Indian Polity & Constitution 82 YCT
Ans. (d) Kindly refer the explanation of the above Which of the statements given above is/are
question. correct?
34. In the rules of Business of Indian Parliament, if (a) 1 and 2 only (b) 3 only
the Speaker admits notice of a motion but no (c) 2 and 3 only (d) 1, 2 and 3
date is fixed for its introduction-then it is
called. Ans:(a) The law does not clearly define what
(a) Call attention motion constitutes an ‘Office of Profit’ but the definition has
(b) Adjournment motion evolved over the years with interpretations made in
(c) No Confidence motion various court judgments. Hence statement 3 is not
(d) No day-yet named motion correct. An office of profit has been interpreted to be a
TNPSC (Pre) G.S. 2017 position that brings to the office-holder some financial
Ans. (d) : If the Speaker admits notice of a motion and gain or advantage or benefit. Under the provisions of
no date is fixed for its discussion, it is called a "No- Article 102 (1) (a) [for M.P] and Article 191 (1) (a) [for
Day-yet-Named Motion" and a copy of the admitted M.L.A] of the Constitution, an MP or an MLA (or an
motion is forwarded to the Minister concerned with the MLC) is barred from holding any office of profit under
subject matter of the motion. Admitted notices of such
the Central or State Government. President for MP’s
motions may be placed before the Business Advisory
Committee for selecting the motions for discussion in and Governor of the States for MLA’s take final
the House according to the urgency and importance of decision on office of profit. Parliament has also
the subject-matter thereof and allotting time for the enacted the Parliament (Prevention of Disqualification)
same. Act, 1959, which has been amended five times to
35. What is meaning of Guillotine in exempt several posts from disqualification on the
Parliamentary procedure? grounds of 'Office of Profit'. Hence statement 1and 2
(a) To stop the debate on the bill are correct.
(b) To continue debate on the bill 37. Who decides Office of Profit?
(c) Boycott of house by the members
(a) President and Governor
(d) To adjourn the house
(e) To adjourn the house for the day (b) Union Parliament
UPPCS (Pre.) G.S., 2012 (c) Supreme Court
Ans. (a) In legislative parlance, ‘Guillotine’ means to (d) Union Public Service Commission
bunch together and fast-track the passage of financial UPPCS (Pre.) G.S, 2009
business. It is a fairly common procedural exercise in Ans. (a) Kindly refer the explanation of the above
Lok Sabha during the Budget Session. After the Budget
question.
is presented, Parliament goes into recess for about three
weeks during which time the House Standing 38. ‘Office of Profit’ is defined-
Committees examine Demands for Grants for various (a) By the Constitution
Ministries and prepare reports. After Parliament (b) By the Supreme Court
reassembles, members utilize the opportunity to discuss
(c) By the Union Cabinet
the policies and working of Ministries. Once the House
is done with these debates, the Speaker applies the (d) By the Parliament
‘Guillotine’ and all outstanding demands for grants are UPPCS (Pre.) G.S., 2000
put to vote at once. This usually happens on the last day Ans. (b) Kindly refer the explanation of the above
earmarked for the discussion on the Budget. The question.
intention is to ensure timely passage of the Finance Bill,
marking the completion of the legislative exercise with 39. Minimum how many times is it necessary for
regard to the Budget. the meeting of Parliament to be held every
36. Consider the following statements: year?
1. The Parliament (Prevention of Disquali- (a) Once (b) Twice
fication) Act, 1959 exempts several posts (c) Thrice (d) Four times
from disqualification on the grounds of UPPCS (Pre.) G.S., 1995
'Office of Profit'.
2. The above-mentioned Act was amended five Ans. (b) As per the Article 85 of the Constitution, there
times. should not be gap of more than six months between the
3. The term ‘Office of Profit' is well defined in two sessions of the Parliament. Hence meeting of the
the Constitution of India. Parliament is necessary twice to be held every year.

Indian Polity & Constitution 83 YCT


40. Which Article empowers the President to Ans. (c) According to Article 4(2) of the Constitu-tion,
Summon and prorogue either Houses of provisions provided under Article 2 (Admission or
Parliament? establishment of new States) shall not deemed to be an
(a) Article 75 (b) Article 81 amendment of the Constitution under Article 368.
(c) Article 85 (d) Article 88 45. ‘Votes on Account’ permits Union Government
TNPSC (Pre) G.S. 2016 to:
Ans. (c) : Kindly refer the explanation of the above (a) Go for public loan
question. (b) Borrow money from the Reserve Bank of
41. Who amongst the following has the power to India
withhold assent to a Bill, after it has been (c) Give grant-in-aid to States
passed by the two Houses of Parliament? (d) Withdraw money from the Consolidated Fund
(a) The President of India for specific period
(b) The Prime Minister U.P. Lower (Pre.) G.S. SPL. 2008
(c) The Speaker of the Lok Sabha Ans. (d) Article 116 of the Constitution is regarding
(d) The Chairman of the Rajya Sabha votes on account, votes of credit and exceptional grants.
UPPCS (Main) G.S IInd Paper 2011 The vote on account is the special provision given to the
Ans. (a) According to Article 111 of the Constitution, Government to obtain the vote of Parliament to
when a Bill has been passed by the both Houses of withdraw money from Consolidated Fund of India when
Parliament, it shall be presented to the President and the the budget for the new financial year is not released or
President shall declare either that he assents to the Bill the elections is underway. A vote on account stays valid
or that he withholds assent. Such provision is not for two months but however it can be extended, if the
applicable to Money Bill. year is an election year.
42. Which one of the following is empowered to 46. Vote on Account is meant for
alter the boundaries of States under the (a) Vote on the report of CAG
Constitution of India? (b) To meet unforeseen expenditure
(a) Parliament (b) Lok Sabha (c) Appropriating funds pending passing of
(c) President (d) Supreme Court budget
UPPCS (Main) G.S.IInd Paper 2015 (d) Budget
Ans. (a) According to the Article 3 of the Constitution, (e) None of the above/More than one of the
Parliament by law form a new State, increase the area of above
any State, diminish the area of any State, alter the BPSC (Pre) G.S.,2016
boundaries of any State and alter the name of any State. Ans: (c) Kindly refer the explanation of the above
43. Which of the following has executive power to question.
form a new State?
45. Any statutory bill can be placed on the floor of
(a) Lok Sabha (b) Parliament which House?
(c) Political Parties (d) Union Government (a) Lok Sabha
UP RO/ARO (Main) G.S., 2014 (b) Rajya Sabha
UPPCS (Main) G.S. IInd Paper 2013 (c) Floor of the one of the House
Ans. (b) Kindly refer the explanation of the above (d) None of the above
question.
MPPCS (Pre) G.S,2010
44. Which one of the following is not correct in the
Ans. (c) Money bill could only be placed on the floor of
matter of formation of new States?
the Lok Sabha and rest of the Bills could be placed at
(a) Parliament may be law form a new State. floor of the both the Houses.
(b) Such law shall contain provisions for the
46. Who among the following can participate in
amendment of the First Schedule and the
the deliberations of Lok Sabha while being
Fourth Schedule of the Constitution.
member of the Rajya Sabha?
(c) Such law shall be deemed to be an
amendment of the Constitution for the (a) Nominated member of the Rajya Sabha who
purpose of article 368. is an expert in a field
(d) No Bill for enacting such law shall be (b) Deputy Chairperson of the Rajya Sabha
introduced in the Parliament unless it has (c) Minister who is a member of the Rajya Sabha
been referred to the legislature of the State, (d) Leader of the House in Rajya Sabha
whose area, boundaries or name is affected. (e) None of these
UPPCS (Main) G.S. IInd Paper 2011 Chattisgarh PSC (Pre) 1st G.S., 2013

Indian Polity & Constitution 84 YCT


Ans. (c) Article 88 of the Constitution is regarding 50. Consider the follwoing statements-
Rights of Ministers and Attorney General of India to When Lok Sabha is dissolved-
speak and take part in the proceedings of either House 1. A Bill pending in Rajya Sabha which has
of the Parliament but not entitled to vote. not been passed by the Lok Sabha lapses.
47. Being a non member of both the Houses, which 2. A Bill pending in Lok Sabha lapses.
one of the following can take part in the 3. A Bill passed by both houses but pending
proceedings of the House? assent of the President of India does not
(a) Chief Justice of India lapes.
(b) Attorney General of India Choose the correct answer from the following :
Codes-
(c) Chief Election Commissioner
(a) 1 only (b) 1 and 2
(d) Chairman of National Human Rights
(c) 2 and 3 (d) 1,2 and 3
Commission
UPPSC ACF-RFO Mains (IInd Paper), 2019
Chattisgarh PSC (Pre) 1st G.S., 2003
Ans. (c) Kindly refer the explanation of the above
Ans. (b) Kindly refer the explanation of the above
question.
question.
51. Consider the following statements:
48. Which of the following is not authorized to take
(i) Bill pending in the Rajya Sabha which has
part in the proceedings of the Lok Sabha? not been passed by the Lok Sabha shall lapse
(a) Comptroller and Auditor General of India on the dissolution of the Lok Sabha.
(CAG) (ii) A Bill pending in Parliament shall lapse by
(b) Attorney General of India reason of Prorogation of the houses.
(c) Solicitor General of India Which of the statement/s given above is/are
(d) Secretary to the President of India correct?
BPSC (Pre) G.S, 2003 (a) Only (i) (b) Neither (i) nor (ii)
Ans. (d) Secretary to the President of India is not (c) Both (i) and (ii) (d) Only (ii)
entitled to take part in the proceedings of the Lok Sabha Maharashtra PSC (Pre) G.S 2014
while rest of the given options are entitled to take part Ans. (b) Kindly refer the explanation of the above
in the proceedings of the Lok Sabha. question.
49. Which of the following statements is/are 52. Consider the following statements regarding
correct? lapsing of bills in the Parliament:
1. A Bill pending in the Lok Sabha lapses on its 1. A bill pending in the parliament lapses by
prorogation. reason of the prorogation of the Houses.
2. A Bill pending in the Rajya Sabha, which has 2. A Bill pending in the Rajya Sabha but not
not been passed by the Lok Sabha, shall not passed by the Lok Sabha lapses when Lok
lapse on dissolution of the Lok Sabha. Sabha dissolves.
Select the correct answer using the code given Which of the above statements above is/ are
below. correct?
(a) Only 1 (b) Only 2
(a) 1 only (b) 2 only
(c) Both 1 and 2 (d) Neither 1 nor 2
(c) Both 1 and 2 (d) Neither 1 nor 2
Punjab PSC (Pre) G.S 2013
IAS (Pre) G.S., 2016
Ans. (d) Kindly refer the explanation of the above
Ans. (b) A Bill pending in the Lok Sabha does not lapse question.
on its prorogation rather it is tabled for discussion for
53. What is correct about starred question?
next session of the Lok Sabha. Hence statement 1 is
(i) Answer is given oral
incorrect .According to Article 107(5) of the
Constitution, a Bill which is pending in the Lok Sabha or (ii) Answer is given in written
which having been passed by the Lok Sabha and is (iii) Supplementary questions can be asked
pending in the Rajya Sabha, lapse on a dissolution of the (iv) Supplementary questions cannot be asked
Lok Sabha. According to Article 107 (4) of the (v) May be discussed on answer
Constitution, a Bill pending in the Rajya Sabha which has (vi) Speaker controls on answers
not been passed by the Lok Sabha shall not lapse on (a) (ii) (iv) (b) (i) (iii)
dissolution of the Lok Sabha. Hence statement 2 is (c) (v) (vi) (d) (iii) (v)
correct. Further, a bill passed by both Houses but pending (e) (ii) (vi)
assent of the President does not lapse. Chattisgarh PSC (Pre) 1st G.S., 2014
Indian Polity & Constitution 85 YCT
Ans. (b) A Starred Question is the one to which a Ans. (a) Article 109 of the Constitution is regarding
member desires an oral answer in the house, having a special procedure with respect to Money Bill. Article
distinguishing mark of an asterisk. When a question is 109(1) asserts that a Money Bill shall not be introduced
answered orally, Members of Parliament have the in the Rajya Sabha. According to Article 109 (2), after a
option to raise the supplementary questions based on the Money Bill has been passed by the Lok Sabha, it shall
replies to the starred questions. Per day, only 20 be transmitted to the Rajya Sabha for its
questions can be listed for oral answer. An Unstarred recommendations and the Rajya Sabha shall within a
Question is the one which does not called for an oral period of fourteen days from the date of its receipt of
the Bill return the Bill to the Lok Sabha with its
answer in the House and hence on which no
recommendations and the Lok Sabha may thereupon
supplementary questions can be asked. To an Un-starred
either accept or reject all or any of the recommendations
Question, a written answer is mandated to be laid on the
of the Rajya Sabha.
table after the Question Hour by the concerned
56. Sovereignty of Indian Parliament is restricted
Minister. A Short Notice Question is the one which
by
relates to a matter of urgent public importance and can
(a) Powers of the President of India
be asked with a shorter notice. It is answered orally
(b) Judicial review
followed by supplementary questions.
(c) Leader of the opposition
54. Who among the following have the right to vote
(d) Powers of the Prime Minister of India
in the elections of both Lok Sabha and Rajya
UPPCS (Main) G.S. Ist Paper, 2004
Sabha?
Ans. (b) Sovereignty of Indian Parliament is restricted
(a) Elected members of the Lower House of the
by judicial review. Judicial review is the power of the
Parliament
courts to consider the acts of organs of Government
(b) Elected members of the upper House of the (both executive and legislature) constitutionally and
Parliament declare it unconstitutional or null and void, if it violates
(c) Elected members of the Upper House of the the basic principles of Constitution.
State Legislature 57. The Secretariat of the Indian Parliament-
(d) Elected members of the Lower House of the (a) Is under Minister of Parliamentary Affairs
State Legislature (b) Is under President
IAS (Pre.) G.S. 1995 (c) Is independent of Government
Ans. (d) Elected Members of the Lower House of the (d) Is under Supreme Court
State Legislature (Vidhan Sabha) have the voting right MPPSC (Pre.) G.S, 2010
in the elections of both Lok Sabha and Rajya Sabha. Ans. (c) Article 98 of the Constitution is regarding
The members of the Rajya Sabha are elected by the Secretariat of Parliament which in turn is independent
Legislative Assembly of States and Union territories of Government.
by means of single transferable vote through 58. Which one of the following is not a collective
proportional representation. The members of the privilege of the members of Parliament?
Lower House of the State Legislature (Vidhan Sabha) (a) Freedom of debates and proceedings
also have voting right in the elections of the Lok (b) The Right to regulate the internal matters of
Sabha as an ordinary citizen of India. the Parliament
55. What will follow if a Money Bill is substantially (c) Freedom from attendance as Witness
amended by the Rajya Sabha? (d) The privilege of excluding strangers from the
(a) The Lok Sabha may still proceed with the house
Bill, accepting or not accepting the UPPCS (Pre) G.S, 2017
recommendations of the Rajya Sabha Ans. (c) Article 105 of the Constitution defines the
(b) The Lok Sabha cannot consider the Bill power and privileges of Member of Parliament and
further Article 194 of the Constitution deals with power and
(c) The Lok Sabha may send the Bill to the Rajya privileges of Members of the State Legislatures.
Sabha for reconsideration Freedom from attendance as witness is a personal
(d) The President may call a joint sitting for privilege of members of Parliament while rest of the
passing the Bill options given are collective privilege.
Indian Polity & Constitution 86 YCT
15.
President/Vice-President
1. With reference to the election of the President 2. The value of the vote of an elected Member
of India, consider the following statements: of Parliament equals Total value of the votes
(i) The value of the vote of each MLA varies of all elected MLAs/Total number of elected
from State to State.
MPs
(ii) The value of the vote of MPs of the Lok
Sabha is more than the value of the vote of 3. There were more than 5000 members in the
MPs of the Rajya Sabha latest election.
Which of the following statements given above Which of these statements is/are correct?
is/are correct? (a) 1 and 2 (b) Only 2
(a) 1 only (b) 2 only
(c) 1 and 3 (d) Only 3
(c) Both 1 and 2 (d) Neither 1 nor 2
IAS (Pre) G.S., 2003
IAS (Pre) G.S, 2018
Ans. (a) Article 55 of the Constitution provides manner Ans. (b) Kindly refer the explanation of the above
of election of President which shall be held in question.
accordance with the system of proportional 3. Who amongst the following are not included in
representation by means of the single transferable vote
the Electoral College for the election of the
and the voting shall be by secret ballot. The President is
elected by the members of an electoral college President of India?
consisting of the elected members of both the Houses of (a) Elected members of the House of the People.
Parliament and the elected members of the Legislative (b) Elected members of the Council of States.
Assemblies of States and the Union Territories of Delhi (c) Elected members of the Legislative
and Pondicherry. For securing uniformity among the
Assemblies of the States.
States as per Article 55, the number of votes to which
each State is entitled is determined as follows:- (d) Elected members of the Legislative Councils
(a) Every elected member of the legislative assembly of the States.
of a State shall have as many votes as there are UPPCS (Pre.) Re-exam G.S., 2015
multiples of one thousand in the quotient obtained
Ans. (d) Kindly refer the explanation of the above
by dividing the population of the state by the total
number of elected members of the Assembly. question.
(b) If after taking the said multiples of one thousand, 4. Which of the following are the members of the
the remainder is not less than five hundred and then Electoral College for electing the President of
the vote of each member shall be further increased India? Select the correct answer from the codes
by one. given below:
(c) Each elected member of either House of Parliament
1. All the members of the two Houses of
shall have such number of votes as may be obtained
by dividing the total number of votes assigned to Parliament.
the members of the State Legislation Assemblies by 2. All the elected members of the State
the total number of elected members of both the Legislative Assemblies.
House of Parliament fractions exceeding one-half 3. All the elected members of the U.T.
being counted as one and other fractions being
Assemblies,
disregarded. Hence statement 1 is correct while
statement 2 is incorrect. 4. All the Governors and Lt-Governors.
2. Consider the following statements: Codes:
In the Electoral College for Presidential (a) 1 and 2 only (b) 2 and 3 only
Election in India, (c) 1, 2 and 3 only (d) 1,2, 3 and 4
1. The value of the vote of an elected Member
UPPCS (Main) G.S. IInd Paper 2012
of Legislative Assembly equals State
Population Number of elected MLAs of the Ans. (c) Kindly refer the explanation of the above
State x 100 question.
Indian Polity & Constitution 87 YCT
5. By the 70th Constitution amendment, which of 8. Which of the following is part of the Electoral
the following two Union territories have right College for the election of the President of
to vote in the election of the President? India but does not form part of forum for his
(a) Delhi and Chandigarh impeachment?
(b) Delhi and Puducherry (a) Lok Sabha
(c) Delhi and Daman and Diu (b) Rajya Sabha
(c) State Legislative Councils
(d) Chandigarh and Puducherry
(d) State Legislative Assemblies
UPPSC RO/ARO G.S.IInd Paper 2009
IAS (Pre) G.S., 1996
Ans. (b) Kindly refer the explanation of the above
Ans. (d) According to Article 54 of the Constituion,
question.
Electoral college of President consisting of the elected
6. In the Presidential election in India, every members of both the Houses of Parliament and the
elected member of the Legislative Assembly of elected members of the Legislative Assemblies of States
a State shall have as many votes as there are and the Union Territories of Delhi and Pondicherry. As
multiples of one thousand in the quotient per the Article 61(1) of the Constitution, when a
obtained by dividing the population of the President is to be impeached for “violation of the
State by the total number of elected members Constitution”, the charge shall be preferred by either
of the Assembly. As at present (1997) the House of Parliament. Therefore State Legislative
expression “population” here means the Assemblies are not involved in the impeachment of the
population as ascertained by the President. Hence d is the correct answer.
(a) 1991 Census (b) 1981 Census 9. The President can be impeached on the
(c) 1971 Census (d) 1961 Census grounds of violating the Constitution by
IAS (Pre) G.S., 1997 (a) The Chief Justice of India.
Ans. (c) Under Article 82 of the Constitution, the (b) The Vice-President of India.
Parliament by law enacts a Delimitation Act after every (c) The Speaker of the Lok Sabha.
census. After commencement of the Act, the Central (d) The two Houses of Parliament.
Government constitutes a Delimitation Commission. UPPCS (Main) G.S. IInd Paper 2008
This Delimitation Commission demarcates the Ans. (d) Kindly refer the explanation of the above
boundaries of the Parliamentary Constituencies as per question.
provisions of the Delimitation Act. The present 10. The Chief Minister of a State in India is NOT
delimitation of constituencies has been done on the eligible to vote in Presidential election if
basis of 2001 census figures under the provisions of (a) He himself is a candidate
Delimitation Act, 2002. The Constitution of India was (b) He is yet to prove his majority on the floor of
specifically amended in 2002, not to have delimitation the Lower House of the State Legislature
of constituencies till the first census after 2026. Thus (c) He is a member of the Upper House of the
the present constituencies carved out on the basis of Legislature
2001 census shall continue to be in operation till the (d) He is a caretaker Chief Minister
first census after 2026. IAS (Pre) G.S., 2003
7. On which matter, it is not essential to take Ans. (c) As per the Article 54 of the Constitution, the
advice from Council of Ministers for electoral college of President of India consist of elected
President? members of both Houses of Parliament and the elected
(a) Appointment of Ambassadors members of the Legislative Assemblies of the States.
Hence member of the Upper House of the Legislature
(b) To assent on Bills
(Vidhaan Parishad) cannot take part in the election of
(c) To assent on Bills referred from Governor the President.
(d) To dissolve Lok Sabha
11. Election to the Office of the President is
(e) To assent on constitutional amendment bill conducted by
Chattisgarh P.C.S. (Pre.) 2016 (a) The Speaker of the Lok Sabha
Ans. (b) It is not essential for the President to take (b) The Prime Minister's Office
advice from Council of Ministers to assent on Bills. For (c) The Minister for Parliamentary Affairs
rest of the given options, it is necessary for the (d) The Election Commission of India
President to take advice from Council of Ministers. UPPCS (Main) G.S. IInd Paper 2009
Indian Polity & Constitution 88 YCT
Ans. (d) As per the Article 324(1) of the Constitu- 15. In the absence of the President of India, if Vice-
tion, superintendence, direction and control of the President is not available, who of the following
preparation of the electoral rolls and the conduct of all can act as the President?
elections to Parliament and to the Legislature of every (a) Chief Justice of India
State and of elections to the offices of President and (b) Prime Minister
Vice President held under Constitution shall be vested (c) Speaker of the Lok Sabha
in Election Commission of India. (d) Attorney General of India
12. Who settle the dispute regarding the election of UPPCS (Pre.) Re-exam. G.S., 2015
the President of India? Ans. (a) As per the President (Discharge of Funct-ions)
(a) Supreme Court (b) Election Commission Act 1969, in the event of the occurrence of vacancies in
(c) Parliament (d) None of the above the offices of both the President and the Vice-President
Uttarakand RO/ARO, 2016 by reason in each case of death, resignation or removal
or otherwise, the Chief Justice of India or in his
Ans. (a) As per the Article 71(1) of the Constitution, all
absence, the senior most Judge of the Supreme Court of
doubts and disputes in connection with the election of
India available, shall discharge the functions of the
the President or Vice President shall be inquired into
President until a new President is elected or a new Vice-
and decided by the Supreme Court, whose decision
President so elected begins to act as President under
shall be final. article 65 of the Constitution, whichever is earlier.
13. A Member of Parliament or State Legislature 16. As per the Article 70 of the Constitution of
can be elected as President of India, but- India, who will be the President in absence of
(a) He has to resign before contesting election President and Vice President?
(b) He has to vacate the seat within six months of (a) Speaker of the Lok Sabha
his election (b) Prime Minister of India
(c) He has to vacate the seat immediately after (c) Chief Election Commissioner of India
getting elected (d) Chief Justice of India
(d) A Member of Parliament can contest for the Uttarakhand RO/ARO (M) 2016
election, but member of State Legislature can Himanchal PSC (Pre) G.S 2011
not
Ans. (d) Kindly refer the explanation of the above
UPPCS (Main) Spl. G.S. IInd Paper 2008 question.
Ans. (c) The President shall not be a member of either 17. The procedure of the impeachment of the
House of Parliament or of a House of the Legislature of President of India is
any State and if any such member is elected as (a) Judicial procedure
President, he shall be claimed to have vacated his seat (b) Quasi-Judicial procedure
in that House on the date on which he enters upon office
(c) Legislative procedure
as President.
(d) Executive procedure
14. The proposal for the candidate of the President RAS/RTS (Pre.) G.S., 2016
of India should be ratified by atleast-
Ans. (b) Quasi-Judicial procedure means a proceed-ing
(a) By ten electors (b) By twenty electors
conducted by legislature or executive, similar to court
(c) By five electors (d) By fifteen electors proceedings and court may review the decision. As per
UPPCS (Mains) G.S IInd Paper, 2012 the Article 61 of the Constitution, President can be
Ans. (*) As per Presidential and Vice-Presidential impeached on the grounds of “Violation of the
Elections (Amendment) Act, 1974, there should be Constitution” and charges can be framed by either
atleast 10 electors as proposers and atleast 10 electors as House of the Parliament. The proposal to prefer such
seconders for the election of the President of India and charge is contained in a resolution which has been
in case of Vice President, it was atleast twenty electors. moved after at least fourteen days notice in writing
This Act was amended on 5th June, 1997 and as per the signed by not less than one fourth of the total number of
new provision in the case of Presidential election, at members of the House. Such resolution has been passed
least fifty electors as proposers and at least fifty electors by a majority of not less than two third of the total
as seconders and in the case of Vice-Presidential membership of the House. When a charge has been so
election, at least twenty electors as proposers and at preferred by either House of Parliament, the other
least twenty electors as seconders. Hence none of the House shall investigate the charge and the President
given options are correct. shall have the right to appear and to be represented as

Indian Polity & Constitution 89 YCT


such investigation .President can defend himself by Ans. (a) According to Article 56(a) of the Constitution,
taking service of Attorney General of India or any other President shall submit his resignation to the Vice
lawyer of his choice. If as a result of the investigation, a President and shall communicate to the Speaker of the
resolution is passed by a majority of not less than two Lok Sabha.
thirds of the total membership of the House by which
23. According to the Constitution of India, it is
the charge was investigated, the President stands
impeached. Hence impeachment of President in India is duty of the President of India to cause to be
a quasi-judicial procedure. laid before the Parliament which of the
following?
18. At least how many days, of prior notice is
required for the impeachment of the President 1. The Recommendations of the Union Finance
of India? Commission
(a) 7 days (b) 14 days 2. The Report of the Public Accounts Committee
(c) 21 days (d) 30 days 3. The Report of the Comptroller and Auditor
UPPCS (Pre) G.S. IInd Paper, 2006 General
Ans. (b) Kindly refer the explanation of the above 4. The Report of the National Commission for
question. the Scheduled Castes
19. Under which Article of the Constitution of Select the correct answer by using the codes
India, impeachment on President can be given below:
imposed? (a) 1 only (b) 2 and 4 only
(a) Article 356 (b) Article 75 (c) 1, 3 and 4 only (d) 1, 2, 3 and 4
(c) Article 76 (d) Article 61 IAS (Pre) G.S., 2012
UPPCS (Main) G.S. IInd 2017 Ans. (c) Reports to be laid down before the Parliament
Ans. (d) Kindly refer the explanation of the above by the President are-
question. (i) The Annual Financial Statement (Budget) and the
20. The vacant post of the president is to be filled Supplementary Statement, if any.
up within (ii) The report of the Auditor-General relating to the
(a) 30 days (b) 60 days accounts of the Government of India.
(c) Six months (d) One year (iii) The recommendations made by the Finance
UPPCS (Pre.) G.S., 2006 Commission.
UPPCS (Main) G.S. IInd 2013 (iv) The report of the Union Public Service
Ans. (c) According to the Article 62(2) of the Commission.
Constitution, the vacancy for the President shall be (v) The report of the Special Officer for Scheduled
filled in no case later than six months from the date of Castes and Tribes.
the occurrence of the vacancy. (vi) The report of the Commission on backward
21. How long can the Vice President act as classes.
President when the President cannot perform (vii) The report of the Special Officer for linguistic
his duties for reasons of death, resignation minorities.
removal or otherwise?
24. Who among the following are appointed by
(a) 5 years (b) 1 and half years
President of India?
(c) 1 year (d) 6 months
I. Chairman, Finance Commission
Jharkhand PSc (Pre) G.S. Ist Paper, 2013
II. The Deputy Chairman, Planning Commission
Ans. (d) Kindly refer the explanation of the above
III. The Chief Minister of a Union Territory
question.
Choose the correct answer from the codes
22. To whom does the President of India submit
given below.
his resignation?
(a) Vice-President Codes:
(b) Chief- Justice (a) I only (b) I and II only
(c) Speaker of the Lok Sabha (c) I and III only (d) II and III only
(d) Prime Minister IAS (Pre) G.S., 1994
(e) Chief Election Commissioner UPPCS(Mains) G.S. IInd Paper,2005
Chattisgarh PSC (Pre.) 1st, 2012 Jharkhand PSC (Pre) G.S,2006
Indian Polity & Constitution 90 YCT
Ans. (c) Article 280 of the Constitution is regarding Ans. (b) According to the Article 143 of the Indian
Finance Commission. It asserts that the President, on Constitution. The President can seek advice from the
every fifth year, constitute a Finance Commission Supreme Court (Appex Court) on any question of law
which shall consist of a Chairman and four other or fact. However the advice tendered by Supreme Court
members to be appointed by the President. Section 45 in is not binding to the President.
the Government of Union Territories Act, 1963 asserts 29. Which article of the Indian Constitution
that the Chief Minister of a Union Territory which has empowers the president to consult the Supreme
Legislative Assembly shall be appointed by the Court?
President. Planning Commission was established in (a) Article 129 (b) Article 132
1950 to oversee the country’s economic and social (c) Article 143 (d) Article 32
development, mainly through the formulation of five- UPPCS (Pre.) G.S. Spl., 2004
year plans. The commission was chaired by Prime
Ans. (c) Kindly refer the explanation of the above
Minister and a deputy chairman and other members
question.
were appointed by the Prime Minister. In January 2015,
30. Who among the following has the
Planning Commission was transformed to NITI Aayog
constitutional power to declare any
(National Institute for Transforming India).
geographical area as Scheduled area?
25. Which one of the following Articles of the
(a) Governor (b) Chief Minister
Constitution vests the executive power of the
Union in the President? (c) Prime Minister (d) President
(a) Article 51 (b) Article 52 UPPCS (Pre) G.S., 2006
(c) Article 53 (d) Article 54 Ans. (d) Article 244 of the Constitution is regarding
UPPCS (Mains) G.S. IInd Paper,2010 administration of Scheduled Areas and Tribal Areas.
Ans. (c) Article 53 of the Constitution is regarding As per the Constitutional provision under Article 244
executive power of the Union which asserts that the (1) of the Constitution, the ‘Scheduled Areas’ are
executive power of the Union shall be vested in the defined as ‘such areas as the President declare to be
President and shall be exercised by him either directly Scheduled Areas’ as per the Fifth Schedule of the
or through officers subordinates to him. Constitution. The specification of ‘Scheduled Areas’ in
26. The executive power of the Union vests in: relation to a State is by a notified order of the President
(a) The Prime Minister after consultation with the Governor of that State. Such
(b) The President provision is not applicable to the States of Assam
(c) The Council of Ministers Meghalaya, Tripura and Mizoram.
(d) The Parliament 31. The constitutional authority, vested with the
UPPCS (Pre.) Re-exam G.S., 2015 power of declaring castes and tribes as the
Ans. (b) Kindly refer the explanation of the above Scheduled Castes and Scheduled Tribes, is the
question. (a) President of India
27. Under which article of the Indian Constitution (b) Prime Minister of India
President of India can withhold his assent to a (c) Minister of Social Welfare
bill passed by both the house of parliament? (d) Chairman, SC/ST Commission
(a) Article 63 (b) Article 108 UPPCS (Pre) G.S., 2016
(c) Article 109 (d) Article 111 Ans. (a) According to Article 341 (1) of the
UPPCS (Pre) G.S, 1993 Constitution, the President may with respect to any
Ans. (d) Article 111 of the Constituion asserts that State or Union Territory after consultation with the
when a Bill has been passed by the both Houses of Governor of the State specify caste, races or tribes be
Parliament, it shall be presented to the President and the deemed to be Schedule Caste in relation to that State/
President shall declare either that he assents to the Bill Union Territory. Same is applicable to the Schedule
or that he withholds assent.
Tribes as per the Article 342 (1) of the Constitution.
28. For the Constitutional matters, who has
32. Given below are two statements:
authority to consult the Supreme Court of
India? Assertion (A) The Union Executive is headed
(a) Prime Minister (b) President by the President of India.
(c) High Court (d) All of above Reason (R) There are no limitations on the power
UPPCS (Pre.) G.S., 2004 of the President.
Indian Polity & Constitution 91 YCT
In the context of the above which are off the 36. How many members may be nominated by the
following is correct? President from Anglo-Indian Community, if he
(a) Both (A) and (R) are true and (R) is the is of opinion that the community is not
correct explanation of (A). adequately represented in the House of the
(b) Both (A) and (R) are true, but (R) is not the People?
correct explanation of (A). (a) 3 (b) 5
(c) (A) is true, but (R) is false. (c) 1 (d) 2
(d) (A) is false, but (R) is true. UPPCS (Pre) G.S.,2014
UPPCS (Main) G.S. IInd Paper 2012 Ans. (d) According to Article 331 of the Constitution,
Ans. (c) As per the Article 53 of the Constitution, the the President may, if he is of opinion that the Anglo
executive power of the Union shall be vested in the Indian community is not adequately represented in the
President and shall be exercised by him either directly Lok Sabha, nominate not more than 2 members of that
or through officers subordinates to him. Hence community to the Lok Sabha.
Assertion (A) is correct. The power of President in India
37. Which of the following are the discretionary
is checked and balanced by constitutional provisions.
powers of President?
Hence it is not unlimited. Therefore reason (R) is not
(i) To appoint Prime Minister to the leader of
correct.
majority party.
33. The Constitution authorizes the President to
(ii) To summon the Parliament.
make provisions for discharge of duties of
Governor in extraordinary circumstances (iii) To return a bill to the Parliament with
under the objections.
(a) Article 160 (b) Article 162 (iv) To appoint the members of Council of
(c) Article 165 (d) Article 310 Ministers.
UPPCS (Main) G.S. IInd Paper 2007 (v) To assent the bills refer by the Governor.
Ans. (a) According to Article 160 of the Constitution, (vi) To detain a bill.
the President may make such provision as he thinks fit (vii) To ask the Prime Minister for such
for the discharge of the functions of the Governor of a information related to any matter.
State in any contingency. (viii) To say the Council of Ministers for
34. Which one of the following is not a constitutional reconsideration on any advise.
prerogative of the President of India? (a) (i) (ii) (iv) (vii)
(a) Returning an ordinary Bill for reconsideration (b) (ii) (vi) (vii) (viii)
(b) Returning a financial bill for reconsideration (c) (i) (iii) (vi) (viii)
(c) Dissolving the Lok Sabha (d) (ii) (v) (vii) (viii)
(d) Appointing the Prime Minister (e) (iii) (vi) (vii) (viii)
UPPCS (Main) G.S. IInd Paer 2007 Chattisgarh PSC (Pre.) G.S. 2017
Ans. (b) Financial Bill cannot be returned by the President
Ans. (e) : Discretionary powers of the President are
to the Parliament for its reconsideration as it is presented in
suspensive veto i.e. he can return a bill (not Money Bill)
the Lok Sabha with his permission. Rest are the to the Parliament for reconsideration, pocket veto;
constitutional prerogative of the President of India. which is not mentioned in Constitution but the President
35. Who nominates the 12 members for Rajya neither ratifies nor reject nor return the bill but simply
Sabha in India? keeps the bill pending for an indefinite period. Under
(a) Prime Minister (b) Chief Justice article 78 of the Constitution, the President enjoys the
(c) President (d) Vice President right to seek information from the Prime Minister
Uttarakhand RO/ARO, 2016 regarding the administration of the affairs of the Union.
Ans. (c) As per the Article 80 of the Constitution, Under Article 85 of the Constitution, the President can
President can appoint 12 members to the Rajya Sabha summon each House of Parliament. According to
having special knowledge or practical experience in the Article 74(1) of the Constitution, President may ask
field of literature, art, science and social service. Council of Ministers for reconsideration on any advice.

Indian Polity & Constitution 92 YCT


38. The only instance when the President of India 41. Which Article of the Constitution of India
exercised his power of veto related to empowers the President to issue ordinance?
(a) The Hindu code bill (a) Article 74
(b) The PEPSU appropriation bill
(b) Article 78
(c) The Indian Post Office (Amendment Bill)
(c) Article 123
(d) The Dowry Prohibition Bill
IAS (Pre) G.S., 1993 (d) Article 124(2)
Ans. (c) In 1986 President Gryani Zail Singh (1982-87) UPPCS (Pre) G.S., 2015
exercised pocket veto to prevent the Indian Post Office Ans. (c) According to Article 123 of the Constitution, if
(Amendment) Bill from becoming law. In India, at any time except when both Houses of Parliament are
President has three veto powers namely absolute, in session, the President is satisfied that circumstances
suspension and pocket. In absolute veto, President has
exist which render it necessary for him to take
power to withhold his approval to any bill passed by the
immediate action, he may promulgate Ordinance as the
Parliament. Once the bill is withheld, it ends and does
not become an Act. In suspensive veto, President has circumstances appear to him to require and such
power to return the bill to the Parliament for Ordinance shall have the same force and effect as an
reconsideration and in pocket veto; the President neither Act of Parliament but every such Ordinance shall be
ratifies nor rejects nor returns the bill but simply keeps laid before both House of Parliament and shall cease to
the bill pending for an indefinite period. operate at the expiration of six weeks from the
39. Match the pairs regarding types of veto: reassemble of Parliament.
A. Absolute veto I. neither ratifies nor 42. The power of the President to issue ordinance
rejects nor returns the
is a relic of
bill
(a) G.O.I. Act of 1919
B. Qualified veto II. Can be overridden by
the legislature with an (b) G.O.I. Act of 1935
ordinary majority (c) G.O.I. Act of 1909
C. Suspensive veto III. Can be overridden by (d) Indian Independence Act, 1947
the legislature with a UPPCS (Main) G.S. IInd Paper 2008
higher majority
Ans. (b) The concept of Ordinance emerged in the
D. Pocket veto IV. Withholding of assent
to the bill Government of India Act, 1935 in which the Governor
A B C D General was given power to issue Ordinance. Similar
(a) I II III IV provisions are available in the Constitution of Pakistan
(b) IV III II I and Bangladesh.
(c) IV III I II 43. Power of the President to grant pardons etc. is a
(d) III IV II I (a) Legislative Power
Maharashtra PSC (Pre) G.S 2016 (b) Judicial Power
Ans. (b) Kindly refer the explanation of the above (c) Executive Power
question.
(d) None of the above
40. Which of the following President has exercised
UPPCS (Main) G.S. IInd Paper 2010
a power which is called ‘Pocket Veto’ in
constitutional terminology? Ans. (b) The Constitution of India has provided the
(a) Dr. Rajendra Prasad judicial power to the President of India (Article 72) and
(b) V.V Giri the Governors of States (Article 161) to grant pardons,
(c) Gyani Zail Singh reprieves, respites or remissions of punishment or to
(d) Dr. A.P.J Abdul Kalam suspend or commute the sentence of any person
(e) None of the above convicted of any offence. Governor of the State cannot
Chattisgarh P.C.S. 2015 grant pardon to somebody convicted and sentenced to
Ans. (c) Kindly refer the explanation of the above death, although he can commute such sentence. Only
question. President of India has power to pardon death sentence.
Indian Polity & Constitution 93 YCT
44. Which one of the following statements about 47. Who amongst the following elects the Vice
the President of India is not correct? President of India?
(a) He is a constituent part of Parliament. Select the correct answer from the codes given
(b) He participates in the discussions in the two below:
Houses. 1. Member of Lok Sabha
(c) He addresses the two Houses at a joint 2. Member of Rajya Sabha
meeting every year 3. Members of Legislative Assembly
(d) He can promulgate ordinances in certain 4. Members of Legislative Council
situations.
Codes:
UPPCS (Main) G.S. IInd Paper 2010
(a) 1 and 2 only
Ans. (b) According to Article 85 of the Constitution,
(b) 1 and 3 only
President summons each house of Parliament from time
(c) 1, 2 and 3 only
to time but he does not participate in the discussions in
the two Houses. (d) 1, 2, 3 and 4
45. Consider the following statements: Punjab PSC (Pre) G.S 2013
1. The President shall make rules for the more UPPCS (Pre) G.S., 2012
convenient transaction of the business of the Ans. (a) According to Article 66(1) of the Constitution,
Government of India, and for the allocation the Vice President shall be elected by the members of
among Ministers of the said business. an electoral college consisting of the members of both
2. All executive actions of the Government of Houses of Parliament in accordance with the system of
India shall be expressed to be taken in the proportional representation by means of the single
name of the Prime Minister. transferable vote and the voting at such election shall be
Which of the statements given above is / are by secret ballot.
correct? 48. The Vice-President of India is elected by the
(a) 1 only (b) 2 only members of an Electoral College comprising
(c) Both 1 and 2 (d) Neither 1 nor 2 (a) Only elected members of both Houses of
IAS (Pre) G.S., 2014 Parliament.
Ans. (a) According to Article 77(3) of the Constitution, (b) All the members of both Houses of
the President shall make rules for the more convenient Parliament.
transaction of the business of the Government of India (c) Only elected members of Rajya Sabha and
and for the allocation among Ministers of the said State Assemblies.
business. Hence statement 1 is correct. Article 77(1) of
(d) Only elected members of State Assemblies.
the Constitution asserts that all executive action of the
UPPCS (Main) G.S.IInd Paper 2012
Government of India shall be expressed to be taken in
the name of the President. Therefore statement 2 is UPPCS (Pre) G.S., 2010
incorrect. Ans. (b) Kindly refer the explanation of the above
46. "He represents the nation but does not lead the question.
nation"- This dictum applies on whom of the 49. Which one of the following statements about
following? the Vice-President of India is incorrect?
(a) Speaker of the Lok Sabha (1) The electoral college for election to this
(b) President office consists of the elected members of both
(c) Prime Minister the Houses of Parliament.
(d) Chief Justice of India (2) He can hold the office of the President for a
UPPCS (Main) G.s. IInd Paper 2007 period not exceeding six months in the
Ans. (b) Dr. B.R. Ambedkar said, “Our President is the absence of the President.
Head of the State but not of the Executive. (3) For election to this office a candidate's
He represents the Nation, but does not rule the Nation.” nomination paper must be subscribed by at
The President of India is a ceremonial head and the least 20 electors as proposers and 20 electors
actual power lies with elected Government. as seconders.

Indian Polity & Constitution 94 YCT


Codes : 52. Consider the following statements:
(a) 1 (b) 2 1. The original Constitution provided that the
(c) 3 (d) None of the above Vice-President would be elected by the two
Maharashtra PSC (Pre) G.S 2018 Houses of Parliament assembled at a joint
Ans. (d) According to Article 66(1) of the Constitution, meeting.
the Vice President shall be elected by the members of 2. The 11th Constitutional Amendment, 1961
an electoral college consisting of the members of both changed the procedure of election of the
Houses of Parliament. Hence statement 1 is correct. Vice-President.
According to Article 65 of the Constitution, the Vice- 3. The Vice- President is elected by the
President acts as President during casual vacancy in the members of an electoral college consisting of
office of the President by reason of death, resignation or the elected members only of both the Houses.
removal or otherwise until a new President is elected as 4. The election of the Vice-President cannot be
soon as practicable and in no case later than six months challenged on the grounds that the electoral
from the date of occurrence of the vacancy. Hence college was incomplete.
statement 2 is correct. As per Presidential and Vice- (a) Statements 1, 2 are correct, 3 and 4 are false
Presidential Elections (Amendment) Act, 1974, which (b) Statements 2, 3, 4 are correct, 1 is false
was amended on 5th June, 1997 and as per the new (c) Statements 1, 2, 4 are correct and 3 is false
provision, in the case of Vice-Presidential election, at (d) Statements 2, 3 are correct, 1 and 4 are false
least twenty electors as proposers and at least twenty Maharashtra PSC (Pre) G.S, 2019
electors as seconders. Hence statement 3 is also correct. Ans. (c) : Under Article 66 (1) of the Constituion, the
50. The resolution for removing the Vice- Vice- President has to be elected by members of the
President of India can be moved in the: both the Houses of Parliament assembled at a joint
(a) Lok Sabha alone meeting. Hence statement 1 is correct. The Constituion
(b) Either House of Parliament (Eleventh Amendment Act), 1961 amended Article
(c) Joint sitting of the Parliament 66(1) of the Constituion due to the reason that the
(d) Rajya Sabha alone requirement that members of the two Houses should
IAS (Pre) G.S., 2004 assemble at a joint sitting for the election of the Vice-
President seems to be totally unnecessary and may also
UPPCS (Main) G.S.Ist Paper 2004
cause practical difficulties. Therefore the Constituion
Ans. (d) According to Article 67(b) of the Constitu-
(Eleventh Amendment Act), 1961 amended Article 66
tion, a Vice President may be removed from his office
(1) as Vice-President will be elected by members of an
by a resolution of the Rajya Sabha passed by a majority
electoral college consisting of members of both Houses
of all the then Members of the Rajya Sabha and agreed
of Parliament. Further the Constituion (Eleventh
to by the Lok Sabha but no resolution shall be moved
Amendment Act), 1961 amended Article 71 of the
unless at least fourteen days notice has been given of
Constituion and inserted clause 4 in Article 71 as the
the intention to move the resolution.
election of a person as President or Vice President shall
51. The Proposal relating to dismissal of the Vice not be called in question on the ground of the existence
President can be presented in of any vacancy for whatever reason among the members
(a) Any House of the Parliament of the electoral college electing him. Hence statement 2
(b) Rajya Sabha and 4 are also correct. The nominated members of
(c) The Lok Sabha Rajya Sabha as well as of Lok Sabha are also eligible to
(d) None of the above be included in the electoral college of Vice-President.
UPPCS (Main) G.S.IInd 2007 Hence statement 3 is not correct.
UPPCS (Main) G.S. 2003 53. The Vice President of India is elected by-
Ans. (b) Kindly refer the explanation of the above I. The members of Lok Sabha
question. II. The members of Rajya Sabha out of these

Indian Polity & Constitution 95 YCT


(a) Neither I nor II 3. Neelam Sanjiva
(b) I only 4. Shankar Dayal Sharma
(c ) II only Select the correct answer using the code given
(d) Both I and II below:-
TNPSC (Pre) G.S. 2014 (a) 1,2,3 and 4 (b) 1 and 4 only
Ans. (d) : Kindly refer the explanation of the above (c) 2 and 3 only (d) 3 and 4 only
question. IAS (Pre.) G.S.,2008
54. Who decides the dispute regarding the election Ans. (b) List of former Vice Presidents of India is- Dr.
of the Vice-President in India? Sarvepalli Radhakrishnan, Dr.Zakir Hussain, Sh. V.V. Giri,
(a) President Sh. Gopal Swarup Pathak, Sh. B.D.Jatti, Sh. M.
(b) Election Commission Hidayatullah, Sh. R. Venkataraman, Dr. Shanker Dayal
(c) Supreme Court Sharma, Sh. K.R. Narayanan, Sh. Krishan Kant, Sh.
(d) Chairperson Rajya Sabha Bhairon Singh Shekhawat, Sh. M. Hamid Ansari and
Chhatisgarh PSC (Pre.) 1st, 2012 currently Sh. M. Venkaiah Naidu is Vice President of India.
Ans: (c) According to Article 71(1), all doubts and 57. Which of the following statements about the
disputes arising out of or in connection with the election Vice-President of India are not correct?
of the President or Vice President shall be inquired into 1. In order to be the Vice-President, a person
and decided by the Supreme Court, whose decision must be qualified for election as a member of
shall be final. the House of the People.
55. The Vice President of India, 2. A member of the State Legislature can be
1. Is the second highest dignitary of India. candidate for his office.
2. Has no formal function attached to his office. 3. The term of the office of the Vice- President
3. Discharges the functions of the President is the same as that of the President.
during his absence. 4. He can be removed through a formal
4. Acts as President, if the President resigns or impeachment.
is removed or dies. Select the correct answer using the codes given
Select the correct answer from the code given below:
below: (a) 1 and 2 (b) 2 and 3
Codes: (c) 1 and 4 (d) 3 and 4
(a) Only 1 and 2 (b) Only 1, 2 and 3 UPPCS (Main) G.S.IInd Paper 2005
(c) Only 1, 3 and 4 (d) All the four Ans. (c) According to Article 66(3) of the Constitution,
U.P. Lower (Pre.) G.S. 2013 a person is eligible for election as Vice-President if he is
UPUDA/LDA (Pre.) G.S., 2010 citizen of India, has completed the age of thirty five
Ans. (d) As per the order of precedence in India, Vice years and is qualified for election as a member of the
President is the second highest dignitary of India after Council of States (Rajya Sabha). The process of
President. The only provision with regard to the Vice- impeachment of Vice-President of India has been
President's function in the Constitution is as the provided in Article 67(b) which asserts that a Vice
Chairperson of the Rajya Sabha (Article 64). As per the President may be removed from his office by a
Article 65 of the Constitution, the Vice-President acts as resolution of the Council of States (Rajya Sabha) passed
President, during casual vacancy in the office of the by a majority of all the then members of the Council
President by reason of death, resignation or removal. and agreed to by the House of the People (Lok Sabha),
56. Who among the following have held the office but no resolution for the purpose of this clause shall be
of the Vice President of India? moved unless at least fourteen days notice has been
1. Mohammad Hidayatullah given of the intention to move the resolution. Hence
2. Fakhruddin Ali Ahmed statement 1 and 4 are incorrect.

Indian Polity & Constitution 96 YCT


16.
Lok Sabha/Rajya Sabha
1. Rajyasabha has equal power with Loksabha in 3. For election to the Lok Sabha, a nomination
(a) The matter of creating new all India services paper can be filed by
(b) Amending the constitution (a) Anyone residing in India.
(c) The removal of the government (b) A resident of the constituency from which the
(d) Making cut motions election is to be contested.
IAS (Pre) G.S. 2020 (c) Any citizen of India whose name appears in
Ans. (b) Rajyasabha has equal power with Loksabha in the electoral roll of a constituency.
matters related to introduction and passage of ordinary (d) Any citizen of India.
and Constitutional Amendment Bills, financial Bills IAS (Pre) G.S, 2017
related to expenditure from Consolidated Fund of India, Ans. (c) For contesting an election as a candidate, a
election and empeachment of the President, election of person must be registered as a voter in the voter list.
the Vice President. Section 4 (d) of Representation People Act, 1951
2. Consider the following statements: prohibits a person from contesting unless he is an
1. In the first Lok Sabha, the single largest party elector in any parliamentary constituency.
in the opposition was the Swatantra Party. 4. Consider the following statements:
2. In the Lok Sabha, a “Leader of the 1. In the election for Lok Sabha or State
Opposition” was recognised for the first time Assembly, the winning candidate must get at
in 1969. least 50 percent of the votes polled, to be
3. In the Lok Sabha, if a party does not have a declared elected.
minimum of 75 members, its leader cannot be 2. According to the provisions laid down in the
recognised as the Leader of Opposition. Constitution of India, in Lok Sabha, the
Which of the statements given above is/are Speaker’s post goes to the majority party and
correct? the Deputy Speaker’s to the Opposition.
(a) 1 and 3 only (b) 2 only Which of the statements given above is/are
(c) 2 and 3 only (d) 1, 2 and 3 correct?
IAS (Pre) G.S, 2018 (a) 1 only
Ans. (b) For the first Lok Sabha (1952-57), elections (b) 2 only
were held for 489 seats. Indian National Congress won (c) Both 1 and 2
364 seats followed by Communist Party of India with 16 (d) Neither 1 nor 2
seats and the Socialist Party with 12 seats. Hence IAS (Pre) G.S, 2017
statement 1 is not correct. In the Lok Sabha, a “Leader of Ans. (d) In the election of Lok Sabha or State
the Opposition” was recognised for the first time in 1969. Assembly, majority of votes is sufficient to get elected
Dr. Ram Subhag Singh was the first leader of opposition even if it is less than 50% of the total votes polled.
in Lok Sabha from 17 Dec 1969 to 27 Dec 1970. Hence Hence statement 1 is incorrect. Speaker and Deputy
statement 2 is correct. According to Parliament rules, the Speaker of the Lok Sabha are elected by the members of
biggest Opposition party in the House has to have at least the Lok Sabha. This is a convention (not constitutional
10 per cent of the total strength of the Lok Sabha or 55 provision) that usually Speaker’s post goes to the
seats, to be eligible for the post of Leader of Opposition. majority party and the Deputy Speaker’s post to the
Hence statement 3 is wrong. Opposition. Hence statement 2 is also incorrect.

Indian Polity & Constitution 97 YCT


5. Which one of the following is not correct with Tribes in the State concerned to that of the total
regard to the elections to the Lok Sabha? population as per the provision contained in Article 330
(a) The elections are on the basis of adult of the Constitution. For Scheduled Castes, 84 seats are
suffrage reserved in Lok Sabha and for Scheduled Tribes, 47
(b) The Election Commission is required to seats are reserved in Lok Sabha.
conduct the elections. 10. In order to get elected for Lok Sabha,
(c) The electoral rolls are prepared under the minimum age required is?
direction and control of the President of India. (a) 18 years (b) 21 years
(d) On the request of the Election Commission, (c) 25 years (d) None of the above
the President is required to make available to
BPSC (Pre.) G.S. 2002
the Election Commission such staff as may be
Ans. (c) Article 84 (b) of Constitution provides that the
necessary for the discharge of its functions.
minimum age for becoming a candidate for Lok Sabha
UPPCS (Main) G.S. IInd Paper 2011
election shall be 25 years. Similar provision exists for a
Ans. (c) The electoral rolls are prepared under the
candidate to the Legislative Assemblies.
direction and control of the Election Commission of
India. Rest all are correct. 11. After how many days of absence from
Parliament without permission can a M.P. be
6. Maximum strength of Lok Sabha as stipulated
disqualified?
in the Constitution of India is—
(a) 30 days (b) 60 days
(a) 547 (b) 545
(c) 90 days (d) 120 days
(c) 552 (d) None of these
MPPCS (Pre) G.S. 2008 UPPCS (Pre) G.S, 2018
Ans: (c) Maximum strength of the Lok Sabha is 552 Ans: (b) According to Article 101 (4), if for a period of
members. 530 members to represent the States, 20 sixty days a member of either House of Parliament is
members to represent the Union Territories and 2 without permission of the House absent from all
members to be nominated by the President from the meetings thereof, the House may declare his seat
Anglo-Indian Community. For 17th Lok Sabha, total vacant, provided that in computing the said period of
strength of the members of Lok Sabha is 542. sixty days no account shall be taken of any period
7. What can be the maximum representation during which the House is prorogued or is adjourned for
from the Union Territories to the Lok Sabha? more than four consecutive days.
(a) 15 (b) 20 12. The deposit is forfeited for any candidate
(c) 25 (d) None of the above contesting the election for Lok Sabha if he does
UPPCS (Pre) G.S, 2018 not get-
Ans: (b) Kindly refer the explanation of the above (a) 1/3rd of valid votes
question. (b) 1/4th of the valid votes
8. How many seats have been reserved in Lok (c) 1/5th of the valid votes
Sabha for Union Territories? (d) None of the above
(a) 30 (b) 27 BPSC (Pre.) G.S, 1995
(c) 25 (d) 20 Ans. (d) As per the Election Commission of India rules,
nd
UPPCS (Main) Spl. G.S.II 2008 if the candidate fails to get a minimum of one-sixth
Ans. (d) Kindly refer the explanation of the above (1/6) of the total valid votes polled, the deposit goes to
question. the treasury i.e. forfeited.
9. How many seats are reserved for schedule caste 13. The term of Lok Sabha
in the Lok Sabha? (a) Cannot be extended at any circumstances
(a) 59 (b) 69 (b) Can be extended by six months at a time
(c) 79 (d) 84 (c) Can be extended by one year at a time during
MPPCS (Pre) 2018 the proclamation of emergency
Ans. (d) Allocation of seats for Scheduled Castes and (d) Can be extended for two years at a time
Scheduled Tribes in the Lok Sabha are made on the during the proclamation of emergency
basis of proportion of Scheduled Castes and Scheduled IAS (Pre) G.S., 2002
Indian Polity & Constitution 98 YCT
Ans. (c) According to Article 83(2), Lok Sabha, unless 15. Name the Prime Minister when for the first
sooner dissolved, shall continue for five years from the time the No-Confidence was moved in the
date appointed for its first meeting while a proclamation Parliament?
of emergency is in operation, be extended by Parliament (a) Jawahar Lal Nehru
by law for a period not exceeding one year at a time and (b) Lal Bahadur Shastri
not extending in any case beyond a period of six months (c) Indira Gandhi
after the proclamation has ceased to operate. (d) Morarji Desai
14. Which one of the following statements is/are TNPSC (Pre) G.S. 2019
not correct? Ans. (a) Kindly refer the explanation of the above
1. In Lok Sabha, a no-confidence motion has to question.
set out the grounds on which it is based.
16. Consider the following statements regarding a
2. In the case of a no-confidence motion in Lok
No-Confidence Motion in India:
Sabha procedures of admissibility have been
1. There is no mention of a No-Confidence
laid down in Rule 198 of Lok Sabha.
Motion in the Constitution of India.
3. A motion of no-confidence once admitted has
2. A Motion of No-Confidence can be
to be taken up within ten days of the leave
introduced in the Lok Sabha only.
being granted
Which of the statements given above is / are
4. In Italy, the government requires the support
correct?
of both houses of Parliament.
(a) 1 only (b) 2 only
(a) Only 1 (b) Only 3
(c) Both 1 and 2 (d) Neither 1 nor 2
(c) 2 and 3 (d) 1 and 4
IAS (Pre) G.S., 2014
Maharashtra PSC (Pre) G.S, 2019
Ans. (c) Kindly refer the explanation of the above
Ans. (a) : The Constitution of India does not mention
question.
either a confidence motion or a no-confidence motion.
It can be moved only in the Lok Sabha. If the 17. Who was the prime Minister when for the fist
Government has to demonstrate its strength on the floor time No-Confidence motion was moved in the
of the House, it can have a motion of confidence. Indian parliament?
However, the opposition parties or any member can (a) Jawaharlal Nehru
move a motion expressing want of confidence (no (b) Indira Gandhi
confidence) in the Lok Sabha. The procedure is laid (c) Morarji Desai
down under Rule 198 of the rules of procedure and (d) Lal Bahadur Shastri
conduct of business of the Lok Sabha. A no-confidence TNPSC (Pre) G.S. 2014
motion need not set out any grounds on which it is Ans. (a) : Kindly refer the explanation of the above
based. Even when grounds are mentioned in the notice question.
and read out in the House, they do not form part of the
18. Which one of the following statements is NOT
no-confidence motion. The member has to give a
correct?
written notice of the motion before 10 am which will be
(a) In Lok Sabha, a no - confidence motion has to
read out by the Speaker in the House. A minimum of 50
set out the grounds on which it is based
members have to accept the motion and accordingly, the
Speaker will announce the date for discussion for the (b) In the case of a no - confidence motion in Lok
motion. The allotted date has to be within 10 days from Sabha, no conditions of admissibility have
the day the motion is accepted, otherwise the motion been laid down in the Rules
fails and the member who moved the motion will be (c) A motion of no - confidence once admitted,
informed about it. Acharya Kripalani moved the first no has to be taken up within ten days of the leave
confidence motion against Jawaharlal Nehru in August, being granted
1963. In Italy, the Government requires the support of (d) Rajya Sabha is not empowered to entertain a
both houses of Parliament. Hence only statement 1 is motion of no - confidence
not correct. IAS (Pre) G.S., 2004
Indian Polity & Constitution 99 YCT
Ans. (a) Kindly refer the explanation of the above 21. Regarding Money Bill, which of the following
question. statements is not correct?
19. How many members can table the “No (a) A bill shall be deemed to be a Money Bill if it
Confidence Motion” against the Council of contains only provisions relating to
Ministers in the Lok Sabha? imposition, abolition, remission, alteration or
(a) 30 members (b) 40 members regulation of any tax.
(c) 50 members (d) 60 members (b) A Money Bill has provisions for the custody
UPPCS (Main) G.S. IInd Paper 2013 of the Consolidated Fund of India or the
Ans. (c) Kindly refer the explanation of the above Contingency Fund of India.
question. (c) A Money Bill is concerned with the
20. Consider the following statements: appropriation of moneys out of the
(1) If a person is elected to both the House of Contingency Fund of India.
Parliament, he must intimate within 10 days (d) A Money Bill deals with the regulation of
in which House he desires to serve, otherwise borrowing of money or giving of any
both seats become vacant. guarantee by the Government of India.
(2) If a person is elected to two seats in a House IAS (Pre) G.S, 2018
he should exercise his option for one, Ans: (c) Article 110 of the Constitution provides
otherwise both seats become vacant.
definition of the Money Bill which asserts that a Bill
(3) If a sitting member of one House is also shall be deemed to be a Money Bill if it contains
elected to the other House, his seat in the first
imposition, abolition, remission, alteration or regulation
House becomes vacant.
of any tax, the regulation of the borrowing of money or
Which of the above statements is/are not
the giving of any guarantee by the Government of India,
correct?
the custody of the Consolidated Fund or the
(a) Only 1 and 2 (b) Only 2 and 3
Contingency Fund of India, the payment of moneys into
(c) Only 1 (d) Only 3
or the withdrawal of moneys from Contingency Fund of
Maharashtra PSC (Pre) G.S. 2017 India.
Ans. (c) According to Article 101(1) of the
22. Money Bill can be presented
Constitution, a person cannot be member of both
(a) Only in the Lok Sabh
Houses of Parliament. If a person is elected to both
Houses of Parliament but have not taken his/her seat in (b) Only in the Rajya Sabha
either House, he/she should give notice in writing (c) In both the Houses
signed by him and delivered to the Secretary to the (d) In joint meeting of both the Houses
Election Commission within ten days from the date, (e) None of the above
intimating in which of the Houses he/she wish to serve. BPSC (Pre) G.S. 2018
His/her seat in the other House will then become vacant. UPPCS (Main) G.S. IInd Paper 2015
If the elected person fails to send such intimation within
Ans. (a) Money Bill can only be introduced in Lok
that time-limit, his/her seat in the Rajya Sabha will
Sabha and if any question arises whether a Bill is a
become vacant automatically. Hence statement (a) is
Money Bill or not, the decision of the Speaker of the
incorrect. Further if a person is member of the Lok
Sabha and subsequently elected to Rajya Sabha, his/her Lok Sabha shall be final.
seat in Lok Sabha will become vacant on the date of 23. Which one of the following is authorized to
his/her election to the Rajya Sabha. Same is applicable prorogue the session of the house of the people?
for a member of Rajya Sabha elected to Lok Sabha. (a) Speaker
Hence statement (c) is correct. If a person is elected to (b) Attorney General
more than one seat in the Lok Sabha or the Rajya Sabha (c) Minister of Parliamentary affairs
or in the Legislature of a State, in such a case, he/she
(d) President
have to resign one of such seats within fourteen days
UPPCS (Main) G.S. IInd Paper 2015
from the date of such election. If the elected person fails
to do so, both of his/her seats become vacant. Hence Ans. (d) According to Article 85(2) of the Constitution,
statement (b) is correct. President may prorogue either House of Parliament.

Indian Polity & Constitution 100 YCT


24. Consider the following: 26. Regarding the office of the Lok Sabha Speaker,
Assertion (A) Recommendations of Rajya Sabha, consider the following statements:
after a Money Bill is passed by the Lok Sabha and 1. He/She holds the office during the pleasure of
transmitted to the Rajya Sabha have to be the President.
accepted by the Lok Sabha within 14 days from 2. He/She need not be a member of the House at
the receipt of recommendations and then get them the time of his/her election but has to become
incorporated in the Bill. a member of the House within six months
Reason (R) Money Bill cannot be introduced in from the date of his/her election.
the Rajya Sabha. 3. If he/she intends to resign, the letter of his/her
Select the correct answer from the codes given resignation has to be addressed to the Deputy
below: Speaker.
Codes: Which of the statements given above is /are
(a) Both (A) and (R) are true and (R) is the correct?
correct explanation of (A). (a) 1 and 2 only (b) 3 only
(b) Both (A) and (R) are true and (R) is not the (c) 1, 2 and 3 (d) None
correct explanation of (A). IAS (Pre) G.S., 2012
(c) (A) is true but (R) is false. Ans. (b) According to Article 93 of the Constitution,
(d) (A) is false but (R) is true. the Lok Sabha shall choose two members of the House
UPPCS (Main) 2017 G.S. IInd Paper to be respectively Speaker and Deputy Speaker of the
House. The Speaker may at any time resign from office
Ans. (d) According to Article 109(2) of the
by writing under her hand to the Deputy Speaker. The
Constitution, after a Money Bill has been passed by the
Speaker can be removed from office only on a
Lok Sabha, it shall be transmitted to the Rajya Sabha
resolution of the House passed by a majority of all the
for its recommendations and the Rajya Sabha shall
then members of the House. It is also mandatory to give
within a period of fourteen days from the date of its
a minimum of 14 days notice of the intention to move
receipt of the Bill return the Bill to the Lok Sabha with
the resolution. Hence only statement 3 is correct.
its recommendations and the Lok Sabha may thereupon
either accept or reject all or any of the recommendations 27. The Speaker of the Lok Sabha can be removed by
of the Rajya Sabha. Hence Assertion (A) is false. (a) A resolution of the Lok Sabha passed by a
Money Bill could only be introduced in Lok Sabha. majority of the Members of the House.
Therefore Reason (R) is true. (b) Two third majority of the members of the
House.
25. At which of the following stages general
(c) Order of the President.
discussion on a Bill takes place in the Lok
Sabha? (d) Simple majority of the Parliament.
(a) During introduction of the Bill UPPCS (Main) G.S. IInd Paper 2008
(b) During second reading Ans. (a) Kindly refer the explanation of the above
question.
(c) During report stage
(d) During third reading 28. To whom does the Speaker of the Lok Sabha
address to resign from the office?
UPPCS (Main) G.S. IInd Paper, 2006
(a) The President
Ans. (b) A Bill undergoes three readings in each House
(b) The Prime Minister
before it is submitted to the President for assent.
The first reading refers to motion for leave to introduce (c) The Deputy Speaker of the Lok Sabha
a Bill in the House on the adoption of which the Bill is (d) The Chief Justice of India
introduced. The second reading constitutes discussion UPPCS (Main) G.S. IInd Paper 2014
on the principles of the Bill and its provisions and the UPPCS (Main) G.S. IInd Paper 2008
third reading refers to the discussion on the motion that Ans. (c) Kindly refer the explanation of the above
the Bill, as amended, be passed. question.

Indian Polity & Constitution 101 YCT


29. Which one of the following presides the joint 33. Which of the following doesn’t take oath of the
sitting of the Parliament? office?
(a) President (a) President (b) Vice- President
(b) Speaker of the Rajya Sabha (c) Speaker (d) Prime Minister
(c) Speaker of the Lok Sabha (e) Members of the Council of Ministers
(d) None of the above Chattisgarh PSC (Pre) G.S. 2017
UPPCS (Pre.) G.S., 2014 Ans. (c) Among the given options, the Speaker does not
Ans. (c) As per the Article 118(3) of the Constitution, take oath of the office of Speaker. He/She takes oath as
the joint sitting of the Parliament shall be presided by a member of the Parliament.
the Speaker of the Lok Sabha. 34. The speaker of the Lok Sabha makes use of his
30. The function of the Pro-temp. Speaker is to casting vote only?
(a) Conduct the proceeding of the House in the (a) To save the existing government.
absence of Speaker (b) In Case of constitutional Amendment.
(b) Swear in members (c) In case of emergency.
(c) Officiate as Speaker when the Speaker is (d) In case of a tie when votes are equally
unlikely to be elected divided.
(d) Only check if the election certificates of the
UPPCS (Main) Spl. G.S. IInd Paper 2008
members are in order.
Ans. (d) Speaker can cast his vote when the
UPPCS (Pre.) G.S., 2010
Government and opposition get equal votes on a
Ans. (c) When the Lok Sabha and Legislative
particular.
Assemblies have been elected but the vote for the
35. What is the quorum to constitute a meeting of
Speaker and Deputy Speaker has not taken place, the
Pro-Tem Speaker is chosen for the conduct of the Lok Sabha?
house. The appointment has to be approved by the (a) One sixth of the total members of the House
President. Usually, the senior most member of the house (b) One tenth of the total members of the House
is selected for the post. The main duty of the Pro-Tem (c) One fourth of the total members of the House
Speaker is to administer the oath of office to new (d) One third of the total members of the House
members of the house. He also enables the House to UPPCS (Pre) G.S, 2017
elect the new Speaker. Once the new Speaker is elected,
Ans. (b) According to Article 100 of the Constituition,
the office of the pro tem speaker ceases to exist. the quorum to constitute a meeting of either House of
31. Who is generally appointed as a Protem Parliament is one-tenth (1/10) of the total number of
Speaker? members of the House. If at any time during a meeting of
(a) The speaker of the outgoing Lok Sabha a House, there is no quorum, it shall be duty of the
(b) The Dy.Speaker of the outgoing Lok Sabha Chairman or person acting as such, either to adjourn the
(c) One of the senior most members of the newly House or to suspend the meeting until there is a quorum.
elected Lok Sabha 36. In the case of election to the Lok Sabha, the
(d) The leader of the opposition party in the amount of Security deposited for general
outgoing Lok Sabha category candidates and SC/ST category
UPPCS (Pre) G.S, 2017 candidates respectively is
Ans. (c) Kindly refer the explanation of the above (a) Rs 5,000 and Rs 2,500
question. (b) Rs 10,000 and Rs 2,500
32. Pro-Temp. Speaker has been appointed by- (c) Rs 10,000 and Rs 5,000
(a) President of India (d) Rs 15,000 and Rs 7,500
(b) Vice-President of India IAS (Pre) G.S., 2002
(c) Prime Minister of India Ans. (*) During the time when question was asked,
(d) Chief Justice of India security deposit for general candidates in case of Lok
Uttarakhand RO/ARO (M) 2016 Sabha election was Rs 10,000 and for scheduled caste
Ans. (a) Kindly refer the explanation of the above and scheduled tribes was Rs 5,000. At present, according
question. to Representation of the People Act, 1951, for Lok Sabha
Indian Polity & Constitution 102 YCT
election, security deposit for general candidates is Rs Legislature of a State shall be transacted in the
25,000 and for scheduled caste and scheduled tribes languages of the State or in hindi or in english provided
candidates, security deposite is Rs 12,500. For Assembly that the Speaker of the Legislative Assembly or
elections, security deposite for general candidates is Rs Chairman of the Legislative Council may permit any
10,000 and for scheduled caste and scheduled tribes member who cannot adequately express himself in any
candidates, security deposite is Rs 5,000. of the languages aforesaid to address the House in
37. Who appoints the chairman of Public Accounts his/her mother-tongue.
Committee? 40. Under which provision can the Speaker allow
(a) Speaker of the Lok Sabha any member of the House to speak in his/her
(b) Prime Minister mother language?
(c) President of India (a) Article 110 (b) Article 122
(d) Chairman of the Rajya Sabha (c) Article 120(1) (d) Article 126
Himanchal PSC (Pre) GS, 2016
UP Lower (Mains) G.S., 2013
Ans. (c) Kindly refer the explanation of the above
Ans. (a) The Public Accounts Committee examines the
question.
expenditure of the public money by the Government. It
examines the amount granted by the Parliament and the 41. Which of the following are correct about the
amount actually spent. It consists of 22 members, 15 Rajya Sabha?
from the Lok Sabha and 7 from the Rajya Sabha. A 1. It is not subject to dissolution.
minister cannot be elected as a member of the Public 2. It has a term of five years.
Accounts Committee. The chairman of the Public 3. One third of its members retire after every
Accounts Committee is appointed by the Speaker of the two years.
Lok Sabha from the opposition as a convention. 4. Its member shall not be less than 25 years of
age.
38. Who nominates the chairman of the public
accounts Committee of the Indian Parliament? Select the correct answer using the codes given
below:
(a) The Prime Minister
Codes:
(b) The Speaker of the House of People
(a) 1, 2 and 3 (b) 2, 3 and 4
(c) Minister of Parliament Affairs
(c) 1 and 3 (d) 2 and 4
(d) Committee of Parliamentary Affairs
UPPCS (Pre.) G.S.,2014
TNPSC (Pre) G.S. 2016
Ans. (c) Rajya Sabha is a permanent House and is not
Ans. (b) Kindly refer the explanation of the above subject to dissolution. However one-third Members of
question. Rajya Sabha retire after every second year. A member
39. Which of the following languages have been who is elected for a full term serves for a period of six
declared by the Constitution to be the language years. According to Article 84(b) of the Constitution,
for conducting business in Parliament? minimum age required for the election of Rajya Sabha
(a) English is 30 years. Hence statement 1 and 3 are correct.
(b) Hindi 42. The Rajya Sabha can be dissolved by
(c) English and Hindi (a) The President
(d) English, Hindi and mother tongue (b) The Prime Minister
Himanchal PSC (Pre) G.S, 2013 (c) The Council of Minister
Ans. (d) Article 120 of the Constituion is regarding (d) None of the above
language to be used in Parliament. According to Article UPPCS (Main) G.S. IInd Paper 2012
120 (1) of the Constitution, business in Parliament shall Ans. (d) Rajya Sabha is a permanent House and is not
be transacted in hindi or in english provided that the subject to dissolution.
Chairman of the Rajya Sabha or Speaker of the Lok 43. What is the minimum age to be a Member of
Sabha may permit any member who cannot adequately Parliament?
express himself in hindi or in english to address the (a) 18 years (b) 21 years
House in his/her mother-tongue. Article 210 of the (c) 25 years (d) 30 years
Constitution is regarding language to be used in State (e) More than one of the above
Legislature which asserts that the business in the
BPSC (Pre) G.S. 2017
Indian Polity & Constitution 103 YCT
Ans. (e) Article 84 of the Constitution is regarding the Ans. (a) According to Article 64 of the Constitution, the
qualification for membership of Parliament. Article Vice-President of India is ex-officio Chairman of Rajya
84(b) asserts that in the case of a seat in the Rajya Sabha. Rajya Sabha also chooses amongst its members,
Sabha, not less than thirty years of age and in the case a Deputy Chairman.
of a seat in the Lok Sabha, not less than twenty five
48. Chairman of which House is not its member?
years of age.
(a) Rajya Sabha
44. What is the definite number for the Members
(b) Lok Sabha
of Rajya Sabha?
(c) Legislative Assembly
(a) 220 (b) 230
(c) 210 (d) 250 (d) Legislative Council
MPPSC (Pre) GS 2010 UPPCS (Pre.) G.S., 1992
Ans. (d) Article 80 of the Constitution lays down the UPUDA/LDA (Main) G.S.,2010
maximum strength of Rajya Sabha as 250, out of which BPSC (Pre) 2003, 2007
12 members are nominated by the President and 238 are Ans. (a) The Vice-President of India is ex-
representatives of the States and of the two Union officio Chairman of Rajya Sabha. He is not the member
Territories. The members nominated by the President of the Rajya Sabha.
are persons having special knowledge or practical
49. Consider the following statements:
experience in respect of such matters as literature,
1. The Rajya Sabha has no power either to reject
science, art and social service. However the present
or to amend a Money Bill.
strength of Rajya Sabha is 245, out of which 233 are
representatives of the States and Union territories of 2. The Rajya Sabha cannot vote on the Demands
Delhi and Puducherry and 12 are nominated by the for Grants.
President. 3. The Rajya Sabha cannot discuss the Annual
45. In which year was Rajya Sabha constituted for Financial Statement.
the first time? Which of the statements given above is / are
(a) 1950 (b) 1951 correct?
(c) 1952 (d) 1953 (a) 1 only (b) 1 and 2 only
UPPCS (Pre) G.S, 2017 (c) 2 and 3 only (d) 1, 2 and 3
Ans. (c) Rajya Sabha was constituted on 3rd April, IAS (Pre) G.S., 2015
1952. Rajya Sabha met for the first time on 13th May, Ans. (b) According to Article 109, a Money Bill shall
1952. Dr. S. Radhakrisnan was the first Chairman of the not be introduced in the Council of States (Rajya Sabha)
Rajya Sabha and S.V.Krishnamoorthy Rao was the first and after Money Bill is passed by Lok Sabha and
Deputy Chaiman of the Rajya Sabha. On 23rd August, transmitted to Rajya Sabha, it shall be returned to Lok
1954, the Chairman, Rajya Sabha made an
Sabha with its recommendations within fourteen days
announcement in the House that the Council of States
and Lok Sabha may accept or reject recommendations
would now be called ‘Rajya Sabha’ in Hindi.
provided by Rajya Sabha. Hence statement 1 is correct.
46. The Upper house of Indian Parliament, Rajya Rajya Sabha does not vote on demands for grants of
Sabha held its first seassion on:
various Ministries, a matter exclusively reserved for
(a) 26 January, 1951
Lok Sabha. No money however can be withdrawn from
(b) 28 January, 1950 the Consolidated Fund of India unless the Appropriation
(c) 13 May, 1952 Bill has been passed by both the Houses. Therefor
(d) None of the above statement 2 is also correct. Annual Financial Statement
UPPCS ACF-RFO Mains (IInd Paper), 2019 (Budget) shall be presented to Rajya Sabha on such day
Ans. (c) Kindly refer the explanation of the above as the President may direct. There shall be no discussion
question. on the Budget on the day on which it is presented to the
47. Officiating Chairman of the Rajya Sabha is Rajya Sabha. On the day to be appointed by the
(a) Vice-President (b) President Chairman of the Rajya Sabha, the Rajya Sabha has the
(c) Prime Minister (d) Speaker liberty to discuss the Budget as whole but no motion
nd
UPPCS (Main) G.S. II Paper 2011 shall be moved nor shall the Budget be submitted to
BPSC (Pre) G.S.2017-18 vote of the Rajya Sabha. Hence statement 3 is incorrect.
Indian Polity & Constitution 104 YCT
50. Which of the following special powers have Ans. (a) Kindly refer the explanation of the above
been conferred on the Rajya Sabha by the question.
Constitution of India?
53. A resolution passed under clause (1) of Article
(a) To change the existing territory of a State and
249 shall remain in force for a period not
to change the name of a State
exceeding
(b) To pass a resolution empowering the
Parliament to make laws in the State List and (a) One month (b) Three months
to create one or more All India Services (c) Six months (d) One year
(c) To amend the election procedure of the UPPCS (Pre.) G.S., 2016
President and to determine the pension of the Ans. (d) Article 249(1) of the Constitution asserts that if
President after his/her retirement the Rajya Sabha has declared by resolution supported
(d) To determine the functions of the Election by not less than two thirds of the members present and
Commission and to determine the number of
voting that it is necessary or expedient in national
Election Commissioners
interest that Parliament should make laws with respect
IAS (Pre) G.S., 2012
to any matter enumerated in the State List specified in
Ans. (b) Rajya Sabha being a federal chamber enjoys
the resolution, it shall be lawful for Parliament to make
certain special powers under the Constitution. Rajya
laws for the whole or any part of the territory of India
Sabha through resolution empowers the Parliament to
make laws enumerated in the State List, Rajya Sabha with respect to that matter while the resolution remains
could create one or more All India Services. Rajya in force. Article 249(2) further asserts that a resolution
Sabha enjoys a special power regarding emergency passed under clause (1) shall remain in force for such
provisions. If an emergency proclamation is issued at a period not exceeding one year.
time when Lok Sabha has been dissolved or the 54. Which of the following statements is/are
dissolution of Lok Sabha takes place within the period correct?
allowed for its approval, then the proclamation remains
(1) Only the Rajya Sabha and not Lok Sabha can
effective, if the resolution approving emergency is
have nominated members.
passed by Rajya Sabha within the period specified in
the Constitution. (2) There is a constitutional provision for
nominating two members belonging to the
51. The council of States has exclusive powers in
relation to which of the following? Anglo-Indian Community to the Rajya Sabha.
(a) To initiate proceedings for the removal of the (3) There is no constitutional bar for nominated
President members to be appointed as Union Ministers.
(b) To initiate proceedings for the removal of a (4) A nominated member can vote both in
judges of the Supreme Court Presidential and Vice-Presidential elections.
(c) To recommend creation of a new All India (a) 1 and 2 (b) 3 and 4
Service (c) Only 2 (d) Only 3
(d) None of the above
Maharashtra PSC (Pre) G.S 2018
UPPCS (Pre) G.S, 2018
IAS (Pre) G.S., 2003
Ans: (c) Kindly refer the explanation of the above
Ans. (d) Two members from Anglo Indian community
question.
are nominated by the President to the Lok Sabha hence
52. Consider the following statements:
statement 1 is incorrect.12 members are nominated to the
1. The Rajya Sabha alone has the power to
Rajya Sabha by the President having special knowledge
declare that it would be in national interest for
the Parliament to legislate with respect to a or practical experience in respect of such matters as
matter in the State List. literature, science, art and social service. Therefore
2. Resolution approving the Proclamation of statement 2 is incorrect. There is no constitutional bar for
Emergency is passed only by the Lok Sabha. nominated members to be appointed as Union Minister
Which of the statements given above is/are but it has not been implemented till date. Hence
correct? statement 3 is correct. Nominated members are not
(a) 1 only (b) 2 only entitled to vote in the election of the President but in the
(c) Both 1 and 2 (d) Neither 1 nor 2 election of the Vice President, they have right to vote.
IAS (Pre) G.S.,2016 Hence statement 4 is incorrect.

Indian Polity & Constitution 105 YCT


17.
Prime Minister and Council
of Ministers
1. Out of the following statements, choose the one 3. If the Prime Minister of India belonged to
that brings out the principle underlying the Upper House of the Parliament
Cabinet form of Government : (a) He will not be able to vote in his favour in the
(a) An arrangement for minimizing the criticism event of the no-confidence motion
against the Government whose (b) He will not be able to speak on the Budget in
the Lower House
responsibilities are complex and hard to carry
(c) He can make statements only on the Upper
out to the satisfaction of all.
House
(b) A mechanism for speeding up the activities of (d) He has to become a member of the Lower
the Government whose responsibilities are House within six months after being sworn-in
increasing day by day. as the Prime Minister
(c) A mechanism of parliamentary democracy for IAS (Pre) G.S., 1997
ensuring collective responsibility of the Jharkhand PSC (Pre) G.S., 2008
Government to the people. Ans. (a) No confidence motion could only be moved in
(d) A device for strengthening the hands of the Lok Sabha hence if the Prime Minister is a member of
head of the Government whose hold over the the Upper House (Rajya Sabha), he won’t be able to
people is in a state of decline. cast his vote in case of no-confidence motion. Prime
Minister has power to speak at both the Houses of
IAS (Pre) G.S, 2017
Parliament. As per Article 75(5) of the Constitution,
Ans. (c) In order to insure the collective responsibility Prime Minster has to become a member of either House
of Government towards people, principle of Cabinet of the Parliament within six months after being sworn-
form of Government has been laid down in in as the Prime Minister.
Parliamentary form of democracy. According to Article 4. Consider the following statements in the
75 (3) of the Constituion, the Council of Ministers is context of Indian Constitution:
collectively responsible to the House of the People (Lok 1. The Union Council of Ministers shall be
Sabha). collectively responsible to the Lok Sabha
2. The Prime Minister of India, at the time of 2. Union Minister shall hold office during the
pleasure of the Prime Minister
his/her appointment
Of these:
(a) Need not necessarily be a member of one of (a) Only 1 is correct
the Houses of the Parliament but must (b) Both 1 and 2 are correct
become a member of one of the Houses (c) Neither 1 nor 2 is correct
within six months (d) Only 2 is correct
(b) Need not necessarily be a member of one of UP RO/ARO (Pre) 2016
the Houses of the Parliament but must Ans. (a) According to Article 75(3) of the Constitution,
become a member of the Lok Sabha within the Council of Ministers shall be collectively
six months responsible to the House of the People (Lok Sabha).
(c) Must be a member of one of the Houses of Hence statement 1 is correct. Article 75(2) asserts that
the Parliament the Minister shall hold office during the pleasure of the
(d) Must be a member of the Lok Sabha President. Hence statement 2 is incorrect.
IAS (Pre) G.S., 2012 5. Which one of the following is not explicitly in
the Constitution of India but followed as
Ans. (a) According to Article 75(5) of the Constitution, convention?
the Prime Minister of India, at the time of his/her (a) The Finance Minister is to be member of the
appointment need not necessarily be a member of one Lower House
of the Houses of the Parliament but must become a (b) Prime Minister has to resign if he loses
member of one of the Houses within six months. majority in the Lower House
Indian Polity & Constitution 106 YCT
(c) All parts of India are to be represented in the under these rules. Each of the Ministries is assigned to a
Council of Ministers Minister by the President on the advice of the Prime
(d) In the event of both President and Vice Minister.
President demitting office simultaneously, 8. Consider the following statements and select
prior to their tenure the Speaker of the Lower the correct answer from the code given below:
House of the Parliament will officiate as 1. The executive power of the Union vests in the
President Prime Minister.
IAS (Pre) G.S.,1995 2. The Prime Minister is appointed by the
Ans. (b) The Constitution does not mention either a President.
confidence motion or a no-confidence motion. Article 3. The Prime Minister is the head of the Council
75 does specify that the Council of Ministers shall be of Ministers.
collectively responsible to the House of the People (Lok 4. The Prime Minister, at the time of his
Sabha). This implies that the majority of Lok Sabha appointment, need not be a member of either
MPs must not be against the Prime Minister and his House of Parliament
Cabinet. No-confidence motion could only be Codes:
introduced to Lok Sabha. (a) Only 1, 2 and 4 are correct.
6. Consider the following statements (b) Only 1, 2 and 3 are correct.
1. The Executive Power of the Union of India is (c) Only 2, 3 and 4 are correct.
vested in the Prime Minister. (d) Only 1, 3 and 4 are correct.
2. The Prime Minister is the ex officio UPPCS (Pre) G.S, 2004
Chairman of the Civil Services Board. Ans. (c) The executive power of the Union vests in the
Which of the statements given above is / are President of India. Rest all the given options are correct.
correct?
9. Consider the following statements:
(a) 1 only (b) 2 only
1. The Council of Ministers in the Centre shall
(c) Both 1 and 2 (d) Neither 1 nor 2 be collectively responsible to the Parliament.
IAS (Pre) G.S., 2015 2. The Union Ministers shall hold the office
Ans. (d) According to Article 53(1) of the Constitution, during the pleasure of the President of India.
the executive power of the Union shall be vested in the 3. The Prime Minister shall communicate to the
President and shall be exercised by him either directly President about the proposals for Legislation.
or through officers subordinates to him in accordance Which of the Statements given above is/are
with the Constitution. Hence statement 1 is incorrect. correct?
Cabinet Secretary is the Ex officio Chairman of the (a) 1 only (b) 2 and 3 only
Civil Services Board. Hence statement 2 is also
(c) 1 and 3 only (d) 1, 2 and 3
incorrect.
IAS (Pre) G.S., 2013
7. With reference to Union Government consider
Ans. (b) According to Article 75(3) of the Constitution,
the following statements:
the Council of Ministers shall be collectively
1. The Ministries and Departments of the responsible to the House of the People (Lok Sabha) not
Government of India are created by the Prime the Parliament hence statement 1 is incorrect. Article
Minister on the advice of the Cabinet 75(2) of the Constitution asserts that the Minister shall
Secretary. hold office during the pleasure of the President. Hence
2. Each of the Ministries is assigned to a statement 2 is correct. According to Article 78 of the
Minister by the President of India on the Constitution, it shall be the duty of the Prime Minister
advice of the Prime Minister. to communicate all decisions of the Council of
Which of the statements given above is/ are Ministers relating to the administration of the affairs of
correct? the Union and proposals for legislation to the President.
(a) 1 only (b) 2 only Hence statement 3 is also correct.
(c) Both 1 and 2 (d) Neither 1 nor 2 10. With reference to Union Government, consider
IAS (Pre) G.S., 2009 the following statements:
Ans. (b) The Government of India (Allocation of 1. The Constitution of India provides that all
Business) Rules, 1961 is made by the President of India Cabinet Ministers shall be compulsorily the
under Article 77 of the Constitution for the allocation of sitting members of Lok Sabha only.
business of the Government of India. The 2. The Union Cabinet Secretariat operates under
Ministries/Departments of the Government are created the direction of the Ministry of Parliamentary
by the President on the advice of the Prime Minister Affairs.
Indian Polity & Constitution 107 YCT
Which of the statements given above is/ are Which of the above statements are correct?
correct? (a) 1, and 2 only (b) 1, 2 and 3 only
(a) 1 only (b) 2 only (c) 1, 3 and 4 only (d) 1, 2, 3 and 4
(c) Both 1 and 2 (d) Neither 1 nor 2 UPPCS (Main) G.S., IInd Paper 2012
IAS (Pre) G.S., 2009 Ans. (b) According to Article 75(3) of the Constitution,
Ans. (d) It is the prerogative of the Prime Minister to the Council of Ministers headed by the Prime Minister
select the members of his cabinet and on the advice of the shall be collectively responsible to the House of the
People (Lok Sabha). Where as Article 74 is related with
Prime Minister; the President appoints the Council of
the Council of Ministers to aid and advice President.
Ministers. Article 75(5) asserts that the minister has to
become a member of either House of the Parliament 14. The Prime Minister of India is responsible to
whom?
within six months after being sworn-in as the minister.
(a) The Cabinet (b) The President
Hence statement 1 is incorrect. The Union Cabinet
Secretariat functions directly under the Prime Minister. (c) The Lok Sabha (d) The Rajya Sabha
The administrative head of the Secretariat is the Cabinet (e) None of the above
Secretary who is also the ex-officio Chairman of the Civil BPSC (Pre) G.S. 2017
Services Board. Hence statement 2 is also incorrect. Ans. (c) Kindly refer the explanation of the above
question.
11. A person can remain a member of the Council
of Ministers without being a member of the 15. Which one of the following is not a correct
Parliament for situation of the status of a Central Council of
Ministers at the time of their resignation?
(a) Three years (b) Two years
(a) President's rule will be imposed.
(c) One years (d) Six months
(b) The president will ask to continue till
UPPCS (Pre.) G.S., 1993, 1995 alternative arrangement.
UPPCs (Main) G.S. IInd Paper 2015 (c) Alternative arrangement means earliest
BPSC (Pre) 1994 possible general election to be held to form a
Ans. (d) Kindly refer the explanation of the above new Government.
question. (d) Outgoing Council of Ministers may have to
12. Assertion (A) The Council of Ministers in the hold charge till the formation of new
Government.
Union of India is collectively responsible both to
the lok Sabha and the Rajya Sabha. UPPCS (Main) G.S. IInd Paper 2010
Reason(R) The Member of both the Lok Sabha Ans. (a) When Union Council of Ministers resigns; the
President will ask to continue till alternative
and the Rajya Sabha are eligible to be the
arrangement has been made which means earliest
Ministers of the Union Government.
possible general election to be held to form a new
Code: Government. Further, outgoing Council of Ministers
(a) Both A and R are individually true and R is may have to hold charge till the formation of new
the correct explanation of A. Government. President rule can be proclaimed under
(b) Both A and R are individually true but R is Article 356 in case of failure of constitutional
not the correct explanation of A. Machinery in state.
(c) A is True but R is false 16. With reference to Prime Minister’s Office
(d) A is false but R is true (PMO) which one of the statements is/are
IAS (Pre) G.S., 2007 correct?
1. It came into existence in 1977
Ans. (d) According to Article 75(3) of the Constitution,
2. It is headed by the Cabinet Secretary
the Council of Ministers shall be collectively
responsible to the House of the People (Lok Sabha). Select the correct answer from the codes given
below:
Hence assertion (A) is false. Members of both Lok
Sabha and Rajya Sabha are eligible to be the Ministers Codes:
(a) Only 2 (b) Both 1 and 2
of the Union Government. Hence reason (R) is true.
(c) Only 1 (d) Neither 1 nor 2
13. The Council of Ministers is collectively
UPPCS (Pre) G.S, 2019
responsible:
Ans: (c) Prime Minister’s Office (PMO) is an extra-
1. To the House of the People
constitutional body that has not been mentioned in the
2. Under a Constitutional obligation Indian Constitution. It was given the status of a
3. As per Article 75 (3) department under the Government of India Allocation
4. As per Article 74 (3) of Bussiness Rule, 1961. Since June, 1977, it is known
Indian Polity & Constitution 108 YCT
as Prime Minister’s Office (PMO) and is headed by the 20. Which of the following statements is correct
Secretary to the Prime Minister, designated as Principal about prime Minister of India?
Secretary to the Prime Minister. (a) Prime Minister is the nominal head of the
17. Which of the following is/are the Council of Ministers
function/functions of the Cabinet Secretariat?
(b) He is not responsible to the Parliament.
1.Preparation of agenda for Cabinet Meetings
(c) He cannot dismiss any of his ministers.
2.Secretarial assistance to Cabinet Committees
(d) Prime Minister is the real head of the Council
3.Allocation of financial resources to the
Ministries of Ministers.
Select the correct answer using the code given UPPCS (Main) G.S. IInd Paper 2013
below: Ans. (d) Article 74(1) of the Constitution provides that
(a) 1 only (b) 2 and 3 only there shall be a Council of Ministers with the Prime
(c) 1 and 2 only (d) 1, 2 and 3 Minister as its head to aid and advice the President, who
IAS (Pre) G.S., 2014 shall exercise his/her functions in accordance to the
Ans. (c) The Cabinet Secretariat is under the direct advice. The real executive power is thus vested in the
charge of the Prime Minister. The administrative head Council of Ministers with the Prime Minister as its head.
of the Secretariat is the Cabinet Secretary who is also 21. In India, which one among the following
the ex-officio Chairman of the Civil Services Board.
formulates the fiscal policy?
The functions of Cabinet Secretariat are assistance to
Cabinet, support to Cabinet Committees and promotion (a) Planning Commission
of inter-ministerial coordination. Allocation of financial (b) Finance Commission
resources to the Ministries is being done by Ministry of (c) Ministry of Finance
Finance. (d) Reserve Bank of India
18. Department of Border Management is a UPPCS (Main) G.S. IInd Paper 2012
Department of which one of the following
Ans. (c) Fiscal policy is the means by which a
Union Ministries?
Government adjusts its spending levels and tax rates to
(a) Ministry of Defence
monitor and influence nation’s economy. In India, fiscal
(b) Ministry Of Home Affairs
policy is formulated by Ministry of Finance.
(c) Ministry of Shipping, Road Transport and
Highways 22. Which one or the following expenditure is not
(d) Ministry of Environment and Forests charged on the Consolidated Fund of India?
IAS (Pre) G.S., 2008 (a) Salary and allowances of the Chief Justice of
Ans. (b) Department of Border Management works India
under Union Ministry of Homes which exclusively (b) Salary and allowances of the Comptroller
secure the borders and create infrastructure in the border and Auditor-General of India
areas of country such as Indo-Pakistan, Indo- (c) Salary and allowances of the Prime Minister
Bangladesh, Indo- China, Indo-Nepal and Indo- Bhutan of India.
borders. Activities include construction of fence,
floodlighting, roads, Border Out Posts (BOP) and (d) Salary and allowances of the Chairman of the
Company Operating Bases (BOP’s) etc. Union Public Service Commission.
19. The Office of the Deputy Prime Minister UPPCS (Main) G.S. IInd Paper 2011
(a) Was created under the original Constitution. Ans. (c) Salary and allowances of the Prime Minister of
(b) Is an extra-constitutional growth. India is not charged upon Consolidated Fund of India.
(c) Was created by 44th Amendment. 23. Which one of the following terms does not
(d) Was created by the 85th Amendment. appear in the Constitution of India?
UPPCS (Main) G.S.IInd Paper 2009 (a) Annual Financial Statement
Ans. (b) The Office of the Deputy Prime Minister of (b) Appropriation Bill
India is not a Constitutional Office but a political (c) Budget
compulsion. The first Deputy Prime Minister of India
(d) Consolidated Fund of India
was Sardar Vallabhbhai Patel, who was also home
minister in Jawaharlal Nehru's cabinet. The office has UPPCS (Main) G.S. IInd Paper 2011
since been only intermittently occupied. The seventh Ans. (c) Under Article 112 of the Constitution, ‘Annual
and last Deputy Prime Minister till date was L. K. Financial Statement’ has been mentioned which is
Advani, who took on the role in addition to Minister of usually called as budget. Appropriation Bill is
Homes from 2002 to 2004 in Atal Bihari Vajpayee's mentioned in Article 114 and Consolidated Fund of
Government. India has been mentioned in Article 266.
Indian Polity & Constitution 109 YCT
18.
Parliamentary Committees
1. Consider the following statements : 2. Consider the following statements:
1. While members of the Rajya Sabha are 1. The Chairman of the Committee on public
associated with Committees on Public Accounts is appointed by the Speaker of the
Accounts and Public Undertakings, members Lok Sabha.
of Committee on Estimates are drawn entirely 2. The Committee on public Accounts
from Lok Sabha. comprises Members of Lok Sabha, Members
2. The Ministry of Parliamentary Affairs works of Rajya Sabha and a few eminent persons of
under the overall direction of Cabinet industry and trade.
Committee on parliamentary Affairs. Which of the statements given above is/are
3. The Minister of Parliamentary Affairs correct?
nominates Members of parliament on (a) 1 only (b) 2 only
Committees, Councils, Boards and (c) Both 1 and 2 (d) Neither 1 nor 2
Commissions etc. set up by the Government IAS (Pre) G.S., 2007
of India in the various ministries.
Ans. (a) Kindly refer the explanation of the above
Which of these statements is/are correct? question.
(a) 1 and 2 (b) 2 and 3
3. Who nominates the Chairman of the Public
(c) 1 and 3 (d) 1, 2 and 3 Accounts Committee of Indian Parliament?
IAS (Pre) G.S., 2003 (a) The Speaker of Lok Sabha
Ans. (d) The Committee on Public Accounts is the (b) The Prime Minister
oldest parliamentary committee and was first (c) The President
constituted in 1921. The committee consists of 22
(d) The Chairman of Rajya Sabha
members, 15 members are elected from Lok Sabha and
UPPCS (Pre.) G.S., 2014
7 members of the Rajya Sabha are associated with
UPUDA/LDA (Pre.) G.S., 2006
it. The Speaker is empowered to appoint the Chairman
of the Committee from amongst its members. The Ans. (a) Kindly refer the explanation of the above
Committee on Public Accounts is constituted by the question.
Parliament each year. Estimates Committee is the 4. Tenure of the members of the estimate
largest parliamentary committee consisting of 30 committee is-
members elected every year by the Lok Sabha from (a) 2 years (b) 1 years
amongst its members to examine the Budget estimates (c) 3 years (d) 4 years
of the Union Government. No minister shall be UPPCS (Main) G.S. IInd Paper, 2016
appointed as a member of Estimates Committee. Ans. (b) Kindly refer the explanation of the above
Ministry of Parliamentary affairs is entrusted with the question.
responsibility of coordinating, planning and arranging
5. Which one of the following is the largest
Government business in both the Houses of Parliament.
Committee of the Parliament?
For this purpose, the ministry works under the overall
(a) The Committee on Public Accounts
direction of Cabinet Committee on Parliamentary
(b) The Committee on Estimates
Affairs. The Minister of Parliamentary Affairs
nominates Members of Parliament on committees, (c) The Committee on Public Undertakings
councils, boards and commissions etc set up by the (d) The Committee on Petitions.
Government in various ministries. Therefore all the IAS (Pre) G.S.,2014
statements are correct. UPPCS (Main) G.S. IInd 2017
Indian Polity & Constitution 110 YCT
Ans. (b) The Committee on Estimates (30 members) is These committees are Public Accounts Committee
the largest committee of the Parliament. All the 30 (PAC), Committee on Public Undertaking (COPU),
members of this Committee from Lok Sabha only. The Estimates Committee (EC) and Departmentally Related
Rajya Sabha has no representation in this committee. Standing Committees (DRSC).
6. Which is the largest Committee of the Indian 10. The members of Standing Committee of
Parliament? Parliament are taken from the Lok Sabha and
(a) Public Accounts Committee Rajya Sabha in which ratio?
(b) Estimates Committee (a) Two and one respectively.
(c) Committee on Public Undertakings (b) Three and one respectively.
(d) Committee on Petitions (c) Four and one respectively.
UPPCS (Pre) G.S 2020 (d) In equal numbers from both the Houses
Ans. (b) Kindly refer the explanation of the above Haryana PSC (Pre) G.S. 2014
question. UPPCS (Main) G.S. IInd 2013
7. The Members of the Rajya Sabha cannot be Ans. (a) The proportion of number of members of the
attached to- two Houses on Standing Committee is approximately in
1. Public Accounts committee the ratio of 2 Members of Lok Sabha to 1 Member of
Rajya Sabha.
2. Estimates committee
11. Consider the following statements: The
3. Public sector enterprise committee
Parliamentary Committee on Public Accounts
Select answer from the given code:
1. Consist of not more than 25 Members of the
(a) 1 and 2
Lok Sabha
(b) 3
2. Scrutinizes appropriation finance accounts of
(c) 2 Government
(d) 1 and 3 3. Of the Auditor examines the report
UPPCS (Main) Spl. G.S. IInd 2008 Comptroller and General of India
Ans. (c) Kindly refer the explanation of the above Which of the statements given a above is / are
question. correct?
8. Estimates Committee is constituted from the (a) 1 only
members of (b) 2 and 3 only
(a) Both the Houses (b) The Rajya Sabha (e) 3 only
(c) The Lok Sabha (d) None of these (d) 1, 2 and 3
nd
UPPCS (Main) G.S. II Paper 2010 IAS (Pre) G.S., 2013
Ans. (c) Kindly refer the explanation of the above Ans. (b) The Public Accounts Committee consists of 22
question. members, 15 members are elected from Lok Sabha and
7 members of the Rajya Sabha are associated with
9. In what way does the Indian Parliament
it. The Committee on Public Accounts scrutinizes the
exercise control over the administration?
Appropriation Accounts of the Government of India and
(a) Through Consultative committees of various
the reports of the Comptroller and Auditor General of
ministries India.
(b) Through Parliamentary committee
12. Who amongst the following attends the
(c) By making the administrators send periodic Meetings of the Public Accounts Committee of
reports the Parliament?
(d) By compelling the executive to issue writs (a) Attorney General of India
IAS (Pre) G.S., 2001 (b) Solicitor General of India
Ans. (b) The Parliamentary controls are defined in the (c) Comptroller and Auditor-General of India
Constitution. The operation of controls is ensured (d) None of the above
through committees established by parliamentary rules. UPPCS (Main) G.S. IInd 2011
Indian Polity & Constitution 111 YCT
Ans. (c) The Committee on Public Accounts and other bodies appointed by Central, State and Union
scrutinizes the Appropriation Accounts of the Territory Governments and recommends wehter person
Government of India and the reports of the Comptroller holding these offices should be disqualified from benig
and Auditor General of India. Comptroller and Auditor elected as Member of Parliament or not. Rest of the
General of India (CAG) attend the meeting of the Public given committees are financial committee of the
Accounts Committee of the Parliament. Parliament.

13. The report of Public Accounts Committee is 16. In the Parliament of India, the purpose of an
presented in the adjournment motion is
(a) Lok Sabha (a) To allow a discussion on a definite matter of
urgent public importance
(b) President’s office
(b) To let opposition members collect
(c) Finance Ministry
information from the ministers
(d) Prime Minister’s office
(c) To allow a reduction of specific amount in
UPPCS (Main) G.S. IInd 2015
demand for grant
Himanchal PSC (Pre) 2011
(d) To postpone the proceedings to check the
Ans. (a) The report of Public Accounts Committee is inappropriate or violent behaviour on the part
presented in the Lok Sabha. of some members
14. Which two committees of the Parliament are IAS (Pre) G.S., 2012
described as ‘twin Sister’s’ committees? Ans. (a) The primary object of an adjournment motion
(a) Public Accounts Committee and Estimates is to draw the attention of the House to a recent matter
Committee of urgent public importance having serious
(b) Public Accounts Committee and Committee consequences and with regard to which a motion or a
on Public Undertakings resolution with proper notice will be too late. The
(c) Law Committee and Committee of Delegated matter proposed to be raised should be of such a
Legislation character that something very grave which affects the
(d) Committee on Privileges and Committee on whole country and its security has happened and the
Petitions House is required to pay its attention immediately by
Punjab PSC (Pre) G.S 2018 interrupting the normal business of the House. When
the motion is being discussed, the Speaker has no power
Ans. (a) Public Accounts Committee and Estimate
to adjourn the House for the day. Once the debate on
Committee are called twin-sister committee as Public
adjournment motion starts, it has to be concluded and
Accounts Committee is concerned with the spending of
decision arrived at without interruption of the House.
public funds while Estimate Committee examines the
Rules of procedure and conduct of business in Lok
Budget estimates of the Union Government.
Sabha asserts that "an adjournment motion involves an
15. Which of the following is not a financial
element of census against the Government. In the event
committee of the Parliament?
of an adjournment motion being adopted, the House
(a) Public Accounts Committee automatically stands adjourned."
(b) Estimates committee
17. The provision for Calling Attention Notices has
(c) Committee on Public Undertaking restricted the scope of the following?
(d) Joint committee on offices of profit (a) Short duration discussion
Punjab PSC (Pre) G.S 2015 (b) Question hour
Ans. (d) Joint Committee on Office of Profit is not (c) Adjournment motion
financial committee of the Parliament. This comittee (d) Zero hour
examines the composition and character of committees UPPCS (Pre.) G.S., 2010
Indian Polity & Constitution 112 YCT
Ans. (a) The provision of calling attention notices has Indian economy including inflation, price control, foreign
restricted the scope of the short duration discussion. exchange management, official development assistance,
Calling attention notice is introduced in Parliament by a domestic finance and preparation of the Union Budget,
member to call attention of the minister to a matter of bilateral and multilateral engagement with international
the urgent public importance and seek authentic financial institutions and other countries.
information from him on that matter. In short duration 21. The Consultative Committee of members of
discussion, the Parliament member can raise such Parliament for Railway Zones is constituted by
discussion on the matter of the urgent public the
importance. (a) President of India
18. Which one of the following motions, the (b) Ministry of Railways
Council of Ministers in India can move? (c) Ministry of Parliamentary Affairs
(a) No Confidence Motion (d) Ministry of Transport
(b) Censure Motion UPPCS (Main) G.S. IInd Paper 2015
(c) Adjournment Motion
Ans. (c) The Consultative Committee of Members of
(d) Confidence Motion
Parliament for Railway Zones is constituted by the
UPPCS (Pre.) G.S., 2006 Ministry of Parliamentary Affairs.
UPPCS (Main) G.S., IInd 2010
22. With reference to the Parliament of India, which
Ans. (d) Confidence Motion is moved by the Council of of the following Parliamentary Committees
Ministers to show the majority in the House. scrutinizes and reports to the House whether the
19. Which of the following motions is related to powers to make regulations, rules, sub-rules, by-
Union Budget? laws etc. conferred by the constitution of
(a) Censure Motion delegated by the Parliament are being properly
(b) Calling Attention Motion exercised by the Executive within the scope of
(c) Cut Motion such delegation?
(d) Adjournment Motion (a) Committee on Government Assurances
UPPCS (Pre.) G.S., 2002 (b) Committee on Subordinate Legislation
UP Lower (Pre) Spl. G.S., 2002 (c) Rules Committee
Ans. (c) The Cut Motion is introduced by the members (d) Business Advisory Committee
of the Lok Sabha during the passing of the Budget in IAS (Pre) G.S, 2018
the House. Cut motions are of three types namely Ans. (b) Committee of Subordinate Legislation
Disapproval of policy Cut Motion, Economy Cut examines and reports to the House whether the power to
Motion and Token Cut Motion. make regulation, rules, sub-rules and bye laws
20. Which one of the following is responsible for delegated by parliament or conferred by the constitution
the preparation and presentation of Union to the executive are being properly exercosed by it. This
Budget to the Parliament? committee consists of 15 members in both the houses
(a) Department of Revenue and it was constituted in 1953.
(b) Department of Economic Affairs 23. In India, which of the following review the
(c) Department of Financial Services Independent regulators in sectors like
(d) Department of Expenditure telecommunications, insurance, electricity, etc?
IAS (Pre) G.S., 2010 1. Ad Hoc Committees set up by the
Ans. (b) The Department of Economic Affairs, Ministry Parliament.
of Finance is responsible for advice on economic issues 2. Parliamentary Department Related Standing
having a bearing on internal and external aspects of the Committees
Indian Polity & Constitution 113 YCT
3. Financial commission Sabha to be nominated by that House for being
4. Financial commission Setcor Legislative associated with the Committee. A Minister shall not be
Reforms Commission elected a member of the Committee and that if a
5. NITI Aayog member, after his election to the Committee is
Select the correct answer using the code given appointed a Minister; he/she shall cease to be a member
below. of the Committee from the date of such appointment.
(a) 1 and 2 The term of office of members of the Committee shall
not exceed one year. Government Assurances
(b) 1, 3 and 4
committee consist of 15 members nominated by
(c) 3, 4 and 5
Speaker. Estimates committee consist of 30 members
(d) 2 and 5
who are elected form Lok Sabha every year. For the
IAS (Pre) G.S, 2019
welfare of the minorities, Government has set up a
Ans: (a) In India, parliamentary scrutiny of the National Commission for Minorities under the National
regulators can take place through the following means; Commission for Minorities Act, 1992. Hence it is not a
question hour, discussions in Parliament and parliamentary committee.
parliamentary committees. In India, there are 24
25. Which one of the following is not a Standing
department related Standing Committees that comprise
Committee of Parliament?
members from both Houses of Parliament. These
(a) Committee on Public Accounts
committees are ministry specific and may review the
(b) Estimate Committee
working of regulators within their respective
(c) Committee on Welfare of SCs and STs
departments. Parliament may establish ad-hoc
(d) Committee on Public Undertakings.
committees which may examine the working of
UPPCS (Main) G.S. IInd Paper 2015
regulators. The two parliamentary committees on
Ans.(*) In India, there are two types of Parliamentary
finance which exercise oversight of regulators are; the
Committee namely Ad hoc Committee and Standing
Committee on Estimates and the Public Accounts Committee. Ad hoc Committees are appointed for a
Committee (PAC). The Committee on Estimates specific purpose and they cease to exist when they
reviews budgetary estimates of Government finish the task assigned to them and submit a report.
departments. Such estimates include the budget of Standing Committees are permanent committees. All
regulators. the committees given in the option are Standing
24. On which of the following, a Parliamentary Committee of Parliament.
Committee has not been set up? 26. Which of the following are the Financial
(a) Public Undertakings Committees of Parliament in India?
(b) Government Assurances 1. Public Account Committee
2. Estimates Committee
(c) Estimates
3. Committee on Public Undertakings
(d) Welfare of Minorities
Select the correct answer from the code given
UPPCS (Main) G.S. IInd 2012
below:
Ans. (d) Committee on Public Undertakings examines (a) 1 only
the working of the public undertakings specified in the (b) 1 and 2
Fourth Schedule. The Committee shall consist of not (c) 1 and 3
more than 22 members comprising 15 members who (d) 1, 2 and 3
shall be elected by the Lok Sabha every year from UPPCS (Main) G.S. IInd Paper 2007
amongst its members according to the principle of
Ans. (d) Public Account Committee, Estimates
proportional representation by means of the single Committee and Committee on Public Undertakings are
transferable vote and not more than 7 members of Rajya financial committees of the Parliament of India.
Indian Polity & Constitution 114 YCT
19.
Various Constitutional
Amendments and its procedure
1. Which of the following statements about the the Constitution by the way of addition, variation or
first amendment to the constitution is/are true? repeal any provision in the Constitution as per the
I. The first amendment was enacted in 1952. procedure laid down by the Constitution. An
II. The first amendment was enacted before the Amendment may be initiated by introducing a Bill in
first general elections. either House of the Parliament. Hence statement 1 is
III. It was enacted by provisional Parliament. incorrect. When the Bill is passed in each House by a
majority of the total membership of that House present
(a) I and II are true
and voting, it shall be presented to the President who
(b) II and III are true
shall give his assent to the Bill and thereupon the
(c) I and III are true
Constitution shall stand amended in accordance with the
(d) I, II and III are true
terms of the Bill, provided the Bill is not related to
TNPSC (Pre) G.S. 2019 amend the federal structure of the Constitution. An
Ans. (b) The first Amendment to the Constitution Amendment to change the federal structure of the
carried out in 1951 was related to land and agrarian Constitution requires additional ratification by the
reforms. The First Amendment Act, 1951 inserted Ninth Legislature of not less than one half of the States before
Schedule to the Constitution to protect the land reform presented to the President for assent. Hence statement 2
and other laws present in it from the judicial review. is also incorrect.
Hence statement I is not correct. First general election 3. According to the Indian Constitution, the
in independent India was held between 1951 to 1952. Constitutional Amendment of the following
Hence statement II is correct. The first Amendment to topics requires consent of at least half of the
the Constituion was enacted by provisional Parliament. State Legislatures-
Hence statement III is also correct. 1. Federal provision of the Constitution
2. Consider the following statements 2. Rights of the Supreme Court
1. An amendment to the Constitution of India 3. The process of the Constitutional Amendment
can be initiated by an introduction of a bill in 4. Formation of new States or transformation in
the Lok Sabha only. the borders and names of the States
2. If such an amendment seeks to make changes Code:
in the federal character of the Constitution, (a) 1, 2, 3 (b) 1, 2, 4
the amendment also requires to be ratified by
(c) 1, 3, 4 (d) 2, 3, 4
the legislature of all the States of India.
UP Lower (Pre) G.S, 2003
Which of the statements given above is/are
Ans. (a) Article 368 of the Constitution is regarding
correct?
power and procedure of the Parliament to amend the
(a) 1 only (b) 2 only
Constitution. The various categories of amendment to
(c) Both 1 and 2 (d) Neither 1 nor 2 the Constitution can be summarized as follows:
IAS (Pre) G.S.,2013 Amendment by Simple Majority- As the name
Ans. (d) According to Article 368 of the Constitution, suggests, an Article can be amended in the same way by
Parliament may exercise its constituent power to amend the Parliament as an ordinary law is passed which
Indian Polity & Constitution 115 YCT
requires simple majority. These Articles are specifically Ans. (d) Kindly refer the explanation of the above
excluded from the purview of the procedure prescribed question.
under Article 368. 5. Which of the following parts of the
Amendment by Special Majority- Articles which can Constitution cannot be amended by a simple
be amended by special majority are laid down in Article majority?
368. Amendments in Fundamental Rights, Directive
(a) Creation of a new State
Principles of State Policy come under the procedure of
(b) Process of the election of the President
amendment by special majority. All amendments,
(c) Official language of the Union or States
falling under this category must be affected by a
(d) Provision of Quorum
majority of total membership of each House of
Parliament as well as two third of the members present (e) All of these
and voting. Ans. (b) Kindly refer the explanation of the above
Amendment by Special Majority and Ratification by question.
States- Amendment to certain Articles requires special 6. Creation of a new state requires a majority for
majority as well as ratification by States. Ratification by Constitutional Amendment?
States means that there has to be a resolution to that (a) Simple
effect by atleast one-half of the State legislatures. These (b) Two-third
articles include Article 54 (Election of President), (c) Three-fourth
Article 55 (Manner of election of President), Article 73
(d) Two-thirdplus ratification by half of all states
(Extent of executive power of the Union), Article 162
(e) None of the above/More than one of the
(Extent of executive power of State), Article 124 to
above
Article 147 (The Union Judiciary), Article 214 to
BPSC (Pre) G.S. 2016
Article 231 (The High Courts in the States), Article 241
Chattisgarh PSC (Pre.) 1st Paper, 2012
(High Courts for Union Territories), Article 245 to
Article 255 (Distribution of Legislative powers) and Ans. (a) Kindly refer the explanation of the above
Article 368 (power of the Parliament to amend the question.
Constitution and procedure) itself. Any list of Seventh 7. Consider the following statements:
Schedule or representation of States in Parliament as 1. The 44th Amendment to the Constitution of
mentioned in the Fourth Schedule is also included. India introduced an Article placing the
4. Which is unmatched in the context of election of the Prime Minister beyond judicial
Constitutional Amendment Procedure? review.
i. Extension of executive power of Center and 2. The Supreme Court of India struck down the
State 99th Amendment to the Constitution of India
ii. Supreme and High Court as being violative of the independence of
iii. Election of the President judiciary.
iv. Qualification of President Which of the statements given above is/are
v. Condition of the office of the President correct?
vi. Tenure of President (a) 1 only (b) 2 only
vii. Distribution of Legislative powers in Center (c) Both 1 and 2 (d) Neither 1 nor 2
and State IAS (Pre) G.S, 2019
viii. States representation in Parliament Ans : (b) A challenge to Prime Minister Indira
(a) i ii iii (b) ii iii iv Gandhi's election victory was upheld by the Allahabad
(c) iii iv v (d) iv v vi High Court on grounds of electoral malpractice in 1975.
(e) vi vii viii Meanwhile, Parliament passed the Constituion (Thirty-
Chattisgarh PSC (Pre) G.S. 2017 Ninth Amendment) Act, 1975 which removed the
Indian Polity & Constitution 116 YCT
authority of the Supreme Court to adjudicate petitions categories and introduction of Union Territories. House
regarding elections of the President, Vice President, of the People (Lok Sabha) shall consist of not more than
Prime Minister and Speaker of the Lok Sabha. Instead, 500 members to be chosen by direct election from
a body constituted by Parliament was vested with the territorial constituencies in the States and not more than
power to resolve such election disputes. Hence 25 members to represent the Union territories (Total
statement (1) is incorrect. The Constituion (Ninty-Ninth 525 members). The Constitution (Thirty- first
Amendment) Act, 2014 provided for the formation of a Amendment) Act, 1973, increased the upper limit for
National Judicial Appointments Commission (NJAC). representation of the States from 500 to 525 and
In October 2015, a Supreme Court Bench presided by decreased the limit for the Union territories from 25 to
Justice J.S Khehar in a majority of 4:1 rejected 99th 20 (Total 545 members).
Constitutional Amendment Act (also known as NJAC 10. Which Constitution amendment increased the
Act) on the grounds of violation of independence of numbers of Members of Parliament from Lok
Judiciary. Hence statement 2 is correct. Sabha to 545 from 525?
8. With reference to the Parliament of India, (a) 31st Amendment (b) 35th Amendment
consider the following statements: (c) 42nd Amendment (d) 45th Amendment
1. A private member’s bill is a bill presented by UPPCS (Main) Spl. G.S. IInd Paper, 2004
a Member of Parliament who is not elected Ans. (a) Kindly refer the explanation of the above
but only nominated by the President of India.
question.
2. Recently, a private member’s bill has been
11. Which one of the following has not been
passed in the Parliament of India for the first
provided by the 44th Constitutional Amendment
time in its history.
Act of 1978 relating to National Emergency?
Which of the statements given above is/are
(a) A proclamation of National Emergency will
correct?
not be issued by President unless there is a
(a) 1 only (b) 2 only
written recommendation of the Cabinet.
(c) Both 1 and 2 (d) Neither 1 nor 2
(b) The expression 'internal disturbances' has
IAS (Pre) G.S, 2017
been replaced by 'armed rebellion'.
Ans. (d) A Bill can be introduced either by a minister or
(c) The six fundamental rights under Article 19
a member other than a minister. If a Bill is introduced by
can be suspended only when the National
a minister, it is called a Government Bill and if a Bill is
Emergency is declared on grounds of war or
introduced by a member, then it is called as a Private
external aggression and not on grounds of
Member's Bill. Hence statement 1 is incorrect. 14 Private
Member Bills have been passed by the Parliament since armed rebellion.
independence. Hence statement 2 is also incorrect. (d) The President must issue a proclamation of
revocation any time if any House of
9. Which of the following Constitutional
Amendments are related to raising the number Parliament passes a resolution disapproving
of Members of Lok Sabha to be elected from of the continuance of the proclamation.
the States? Maharashtra PSC (Pre) G.S, 2019
(a) 6th and 22nd (b) 13th and 38th Ans. (d) : The Constituion (Forty-Fourth Amendment)
(c) 7th and 31st (d) 11th and 42nd Act, 1978 provided that an Emergency can be proclaimed
IAS (Pre) G.S., 2003 only on the basis of written advice tendered to the
Uttarakhand PCS (Pre) 2004 President by the Cabinet. Hence statement 1 is correct.
Ans. (c) The Constitution (Seventh Amendment) Act, Before 1978, an emergency could be declared on the
1956, recognized the States on the linguistic basis. It grounds of war, external aggression or internal disturbance,
paved way for classification of States in A, B, C and D which was too vague and broad in sense. The 44th

Indian Polity & Constitution 117 YCT


Constitutional Amendment, substituted the word ‘armed 14. By which Constitutional Amendment, the
rebellion’ for internal disturbance.Hence statement 2 is subject of ‘Education’ was transferred from
also correct. Before the 44th Amendment, it was provided State list to concurrent list?
in the Constituion that during the proclamation of National (a) 5th (b) 9th
Emergency, the Fundamental Rights under Article 19 are (c) 42nd (d) 44th

automatically suspended and this suspension continues till Uttarakhand RO/ARO, 2016

the end of the emergency. But after the 44th Amendment, UPPCS (Main) 2017 G.S. IInd Paper

Freedoms listed in Article 19 can be suspended only in Ans. (c) Through the 42nd Amendment Act of 1976,
five subjects namely education, forests, weights and
case of proclamation on the ground of war or external
measures, protection of wild animals and birds,
aggression and not on grounds of armed rebellion. Hence
administration of Justice were transferred from State
statement 3 is also correct. If the situation improves, the
List to Concurrent List.
emergency can be revoked by another proclamation by the
15. Under the Constitution of India Education as a
President of India. The 44th Amendment of the
subject of legislation is included in:
Constitution provides that ten per cent or more members of
(a) The Union List
the Lok Sabha can requisition a meeting of the Lok Sabha
(b) The State List
and in that meeting; it can disapprove or revoke the (c) The Concurrent List
emergency by a simple majority. In such a case emergency (d) The Residuary Power
will immediately become inoperative. Hence statement 4 is Himanchal PSC (Pre), G.S, 2011
incorrect. Ans. (c) Kindly refer the explanation of the above
12. Which of the following Article was inserted by question.
the 42nd Constitution Amendment to provide 16. The authoritative text of the Indian
for participation of workers in management? Constitution in Hindi was authorized to be
(a) Article 38 (b) Article 39 A published by which of the following
(c) Article 45 (d) Article 43 A constitutional amendment?
UPPCS (Pre) G.S, 2019 (a) 57th Amendment, 1987
(b) 58th Amendment, 1987
Ans: (d) The Constituion (Forty-Second Amendment)
(c) 59th Amendment, 1988
Act, 1976 inserted Article 43A as one of the Direcive
(d) 60th Amendment, 1988
Principle of State Policy regarding participation of the
UP RO/ ARO (Pre) G.S, 2016
workers in management.
Ans. (b) By the Constitution 58th Amendment Act,
13. Which of the following
constitutional 1987, Article 394(A) was included through which the
amendment formally inducted Sikkim as a authoritative text of the Indian Constitution in Hindi
State in to the Union of India? was authorized to be published.
th th
(a) 34 (b) 35 17. By which of the following Amendments of the
(c) 36th (d) 37th Indian Constitution word ‘Co-operative
UPPCS (Main) G.S. IInd Paper 2005 Societies’ was added in Article 19(1) (c)?
UPPCS (Main) G.S., IInd Paper, 2006 (a) 42nd Amendment Act, 1976
(b) 73rd Amendment Act, 1993
U.P. Lower (Pre.) G.S. 2013
(c) 97th Amendment Act, 2011
UPUDA/DA (Pre.) G.S., 2006
th
(d) 36th Amendment Act, 1975
Ans. (c) 36 Constitution Amendment Act, 1975
UPPCS (Main) G.S. IInd Paper, 2016
included Sikkim as a full-fledged State in the First
Ans. (c) The Constitution (Ninety-Seventh
Schedule of the Constitution and allotted Sikkim one Amendment) Act, 2011 inserted the word ‘Co-operative
seat in the Rajya Sabha and one seat in the Lok Sabha. Societies’ in Article 19(1) (c). This Amendment also
Indian Polity & Constitution 118 YCT
added Article 43 B in the Constitution which asserts 20. The words “Socialist” and “Secular” were
that the State shall endeavour to promote voluntary added to the Preamble of Indian Constituion
formation, autonomous functioning, democratic control by:
and professional management of co-operative societies. (a) The Constituion (Sixteeth Amendment) Act
The definition of co-operative Societies has been given 1950
in Article 243 ZH. (b) The Constituion (First Amendment) Act 1963
(c) The Constituion (Forty-First Amendment)
18. The First Amendment to the Constitution
Act 1976
carried out in 1951 related to
(d) The Constituion (Fourty-second Amendment)
(a) Security of the country
Act 1976
(b) Security of the Prime Minister
TNPSC (Pre) G.S. 2019
(c) Protection of agrarian reforms in certain
Ans. (d) Kindly refer the explanation of the above
Stales
question.
(d) Scheduled castes and scheduled tribes
21. Which Constitutional Amendment added
UPPCS (Main) G.S. IInd Paper 2009 ''Integrity" in the Preamble?
Ans. (c) The First Amendment to the Constitution (a) 42th (b) 30th
carried out in 1951 was related to land and agrarian (c) 25 th
(d) 15th
reforms. The First Amendment Act, 1951 inserted Ninth UPPSC ACF-RFO Mains (IInd Paper), 2019
Schedule to the Constitution to protect the land reform Ans. (a) Kindly refer the explanation of the above
and other laws present in it from the judicial review. It question.
inserted articles 31A and 31B. It also amended Articles 22. Which of the following amendment/s in the
15, 19, 85, 87, 174, 176, 341, 342, 372 and 376. Indian Constitution was/were equated with
19. By which Amendment, words socialist, secular ‘Mini Constitution’ ?
and national unity and integrity has been (a) 24th and 25th Amendments
added to the Constitution of India? (b) 42nd Amendment
(a) 42nd Amendment (c) 44th Amendment
(b) 44th Amendment (d) 73rd and 74th Amendments
(c) 52nd Amendment Punjab PSC (Pre) G.S 2018
(d) None of the above Ans. (b) Kindly refer the explanation of the above
BPSC (Pre) 2003 question.
UPPCS (Main) Spl. G.S. IInd 2008 23. Under which constitution amendment, 10
Fundamental Duties of the citizens were
UPPCS (Pre.) G.S. 1991, 2010
included in the constitution?
Uttarakhand UDA/LDA (Pre) 2003, 2006
(a) 24th (b) 38th
st
MPPSC (Pre) GS 1 Paper 2013 (c) 42nd (d) 44th
Ans. (a) The Constitution (Forty-Second Amendment) Haryana PSC (Pre), G.S. 2014
Act, 1976, was enacted during the emergency (25 June Ans. (c) Kindly refer the explanation of the above
1975 – 21 March 1977). This Amendment brought question.
about the most widespread changes to the Constitution 24. Fundamental Duties by the citizens were added
of India in its history and is sometimes called a ‘Mini- to the Constitution of India by which
Constitution’. By this Amendment, three new words Amendment?
Socialist, Secular and Integrity were added in the (a) 38th (b) 41st
Preamble. A new Part IV A (Fundamental Duties by the (c) 42nd (d) 45th
citizens) was added. Four Articles namely 39, 39 A, Uttarakhand UDA/LDA (Pre) 2006
43A and 48A were added to the Directive Principles of UPPCS (Pre) G.S., 2015
nd
the State Policy. Constitution 42 Amendment made MPPSC (Pre) GS 1990
the President bound by the advice of the Union Cabinet. BPSC (Pre) 2007
Indian Polity & Constitution 119 YCT
Ans. (c) Kindly refer the explanation of the above 28. Consider the following statements:
question. I. Through 44th Constitutional Amendment,
25. The word ‘Socialist’ was added to Preamble of 1978, all the Directive Principles of the State
Policy have been given preference over
the Constitution of India by which of the
Fundamental Rights mentioned in Articles 14
following Amendment?
and 19.
(a) 42nd Amendment (b) 44th Amendment
II. Supreme Court has laid down in its verdict in
(c) 46th Amendment (d) 74th Amendment
Minerva Mill case, 1980 that the preference
(e) None of the above given only to Directive Principle of The State
BPSC (Pre) G.S. 2016 Policy mentioned in Article 39(b) and (c)
Ans. (a) Kindly refer the explanation of the above over Fundamental Rights mentioned in
question. Articles 14 and 19.
26. According to which one of the following (a) Only I is correct.
Constitutional Amendments in the Indian (b) Only II is correct.
Constitution the Right to Property has been (c) Both I and II are correct.
abolished? (d) Neither I nor II are correct.
(a) 42nd Amendment Act, 1976 RAS/RTS (Pre) G.S. 2013
(b) 43rd Amendment Act, 1977 Ans. (b) Directive Principles of the State Policy have
(c) 44th Amendment Act, 1978 been given preference over Fundamental Rights
mentioned in Articles 14 and 19 through 42nd
(d) 45th Amendment Act, 1979
Constitutional Amendment, 1976. Hence statement 1 is
MPPSC (Pre) G.S. 1994, 2008
incorrect. However this provision was abolished by
UPPCS (Main) SPl. G.S. IInd 2004 Supreme Court in Minerva Mill case, 1980. Supreme
UPPCS (Main) G.S. IInd 2013, 2014, 2015 Court ruled that the preference given only to Directive
U.P. Lower (Pre.) Spl. G.S. 2004 Principles of the State Policy mentioned in Article 39(b)
UPUDA/LDA Special (Main) G.S., 2010 and (c) over Fundamental Rights mentioned in Articles
Chhattisgarh PSC (Pre) G.S. 2010-11 14 and 19. Hence statement II is correct.
Ans. (c) Constitution 44th Amendment (1978) repealed 29. The President’s power to Veto a Bill from
Article 19(1) (f) and Article 31 of the Constitution Amendment of the Constitution has been taken
which were about the right to acquire, hold and dispose away by substituting the word “shall give the
of property as a Fundamental Right. However in consent” by which Amendment?
another part of the Constitution, Article 300(A) was (1) Twenty third Amendment
inserted to affirm that no person shall be deprived of his (2) Twenty fourth Amendment
property save by authority of law. Right to Property as a (3) Forty second Amendment
Fundamental Right is substituted as Statutory Right. (4) Forty fourth Amendment
27. During which Prime Minister's regime the RAS/RTS (Pre.) G.S., 2015
right to property was excluded from the list of Ans. (c) The President’s power to veto a Bill from
fundamental rights? Amendment of the Constitution has been taken away by
substituting the word “shall give the consent” by the
(a) Rajeev Gandhi
42nd Constitutional Amendment, 1976.
(b) Morarji Desai
30. Which one of the following amendments to the
(c) Indra Kumar Gujral
Indian Constitution empowers the President to
(d) H.D. Devogowda
send back any matter for reconsideration by
Himanchal PSC (Pre) G.S. 2012 the Council of Ministers?
Ans. (b) Right to property was excluded from the list of (a) 39th (b) 40th
Fundamental Rights under the Priministership of (c) 42nd (d) 44th
Morarji Desai in the year 1977. IAS (Pre) G.S., 2002
Indian Polity & Constitution 120 YCT
Ans. (d) Constitution (Forty-Fourth Amendment) Act, Ans. (c) The Constitution (Seventy-Third Amendment)
1978 amended the Article 74 of the Constitution. Article Act, 1992, provided constitutional status to the
74(1) asserts that provided that the President may require Panchayati Raj Institutions in India through insertion of
the Council of Ministers to reconsider advice and the Article 243 to Part IX of the Constitution. This
President shall act in accordance with the advice tendered Amendment also provided 30% reservation for woman
after such reconsideration. in village panchayats.
31. Which one of the following is not properly 34. The 73rd Constitution Amendment Act 1992
matched? refers to the
(a) 42nd Constitutional Amendment–Funda- (a) Generation of gainful employment for the
mental duties unemployed and the underemployed men and
(b) 52nd Constitutional Amendment–Anti women in rural area
Defection law (b) Generation of employment for the ablebodies
(c) 73rd Constitutional Amendment–Panchayati adults who are in need for any desirous of
Raj work during the lean agricultural season
(d) 84th Constitutional Amendment–Centre– (c) Laying the foundation for strong and vibrant
State Relations Panchayati Raj Institutions in the country
Himanchal PSC (Pre) G.S, 2010 (d) Guarantee of right to life, liberty and security
th
Ans. (d) The Constitution 84 Amendment Act, 2001 is of person, equality before law and equal
related to delimitation up to the year of 2026. Rest are protection without discrimination.
properly matched. IAS (Pre.) G.S., 2000
32. By which Constitutional Amendment the Ans. (c) Kindly refer the explanation of the above
voting age was reduced from 21 years to 18 question.
years? 35. The Constitution (Seventy- Third Amendment)
(a) Sixty third Amendment Act, 1992 which aims at promoting the
(b) Sixty second Amendment Panchayati Raj Institutions in country,
(c) Sixty first Amendment provides for which of the following?
(d) Sixtieth Amendment 1. Constitution of district planning committee
UPPCS (Main) G.S. IInd 2007, 2010, 2014 2. State Election Commission conducting
UPPCS (Main) Spl. G.S. IInd 2004 Panchayat elections

MPPSC (Pre) G.S. 2010-11 3. Establishment of State Finance Commission

BPSC (Pre) 2007-08 State the correct answer using the codes given
below:
Ans. (c) The Constitution (Sixty- First Amendment)
(a) 1 only (b) 1 and 2 only
Act, 1988 amended Article 326 of the Constitution. It
reduced the voting age from 21 years to 18 years. (c) 2 and 3 only (d) 1, 2 and 3
IAS (Pre) G.S., 2011
33. Under which of the following Constitutional
Amendments, 30% seats in the village Ans. (d) The Constitution (Seventy-Third Amendment)
panchayats have been reserved for the woman Act, 1992 inserted Article 243ZD which mandates that
in India? a District Planning Committee shall be constituted at the
(a) 70th Amendment district level in every State to consolidate the plans
(b) 71st Amendment prepared by the Panchayats and the Municipalities in
rd the district and to prepare a draft development plan for
(c) 73 Amendment
th
(d) 74 Amendment the district as a whole. Article 243 K of the Constitution
UPPCS (Main) G.S. IInd Paper, 2016 made provision for the State Election Commission to
Indian Polity & Constitution 121 YCT
conduct Panchayat elections at the State level. Article 39. Which one of the following amendments to the
243I prescribed that the Governor should constitute a Constituion of India sought to curb political
Finance Commission to review the financial position of defections?
the Panchayats. (a) 42nd (b) 52nd
36. By which one of the following Constitutional (c) 62nd (d) 70th
Amendments Delhi has become 'National Maharashtra PSC (Pre) G.S 2014
Capital Region'? Ans. (b) Kindly refer the explanation of the above
(a) 61st Amendment (b) 69th Amendment question.
(c) 71st Amendment (d) 79th Amendment
40. The Tenth Schedule added to the Constitution
UPPCS (Main) G.S. IInd Paper 2009
of India by the 52nd Amendment relates to:
Ans. (b) By the commencement of the Constitution (a) Anti-defection law
(Sixty-Ninth Amendment) Act, 1991, the Union
(b) Land Reforms
Territory of Delhi shall be called the National Capital
(c) Panchayati Raj
Territory and the Administrator appointed under Article
239 shall be designated as the Lieutenant Governor. (d) Languages.
There is a Legislative Assembly for the National Himanchal PSC (Pre), G.S, 2011
Capital Territory of Delhi and the seats in such Ans. (a) Kindly refer the explanation of the above
Assembly are filled by members chosen by direct question.
election from territorial constituencies in the National 41. Mizoram has been granted the status of the
Capital Territory. State by which of the following Constitution
37. Delhi has been made National Capital Region Amendment?
by which of the following Amendment? (a) 54th (b) 55th
th th
(a) 67 (b) 68 (c) 52nd (d) 53rd
th th
(c) 69 (d) 70 RAS/RTS (Pre) G.S., 2015
UPPSC ACF-RFO Mains (IInd Paper), 2019
Ans. (d) The Constitution 53rd Amendment Act, 1986
Ans. (c) Kindly refer the explanation of the above provided statehood to the Mizoram.
question.
42. Which one of the following Constitutional
38. 52nd Constitution Amendment is related to
Amendments states that the total number of
(a) Anti Defection
Ministers, including the Prime Minister, in the
(b) Reservation Council of Ministers shall not exceed fifteen
(c) Election percent of the total number of members of the
(d) Reservation of the minorities House of the People?
UPUDA/LDA (Pre.) G.S., 2006 (a) 90th (b) 91st
Ans. (a) The Tenth Schedule, popularly known as Anti- (c) 92nd (c) 93rd
Defection Act was included in the Constitution by 52nd
IAS (Pre) G.S., 2009
Constitution Amendment Act, 1985. Disqualification
Ans. (b) The Constitution (Ninety-First Amendment)
under the grounds of Anti-Defection Law under Article
Act, 2003 inserted clause (1A) in Article 75 of the
102(2) [For Lok Sabha and Rajya Sabha] and 191(2)
Constitution which asserts that the total number of
[For State Legislature and State Legislative Cuncil] is-
ministers including the Prime Minister in the Council of
(i) If an elected member voluntarily gives up his
Ministers shall not exceed fifteen per cent (15%) of the
membership of a political party
total number of members of the House of the People
(ii) If he votes or abstains from voting in such House
(Lok Sabha). Further, clause (1A) has been added to
contrary to any direction issued by his political
Article 164 of the Constitution which asserts that the
party or anyone authorized to do so without
total number of ministers including the Chief Minister
obtaining prior permission.
Indian Polity & Constitution 122 YCT
in the Council of Ministers in a State shall not exceed (c) 92nd Amendment
fifteen per cent of the total number of members of the (d) 29th Amendment
Legislative Assembly of that State provided that the Punjab PSC (Pre) G.S 2015
number of ministers including Chief Minister in a State
Ans. (d) Kindly refer the explanation of the above
shall not be less than twelve.
question.
43. Which Constitutional Amendment has limited
46. With which subject the 97th Constitution
the number of Central Ministers to 15% of the
Amendment Act of 2012 is concerned?
total numbers of the Lok Sabha?
(a) Free and compulsory education of children
(a) 90th (b) 91st
till the age of 14 years
(c) 92nd (d) None of the above
(b) Organizing and working of co-operative
UPPCS (Main) G.S. IInd 2017
societies
UPPCS (Pre.) Re-exam. G.S., 2015
(c) Stringent measure to deal with terrorism
IAS (Pre) G.S., 2007
(d) Provision of Lok Pal to prevent corruption
UPPCS (Main) G.S. 1st Paper 2004
UPPCS (Main) G.S. IInd Paper 2005
Ans. (b) Kindly refer the explanation of the above
question. Ans. (b) The Constitution (Ninety-Seventh
Amendment) Act, 2011 is related to co-operative
44. Under which one of the following Constitution
societies. By this Amendment, a new Article 43B was
Amendments Acts, four languages were added
to the languages under the Eighth schedule of inserted in Part IV (Directive Principles) of the
the Constitution of India, thereby raising their Constitution and in Part III (Fundamental Rights), after
number to 22? word “or Unions” the words “Cooperative Societies”
(a) Constitution (Ninetieth Amendment) Act was added.
(b) Constitution (Ninety-first Amendment) Act 47. The 79th Amendment of the Indian
(c) Constitution (Ninety-second Amendment) Constitution is related to
Act (a) Centre-State Relations
(d) Constitution (Ninety-third Amendment) Act (b) Establishment of two political parties
IAS (Pre) G.S., 2008 (c) Fundamental Rights
Ans. (c) The Constitution (Ninety-Second Amend- (d) Reservation of SCs and STs in the Lok Sabha
ment) Act, 2003 added four languages namely Bodo, and State Assemblies
Dogri, Santhali and Maithili to the Eight Schedule of UPPCS (Main) G.S. IInd Paper 2009
the Constitution thereby raising the total number of Ans. (d) The Constitution (Seventy-Ninth Amendment)
languages to 22 listed in Eight Schedule. Of these Act, 2000, extended reservation for Schedule Caste /
languages, 14 were initially included in the Schedule Tribes and nomination of Anglo Indian
Constitution. Sindhi language was added by the
members in Parliament and State Assemblies for
Constitution (Twenty- First Amendment) Act, 1967.
another ten years i.e. up to 2010.
Thereafter three more languages namely Konkani,
48. Which Constitution Amendment added a new
Manipuri and Nepali were included by the Constitution
Article to the Constitution to provide ‘Right to
(Seventy-First Amendment) Act, 1992. Subsequently
Bodo, Dogri, Maithili and Santhali were added in 2004. Education’?
(a) 86th Amendment
45. Which of these constitutional amendments does
not pertain to addition of more languages in (b) 87th Amendment
the 8th Schedule? (c) 88th Amendment
(a) 21st Amendment (d) 89th Amendment
(b) 71st Amendment UPPSC (Main) G.S. IInd 2006
Indian Polity & Constitution 123 YCT
Ans. (a) The Constitution (Eighty-Sixth Amendment) Scheduled Castes or the Scheduled Tribes in so far as
Act, 2002 inserted Article 21A in the Constitution to such special provisions relate to their admission to
provide free and compulsory education of all children in educational institutions including private educational
the age group of six to fourteen years as a Fundamental institutions, whether aided or unaided by the State, other
Right. Article 21A and Right to Education Act came than the minority educational institutions.
into effect on 1 April 2010. 52. By which Constitutional Amendment OBCs
49. ‘Right to Education’ proposed by 86 th have been given 27 percent reservation in the
Amendment was implemented in which year? admission to educational institutions?
(a) From 2002 (b) From 2004 (a) 92nd (b) 93rd
(c) From 2008 (d) From 2010 (c) 94th (d) 96th

Uttarakhand PCS (Pre) 1st 2014-15 Uttarakhand PCS (Pre) 1st Paper 2014
Ans. (b) Kindly refer the explanation of the above
Ans. (d) Kindly refer the explanation of the above
question.
question.
53. 93rd Constitutional Amendment is related to
50. Which constitutional amendment virtually
whom?
transferred Article 45 from Part IV (Directive
(a) To Panchayati Raj Institutions
Principles) to Part III (Fundamental Rights) in
(b) To Fundamental Duties
the form of Article 21 A?
(c) To the North-Eastern States of India
(a) 86th Amendment (b) 25th Amendment
(d) Reservation of OBCs in educational
(c) 73rd Amendment (d) 90th Amendment
institutions
Himanchal PSC(Pre) G.S. 2012
Uttarakhand RO/ARO (M) 2016
Ans. (a) Article 45 under the Directive Principle of
Ans. (d) Kindly refer the explanation of the above
State Policy from Part IV is regarding provision for free question.
and compulsory education for children. The
54. Match List I with List II and select the correct
Constitution (Eighty-Sixth Amendment) Act, 2002
answer using the codes given below the Lists:
inserted Article 21A in the Constitution to provide free
List-I List-II
and compulsory education of all children in the age
(A) The Constitution (i) Establishment of state
group of six to fourteen years as a Fundamental Right
(sixty-ninth level Rent Tribunals
under Part III of the Constituion.
Amendment)
51. The 93rd Constitution Amendment Bill deals Act, 1991
with the
(B) The Constitution (ii) No reservations for
(a) Continuation of reservation for backward (Seventy-fifth Scheduled Castes in
classes in educational institutions Amendment) Panchayats in
(b) Free and compulsory education for all Act, 1994 Arunachal Pradesh
children between the age of 6 and 14 years (C) The Constitution (iii) Constitution of
(c) Reservation of 30 percent post for women in (Eightieth Panchayats in
Government recruitments Amendment) Villages or at other
(d) Allocation of more number of Parliamentary Act, 2000 local levels
seats for recently created States (D) The Constitution (iv) Accepting the
IAS (Pre) G.S., 2002 (Eighty-third recommendations of
Ans. (a) The Constitution 93rd Amendment, 2006 added Amendment) the Tenth Finance
clause (5) in Article 15 which asserts that nothing shall Act, 2000 Commission
prevent the State from making any special provision by (v) According the Status
law for the advancement of any socially and of National Capital
educationally backward classes of citizens or for the Territory to Delhi

Indian Polity & Constitution 124 YCT


A B C D Ans. (e) :
(a) 5 1 4 2
List-I List-II
(b) 1 5 3 4 th
13 Amendment, Nagaland
(c) 5 1 3 4
1962
(d) 1 5 4 2
IAS (Pre) G.S., 2001 18th Amendment, Redefined State
Ans: (a) 1966
List-I List-II 39th Amendment, The election of
The Constitution According the Status of 1975 President, Vice-
(Sixty-ninth National Capital Territory President,Speaker
Amendments) Act, to Delhi and Prime
1991 Minister cannot
The Constitution Establishment of state be challenged
(Seventy-fifth level Rent Tribunals
52nd Amendment, Anti-defection
Amendment) Act,1994
1985 law
The Constitution Accepting the
(Eightieth recommenddations of the 56. Consider the following statements about the
Amendment) Act,2000 Tenth Finance 108th Constitutional Amendment Bill:
Commission (1) It related to reservation of one-third seats in
The Constitution No reservations for Lok Sabha and State Legislative Assemblies
(Eighty-third Scheduled Castes in
for women.
Amendment) Act,2000 Panchayats in Arunachal
(2) Rajya Sabha passed the Bill on 9th March,
Pradesh
2010.
55. Match the following Constitutional
Amendments. (3) Lok Sabha never voted on the Bill.
List-I List-II (4) It was lapsed after the dissolution of the 15th
(a) 13th (i) Nagaland Lok Sabha.
Amendment Which of the statements given above are
th
(b) 18 (ii) Anti-defection law correct?
Amendment (a) Only 1, 2 and 4
(c) 39th (iii) Redefined State (b) Only 2, 3 and 4
Amendment
(c) Only 1, 2 and 3
(d) 52nd (iv) The election of
(d) 1, 2, 3 and 4
Amendment President, Vice-
President, Speaker Maharashtra PSC (Pre) G.S. 2017
and Prime Minister Ans. (d) The Constitution (108th Amendment) Bill, 2008,
cannot be seeked to reserve one-third of all seats for women in the
challenged
Lok Sabha and the State Legislative Assemblies. One-
A B C D
third of the total number of seats reserved for SC/ST shall
(a) (ii) (iv) (i) (iii)
be reserved for women of those groups in the Lok Sabha
(b) (iv) (iii) (ii) (i)
(c) (iii) (ii) (i) (iv) and the legislative assemblies. Reserved seats may be

(d) (i) (iv) (ii) (iii) allotted by rotation to different constituencies in the State
(e) (i) (iii) (iv) (ii) or Union Territory. Reservation of seats for women shall
st
Chhattisgarh PSC (Pre) 1 G.S., 2014 cease to exist 15 years after the commencement of this
Indian Polity & Constitution 125 YCT
Amendment Act. The Rajya Sabha passed the bill on Ans. (d) : Goods and Serices Tax (GST) was
March 9, 2010. However, the Lok Sabha never voted on introduced by the Constitution (One Hundred and First
the bill. The bill lapsed after the dissolution of the 15th Amendment) Act, 2016. By this amendment, after
Lok Sabha in 2014. Article 246 of the Constitution, the following Article

57. Which one of the following cases prompted shall be inserted, namely:—
Parliament to enact 24th Constituional 246A. (1) Notwithstanding anything contained in
Amendment Act? Articles 246 and 254, Parliament and subject to clause
(2), the Legislature of every State, have power to make
(a) Golaknath case
laws with respect to goods and services tax imposed by
(b) Minerva Mill case
the Union or by such State.
(c) Kesavananda Bharti case
(2) Parliament has exclusive power to make laws with
(d) Shankari Prasad case
respect to goods and services tax where the supply of
Maharashtra PSC (Pre) G.S 2014 goods or of services or both takes place in the course of
Ans. (a) The Supreme Court in the well-known Golak inter-State trade or commerce.
Nath case reversed its own earlier
decisions 60. The Constitutional Amendment Act 104th -2019
upholding the power of Parliament to amend all parts of is related to-
the Constitution including Part III relating to (a) Article-334
Fundamental Rights. Constitution (Twenty-Fourth (b) Article-234
Amendment) Act, 1971 amended Article 368 to clarify (c) Article-134
that when a Constitution Amendment Bill passed by (d) None of above
both Houses of Parliament is presented to the President UPPSC ACF-RFO Mains (IInd Paper), 2019
for his assent, he should give his assent thereto. The 24th
Ans. (a) : Constitution (One Hundred and Fourth
Amedment Act also seeks to amend Article 13 of Amendment) Act, 2019 came into force on 25th January,
the Constitution to make it inapplicable to any 2020 to amend Article 334 of the Constituion. Article
amendment of the Constitution under Article 368. 334 of the Constitution lays down that the provisions of
58. Which of the following cases prompted the the Constitution relating to the reservation of seats for
Indian Parliament to enact 24th Amendment the Scheduled Castes and the Scheduled Tribes and the
Bill? representation of the Anglo-Indian community by
(a) Golaknath Case nomination in the House of the People and Legislative

(b) Shankari Prasad Case Assemblies of the States shall cease to have effect on
the expiration of the period of 70 years from the
(c) Keshvananda Bharati Case
commencement of the Constitution. In other words,
(d) Shah Banu Case
these provisions ceased to have effect on the 25th
TNPSC (Pre) G.S. 2015
January, 2020, if not extended further. Therefore, with a
Ans. (a) : Kindly refer the explanation of the above view to retaining the inclusive character as envisioned
question. by the founding fathers of the Constitution, it is
59. Under which Constitutional Amendment Act proposed through Constitution (One Hundred and
the G.S.T. was introduced? Fourth Amendment) Act, 2019 to continue the
(a) 96th (b) 98th reservation of seats for the Scheduled Castes and the
Scheduled Tribes for another ten years i.e. up to 25th
(c) 100th (d) 101th
January, 2030.
UPPSC ACF-RFO Mains (IInd Paper), 2019
Indian Polity & Constitution 126 YCT
61. The Ninth Schedule to the Indian Constitution Lowering the 61st Amendment
was added by: voting age from
(a) First Amendment 21 to 18
(b) Eighth Amendment Addition of the 42nd Amendment
(c) Ninth Amendment word secular in
(d) Forty second Amendment the preamble
RAS/RTS (Pre) G.S., 2013 63. Match List I with List II and select the correct
Ans. (a) The Constitution (First Amendment) Act, 1951 answer from the code given below:
was introduced to keep certain laws particularly those List-I List-II
on land reforms beyond the judicial review by inserting rd
A. 93 Amendment 1. Parliaments
Article 31B to the Constitution.
power to amend
62. Match List I with List II and select the correct Fundamental
answer from the code given below:
Rights
List-I List-II th
B. 97 Amendment 2. Abolition of
A. Curtailment of the (i) 61st Amendment Princely states,
power of judicial titles and
review
pensions
B. Abolition of Right (ii) 42nd Amendmen th
C. 24 Amendment 3. Reservation of
to property as a t
OBC’s in
fundamental right
unaided
C. Lowering the (iii) 38th Amendment educational
voting age from 21
institutes
to 18 th
D. 26 Amendment 4. New Cooperative
D. Addition of the (iv) 44th Amendment
Act
word secular in the
Codes:
preamble
A B C D
Codes:
(a) 1 3 2 4
A B C D
(b) 3 4 1 2
(a) 1 2 4 3
(c) 4 2 3 1
(b) 2 4 1 3
(d) 3 1 2 4
(c) 3 4 1 2
Punjab PSC (Pre) G.S 2015
(d) 4 1 3 2
UPPCS (Pre) G.S, 2017 Ans. (b) Correctly matched list is give below-
93rd Amendment Reservation of OBC’s in
Ans. (c) :
unaided educational institutions
Curtailment of 38th Amedment
th
the power of 97 Amendment New cooperative Act
th
judicial review 24 Amendment Parliament’s power to amend

Abolition of 44th Amendment Fundamental Rights


th
Right to property 26 Amendment Abolition of special previleges
as a fundamental of Princly states, titles and
right pensions

Indian Polity & Constitution 127 YCT


20.
Sequence of Precedence
1. Which one of the following is the correct 8. Ambassadors, High Commissioners of
sequence in the descending order of precedence Commonwealth countries accredited to India,
in the warrant of precedence? Chief Ministers of States outside their respective
(a) Attorney General of India - Judges of the States, Governors of States outside their
Supreme Court - Members of Parliament- respective States
Deputy Chairman of Rajya Sabha 9. Judges of Supreme Court, Chairperson-Union
(b) Judges of the Supreme Court- Deputy Public Service Commission, Chief Election
Chairman of Rajya Sabha - Attorney General Commissioner, Comptroller and Auditor General
of India - Members of Parliament of India

(c) Attorney General of India- Deputy Chairman 10. Deputy Chairman, Rajya Sabha, Deputy Chief
of Rajya Sabha - Judges of the Supreme Ministers of States, Deputy Speaker- Lok Sabha,
Court - Members of Parliament Members of the Planning Commission, Ministers

(d) Judges of the Supreme Court - Attorney of State of the Union


General of India- Deputy Chairman of Rajya 11. Attorney General of India, Cabinet Secretary,
Sabha - Members of Parliament Lieutenant Governors within their respective

IAS (Pre) G.S., 2004 Union Territories

Ans. (d) Table of Precedence- 12. Chiefs of Staff holding the rank of full General or

1. President equivalent rank.

2. Vice-President 14. Chairmen and Speakers of State Legislatures

3. Prime Minister within their respective States, Chief Justices of

4. Governors of States within their respective States High Courts within their respective jurisdictions

5. Former Presidents, Deputy Prime Minister 15. Cabinet Ministers in States within their respective
States, Chief Ministers of Union Territories.
6. Chief Justice of India, Speaker of Lok Sabha

7. Cabinet Ministers of the Union, Chief Ministers 17. Judges of High Courts within their respective

of States within their respective States, Deputy jurisdictions

Chairman- Planning Commission, Former Prime 21. Members of Parliament

Ministers, Leaders of Opposition in Rajya Sabha 23. Secretary to the President, Secretary to the Prime

and Lok Sabha, Holders of Bharat Ratna Minister, Secretary- Rajya Sabha/Lok Sabha,

decoration Solicitor General

Indian Polity & Constitution 128 YCT


25. Additional Secretaries to the Government of (b) Former President

India, Director- Central Bureau of Investigation , (c) Governor of the State


Director General- Border Security Force, Director (d) Chief Justice of the Supreme Court
General- Central Reserve Police, Director- Maharashtra PSC (Pre) G.S 2016
Intelligence Bureau Ans. (c) Kindly refer the explanation of the above
26. Joint Secretaries to the Government of India question.
2. Consider the following functionaries 5. In the Indian Order of Precedence, who
1. Cabinet Secretary amongst the following comes first?
2. Chief Election Commissioner (a) The Chairman of UPSC
3. Union Cabinet Ministers (b) The Chief Election Commissioner
4. Chief Justice of India (c) The Comptroller and Auditor General
Their correct sequence, in the Order of (d) The Chief Justice of High Court
Precedence is (e) None of the above/ More than one of the
(a) 3, 4, 2, 1 (b) 4, 3, 1, 2 above
(c) 4, 3, 2, 1 (d) 3, 4, 1, 2 BPSC (Pre) G.S. 2019
IAS (Pre) G.S., 2000 Ans. (e) As per the sequence of precedence, Judges of

Ans. (c) Kindly refer the explanation of the above Supreme Court, Chairperson-Union Public Service
question. Commission, Chief Election Commissioner and
3. As per Indian Protocol, who among the Comptroller and Auditor General of India have been
following ranks highest in the order of provided same precedence.
precedence? 6. The status of the Chief Election Commissioner
(a) Deputy Prime Minister of India is equal to the
(b) Former President (a) Chief Justice of India
(c) Governor of a State within his State (b) Governor of a State
(d) Speaker of Lok Sabha (c) Speaker of Lok Sabha
IAS (Pre) G.S., 2003 (d) Speaker of Legislative Assembly of State
Ans. (c) Kindly refer the explanation of the above UPPCS (Main) G.S., IInd Paper, 2016
question. Ans. (*) As per the table of the precedence, office of the
4. As per Indian protocol, who among the Chief Election Commissioner of India is equivalent to
following ranks highest in a State in order of the Judges of Supreme Court, Comptroller and Auditor
precedence? General, Chief Election Commissioner. Hence none of
(a) Deputy Prime Minister the given options are correct.

Indian Polity & Constitution 129 YCT


21
Governor of the State/
Leiutent Governor
1. Consider the following statements: appointment of minister is prerogative of the Chief
1. No criminal proceedings shall be instituted Minister. Governor of the State appoints the ministers
against the Governor of a State in any court on the advice of the Chief Minister. Therefore this is not
during his term of office. the discretionary power of the Governor. Hence
2. The emoluments and allowances of the statement 2 is incorrect. According to Article 200 of the
Governor of a State shall not be diminished Constitution, Governor may reserve certain Bills passed
during his term of office. Which of the by the State Legislature for consideration of the
statements given above is/are correct? President of India which is discretionary power of the
(a) 1 only (b) 2 only Governor. Hence statement 3 is correct. According to
(c) Both 1 and 2 (d) Neither 1 nor 2 Article 166 of the Constitution, the Governor shall
IAS (Pre) G.S.2018 make rules for the more convenient transaction of the
business of the Government of the State which in turn is
Ans. (c) Like the President of India, a Governor of a
not being done by the Governor on his own discretion
State is also entitled to a number of privileges and
but on the advice of the State Government. Hence
immunities. During his term of office, he is immune from
statement 4 is incorrect.
any criminal proceedings, even in respect of his personal
acts. Hence statement 1 is correct. The Governor is 3. The Governor of the State is appointed under
entitled to such emoluments, allowances and privileges as the Article of the Constitution
may be determined by Parliament. His emoluments and (a) 153 (b) 154
allowances cannot be diminished during his term of (c) 155 (d) 156
office. Hence statement 2 is also correct. UPPCS (Pre) G.S., 2015
2. Which of the following are the discretionary Ans. (c) According to Article 155 of the Constitution,
powers given to the Governor of a State? the Governor of a State shall be appointed by the
1. Sending a report to the President of India for President.
imposing the President's rule 4. Which of the following appoints the Governor?
2. Appointing the Ministers (a) Union Ministry
3. Reserving certain bills passed by the State (b) Chief Justice of Supreme Court
Legislature for consideration of the President (c) Speaker of the Lok Sabha
of India (d) President of India
4. Making the rules to conduct the business of UPPCS (Pre.) G.S., 2012
the State Government Ans. (d) Kindly refer the explanation of the above
Select the correct answer using the code given question.
below. 5. Whether Governor of a State has the power to
(a) 1 and 2 only (b) 1 and 3 only dissolve the State Legislature?
(c) 2, 3 and 4 only (d) 1, 2, 3 and 4 (a) Yes
IAS (Pre) G.S., 2014 (b) No
Ans. (b) According to Article 356 of the Constitution, (c) He/She can only recommend
the President on the report of the Governor of the State (d) Only President Can
may impose President’s rule in the State. This is TNPSC (Pre) G.S. 2017
discretionary power of the Governor and he is not Ans. (a) : According to Article 174(2) (b) of the
bound to take advice from the Council of Ministers on Constituion, Governor may dissolve the Legislative
such matter. Hence statement 1 is correct. In any State, Assembly.

Indian Polity & Constitution 130 YCT


6. When the same person is appointed as a Union Territory, administered by the President through
Governor of two or more States, the an administrator (Article 239).
emoluments and allowances payable to the
10. Consider the following and select the correct
Governor shall be
answer by using code given below:
(a) As expressed wish of the Governor
1. The Governor shall take oath or affirmation
(b) As decide by the President
before entering upon his office
(c) As decided by the Home Ministry
(d) It shall be allocated among the States in such 2. The format of oath or affirmation is given in
proportion as the President may by order the Third Schedule of the Indian Constitution
determine 3. The Governor shall make and subscribe the
UPPCS (Main) G.S. IInd Paper, 2016 oath or affirmation in the presence of Chief
Ans. (d) According to Article 158(3A) of the Justice of the High Court or in his absence
Constitution, when the same person is appointed as senior most Judge of the High Court
Governor of two or more States, the emoluments and 4. The process of oath or affirmation is given in
allowances payable to the Governor shall be allocated the Article 159 of the Indian Constitution
among the States in such proportion as the President Code:
may by order determine. (a) 1, 2 and 3 (b) 1, 3 and 4
7. Choose the correct answer: Governor of the (c) 1, 2 and 4 (d) 1, 2, 3 and 4
Rajasthan is Chancellor of
UPPCS (Pre.) G.S., 2009
(a) All State universities
(b) All State universities and central universities Ans. (d) All the options given about the Governor of a
in the State State is correct.
(c) All State universities as well as private 11. Which one of the following States is an
universities in the State exception to the Article 170 as a subject to the
(d) All State universities, central universities in provision of the composition of the Legislative
the State and all private universities in the Assembly, a State shall consist of not less than
State sixty members?
RAS/RTS (Pre) G.S., 2016 (a) Sikkim (b) Jammu & Kashmir
Ans. (a) The Governor of the State is the Chancellor of (c) Haryana (d) Uttaranchal
the all State Universities and he appoints the Vice-
UPPCS (Main) G.S. IInd Paper 2015
Chancellors of the State Universities.
Ans.(a) Sikkim became the 22nd State of India by
8. According to which Article of Constitution of
India, the Chief Minister is appointed by the Constitution (Thirty-Sixth Amendment) Act, 1975. The
Governor of a State? Act provides that the Legislative Assembly of Sikkim
(a) Article 163 (b) Article 164 shall consist of not less than thirty two
(c) Article 165 (d) Article 166 members Therefore Sikkim has been provided
Uttarakhand PSC (Pre) G.S, 2016 exemption from the provisions mentioned in Article 170
Ans. (b) According to Article 164(1) of the (Legislative Assembly of each State shall consist of not
Constitution, the Chief Minister shall be appointed by more than five hundred and not less than sixty
the Governor and the other Ministers shall be appointed members) of the Constitution.
by the Governor on the advice of the Chief Minister. 12. The provision that Governor shall address the
9. Consider the following statements: Legislative Assembly of the State is in which
1. The Governor of Punjab is concurrently the Article of the Constitution of India?
Administrator of Chandigarh (a) 172 (b) 176
2 The Governor of Kerala is concurrently the
(c) 182 (d) 183
Administrator of Lakshadweep
(e) 187
Which of the above statements is/ are correct?
(a) 1 only (b) 2 and 3 only Chattisgarh PSC (Pre.) 1st Paper, 2012
(c) Both 1 and 2 (d) 1, 2 and 3 Ans. (b) According to Article 176 of the Constitution,
IAS (Pre) G.S., 2009 at the commencement of the first session after each
Ans. (a) Ever since June 1984, the Governor of Punjab general election to the Legislative Assembly and at the
has been functioning as the Administrator of Union commencement of the first session of each year, the
Territory of Chandigarh. Lakshadweep is uni-district Governor shall address the Legislative Assembly.
Indian Polity & Constitution 131 YCT
13. Which among the following Articles of the correct. According to Section 45 in the Government of
Indian Constitution empowers the Governor to Union Territories Act, 1963, the Chief Minister of a
issue the ordinance? Union Territory shall be appointed by the President and
(a) Article 208 (b) Article 212 the other Ministers shall be appointed by the President
(c) Article 213 (d) Article 214 on the advice of the Chief Minister. Hence statement 4
is incorrect.
UPPCS (Main) ,G.S. IInd Paper 2017
16. Which of the following statements is correct?
Ans. (c) Under Article 213 of the Constitution, when
(a) The same person cannot be appointed as
the Legislature is not in session and if the Governor is
Governer for two or more States at the same
satisfied that circumstances exist which require him to
time.
take immediate action, he may legislate by Ordinance.
(b) No procedure has been laid down in the
An Ordinance promulgated under Article 213 shall have
Constitution of India for the removal of a
the same force and effect as an Act of the Legislature of
Governor from his post.
the State. The Ordinance expires after six weeks of the
(c) The Governor should be a member of either
reassembly of the Legislature unless it is approved by
House of the Parliment.
the Legislature. The Ordinance may be withdrawn at
(d) The Governor appoints the Chancellors of the
any time by the Governor.
Universities in the State.
14. Ordinance issued by the Governor of the any
Maharashtra PSC (Pre) G.S. 2017
State should be necessarily approved by?
Ans. (b) As per the Article 153 of the Constitution, the
(a) President
same person can be appointed as Governor for two or
(b) Legislature of the State more States at the same time. Hence statement 1 is
(c) Council of Ministers of the State incorrect. No procedure has been laid down in the
(d) None of the above Constitution for the removal of a Governor from his/her
UPPCS (Main) Spl. G.S. IInd 2008 post. Hence statement 2 is correct. According to Article
Ans. (b) Kindly refer the explanation of the above 158 of the Constitution, the Governor shall not be a
question. member of either House of Parliament or of a House of
the Legislature of any State specified in the First
15. Which one of the following statements is
Schedule and if a member of either House of Parliament
correct?
or of a House of the Legislature of any such State is
(a) In India, the same person cannot be appointed appointed Governor, he shall be deemed to have
as Governor for two or more States at the vacated his seat in that House on the date on which he
same time enters upon his office as Governor. Therefore statement
(b) The Judges of the High Court of the States in 3 is incorrect. Governor himself is the Chancellor of the
India are appointed by the Governor of the State Universities. Hence statement 4 is also incorrect.
State just as the Judges of the Supreme Court 17. Article 156 of the Constitution of India
are appointed by the President provides that the Governor shall hold the office
(c) No procedure has been laid down in the for a term of five years from the date on which
Constitution of India for the removal of a he enters upon his office. Which of the
Governor from his/her post following can be deduced from this?
(d) In the case of a Union Territory having a I. No Governor can be removed from the office
legislative setup, the Chief Minister is till completion of his term
appointed by the Lt. Governor on the basis of II. No Governor can continue in office beyond
majority support the period of five years.
IAs (Pre) G.S., 2013 Select the correct answer from the codes given
Ans. (c) As per the Article 153 of the Constitution, the below:
same person can be appointed as Governor for two or Codes:
more States at the same time. Hence statement 1 is (a) I only (b) II only
incorrect. The Judges of the High Court of the States in (c) Both I and II (d) Neither I nor II
India are appointed by the President. Hence statement 2 IAS (Pre) G.S., 1995
is also incorrect. No procedure has been laid down in Ans. (d) According to Article 156 (1) of the
the Constitution for the removal of a Governor from Constitution, the Governor shall hold office during the
his/her post. Governor of the State holds the post during pleasure of the President therefore he can be removed
the pleasure of the President. Hence statement 3 is from his post any time. Article 156 of the Constitution
Indian Polity & Constitution 132 YCT
asserts that a Governor shall hold office for a term of • State chief minister should have a say in the
five years from the date on which he enters upon his appointment of Governor.
office and continue to hold office until his successor • Governor should be a detached figure without
enters upon his office. Hence both the statements are intense political links or should not have taken part
incorrect. in politics in recent past.
18. Which of the following statements regarding • Governor should not be a member of the ruling
Governor is not correct?
party.
(a) He is part of the State Legislature
(b) He can pardon death penalty 21. The Chairman of Sarkaria Commission was-
(c) He does not appoint the Judges of the State (a) Ranjit Singh Sarkaria
High Court (b) Rajiv Singh Sarkaria
(d) He does not hold emergency powers (c) Sivaraman Singh Sarkaria
UPPCS (Main) G.S. IInd Paper 2005 (d) Ramesh Singh Sarkaria
Ans. (b) According to Article 161 of the Constitution, TNPSC (Pre) G.S. 2014
Governor has power to grant pardons, to suspend, Ans. (a) : Kindly refer the explanation of the above
remit or commute sentences in certain cases but in question.
case of pardoning death penalty, power lies
22. Which one of the following statements about
exclusively with President of India under Article 72 of
the Constitution. Rest of the options are correct the Governor of Maharashtra is true?
regarding Governor of a State. (a) He can nominate one member to the
19. Which among the following constitutes Legislative Council from the Anglo-Indian
‘Finance Commission’ to review the financial community.
condition of the Panchayats? (b) He can nominate at least two members to the
(a) Chief Minister of the concerned State Legislative Assembly from the Anglo-Indian
(b) Finance Minister of the concerned State community
(c) Governor of the concerned State (c) He has been given special responsibility
(d) Panchayati Raj Minister of the concerned under Article 371 (2).
State (d) He cannot reserve the bill for the
UPPCS (Pre) G.S., 2015 consideration of President, dealing with
Ans. (c) Article 243I of the Constitution prescribes that compulsory acquisition of property under
the Governor of a State shall constitute a Finance Article 31A of the Constitution
Commission to review the financial position of the Maharashtra PSC (Pre) G.S, 2019
Panchayats. Ans. (c) : According to Article 333 of the Constituion,
20. Which one of the following suggested that the the Governor of a State may, if he is of opinion that the
Governor should be an eminent person from Anglo Indian community needs representation in the
outside the State and should be a detached figure Legislative Assembly of the State and is not adequately
without intense political links or should not have
represented therein, nominate one member of that
taken part in politics in the recent past?
community to the Assembly. Hence both statement 1
(a) First Administrative Reforms Commission
and 2 are incorrect. According to Article 371(2) of the
(1966)
Constituion, Governor has been given special
(b) Rajamannar Committee (1969)
responsibility by the President for the establishment of
(c) Sarkaria Commission (1983)
separate development boards for Vidarbha, Marathwada
(d) National Commission to Review the Working
and the rest of Maharashtra, the equitable allocation of
of the Constitution (2000)
funds for developmental expenditure over the said
IAS (Pre) G.S, 2019
areas, an equitable arrangement providing adequate
Ans: (c) Government on June 9, 1983, constituted a
facilities for technical education and vocational training
Commission under the Chairmanship of Justice Ranjit
and adequate opportunities for employment in service
Singh Sarkaria with Shri B. Sivaraman and Dr. S.R. Sen
as its members to review the working of the existing under the control of the State Government, in respect of
arrangements between the Union and the States in the all the said areas. Hence statement 3 is correct. Under
changed socio-economic scenario. Chapter IV of the Article 31A (1) of the Constituion, Governor can
Sarkaria Commission report (1988) illustrated the role reserve the bill for the consideration of President,
of the Governor. Sarkaria Commission recommeded- dealing with compulsory acquisition of property under
• Governor should be an eminent person and not Article 31A of the Constitution. Hence statement 4 is
belong to the state where he is to be posted. incorrect.

Indian Polity & Constitution 133 YCT


22.
State Legislature
1. With reference to the Legislative Assembly of a 2. Consider the following statement : The
State in India, consider the following Constitution of India provides that
statements : 1. The Legislative Assembly of each State shall
1. The Governor makes a customary address to consist of not more than 450 members chosen
Members of the House at the commencement by direct election from territorial
of the first seasion of the year. constituencies in the State
2. A person shall not be qualified to be chosen
2. When a State Legislature does not have a rule
to fill a seat in the Legislative Assembly of a
on a particular matter it follows the Lok
State if he/she is less than 25 years of age
Sabha rule on that matter.
Which of the statements given above is/are
Which of the statements given above is/are
correct?
correct?
(a) 1 Only (b) 2 Only
(a) 1 only
(c) Both 1 and 2 (d) Neither 1 nor 2
(b) 2 only IAS (Pre) G.S., 2008
(c) Both 1 and 2 Ans. (b) According to Article 170 of the Constitution,
(d) Neither 1 nor 2 the Legislative Assembly of each State shall consist of
IAS (Pre) G.S, 2019 not more than five hundred and not less than sixty
Ans : (a) According to Article 176 (1) of the members chosen by direct election. Special provisions
Constituion, at the commencement of the first session have been made through amendments for some of the
after each general election to the Legislative Assembly States like Sikkim, Arunanchal, Mizoram and Goa.
and at the commencement of the first session of each Hence statement 1 is incorrect. A person shall not be
qualified to be chosen to fill a seat in the Legislative
year, the Governor shall address the Legislative
Assembly of a State if he/she is less than 25 years of
Assembly. Hence statement 1 is correct. Article 208 of
age and for the State Legislative Council, minimum age
the Constituion lays down rules of procedure. Article
is 30 years. Hence statement 2 is correct.
208 (1) asserts that a House of the Legislature of a State
3. According to Article 170 of the Indian
may make rules for regulating subject to the provisions
Constitution, the minimum and maximum
of the Constitution, its procedure and the conduct of its
number of members of the Legislative
business. Further, Article 208 (2) asserts that until rules
Assemblies of any State can be respectively-
are made under clause (1), the rules of procedure and
(a) 40 and 400 (b) 50 and 540
standing orders in force immediately before the
(c) 50 and 500 (d) 60 and 500
commencement of the Constitution with respect to the
UPPCS (Main) G.S. IInd Paper 2014
Legislature for the corresponding Province shall have
UPPSC RO/ARO (Pre) 2017
effect in relation to the Legislature of the State subject
Ans. (d) Kindly refer the explanation of the above
to such modifications and adaptations as may be made
question.
therein by the Speaker of the Legislative Assembly or
4. What can be the minimum strength of Council
the Chairman of the Legislative Council. It is nowhere
of Ministers including Chief Minister in the
mentioned in the Constituion that When a State
States?
Legislature does not have a rule on a particular matter it
(a)10 (b)12
follows the Lok Sabha rule on that matter. Hence
(c)13 (d)14
statement 2 is not correct.
UPPCS (Pre) G.S 2020
Indian Polity & Constitution 134 YCT
Ans. (b) Article 164(1A) of the Constituion asserts that Ans. (c) Legislative Council (Vidhan Parishad) is the
the total number of Ministers including the Chief upper house in the State. Just like Rajya Sabha, it is a
Minister in the Council of Ministers in a State shall not permanent house. It cannot be dissolved. The term of
exceed fifteen percent of the total number of members each member is 6 years and 1/3 members of the House
retire after every two years. Article 171of the
of the Legislative Assembly of that State provided that
Constitution is regarding composition of the Legislative
the number of Ministers, including the Chief Minister in
Councils which asserts that the total number of
a State shall not be less than twelve.
members in the Legislative Council of a State shall not
5. Consider the following statements: exceed one third (1/3) of the total number of members
1. The Legislative Council of a State in India in the Legislative Assembly of that State. Of the total
can be larger in size than half of the number of members of the Legislative Council of a
Legislative Assembly of that particular State State, one third (1/3) shall be elected by electorates
2. The Governor of a State nominates the consisting of members of municipalities, district boards
and such other local authorities in the State, one twelfth
Chairman of Legislative Council of that
(1/12) shall be elected by electorates consisting of
particular State.
persons residing in the State who have been for at least
Which of the statements given above is/are three years graduates of any university in the territory of
correct? India and one twelfth (1/12) shall be elected by
(a) 1 only (b) 2 only electorates consisting of persons who have been for at
(c) Both 1 and 2 (d) Neither 1 nor 2 least three years engaged in teaching in such
IAS (Pre) G.S., 2015 educational institutions within the State, not lower in
standard than that of a secondary school, one third shall
Ans. (d) According to Article 171 of the Constitution,
be elected by the members of the Legislative Assembly
the total number of members in the Legislative Council
of the State from amongst persons who are not members
(Vidhan Parishad) of a State shall not exceed one third of the Assembly and the remainder (1/6) shall be
(1/3) of the total number of members in the Legislative nominated by the Governor.
Assembly (Vidhan Sabha) of that State. Hence 7. The number of nominated members in the
statement 1 is incorrect. According to Article 182 of the Legislative Council is
Constitution, members of the Legislative Council (a) 1/3 of the total strength
choose Chairman of the Council. Hence statement 2 is (b) 1/4 of the total strength
also incorrect. (c) 1/6 of the total strength
6. What is correct about State Legislative (d) None of the above
Council? UPPCS (Mains) G.S. IInd 2017
(i) Its tenure is 6 years Ans. (c) Kindly refer the explanation of the above
(ii) It is a permanent house question.
(iii) It cannot be dissolved 8. Consider the following statements:
(iv) 1/6 members are elected by local institutions 1. Odisha Legislative Assembly passed
(v) 1/6 members are elected by Legislative resolution for creation of Legislative Council
recently.
Assembly
2. Currently, five states have Legislative
(vi) On each two years 1/3 members are retired
Councils.
(vii) Deputy Governor is Chairman of the house
3. Proposals to create Legislative Councils in
(viii) The tenure of its member is 6 years Kerala and Gujarat are pending in Parliament.
(a) (i) (iii) (iv) (v) 4. 1/12th members of Legislative Councils are
(b) (iii) (vi) (vi) (viii) elected by registered graduates.
(c) (ii) (iii) (vi) (viii) Which of the above statements are correct?
(d) (ii) (v) (vi) (viii) (a) 1, 2 and 3 (b) 2 and 3
(e) (i) (iii) (vi) (vii) (c) 3 and 4 (d) 1 and 4
st Maharashtra PSC (Pre) G.S, 2019
Chattisgarh PSC (Pre) 1 G.S., 2014
Indian Polity & Constitution 135 YCT
Ans. (d) : On 6th September, 2018, Odisha Assembly Ans. (c) Kindly refer the explanation of the above
passed a resolution for the creation of Legislative question.
Council. Hence statement 1 is correct. As of January 12. No Money Bill can be introduced in the
2020, six out of 28 states have a Legislative Council. Legislative Assembly of a State, except on the
The States with bicameral legislature include Andhra recommendations of
Pradesh, Bihar, Karnataka, Maharashtra, Telangana and (a) The Parliament
Uttar Pradesh. These states have both the Legislative (b) The Governor of the State
Council and Legislative Assembly. Hence statement 2 is (c) The President of India
not correct. Proposals to create Legislative Councils in (d) A special committee of ministers
Rajasthan and Assam are pending in Parliament. Hence UPPCS (Pre) G.S., 2012
statement 3 is also incorrect. 1/12th members of Ans. (b) No Money Bill can be introduced in the
Legislative Councils are elected by registered graduates. Legislative Assembly of a State except on the
Hence statement 4 is correct. recommendations of the Governor of the State.
9. Which of the following states does not have a 13. Which of the following States in India does not
bicameral legislature? have a Legislative Council so far even though
the Constitution (Seventh Amendment) Act,
(a) Tammu and Kashmir
1956 provides for it?
(b) Telangana
(a) Maharashtra (b) Bihar
(c) Karnataka
(c) Karnataka (d) Madhya Pradesh
(d) West Bengal IAS (Pre) G.S., 1995
Himanchal PSC (Pre) G.S 2019 Ans. (d) Madhya Pradesh does not have a Legislative
Ans. (d) : Kindly refer the explanation of the above Council so far even though the Constitution (Seventh
question. Amendment) Act, 1956 made provision for it.
10. Under which one of the following Articles of Currently, Andhra Pradesh, Bihar, Karnataka,
the Indian Constitution, provision of creation Maharashtra, Telangana and Uttar Pradesh have
or abolition of Legislative Councils in States is Legislative Councils.
made? 14. Which one of the following States does not have
(a) Article 168 (b) Article 169 a bicameral legislature?
(a) U.P. (b) M.P.
(c) Article 170 (d) Article 171
(c) Bihar (d) Karnataka
UPPCS (Pre.) G.S. 2011, 2013
Ans. (b) Kindly refer the explanation of the above
Ans. (b) Article 169 of the Constitution is regarding
question.
abolition or creation of Legislative Councils in States.
15. If in an election to a State Legislative
According to Article 169, Parliament may by law
Assembly, the candidate who is declared
provide for the abolition of the Legislative Council of a
elected loses his deposit, it means that
State having such a Council or for the creation of such a
(a) The polling was very poor
Council in a State having no such Council, if the
(b) The election was for a multi-member
Legislative Assembly of the State passes a resolution to constituency
that effect by a majority of the total membership of the (c) The elected candidate’s victory over his
Assembly and by a majority of not less than two third of nearest rival was very marginal
the members of the Assembly present and voting. (d) A very large number of candidates contested
11. Which of the following Legislative house can be the election
abolished? IAS (Pre.) G.S., 1995
(a) Lok Sabha (b) Rajya Sabha Ans. (d) As per the Election Commission of India rules,
(c) Vidhan Parishad (d) Vidhan Sabha if the candidate fails to get a minimum of one-sixth
RAS/RTS (Pre) G.S., 2016 (1/6) of the total valid votes polled, the deposit goes to
Indian Polity & Constitution 136 YCT
the treasury. Such situation may arise if a very large from the date on which the Bill is laid before the
number of candidates contested the election and votes Council without the Bill being passed by it or the Bill is
polled are divided among them. Victory of any passed by the Council with amendments to which the
candidate is decided on numbers of votes polled in Legislative Assembly does not agree; the Bill shall be
favour of him irrespective of condition of the deposit. deemed to have been passed by the Houses of the
Legislature of the State in the form in which it was
16. Which one of the following is responsible for
passed by the Legislative Assembly for the second time
the establishment of ‘Contingency Fund’? with such amendments as have been made or suggested
(a) Legislature of any State by the Legislative Council and agreed to by the
(b) Parliament Legislative Assembly. Therefore the Legislative
(c) President Council can withhold a bill only for 4 months (3 months
(d) None of the above on first instance and 1 month on second). Also if a bill
UP RO/ARO (Main) G.S., 2014 passed by Legislative Council is disapproved by
Legislative Assembly, then it becomes dead.
Ans. (a) Contingency Fund is created as an impress
account to meet some urgent or unforeseen expenditure 19. In the case of a bill passed by the State
Legislature, Governor can reserve it for the
of the Government. According to Article 267(2) of the
consideration of the President.
Constituion, the Legislature of a State may by law
1. The President may declare or withhold his
establish a Contingency Fund.
assent to the bill.
17. State Legislature takes parts in the election of 2. The President may return the bill for
which of the following? reconsideration to the State Legislature.
I. President of India 3. State Legislature must reconsider the bill
II. Vice- President of India within three months and if passed again it is
III. Members of the Rajya Sabha obligatory upon the President to give his
IV. Members of the State Legislative Council assent.
Choose the correct answer- 4. In case of reconsidered bill which is passed
(a) I, II and III (b) I, III and IV again by State Legislature, President shall
(c) I and II (d) I, II and IV declare his assent within a period of six
Uttarakhand PCS (Pre)1st Paper 2014 months.
Himanchal PSC (Pre) G.S 2014 Which of the statements given above are
correct?
Ans. (b) State Legislature takes parts in the election of
(a) 1 and 2 (b) 1, 2 and 3
President of India, Members of the Rajya Sabha and
(c) 1, 2 and 4 (d) 1, 3 and 4
Members of the State Legislative Council.
Maharashtra PSC (Pre) G.S, 2019
18. State Council can hold an ordinary Ordinance
up to which of the following time period? Ans. (a) : According to Article 201 of the Constituion,
(a) 3 months (b) 4 months when a Bill is reserved by a Governor for the
consideration of the President, the President shall
(c) 6 months (d) 14 days
declare either that he assents to the Bill or that he
UPPCS (Main) G.S. IInd Paper 2005
withholds assent provided that where the Bill is not a
Ans. (b) According to Article 197 of the Constitution, if
Money Bill. Further, the President may direct the
after a Bill has been passed by the Legislative Assembly
Governor of the State to return the Bill to the State
of a State having a Legislative Council and transmitted
Legislature for reconsideration. Hence both statement 1
to the Legislative Council, the Bill may be rejected by
and 2 are correct. Article 201 asserts that when a Bill is
the Council or more than three months elapse from the
returned by the Presient to the State Legislature for
date on which the Bill is laid before the Council without
reconsideration, the House or Houses shall reconsider it
the Bill being passed by it or the Bill is passed by the
Council with amendments to which the Legislative accordingly within a period of six months from the date
Assembly does not agree; the Legislative Assembly of receipt of such message and if it is again passed by
may pass the Bill again with or without such the House or Houses with or without amendment, it
amendments as have been made by the Legislative shall be presented again to the President for his
Council and then transmit the Bill to the Legislative consideration. It is not mandatory for the President to
Council for the second time . Then the Bill may be give his assent on such Bill. Hence both statement 3 and
rejected by the Council or more than one month elapses 4 are not correct.
Indian Polity & Constitution 137 YCT
20. Consider the following statements: III. The Chief Minister participates in the meeting
1. The Speaker of the Legislative Assembly of National Development Council
shall vacate his/her office if he/she ceases to IV. The Chief Minister submits for the
be a member of the assembly.
consideration of the Council of Ministers any
2. Whenever the Legislative Assembly is
matter on which decision has been taken by a
dissolved, the Speaker shall vacate his/her
minister but which has not been considered
office immediately.
by the council as if the Governor requires
Which of the statements given above is/are
correct? Code:
(a) 1 only (b) 2 only (a) I and II (b) I and IV
(c) Both 1 and 2 (d) Neither 1 nor 2 (c) I, II and III (d) I, II and IV
IAS (Pre.) G.S, 2018 RAS/RTS (Pre) G.S, 2013
Ans. (a) Usually, the Speaker remains in office during Ans. (d) Article 167 of the Constitution is regarding
the life of the particular Assembly. However Speaker duties of the Chief Minister. It asserts that it is the duty
vacates his office earlier in any of the following three of the Chief Minister to communicate to the Governor
circumstances:
of the State all decisions of the Council of Ministers
1. If he ceases to be a member of the assembly.
relating to the administration of the affairs of the State
2. If he resigns by writing to the Deputy Speaker.
and proposals for legislation and if the Governor so
3. If he is removed by a resolution passed by a
requires, to submit for the consideration of the Council
majority of all the then members of the assembly.
Such a resolution can be moved only after giving of Ministers any matter on which a decision has been
14 days advance notice. Hence statement 1 is taken by a Minister but which has not been considered
correct. Article 179 of the Constitution provides by the Council. It is not mandatory for the Chief
that whenever the Assembly is dissolved, the Minister to attain the meeting of National Development
Speaker shall not vacate his office until Council. He may send his representative to participate
immediately before the first meeting of the newly in the meeting of the National Development Council.
elected Assembly after the dissolution. Hence,
23. Salary and allowances of the ministers of the
statement 2 is not correct.
State Government is determined by-
21. If budget is disclosed before introducing in the
(a) Chief Minister (b) Governor
Legislative Assembly, what will happen?
(a) Council of Ministers will have to resign (c) Chief Secretary (d) State Legislature
(b) Chief Minister will have to resign UPPCS (Mains) Spl. G.S., 2008
(c) Finance Minister will have to resign Ans. (d) Salary and allowances of the ministers of the
(d) All of the above State Government is determined by the respective State
(e) None of the above Legislature.
Chhatisgarh PSC (Pre) G.S, 2016 24. Consider the following statements:
Ans. (e) Highest level of secrecy is maintained to 1. The Chief Secretary in a State is appointed by
prevent the leakage of budget before presenting it to the Governor of that State.
Lok Sabha/ State Legislature by the Finance Minister.
2. The Chief Secretary in a State has a fixed
Nevertheless, if it is disclosed, there is no effect on
Chief Minister, Finance Minister or the Council of tenure.
Ministers. Which of the statements given above is/are
22. Select the Constitutional duties of the Chief correct?
Minister from the following by using codes (a) 1 only (b) 2 only
given below: (c) Both 1 and 2 (d) Neither 1 nor 2
I. The Chief Minister communicates to the IAS (Pre) G.S., 2016
Governor all decisions of the Council of
Ans. (d) The Chief Secretary in a State is appointed by
Ministers related to the administration of the
the Chief Minister of that State and the Chief Secretary
affairs of the State
II. The Chief Minister communicates to the in a State has no fixed tenure as he holds the office upon
Governor the proposal for the legislation the pleasure of the Chief Minister.

Indian Polity & Constitution 138 YCT


23.
Special Provisions for some States
1. Arrange the following events in a chronological 2. The tenure of the Chief Minister of the Jammu
order and select the correct answer from the & Kashmir is-
codes given below: (a) Four years (b) Five years
I. Maharaja Hari Singh signed the instrument of (c) Six years (d) Seven years
Accession UPPCS Spl. (Pre.) G.S., 2008
II. Constitution of India was enforced Ans. (*) Kindly refer the explanation of the above
III. Accession to India ratified by the sovereign question.
Constituent Assembly of Jammu and Kashmir 3. The Governor has the power to impose
IV. New Constitution of the State of Jammu and Governor's rule in the State of:
Kashmir came into force (a) Tripura (b) Nagaland
(c) Mizoram (d) Jammu and Kashmir
Code :
Himanchal PSC (Pre) G.S, 2011
(a) I, III, IV, II (b) III, IV, I, II
Ans. (*) Kindly refer the explanation of the above
(c) I, II, III, IV (d) IV, III, II, I
question.
UPPCS (Pre) G.S, 2019
4. Among which of the following States nominates
Ans. (c) Maharaja Hari Singh of Jammu and Kashmir two woman representatives to the State
signed the Instrument of Accession with the Indian Assembly?
Government on October 26, 1947. The Constituent (a) Himanchal Pradesh
Assembly of Jammu and Kashmir convened on October (b) Kerala
31, 1951. The Constitution of the Indian State of Jammu (c) Jammu and Kashmir
and Kashmir was formally adopted by Constituent (d) Uttar Pradesh
Assembly of Jammu and Kashmir on November 17, 1956 UPPCS (Pre.) G.S., 2004
and the Constitution entered into force on January 26, Ans. (*) Kindly refer the explanation of the above
1957. Article 370 of the Constitution of India was a question.
temporary provision which grants special autonomous
5. The nomenclature of the Executive Head of the
status to Jammu and Kashmir. The Jammu and Kashmir Government of Jammu and Kashmir was
Assembly was bicameral legislature consist of Assembly changed From Sadar-i-Riyasat to Governor in
and Council. The tenure of the Jammu and Kashmir 1965 by
Assembly was six years. Hence the tenure of the Chief (a) Lok Sabha Resolution
Minister of Jammu and Kashmir was six years. (b) The Executive Order of the President
Note: On 5th August, 2019, Government of India (c) The 6th Amendment in the State Constitution
introduced Jammu and Kashmir (Reorganization) Bill, of J&K.
2019, to repeal Article 370 (Except the first clause of (d) The State Government under Article 371.
Article 370) of the Constituion. Under article 370(3), UPPCS (Main) G.S. IInd Paper 2007
there is a provision that President on recommendation Ans. (c) The Constitution of Jammu and Kashmir (Sixth
of the Parliament has the power to amend or cease the Amendment) Act, 1965 amended the State Constitution
implementation of article 370, through a public and replaced the words ‘Sadar-i-Riyasat’ with Governor
notification. President of India gave his assent to and ‘Prime Minister’ with Chief Minister.
Jammu and Kashmir (Reorganisation) Act, 2019 on 9th 6. Which Article of the Constitution of India
August, 2019 which made Jammu and Kashmir a Union provided special status to the State of Jammu
territory with a Legislative Assembly along with Union and Kashmir?
Territory status to Ladakh without a Legislative (a) Article 360 (b) Article 370
Assembly. Two Union Territories (Jammu and Kahmir, (c) Article 375 (d) Article 378
Ladakh) came into existence on 31st October, 2019. Uttarakhand- PSC (Pre) - 2016
Indian Polity & Constitution 139 YCT
Ans. (*) Kindly refer the explanation of the above Pradesh and 371I is regarding special provisions with
question. respect to Goa. Hence statement 1 is correct. Constitution
7. Articles of the Constitution of India that apply of India and United States of America envisage a dual
on their own to the State of Jammu and policy (The Union and the States) but United States of
Kashmir, are America has dual citizenship while Republic of India has
(a) Articles 1 and 241 single citizenship. Hence statement 2 is incorrect. Any
(b) Articles 2 and 356 citizen of India can be deprived of his/her citizenship by
(c) Articles 1 and 370 the provisions (naturalization, registration, by voluntarily
(d) Articles 370 and 371A acquiring citizenship of another country) mentioned in the
UPPCS (Main) G.S. II Paper, 2016 Constitution of India. Hence statement 3 is also incorrect.
nd

Ans. (c) Article 1 and 370 of the Constitution of India is 9. Article 371 B of the Constitution of India
applicable on their own to the State of Jammu and makes special provision for which of the
Kashmir. Article 1 is name and territory of the Union following State(s)?
and Article 370 was temporary provisions with respect (a) Maharashtra and Gujarat
to the State of Jammu and Kashmir which granted (b) Assam
special autonomous status to Jammu and Kashmir. (c) Nagaland
8. Consider the following statements: (d) Manipur
1. Article 371A to 371I were inserted in the UPPCS (Main) G.S. IInd Paper 2010
Constitution of India to meet regional Ans. (b) Kindly refer the explanation of the above
demands of Nagaland, Assam, Manipur question.
Andhra Pradesh, Sikkim, Mizoram,
10. Which region of the country conferred special
Arunanchal Pradesh and Goa.
status under Article 371-J of the Constitution
2. Constitution of India and United States of of India?
America can envisage a dual policy (The
(a) Nagaland
Union and the States) but a single citizenship.
(b) Hyderabad and Karnataka
3. A neutralized citizen of India can never be
(c) Maharashtra and Gujarat
deprived of his citizenship.
(d) Ladakh
Which of the following statements given above
UPPCS (Pre) G.S 2020
is/are correct?
Ans. (b) By the Constituion (Ninety-Eighth
(a) 1, 2 and 3
Amendment)Act, 2013, Article 317-J was inserted into
(b) 1 and 3
Constituion which is regarding establishment of a
(c) 3 only separate development board for Hyderabad- Karnataka
(d) 1 only region.
IAS (Pre) G.S., 2005 11. The reference to National Capital Territory of
Ans. (d) Article 371A to 371 I were inserted in the Delhi is found in
Constitution of India to meet the regional demands of (a) Article 239 A (b) Article 239 AA
Nagaland, Assam, Manipur Andhra Pradesh, Sikkim, (c) Article 239 AB (d) Article 239 B
Mizoram, Arunanchal Pradesh and Goa. Article 371 is Maharashtra PSC (Pre) G.S. 2017
regarding special provision with respect to the Nagaland,
Ans. (b) According to Article 239 AA of the
371B is regarding special provision with respect to the Constitution, as from the date of commencement of
Assam, 371C is regarding Manipur, 371 D is regarding the Constitution (Sixty-Ninth Amendment) Act, 1991,
Andhra Pradesh, 371 E is regarding establishment of the Union territory of Delhi shall be called the National
central university in Andhra Pradesh, 371 F is regarding Capital Territory of Delhi and the administrator
special provisions with respect to Sikkim, 371 G is appointed under Article 239 shall be designated as the
regarding Mizoram, 371H is regarding Arunanchal Lieutenant Governor.
Indian Polity & Constitution 140 YCT
24.
Supreme Court-Establishment,
Empowerment and Jurisdiction
1. Consider the following statements: 2. With reference to the establishment of
1. The Constitution of India defines its ‘basic Supreme Court, which of the statement is/are
structure’ in terms of federalism, secularism, correct?
fundamental rights and democracy. 1. Supreme Court at Calcutta was established
2. The Constitution of India provides for by the Regulating Act of 1773
‘Judicial review’ to safeguard the citizens’ 2. Lemaister was the first Chief Justice of this
liberties and to preserve the ideals on which court
the Constitution is based. Select the correct answer using codes given
Which of the statements given above is/are below:
correct? Codes:
(a) 1 only (a) 1 only (b) 2 only
(b) 2 only (c) Both 1 and 2 (d) Neither 1 nor 2
(c) Both 1 and 2 UPPSC RO/ARO (Pre), 2016
(d) Neither 1 nor 2
(Held on 20/09/2020)
IAS (Pre) G.S. 2020
Ans. (a) The promulgation of Regulating Act of 1773
Ans. (b) The Supreme Court recognized Basic Structure by the King of England paved the way for establishment
concept for the first time in the historic Kesavananda of the Supreme Court of Judicature at Calcutta. The
Bharati case in 1973. The Supreme Court declared that
Supreme Courts at Madras and Bombay was established
Article 368 did not enable Parliament to alter the basic
by King George – III on 26 December 1800 and on 8
structure or framework of the Constitution and
December 1823 respectively. Hence statement 1 is
parliament could not use its amending powers under
correct. Sir Elijah Impey was appointed as the first
Article 368 to damage, emasculate, destroy, abrogate,
Chief Justice of Supreme Court at Calcutta. Hence
change or alter the basic structure or framework of the
statement 2 is not correct.
Constitution. Various features like independent
3. The Supreme Court of India enunciated the
Judiciary, rule of law, free and fair elections, separation
doctrine of “Basic Structure of the
of powers etc. emerged as basic structure of the
Constituion from various judgements given by Supreme Constitution” in
Court. Hence statement 1 is not correct. Judicial review (a) The Golaknath Case in 1967
is defined as ‘the power of the Court to determine (b) The Keshavanand Bharati Case in 1973
whether the acts of Legislature and Executive are (c) The Shankari Prasad Case in 1951
consistent with the Constitution or the Constitutional (d) The Sajjan Singh Case in 1965
values’. The concept of judicial review lies in the UPPCS (Pre) G.S 2020
supremacy of the Constitution of the land. Therefore Ans. (b) The Supreme Court recognized Basic Structure
judicial review safeguards the citizen’s liberty and concept for the first time in the historic Kesavananda
preserves the ideals of the Constituion. Hence statement Bharati case in 1973. The Supreme Court declared that
2 is correct. Article 368 did not enable Parliament to alter the basic

Indian Polity & Constitution 141 YCT


structure or framework of the Constitution and 5. In India, the power of judicial review is
parliament could not use its amending powers under enjoyed by
Article 368 to damage, emasculate, destroy, abrogate, (a) The Supreme Court alone
change or alter the basic structure or framework of the (b) The Supreme Court as well as High Courts
Constitution. (c) All the Courts
4. In India, Judicial Review implies (d) None of the above
(a) The power of the judiciary to pronounce upon UPPCS (Main) G.s. IInd 2017
the constitutionality of laws and executive Ans. (b) Kindly refer the explanation of the above
orders. question.
(b) The power of the judiciary to question the
6. Federal Court of India was established in
wisdom of the laws enacted by the
which year?
legislatures.
(a) 1935 (b) 1937
(c) The power of the judiciary to review all the
(c) 1946 (d) 1947
legislative enactments before they are
UPPCS (Main) G.S. IInd Paper 2015
assented to by the President.
(d) The power of the judiciary to review its own Ans. (b) Federal Court of India was established in 1937
judgments given earlier in similar or different under the provisions of Government of India Act, 1935.
cases It was instituted with a Chief Justice and two additional
Judges. The first Chief Justice was Sir Maurice
Maharashtra PSC (Pre) G.S, 2019
Gwyer and the other two judges were Sir Shah
IAS (Pre) G.S, 2017
Muhammad Sulaiman and M. R. Jayakar.
Ans. (a) : The concept of judicial review has been taken
7. The Supreme Court in India was inaugurated
from U.S.A. Judicial review is defined as ‘the power of
on which date?
the Court to determine whether the acts of Legislature
and Executive are consistent with the Constitution or (a) 27th January, 1950
the Constitutional values’. The concept of judicial (b) 28th January, 1950
review lies in the supremacy of the Constitution of the (c) 29th January, 1950
land. Since the Judiciary is the guardian of the (d) 30th January, 1950
Constitution, therefore it is under its purview to check MPPSC (Pre) GS 1st Paper 2013
actions which are inconsistent with the Constitution. Ans. (b) The Supreme Court of India was inaugurated
The tool of judicial review empowers the judiciary to on 28th January, 1950. It succeeded the Fedral Court of
struck down any action which is in conflict with the India estabilished under the Government of India Act
Constitution. In the Constitution of India, the principle 1935.
lies under Article 13. Article 13 of the Constituion of H.J. Kania was the first Chief Justica of India.
India provides that the law to be made should be in line
8. When was Supreme Court of India
with the norms laid down in the Constitution of India.
inaugurated?
In the Keshvananda Bharati Case, the judicial review
(a) Jan 26, 1950 (b) Jan 26, 1950
was ultimately held to be the basic structure of the
(c) Jan 28, 1950 (d) April 26, 1950
Indian Constitution. In another case, Supreme Court
TNPSC (Pre) G.S. 2016
held that power of judicial review under Article 32
(Supreme Court) and 226 (High Courts) is an integral Ans. (c) Kindly refer the explanation of the above
and essential feature of the basic structure of our question.
Constitution. Further, Article 137 of the Constituion 9. The current approved number of Judges of
asserts that the Supreme Court shall have power to Supreme Court of India is
review any judgment pronounced or order made by it. (a) 20 (b) 25
Hence it does not come under judicial review. Therefore (c) 30 (d) 31
only statement 1 is correct. UPPCS (Main) G.S. IInd Paper 2015
Indian Polity & Constitution 142 YCT
Ans. (*) After the Supreme Court (Number of Judges) which asserts that the Supreme Court shall have such
Amendment Bill was passed by Parliament in August further jurisdiction and powers with respect to any of
2019, the sanctioned strength of Supreme Court will be the matters in the Union List as Parliament may by law
33, besides the Chief Justice of India. confer. Hence statement 1 is incorrect. Article 146
10. The Supreme Court of India, consits of one asserts that the appointments of officers and servants of
Chief justice and- the Supreme Court shall be made by the Chief Justice of
(a) 32 other Judges India and administrative expenses of the Supreme
(b) 33 other Judges Court, including all salaries, allowances and pensions
(c) 34 other Judges payable to or in respect of the offices and servants of
(d) None of the above the Court shall be charged upon the Consolidated Fund
UPPSC ACF-RFO Mains (IInd Paper), 2019 of India. Article 229 asserts that appointments of
officers and servants of a High Court shall be made by
Ans. (b) : Kindly refer the explanation of the above
the Chief Justice of the respective High Court and the
question.
administrative expenses of a High Court, including all
11. Who /which of the following is the custodian of
salaries, allowances and pensions payable to or in
the Constitution of India?
respect of the officers and servants of the court shall be
(a) The President of India
charged upon the Consolidated Fund of the State. Hence
(b) The Prime Minister of India statement 2 is also incorrect.
(c) The Lok Sabha Secretariat
13. The minimum number of Judges of the
(d) The Supreme court of India
Supreme Court required for hearing any case
UPPCS (Main) G.S. IInd Paper 2015
involving interpretation of the Constitution is
MPPSC (Pre) GS 1st Paper 2015-16
(a) Ten
IAS (Pre) G.S., 2015
(b) Nine
Ans. (d) Supreme Court of India is the highest court of (c) Seven
the country which acts as the custodian of the
(d) Five
Constitution and serves as the final court of appeal. The
UPPCS (Pre.) G.S., 2012
Supreme Court and High Courts can declare a law as
UPPCS (Main) G.S. IInd Paper 2012
unconstitutional if it contravenes any provisions of the
Constitution. Ans. (d) According to the Article 145(3) of the
Constitution, The minimum number of Judges who are
12. Consider the following statements-
to sit for the purpose of deciding any case involving a
1. The Parliament cannot enlarge the
substantial question of law as to the interpretation of
jurisdiction of the Supreme Court of India as
Constitution shall be five (5).
its jurisdiction is limited to that conferred by
the Constitution. 14. The Supreme Court does hear the cases in New
2. The officers and servants of the Supreme Delhi but can also hear it any other place
Court and High Courts are appointed by the (a) On the recommendation of the President
concern Chief Justice and the administrative (b) If the Judges of the Supreme Court decides it
expenses are charged on the Consolidated by majority
Fund of India. (c) On the recommendation of the Parliament
Which of the statements is/are correct? (d) On the request of the State Legislature
(a) 1 only (b) 2 only UPPCS (Main) Spl. G.S. IInd 2008
(c) Both 1 and 2 (d) Neither 1 nor 2 Ans. (a) According to Article 130 of the Constitution,
IAS (Pre.) G.S., 2005 the Supreme Court shall sit in Delhi or in such other
Ans. (d) Article 138 of the Constitution is regarding the place as the Chief Justice of India may with the
enlargement of the jurisdiction of the Supreme Court approval of the President.

Indian Polity & Constitution 143 YCT


15. Who appoints the Judges of the Supreme the previous consent of the President and after
Court of India? consultation with the Chief Justice of the High Court
(a) Prime Minister concerned, request in writing the attendance at the
(b) President sittings of the Court as an ad hoc Judge for such period
(c) Chief Justice of India as may be necessary of a Judge of a High Court duly
(d) Lokpal qualified for appointment as a Judge of the Supreme
(e) None of the above/ more than one from above Court to be designated by the Chief Justice of India.
Further Article 224 (1) of the Constitution asserts that if
BPSC (Pre) G.S., 2017
by reason of any temporary increase in the business of
Ans. (b) According to Article 124 of the Constitution,
the High Court, it appears to the President that the
every Judge of the Supreme Court shall be appointed by
number of the Judges of that court should be for the
the President after consultation with the Judges of the
time being increased, the President may appoint duly
Supreme Court.
qualified persons to be additional Judges of the High
16. When the office of the Chief Justice of India is Court for such period not exceeding two years.
vacant or when the Chief Justice is, by reason
18. In order to appoint as the Judge of the
of absence or otherwise, unable to perform the
Supreme Court, a person should be the
duties of his office, the duties of the office are
advocate of the High Court for atleast
performed by an acting Chief Justice. Who
(a) 20 years (b) 10 years
appoints the acting Chief Justice?
(c) 8 years (d) 25 years
(a) President of India
Chattisgarh PSC (Pre) G.S. 2004
(b) Seniormost Judge of the Supreme Court (Ex-
officio) Ans. (b) The norms relating to the eligibility for the
Judge of the Supreme Court has been envisaged in the
(c) Prime Minister of India
Article 124 (3) of the Constitution. In terms of age, a
(d) Vice-President of India
person should not exceed 65 years of age. Article 124
Punjab PSC (Pre) G.S 2013
(3) further laid out that to become elevated as a Judge of
Ans. (a) According to the Article 126 of the the Supreme Court, a person should serve as a Judge of
Constitution, when the office of Chief Justice of India is the one of the High Courts or the person should be an
vacant or when the Chief Justice is, by reason or advocate in the Supreme Court for at least 10 years or a
absence or otherwise, unable to perform the duties of distinguished jurist.
his office, the duties of the office shall be performed by
19. What is the age of the retirement of the Judge
such one of the other Judges of the Supreme Court as
of the Supreme Court?
the President may appoint for the purpose.
(a) 60 years (b) 62 years
17. Who can appoint an ad hoc judge in the
(c) 65 years (d) 70 years
Supreme Court?
(e) None of the above/ more than one of above
(a) President of India
BPSC (Pre) G.S. 2017
(b) Minister of Law
Ans. (c)The age of retirement of a Supreme Court Judge
(c) Collegium of the Supreme Court
is 65 years.
(d) Chief Justice of India with the Permission of
20. After retirement, Supreme Court Judge could
the President
practice-
Himanchal PSC (Pre) G.S. 2012
(a) Only in Supreme Court
Ans. (d) Article 127 of the Constitution is regarding
(b) Only in High Court
appointment of ad hoc Judges. Article 127 asserts that if
(c) In Supreme Court and High Court
at any time there should not be a quorum of the Judges
of the Supreme Court available to hold or continue any (d) Not in any court
session of the Court, the Chief Justice of India may with UPPCS (Pre.) G.S., 1997

Indian Polity & Constitution 144 YCT


Ans. (d) According to Article 124 (7) of the Which of the statements given above is/are
Constitution, no person who has held office as a Judge correct?
of the Supreme Court shall plead or act in any court or (a) 1 and 2 (b) 3 only
before any authority within the territory of India. (c) 3 and 4 only (d) 1, 3 and 4
IAS (Pre) G.S, 2019
21. A judge of the Supreme Court may resign his
office by writing a letter to Ans: (c) The Judges (Inquiry) Act, 1968 is an Act to
regulate the procedure for the investigation and proof of
(a) The Chief Justice
misbehavior or incapacity of a Judge of Supreme Court
(b) The President or High Courts and for the presentation of an address by
(c) The Prime Minister Parliament to the President. It provides details of the
(d) The Law Minister process of impeachment of the Judges of the Supreme
UPPCS (Pre.) G.S., 2014 Court and High Courts of India on the grounds of
Ans. (b) According to Article 124(2) (a) of the ‘proved misbehaviour or incapacity’. Hence statement 3
is correct. Accoring to clause 3 (b) of the Judges
Constitution, a Judge may by writing under his hand
(Inquiry) Act, 1968, Speaker of the Lok Sabha either
addressed to the President, resign his office.
admit the motion to impeach the Judge or refuse to
22. Salaries of the Judges of the Supreme Court admit the same. Hence statement 1 is incorrect. The
are determined by Judges of the Supreme Court and High Courts can be
(a) Pay Commission appointed by then President. removed on the ground of proved misbehaviour or
(b) Law Commission incapacity but the Constitution of India neither defines
(c) Parliament nor gives details of what constitutes 'incapacity and
proved misbehaviour' of the Judges of the Supreme
(d) Council of Ministers
Court of India. Hence statement 2 is also incorrect. The
UPPCS (Main) G.S. IInd Paper 2008
Constitution also provides checks against misbehaviour
Ans. (c) According to Article 125 of the Constitution, by Judges. It states that a Judge may be removed only
Salaries of the Judges of the Supreme Court are through a motion in Parliament with a two third support
determined by the Parliament. in each House. The process is laid down in the Judges
23. Consider the following statements: (Inquiry) Act, 1968. A motion has to be moved by
either 100 Lok Sabha members or 50 Rajya Sabha
1. The motion to impeach a judge of the
members. If the motion is admitted, the Speaker of Lok
Supreme Court of India cannot be rejected by
Sabha or Chairman of Rajya Sabha constitutes an
the Speaker of the lok Sabha as per the
inquiry committee. The committee has three members: a
Judges (Inquiry) Act 1968 Supreme Court judge, a High Court Chief Justice and an
2. The Constitution of India defines and gives eminent jurist. Hence statement 4 is correct.
details of what constitutes 'incapacity and 24. What is the provision to safeguard the
proved misbehaviour' of the Judges of the autonomy of the Supreme Court of India?
Supreme Court of India. 1. While appointing the Supreme Court Judges,
3. The details of the process of impeachment of the President of India has to consult the Chief
the Judges of the Supreme Court of India are Justice of India.
given in the Judges (Inquiry) Act. 1968. 2. The Supreme Court Judges can be removed
4. If the motion for the impeachment of a Judge by the Chief Justice of India only.
is taken up for voting, the law requires the 3. The salaries of the Judges are charged on the
motion to be backed by each House of the Consolidated Fund of India to which the
Parliament and supported by a majority of legislature does not have to vote.
4. All appointments of officers and staffs of the
total membership of that House and by not
Supreme Court of India are made by the
less than two-thirds of total members of that
Government only after consulting the Chief
House present and voting.
Justice of India.
Indian Polity & Constitution 145 YCT
Which of the statements given above is/are Article 368 to damage, emasculate, destroy, abrogate,
correct? change or alter the basic structure or framework of the
(a) 1 and 3 only (b) 3 and 4 only Constitution.
(c) 4 only (d) 1, 2, 3 and 4 26. In which of the following cases did the
IAS (Pre) G.S., 2012 Supreme Court of India pronounce the verdict
Ans. (a) According to Article 124 of the Constitution, that the basic structure of the Constitution
cannot be amended by the Parliament ?
while appointing the Supreme Court Judges, the
President of India has to consult the Chief Justice of (a) Shankari Prasad vs Union of India
India. Hence statement 1 is correct. The process of (b) Golaknath vs State of Punjab
removal of Judges of the Supreme Court and High (c) Kesavananda Bharti vs State of Kerla
Courts is mentioned in Article 124(4) of the (d) Minerva Mills Ltd. vs Union of India.
Constitution which asserts that the Judges of the Maharashtra PSC (Pre) G.S 2018
Supreme Court and High Courts shall not be removed Ans. (c) Kindly refer the explanation of the above
from his office except by an order of the President question.
passed after an address by each House of Parliament 27. The Supreme Court of India enunciated the
supported by a majority of the total membership of that doctrine of ‘Basic Structure of the
House and by a majority of not less than two third of the Constitution' in
members of that House present and voting has been (a) The Golaknath Case in 1967
presented to the President in the same session for such (b) The Sajjan Singh Case in 1965
removal on the ground of proved misbehaviour or (c) The Shankari Prasad Case in 1951
incapacity. Hence statement 2 is incorrect. The salary of (d) The Keshavanand Bharati case in 1973
the Judges of the Supreme Court is charged upon the UPPCS (Main) G.S. IInd 2012
Consolidated Fund of India (salary of the Judges of the UPPCS (Pre.) Re-exam G.S., 2015
High Courts are charged upon the Consolidated Fund of
Ans. (d) Kindly refer the explanation of the above
respective State) to which the Legislature does not have
question.
to vote. Hence statement 3 is correct. All appointments
28. Consider the following statements and state
of officers and staffs of the Supreme Court of India is
which one of them is correct?
absolute discretion of Chief Justice of India and
(a) Supreme Court of India has only original
Government has nothing to do with that. Hence
jurisdiction.
statement 4 is incorrect.
(b) It has only original and appellate jurisdiction
25. In which case did the Supreme Court give the
(c) It has only advisory and appellate jurisdiction
doctrine of ‘Basic Structure’ of the
(d) It has original, appellate as well as advisory
Constitution?
jurisdiction
(a) Keshavanand Bharti case
UPPCS (Main) G.S. IInd Paper 2010
(b) Golakhnath case
Ans. (d) The Supreme Court of India has original,
(c) Indira Swahney case
appellate and advisory jurisdiction. Its exclusive
(d) Sankari Prasad case original jurisdiction extends to any dispute between the
Himanchal PSC (Pre) G.S., 2018 Government of India and one or more States or between
Ans: (a) The Supreme Court recognized Basic Structure two or more States. In addition, Article 32 of the
concept for the first time in the historic Kesavananda Constitution gives an extensive original jurisdiction to
Bharati case in 1973. The Supreme Court declared that the Supreme Court in regard to enforcement of
Article 368 did not enable Parliament to alter the basic Fundamental Rights. It is empowered to issue
structure or framework of the Constitution and directions, orders and writs in the nature of habeas
parliament could not use its amending powers under corpus, mandamus, prohibition, quo warranto and

Indian Polity & Constitution 146 YCT


certiorari to enforce them. The Supreme Court has been Select the correct answer using the codes given
conferred with power to direct transfer of any civil or below:
criminal case from one State High Court to another (a) 1 and 2 (b) 2 and 3
State High Court. The Supreme Court, if satisfied that (c) 1 and 4 (d) 3 and 4
cases involving the same questions of law are pending IAS (Pre) G.S., 2012
before it and one or more High Courts and that such
Ans. (c) Kindly refer the explanation of the above
questions are substantial questions of general
question.
importance, may withdraw a case or cases pending
before the High Court or High Courts and dispose of all 30. In order to settle the dispute between the
such cases itself. Under the Arbitration and Conciliation Center and the State, the jurisdiction of the
Act, 1996, International Commercial Arbitration can Supreme Court comes under
also be initiated in the Supreme Court. The appellate (a) Consultant (b) Appellate
jurisdiction of the Supreme Court can be invoked by a (c) Constitutional (d) Original
certificate granted by the High Court concerned under IAS (Pre) G.S., 1996, 2014
Article 132(1), 133(1) or 134 of the Constitution in UPPCS (Pre.) G.S., 2004
respect of any judgement, decree or final order of a Ans. (d) Kindly refer the explanation of the above
High Court in both civil and criminal cases involving question.
substantial questions of law as to the interpretation of
31. All the disputes related to the interpretation of
the Constitution. Parliament is authorised to confer on
the Constitution could be taken to the Supreme
the Supreme Court any further powers to entertain and
Court -
hear appeals from any judgement, final order or
(a) Under original jurisdiction
sentence in a criminal proceeding of a High Court. The
Supreme Court has also a very wide appellate (b) Appellate jurisdiction
jurisdiction over all Courts and Tribunals in India in as (c) Consultant jurisdiction
much as it may, in its discretion, grant special leave to (d) None of the above
appeal under Article 136 of the Constitution from any UP Lower (M) G.S. 2013
judgment, decree, determination, sentence or order in Ans. (b) Kindly refer the explanation of the above
any cause or matter passed or made by any Court or question.
Tribunal in the territory of India. The Supreme Court 32. Which Article of the Constitution of India
has special advisory jurisdiction in matters which may deals with the Appellate Jurisdiction of the
specifically be referred to it by the President of India Supreme Court in connection with
under Article 143 of the Constitution. Additionally,
Constitutional cases?
Under Articles 129 and 142 of the Constitution, the
(a) Article 131
Supreme Court has been vested with power to punish
(b) Article 132
for contempt of Court. For High Courts, this provision
(c) Article 132 read with Article 134A
has been provided under Article 215.
(d) Article 133 read with Article 134A
29. Which of the following are included in the
original jurisdiction of the Supreme Court? UPPCS (Pre.) G.S., 2001
1. A dispute between the Government of India UPUDA/LDA (Pre.) G.S., 2001
and one or more States Ans. (c) Article 131 of the Constitution deals with
2. A dispute regarding elections to either House original jurisdiction of the Supreme Court while 134 A
of the Parliament or that of Legislature of a is related to the certification by the High Court that the
State matter is related to the interpretation of the Constitution
3. A dispute between the Government of India and must hear by the Supreme Court. Hence Article 132
and a Union Territory read with Article 134A reflects the Appellate
4. A dispute between two or more States Jurisdiction of the Supreme Court.
Indian Polity & Constitution 147 YCT
33. The Supreme Court of India tenders advice to Ans. (a) Kindly refer the explanation of the above
the President on a matter of law or fact question.
(a) On its own initiative 36. Which one of the following subjects comes
(b) Only if he seeks such advice under the common jurisdiction of the Supreme
(c) Only if the matter relates to the Fundamental Court and the High Court?
Rights of the citizen (a) Mutual disputes among States
(d) Only if the issue poses a threat to the unity (b) Dispute between Centre and State
and integrity of the country (c) Protection of the Fundamental Rights
IAS (Pre) G.S., 2001 (d) Protection from the violation of the
Ans. (b) According to Article 143 of the Constitution, If Constitution
at any time it appears to the President that a question of UPPCS (Main) G.S.-IInd Paper, 2006
law or fact has arisen which is of such a nature and of Ans. (c) Protection of Fundamental Rights comes under
such public importance that it is expedient to obtain the the common jurisdiction of the Supreme Court and the
opinion of the Supreme Court upon it, he may refer the High Courts. Article 32 of the Constitution empowers
question to the Supreme Court for consideration and the Supreme Court to provide remedies for the enforcement
Supreme Court may after such hearing as it thinks fit, of the Fundamental Rights and Article 226 of the
report to the President its opinion. Constitution empowers the High Courts for the
34. Consider the following statements: protection of Fundamental Rights.
The Supreme Court of India tenders advice to 37. The Supreme Court of India is a ‘Court of
the President of India on matters of law or fact Record’. It implies that-
1. On its own initiative (on any matter of larger (a) It has to keep a record of its decisions
public interest). (b) All its decisions have evidentiary value and
2. If he seeks such an advice. cannot be questioned by any court
3. Only if the matters relate to the Fundamental (c) It has the power to punish for its contempt
Rights of the citizens. (d) No appeal can be made against its decisions
Which of the statements given above is/are UPPCS (Pre.) G.S., 2008
correct? Ans. (b) A ‘Court of Record’ is a court, the records of
(a) 1 only (b) 2 only which are admitted to be of evidentiary value and they
(c) 3 only (d) 1 and 2 are not to be questioned when they are produced in any
IAS (Pre) G.S., 2010 court. According to Article 129 of the Constitution, the
Ans. (b) Kindly refer the explanation of the above Supreme Court shall be a court of record. Article 215 of
question. the Constitution contains similar provision with respect
35. The advisory powers of the Supreme Court of to High Courts.
India imply that it may 38. Which of the following courts in India is/are
(a) Advice to the President on the issues of law known as the court(s) of Records?
or fact which is of public importance. (a) The High Courts only.
(b) Advice to the Government of India on all (b) The Supreme Court only.
Constitutional matters (c) The High Courts and the Supreme Court.
(c) Advice to the Prime Minister on legal (d) The District Courts
matters. UPPCS (Main) G.S. IInd Paper 2008
(d) Advice to all the above persons. Ans. (c) Kindly refer the explanation of the above
nd
UPPCS (Main) G.S. II Paper, 2016 question.
Indian Polity & Constitution 148 YCT
39. Which one of the following is correct with 42. Of the following statements, which one is not
regard to the power to review any judgment correct?
pronounced or order made by the Supreme (a) Supreme Court was constituted in 1950
Court of India? (b) Supreme Court is the highest court of appeal
(a) The President of India has the power of in the country
review such a judgement or order. (c) Supreme Court can hear from any High
(b) The Supreme Court has the power to review Court/Tribunals except from Court-martial
its own judgement or order. (d) Supreme Court can hear from any High
Court/ Tribunals as well as from Court-
(c) The Cabinet has the power to review such a
martial
judgement or order with the permission of the
President of India. Ans. (d) According to Article 136 of the Constitution, the
Supreme Court may in its discretion grant special leave to
(d) The Supreme Court it does not have the
appeal from any judgment, decree, determination,
power to review its own judgement or order.
sentence or order in any cause or matter passed or made
UPPCS (Main) G.S. IInd Paper 2011
by any court or tribunal in the territory of India except to
Ans. (b) According to Article 137 of the Constitution, any judgment, determination, sentence or order passed or
the Supreme Court shall have power to review any made by any court or tribunal constituted, related to the
judgment pronounced or order made by it. Armed Forces. Hence statement d is incorrect. Rest of the
40. Which of the following is correct about the given options are correct.
Supreme Court regarding its judgement? 43. Which Article of the Constitution of India
(a) It can change the judgement Conferred special leave petitions to the
(b) It cannot change the judgement Supreme Court?
(c) Only the Chief Justice of India can change the (a) Article 136 (b) Article 32
judgement (c) Article 139 (d) Article 226
(d) Only the Ministry of Law can change the TNPSC (Pre) G.S. 2017
judgement Ans. (a) : Kindly refer the explanation of the above
UPPCS (Main) G.S. IInd Paper 2005 question.
Ans. (a) Kindly refer the explanation of the above 44. The power to increase the number of judges in
the Supreme Court of India is vested in
question.
(a) The President of India
41. According to the latest Supreme Court
(b) The Parliament
judgement on Armed Forces Tribunal (AFT),
(c) The Chief Justice of India
which of the following is true?
(d) The Law Commission
(a) High Courts cannot entertain writ petitions
IAS (Pre) G.S., 2014
against verdict of AFT.
Ans. (b) According to Article 124(1) of the
(b) High Courts can entertain writ petitions
Constitution, power to increase the number of Judges in
against verdict of AFT.
the Supreme Court of India is vested in the Parliament.
(c) High Courts can entertain writ petitions
45. The jurisdiction of the Supreme Court can be
against verdict of AFT concerning officials
increased by which of the following?
below the rank of Major only.
(a) By the President of India
(d) None of the above
(b) With the resolution passed by the Parliament
UP RO/ARO (Pre) G.S., 2014
(c) Law made by the Parliament
Ans. (a) As per the recent Supreme Court verdict, High (d) By the President of India after consultation
Courts cannot entertain writ petitions against verdict of with Chief Justice of India
Armed Forces Tribunal (AFT). Uttarakhand PCS (Pre) 1st 2014
Indian Polity & Constitution 149 YCT
Ans. (c) According to Article 138 of the Constitution, (c) Judicial initiative
Parliament by law may increase the jurisdiction of the (d) None of the above
Supreme Court. UPPCS (Main) G.S. IInd Paper 2008
46. The power to enlarge the jurisdiction of the Ans. (c) Kindly refer the explanation of the above
Supreme Court of India with respect to any question.
matter included in the Union List of Legislative 49. Where Public Interest litigation (PIL) can be
Powers rests with: introduced?
(a) The President of India (a) Only in the Supreme Court of India
(b) The Chief Justice of India (b) Only in the High Courts of States
(c) The Parliament (c) Only in the Administrative Tribunals
(d) The Union Ministry of Law, Justice and (d) Both in Supreme Court and High Courts
Company Affairs Uttarakhand PCS (Pre) 1st 2014
IAS (Pre) G.S., 2003 Ans. (d) Kindly refer the explanation of the above
Ans. (c) Kindly refer the explanation of the above question.
question. 50. ‘Judicial Activism’ in India is related to-
47. Who was the Chief Justice of India when (a) Committed judiciary
public interest litigation (PIL) was introduced (b) Public interest litigation
to the Indian Judicial System? (c) Judicial review
(a) M. Hidaatullah (d) Judicial freedom
(b) A.M. Ahmadi UPRO/ ARO (Pre) G.S., 2014
(c) A.S. Anand Ans. (b) Kindly refer the explanation of the above
(d) P.N. Bhagwati question.
IAS (Pre) G.S., 2006 51. Which one of the following writs is issued
UPPCS (Pre) G.S, 2018 during the pendency of proceedings in a court?
Ans. (d) Public interest Litigation (PIL) means litigation (a) Mandamus (b) Certiorari
filed in a court of law for the protection of ‘Public (c) Prohibition (d) Quo warranto
Interest’ such as pollution, terrorism, road safety, UPPCS (Pre.) G.S., 2010
constructional hazards etc. Any matter where the interest
Ans. (c) Prohibition writ is an order from a superior
of public at large is affected can be redressed by filing a
court to a lower court or tribunal directing the Judge and
Public Interest Litigation in a court of law. Public interest
the parties to cease the litigation because the lower
litigation is not defined in any statute or in any Act. It has
court does not have proper jurisdiction to hear or
been interpreted by Judges to consider the intent of public
determine the matters before it.
at large. The term ‘PIL’ originated in the United States.
52. A writ which is issued by the judiciary and
PIL had begun in India towards the end of 1970s and
through which executive has been asked to
came into full bloom in the 80s. In 1986, the then Chief
perform under authorized powers, is called as-
Justice of India, Justice P.N. Bhagwati introduced public
(a) Habeas Corpus
interest litigation (PIL) to the Indian judicial system. Any
(b) Mandamus
public spirited citizen can move/approach the court for
the public cause (in the interest of the public or public (c) Prohibition
welfare) by filing a petition in Supreme Court under (d) Quo Warranto
Article 32 of the Constitution and in High Court under UPPCS (Pre.) G.S., 2003
Article 226 of the Constitution. Ans. (b) Mandamus is a command issued by a court to
48. Public Interest Litigation has been introduced an authority directing it to perform a public duty
by imposed upon it by law. The function of mandamus is
to keep the public authorities within the limits of their
(a) A Parliamentary Act
jurisdiction while exercising public functions.
(b) A Constitutional Amendment
Indian Polity & Constitution 150 YCT
53. When the Supreme Court issues a writ to a 56. Which one of the following constitutional
person or to an institution to perform its duty, remedies is also known as 'postmortem'?
it is called (a) Prohibition
(a) Certiorari (b) Mandamus
(b) Mandamus (c) Certiorari
(c) Quo Warranto
(d) Qua warranto
(d) Habeas Corpus
(e) None of the above/ More than one of the
UPPCS (Main) G.S. IInd Paper 2007
above
Ans. (b) Kindly refer the explanation of the above
BPSC (Pre) G.S. 2019
question.
Ans : (c) A writ of certiorari can only be issued by the
54. Which of the following writs can be filed for
Supreme court under Article 32 and a High court under
the sack of personal liberty or freedom?
Article 226 to direct , inferior courts , tribunals or
(a) Mandamus
authorities to transmit to the court the record of
(b) Quo Warranto
proceedings disposed of or pending therein for scrutiny
(c) Habeas Corpus
and if necessary , for quashing the same . But a writ of
(d) Certiorari
certiorari can never be issued to call for the record or
UPPCS (Pre.) G.S., 2005
papers and proceedings of an Act or Ordinance and for
Ans. (c) By Habeas Corpus writ, the court directs the quashing such an Act or Ordinance. Hence writ of
person or authority, who has detained another person, to certiorari can be considered as post-mortem.
bring the prisoner in person before the court so as to
57. Which of the following is NOT correctly
enable the court to decide the validity, jurisdiction or
matched?
justification for such detention. Under Article 22 of the
(a) Habeas corpus To have the body of
Constitution, a person arrested is required to be
produced before a magistrate within 24 hours of his (b) Mandamus We command
arrest and failure to do so would entitle the arrested (c) Prohibition To be certified
person to be released. (d) Quo-Warranto By what authority
55. Under the constitution of India, the power to UPPCS (Pre) G.S, 2019
issue a writ of Haboas Corpus is vested in: Ans : (c)
(a) The District Courts Habeas corpus To have the body of
(b) The High Courts only Mandamus We command
(c) The Supreme Court only Prohibition Writ of Prohibition
(d) Both the High Court and the Supreme Court Quo-Warranto By what authority
Himanchal PSC (Pre), G.S, 2011
58. Under which law is it prescribed that all
Ans. (d) The Supreme Court has been empowered to
proceeding in the Supreme Court or India shall
issue writs in the nature of habeas corpus, mandamus,
be in English language?
prohibition and certiorari and quo warranto for
(a) The Supreme Court rules, 1966
protecting the Fundamental Rights (Article 32(2)).
Similar power has been conferred on the High Courts (b) Article 145 of the Constitution of India.
via Article 226. The High Court can issue the above (c) A Legislation made by Parliament.
writs for protecting the fundamental as well as statutory (d) Article 348 of the Constitution of India
and common law rights. UPPCS (Main) G.S. IInd Paper 2013
Indian Polity & Constitution 151 YCT
Ans. (d) According to Article 348 of the Constitution, Constituion. Hence Both (A) and (R) are true and (R) is
all proceeding in the Supreme Court and High Courts of the correct explanation of (A).
India shall be in English language. 61. Supreme Court's Judgement in Indira

59. The official language of the Supreme Court is- Sawhney Vs. Union of India is related to which

(a) Hindi-Devnagri issue?


(a) Reservation for OBC
(b) Urdu/Parasi
(b) Reservation for Scheduled Castes
(c) English
(c) Reservation for Minorities
(d) 18 languages described in the Constitution of
(d) Reservation for Scheduled Tribes
India
Himanchal PSC (Pre) G.S. 2016/ 2018
Uttarakhand RO/ARO (M) 2016
Ans. (a) Indira Swahney Vs Union of India case is
Ans. (c) Kindly refer the explanation of the above
related to reservation for other backward classes (OBC)
question.
based on the recommendation made by Mandal
60. Given below are two statements, one is labelled Commission.
as Assertion (A) and the other as Reason (R).
62. In which year was the Collegium System for
Assertion (A) : The Supreme Court of India has the appointment of Judges adopted by the
exclusive Jurisdiction regarding Constitutional Supreme Court of India?
validity of Central laws. (a) 1993 (b) 1996
Reason (R) : The Supreme Court is the Guardian (c) 2000 (d) 2004
of the Indian Constitution. Uttarakhand-PSC (Pre.) -2016
Select the correct answer from the codes given Ans. (a) The Collegium method was created as a result
below : of judgment of the Supreme Court in 1993 (Supreme
Codes: Court Advocate-on-Record Association case). Under
(a) Both (A) and (R) are true and (R) is the the Collegium system of appointment and transfer of
correct explanation of (A) Judges of High Courts, the Chief Justice of the

(b) Both (A) and (R) are true but (R) is not the respective High Court and two other senior most Judges
correct explanation of (A) of the same High Court comprise a Collegium to
identify the suitable candidates and recommend them
(c) (A) is true, but (R) is false
for the appointment as a Judge of the High Court. The
(d) (A) is false, but (R) is true
shortlisted candidates are scrutinized by a Collegium of
UPPCS (Pre) G.S, 2019
the five senior most Judges of the Supreme Court
Ans: (a) By the Constitution (42nd Amendment) Act, headed by the Chief Justice of India. The same
1976,Article131A was inserted which conferred on the Collegium of the Supreme Court identifies serving
Supreme Court exclusive jurisdiction in regard to Judges and Chief Justice of the High Courts for the
the constitutional validity of Central laws. Supreme elevation to the Supreme Court. There is no mention of
Court is called the Gurdian of the Constituion as the Collegium either in the original Constitution or in
Supreme Court is the final interpreter of the successive Amendments.

Indian Polity & Constitution 152 YCT


25.
High Court- Establishment,
Empowerment and Jurisdiction
1. High Courts are instituted as; misbehavior or incapacity’. Hence statement 1 is
(a) Constitutional courts under Part VI, chapter correct. According to Article 220 of the Constitution, a
V, Article 214 of the Indian Constitution Judge of the High Court after its retirement shall not
(b) Constitutional courts under Part VII, Chapter plead or act in any court or before any authority in India
V, Article 214 of the Indian Constitution except the Supreme Court and other High Courts.
(c) Constitutional courts under Part VI, chapter Hence statement 2 is incorrect.
VI, Article 214 of the Indian Constitution 3. Consider the following statements:
(d) Constitutional courts under Part VIII, Chapter 1. A person who has held office as a permanent
V, Article 214 of the Indian Constitution Judge of a High Court cannot plead or act in
any court or before any authority in India
Haryana PSC (Pre) G.S. 2014
except the Supreme Court.
Ans. (a) Part VI, chapter V, Article 214 of the
2. A person is not qualified for appointment as a
Constitution defines High Courts for States.
Judge of a High Court in India unless he has
2. Consider the following statements: for at least five years held a judicial office in
1. The mode of removal of a Judge of a High the territory of India.
Court in India is same as that of removal of a Which of the statements given above is/are
Judge of the Supreme Court. correct?
2. After retirement from the office, a permanent (a) 1 only (b) 2 only
Judge of a High Court cannot plead or act in (c) Both 1 and 2 (d) Neither 1 nor 2
any court or before any authority in India. IAS (Pre) G.S., 2006
Which of the statements given above is/are Ans. (d) According to Article 220 of the Constitution, a
correct? Judge of the High Court after its retirement shall not
(a) 1 only (b) 2 only plead or act in any court or before any authority in India
(c) Both 1 and 2 (d) Neither 1 nor 2 except the Supreme Court and other High Courts.
Hence statement 1 is incorrect. According to Article
IAS (Pre) G.S., 2007
217 of the Constitution, a person shall be qualified for
Ans. (a) The process of removal of the Judges of
appointment as a Judge of a High Court if he held a
Supreme Court and High Courts has been mentioned in judicial office in the territory of India for at least ten
the Article 124(4) of the Constitution which is same for years or has for at least ten years been an advocate of a
the Judges of the Supreme Court as well as High High Court or of two or more such Courts in
Courts. It asserts that a Judge of Supreme Court shall succession. Hence statement 2 is also incorrect.
not be removed from his office except by an order of 4. The salaries and allowances of the Judges of
the President passed after an address by each House of the High Court are charged to the
Parliament supported by the majority of the total (a) Consolidated Fund of India
members of the House and by the majority of not less (b) Consolidated Fund of State
than two-third of the members of the House present and (c) Contingency Fund of India
voting has been presented to the President in the same (d) Contingency Fund of the State
session for such removal on the ground of ‘proved IAS (Pre) G.S., 2002
Indian Polity & Constitution 153 YCT
Ans. (b) According to Article 202 of the Constitution, 7. With reference to the Constitution of India,
the salaries and allowances of the Judges of the High consider the following statements :
Courts are charged upon Consolidated Fund of the 1. No High Court shall have the jurisdiction to
respective State. declare any central law to be constitutionally
5. Where from the pension of a retired High invalid.
Court Judge is charged? 2. An amendment to the Constitution of India
(a) Consolidated fund of India. cannot be called into question by the
(b) Consolidated fund of the state where he last Supreme Court of India.
served. Which of the statements given above is/are
(c) Consolidated fund of the different states correct?
where he has served. (a) 1 only
(d) Contingency fund of India. (b) 2 only
UPPCS (Main) G.S. IInd 2013 (c) Both 1 and 2
(d) Neither 1 nor 2
Ans. (a) The pension of a retired High Court Judge is
charged upon the Consolidated Fund of India. IAS (Pre) G.S, 2019

6. Acting judges are appointed in- Ans : (d) The Constituion Forty-Third Amendment)
Act, 1977 restored the power of High Courts to declear
(a) Supream Court
any central law invalid. Hence statement 1 is incorrect.
(b) High Court
In Keshwanand Bharti case, Supreme Court ruled that
(c) District Court
Parliament can amend any part of the Constituion
(d) Both (a) and (b)
(including Preamble of the Constution) under Article
UPPSC ACF-RFO Mains (IInd Paper), 2019 368 of the Constituion provided such amendment does
UPPSC RO/ARO (Pre), 2016 (Held on 20/09/2020) not alter the ‘Basic Structure’ of the Constituion. So
Ans. (b) : Acting Judges can be appointed by the any alteration in the basic structure of the Constituion
President under clause (2) of Article 224 of the could be question by the Supreme Court of India.
Constitution. Such appointments will not however be Therefore statement 2 is also incorrect.
made for periods of less than three months unless there 8. Consider the following statements regarding
are special reasons for doing so. According to Article the High Courts in India:
224 (2), when any Judge of a High Court other than the I. There are eighteen High Courts in India.
Chief Justice is by reason of absence or for any other II. Three of them have jurisdiction over more
reason unable to perform the duties of his office or is than one State.
appointed to act temporarily as Chief Justice, the
III. No Union Territory has a High Court of its
President may appoint a duly qualified person to act as
own.
a Judge of that Court until the permanent Judge has
IV. Judges of the High Court hold office till the
resumed his duties. According to Article 127 of the
age of 62.
Constitution, if at any time there should not be a
Which of these statements is /are correct?
quorum of the Judges of the Supreme Court available
(a) I, II and IV
to hold or continue any session of the Court, the Chief
(b) II and III
Justice of India may with the previous consent of the
(c) I and IV
President and after consultation with the Chief Justice
of the High Court concerned request in writing the (d) IV only
attendance at the sittings of the Court as an ad hoc IAS (Pre) G.S., 2001
Judge for such period as may be necessary of a Judge Ans. (d) At present there are 25 High Courts in India.
of a High Court duly qualified for appointment as a Hence statement 1 is incorrect. 6 High Courts have
Judge of the Supreme Court to be designated by the jurisdiction over more than one State and Union
Chief Justice of India. Territory. They are Bombay High Court (Maharashtra,

Indian Polity & Constitution 154 YCT


Goa, Dadra and Nagar Haveli), Calcutta High Court Court (Punjab and Haryana) have jurisdiction over more
(West Bengal and Andaman and Nicobar Island), than one State (Union Territories not included). The
Guwahati High Court (Assam, Nagaland, Mizoram and President of India on 26th December, 2018, has issued
Arunanchal Pradesh), Kerala High Court (Kerala and orders to establish High Court of Andhra Pradesh with
Lakshadweep), Madras High Court (Tamil Nadu and the principal seat at Amaravati and High Court for the
Pondicherry), Punjab and Haryana High Court (Punjab, State of Telangana with the principal seat at Hyderabad.
Haryana and Chandigarh). Hence statement 2 is The bifurcation and the constitution of separate High
incorrect. Among the Union Territories, Delhi alone has Courts for Telangana and Andhra Pradesh came into
a High Court of its own. Hence statement 3 is also effect from 1st January, 2019. Therefore total number of
incorrect. Judges of the High Court hold office till the High Courts in India is now 25.
age of 62 therefore statement 4 is correct.
12. Which State got a separate High Court on 1st
9. Which of the following places does not fall January 2019, which is the 25th High Court of
under the territorial establishment jurisdiction India?
of Bombay High Court? (a) Telangana
1. Daman and Diu
(b) Mizoram
2. Dadra and Nagar Haveli
(c) Sikkim
3. Goa
(d) Andhra Pradesh
4. Lakshadweep
Himanchal PSC (Pre) G.S. 2019
Choose the correct option from the following:
Ans. (d) : Kindly refer the explanation of the above
(a) 1 only (b) 1 and 2
question.
(c) Only 4 (d) 3 and 4
13. Consider the following statements:
Punjab PSC (Pre) G.S 2013
1. There are 25 High Courts in India.
Ans. (c) Kindly refer the explanation of the above
2. Punjab, Haryana and Union Territory of
question.
Chandigarh have a common High Court.
10. Which one of the following High Courts has the
3. National territory of Delhi has a High Court
Territorial Jurisdiction over Andaman and
of its own.
Nicobar Islands?
Which of the above statements is /are correct?
(a) Andhra Pradesh
(a) 2 and 3
(b) Calcutta
(b) 1 and 2
(c) Madras
(d) Orissa (c) 1, 2 and 3

IAS (Pre) G.S., 2003 (d) 3 only


IAS (Pre) G.S., 2005
Ans. (b) Kindly refer the explanation of the above
question. Ans. (*) At the time when question was asked, there
were 24 High Courts in India. At present, total number
11. How many high Courts in India have
jurisdiction over more than one State (Union of High Courts in India is 25 as explained in above
Territories not included)? question. Statement 2 and 3 are correct.
(a) 2 (b) 3 14. Which of the following High Courts has the
(c) 4 (d) 5 largest number of Benches?
IAS (Pre) G.S., 2008 (a) Kolkata High Court

Ans. (b) Three High Courts in India namely Guwahati (b) Madhya Pradesh High Court
High Court (Assam, Nagaland, Mizoram and (c) Bombay High Court
Arunanchal Pradesh), Bombay High Court (d) Guwahati High Court
(Maharashtra, Goa) and Punjab and Haryana High UPPCS (Main) G.S. IInd 2007

Indian Polity & Constitution 155 YCT


Ans. (d) Guwahati High Court has largest number of unlawful detention on liberty or freedom of the prisoner
Benches. The Guwahati High Court has three outlying or detention. The great value of the writ is that it
Benches namely Kohima Bench for Nagaland, Aizawl enables immediate determination of the right of a
Bench for Mizoram and Itanagar Bench for Arunachal person as to his freedom. Under Article 22 of the
Pradesh. Constitution, a person arrested is required to be
15. When were High Courts of Bombay, Madras produced before a magistrate within 24 hours of his
and Calcutta established? arrest and failure to do so would entitle the arrested
(a) 1861 person to be released.
(b) 1851 18. Which of the following is not correct regarding
(c) 1871 the High Court in India?
(d) 1881 (a) Second appeal to the High Court is within its
UPPCs (Pre.) G.S., 2013 appellate jurisdiction.
UPUDA/LDA Special (Pre.) G.S., 2010 (b) Every Judge of a High Court is appointed by
Ans. (a) High Courts of Bombay, Madras and Calcutta the President.
were established in 1861 under the British Parliament's (c) Governor of the State appoints the Judges of a
Indian High Courts Act, 1861. High Court.
16. A judge of the High Court submits his (d) High Courts may accept a petition related to
resignation to- ‘Public Interest Litigation’.
(a) President
UPPCS (Main) G.S. IInd Paper, 2016
(b) Chief Justice of India
Ans. (c) The Judges of the High Courts are appointed
(c) Chief Justice of the concerned State
by the President. Hence statement c is incorrect. Second
(d) Governor of the State
appeal shall lie to the High Court from every decree
BPSC (Pre) 2007
passed in appeal by any court subordinate to the High
Ans. (a) According to Article 217(1) of the
Court, if the High Court is satisfied that the case
Constitution, a Judge of a High Court may submit his
involves a substantial question of law. Public interest
resignation to the President.
litigation is a litigation which can be file in Supreme
17. Which one of the following writs can be issued Court/ High Court by any public spirited person for the
by a High Court to secure liberty of the
protection of public interest.
individual?
19. Which one of the following is not the main
(a) Mandamus
jurisdiction of the High Court of a State?
(b) Quo-warranto
(a) Advisory jurisdiction
(c) Habeas Corpus
(b) Original jurisdiction
(d) Prohibition
(c) Supervisory jurisdiction
UPPCS (Pre.) Re-exam. G.S., 2015
(d) Appellate jurisdiction
Ans. (c) By Habeas Corpus writ, the court directs the
UPPCS (Main) G.S. IInd Paper 2007
person or authority who has detained another person to
bring the body of the prisoner before the court so as to Ans. (a) Supreme Court and High Courts have original,
enable the court to decide the validity, jurisdiction or supervisory and appellate jurisdiction but Article 143 of

justification for such detention. The principal aim of the the Constitution confers upon the exclusive advisory

writ is to ensure swift judicial review of alleged jurisdiction of the Supreme Court.

Indian Polity & Constitution 156 YCT


26.
Subordinate Court and Tribunals
1. Consider the following statements: 3. Which one of the following writs examines the
1. The highest criminal court of the district is functioning of subordinate courts?
the Court of District and Sessions Judge. (a) Quo Warranto (b) Mandamus
2. The District Judges are appointed by the (c) Certiorari (d) Habeas Corpus
Governor in consultation with the High UPPCS (Main) G.S. IInd Paper 2008
Courts. Ans. (c) Certiorari is a writ issued by a superior court
3. A person to be eligible for appointment as a for the re-examination of an action of a lower court.
District Judge should be an advocate or a Certiorari is also issued by an appellate court to obtain
pleader of seven year's standing or more, or information on a case pending before it.
State.
4. According to Constitution of India, the term
4. When the Sessions Judge awards death
‘district judge’ shall not include
sentence, it must be confirmed by the High
(a) Chief Presidency Magistrate
Court before it is carried out.
(b) Session Judge
Which of these statements given above are
correct? (c) Tribunal Judge
(a) 1 and 2 (b) 2, 3 and 4 (d) Chief Judge of a small cause court
(c) 1 and 3 (d) 1, 2, 3 and 4 IAS (Pre) G.S., 1996
IAS (Pre.) G.S., 2004 Ans. (c) According to Article 236 of the Constitution,
Ans. (d) Article 233 of the Constitution is regarding the expression District Judge includes Judge of a city
appointment of District Judge. Article 233(1) asserts civil court, additional District Judge, Joint District
that the appointments of persons to be and the posting Judge, Assistant District Judge, Chief Judge of a small
and promotion of District Judges in any State shall be cause court, Chief Presidency Magistrate, Additional
made by the Governor of the State in consultation with Chief Presidency Magistrate, Sessions Judge,
the High Court. Article 233(2) asserts that a person not Additional Sessions Judge and Assistant Sessions
already in the service of the Union or of the State shall Judge. Hence Tribunal Judge is not included in the term
only be eligible to be appointed a District Judge if he of District Judge.
has been for not less than seven years an advocate. The 5. With reference to the 'Gram Nyayalaya Act',
highest court in each district is that of the District and which of the following statements is/are correct?
Sessions Judge. This is the principal court of civil 1. As per the Act, Gram Nyayalayas can hear
jurisdiction. This is also a court of Sessions. Sessions- only civil cases and not criminal cases.
triable cases are tried by the Sessions Court. It has the 2. The Act allows local social activists as
power to impose any sentence including capital mediators/reconciliators.
punishment (death sentence) but it must be confirmed Select the correct answer using the code given
by the High Court before it is carried out. Hence all four below.
statements are correct. (a) 1 only (b) 2 only
2. The term District Judge is mentioned in which (c) Both 1 and 2 (d) Neither 1 nor 2
of the following Article of the Constitution? IAS (Pre) G.S., 2016
(a) Article 230 (b) Article 231
Ans. (b) Gram Nyayalayas Act, 2008 extends to the
(c) Article 232 (d) Article 233
whole of India except Jammu and Kashmir, Nagaland,
(e) None of the above
Arunanchal Pradesh, Sikkim and to the tribal areas.
Chhatisgarh PSC (Pre) 2016 According to Gram Nyayalayas Act, 2008, Gram
Ans. (d) Kindly refer the explanation of the above Nyayalaya shall exercise both civil and criminal
question. jurisdiction. Hence statement 1 is incorrect. The Gram
Indian Polity & Constitution 157 YCT
Nyayalaya shall try to settle the disputes as far as in respect of matters relating to divorce or matters
possible by bringing about conciliation between the relating to an offence not compoundable under the law.
parties and for this purpose; it shall make use of the Lok Adalat comprises of a sitting or retired judicial
conciliators to be appointed for this purpose. Hence officer, a member from legal profession, a social worker
statement 2 is correct. engaged in the upliftment of the weaker sections. Hence
6. With reference to Lok Adalats, which of the none of the statements given above is correct.
following statements is correct? 7. Which one of the following statements about
(a) Lok Adalats have the jurisdiction to settle the 'Lok Adalat' is incorrect?
matters at pre-litigation stage and not those (a) Under the Legal Services Authorities Act of
matters pending before any court 1987, Lok Adalats have been given a
(b) Lok Adalats can deal with matters which are statutory status.
civil and not criminal in nature (b) Lok Adalats can be constituted of serving or
(c) Every Lok Adalat consists of either serving or retired judicial persons only.
retired judicial officers only and not any other (c) Every award made by the Lok Adalat is final
person and binding on the parties.
(d) None of the statements given above is correct (d) The award passed by the Lok Adalat is
IAS (Pre) G.S., 2010 deemed to be a decree of a civil Court.
Ans. (d) The modern idea of Lok Adalat was mainly Maharashtra PSC (Pre) G.S, 2019
proposed and implemented by Justice P.N. Bhagwati, Ans. (b) : Kindly refer the explanation of the above
former Chief Justice of India. Lok-Adalat is a non- question.
adversarial system of dispute resolution whereby courts 8. With reference to the Consumer Disputes
(called Lok Adalats) are organised by the National Redressal at district level in India, which one of
Legal Services Authority, State Legal Services the following statements is not correct?
Authority, District Legal Services Authority, Supreme (a) A State Government can establish more than
Court Legal Services Committee, High Court Legal one District Forum in a district if it deems fit
Services Committee and Taluk Legal Services (b) One of the members of the District Forum
Committee. The first Lok Adalat was organised in 1982, shall be a woman
in a village called Una in Junagarh district of Gujarat. (c) The District Forum entertains the complaints
Lok Adalat is one of the alternative dispute redressal where the value of goods or services does not
mechanisms. It is a forum where disputes/cases pending exceed rupees fifty lakhs
in the court of law or at pre-litigation stage are settled/ (d) A complaint in relation to any goods sold or
compromised amicably. There is no court fee payable any service provided may be tiled with a
when a matter is filed in a Lok Adalat. Lok Adalats District Forum by the State Government as a
have been given Statutory Status under the Legal representative of the interests of the
Services Authorities Act, 1987. Under the Legal consumers in general
Services Authorities Act, 1987, the decision made by IAS (Pre) G.S., 2010
the Lok Adalats is deemed to be a decree of a civil court Ans. (c) The Consumer Protection Act, 1986 is a social
and is final and binding on all parties and no appeal legislation that lays down the rights of the consumers
against such an award lies before any court of law. If and provides promotion and protection of the rights of
the parties are not satisfied with the award of the Lok the consumers. To provide inexpensive, speedy and
Adalat, though there is no provision for an appeal summary redressal of consumer disputes, quasi-judicial
against such an award but they are free to initiate bodies have been set up in each district, State and at the
litigation by approaching the court of appropriate National level called the District Forums, the State
jurisdiction by filing a case by following the required Consumer Disputes Redressal Commissions and the
procedure in exercise of their right to litigate. Any case National Consumer Disputes Redressal Commission
pending before any court or any dispute which has not respectively. Each District Forum is headed by a person
been brought before any court and is likely to be filed who is or has been or is eligible to be appointed as a
before the court could be brought to Lok Adalat District Judge, two other members having sound
provided that the Lok Adalat shall have no jurisdiction knowledge of law, economics, commerce, public affairs
Indian Polity & Constitution 158 YCT
etc and one of them must be woman. A State 3. Member of other backward classes(OBC) with
Government can establish more than one District Forum an annual income of less than Rs 3,00,000
in a district, if it deems fit. The provisions of this Act 4. All senior citizens
cover ‘goods’ as well as ‘services’. A written complaint Select the correct answer using the given code
can be filed before the District Consumer Forum for below-
pecuniary value of up to rupees twenty lakh, before (a) 1 and 2 only (b) 3 and 4 only
State Commission for value up to rupees one crore and (c) 2 and 3 only (d) 1 and 4 only
before the National Commission for value above rupees IAS (Pre) G.S 2020
one crore in respect of defects in goods and or Ans. (a) As per the Legal Services Authorities Act,
deficiency in service. Hence statement (c) is not correct. 1987, following are entitled to free legal services:
9. With reference to National Legal Services (a) A member of a Scheduled Caste or Scheduled Tribe
Authority, consider the following statements: (b) A victim of trafficking in human beings or begar as
1. Its objective is to provide free and competent referred to in Article 23 of the Constitution
legal services to the weaker sections of the (c) A woman or a child
society on the basis of equal opportunity. (d) A mentally ill or otherwise disabled person
2. It issues guidelines for the State Legal (e) A person under circumstances of undeserved want
Services Authorities to implement the legal such as being a victim of a mass disaster, ethnic
programmes and schemes throughout the violence, caste atrocity, flood, drought, earthquake or
country. industrial disaster
Which of the statements given above is/are (f) An industrial workman
correct? (g) Income Ceiling to Rs.1,25,000/- p.a. for legal aid
(a) 1 only (b) 2 only before Supreme Court of India and to Rs.1,00,000/- p.a.
(C) Both 1 and 2 (d) Neither 1 nor 2 for legal aid upto High Courts.
IAS (Pre) G.S., 2013 Limit for Transgender is Rs. 2,00,000. Hence statement
Ans. (c) The National Legal Services Authority 1 and 2 are correct. Senior citizens eligibility for free
(NALSA) has been constituted under the Legal Services legal aid depends on the rules framed by the respective
Authorities Act, 1987 to provide free legal services to State Governments in this regard. Hence statement 3
the weaker sections of the society. The Chief Justice of and 4 are not correct.
India is the Patron-in-Chief of National Legal Services 11. Consider the following statements:
Authority and the senior most Judge of the Supreme 1. Central Administrative Tribunal (CAT) was
Court is the Executive Chairman of the Authority. The set up during the Prime Minister ship of Lal
principal objective of NALSA is to provide free and Bahadur Shastri.
competent legal services to the weaker sections of the 2. The Members of CAT are drawn from both
society and to ensure that opportunities for securing judicial and administrative streams.
justice are not denied to any citizen by reason of Which of the statements given above is are
economic or other disabilities and to organize Lok correct?
Adalats for amicable settlement of disputes. Spreading (a) 1 only (b) 2 only
legal literacy and awareness, undertaking social justice (c) Both 1 and 2 (d) Neither 1 nor 2
litigations etc are other objectives of NALSA. NALSA
IAS (Pre) G.S., 2009
works in close coordination with the various State Legal
Ans. (b) The Central Administrative Tribunal (CAT)
Services Authorities, District Legal Services Authorities
had been established (under the Prime Ministership of
and other agencies for a regular exchange of relevant
Rajiv Gandhi) in the year 1985 under Article 323(A) of
information, monitoring and updating on the
the Constitution for adjudication of disputes and
implementation and progress of the various schemes.
complaints with respect to recruitment and conditions of
10. In India, legal service authorities provide free service of persons appointed to public services and
legal services to which of the following type of posts in connection with the affairs of the Union or
citizens? other authorities under the control of the Government.
1. Person with an annual income of less than Rs
There are 17 Benches in the Central Administrative
1,00,000 Tribunal all over India. A Bench consists of one Judicial
2. Transgender with an annual income of less than
Member and one Administrative Member.
Rs 2,00,000
Indian Polity & Constitution 159 YCT
27.
Attorney General of India/
Advocate General of State
1. Consider the following statements in the Ans. (c) Kindly refer the explanation of the above
context of Attorney General of India: question.
1. The President shall appoint a person who is 3. Consider the following statements about the
qualified as to be appointed a Judge of the Attorney General of India:
Supreme Court (i) He is appointed by the President of India
2. The Attorney General shall receive such
(ii) He must have the same qualifications as are
remuneration as the Parliament may determine.
required by for a Judge of the High Court
Select the correct answer using the codes given
(iii) He has right to speak in either House of
below:
Parliament
(a) Only 1 is correct
(iv) He can be removed by following the process
(b) Only 2 is correct
of impeachment
(c) Both 1 and 2 are correct
(v) He has right of audience in all courts in the
(d) Neither 1 nor 2 is correct
territory of India
UPPCS (Pre) G.S 2020
Which of the above mentioned statements are
Ans. (a) According to Article 76 of the Constitution, the
correct?
President shall appoint a person who is qualified to be
(a) (i), (ii) and (iv) (b) (ii), (iii) and (v)
appointed a Judge of the Supreme Court to be Attorney-
(c) (i), (ii), (iii) and (v) (d) (i), (iii) and (v)
General for India. It shall be the duty of the Attorney-
General to give advice to the Government of India upon Punjab PSC (Pre) G.S 2013
such legal matters and to perform such other duties of a Ans. (d) Kindly refer the explanation of the above
legal character. In the performance of his duties, the question.
Attorney-General shall have right of audience in all 4. Consider the following statements about the
courts in the territory of India. He has right to speak in Attorney-General of India:
the both Houses of Parliament .The Attorney-General (i) He is appointed by the President of India.
shall hold office during the pleasure of the President (ii) He must have the same qualifications as are
and shall receive such remuneration as the President required for a Judge of the Supreme Court.
may determine. (iii) He must be a member of either House of
2. Which of the following is not a true statement Parliament.
about the Attorney General of India? (iv) He can be removed by impeachment by
(a) He is not a member of the Cabinet Parliament.
(b) He has Right to speak in the House of Which of these statements are corrects?
Parliament (a) (i) and (ii) (b) (i) and (iii)
(c) He has Right to vote in the Parliament (c) (ii), (iii) and (iv) (d) (iii) and (iv)
(d) None of the above IAS (Pre) G.S., 2000
nd
UPPCS (Main) G.S. II 2017 UP RO/ARO (Main) G.S., 2014
Indian Polity & Constitution 160 YCT
Ans. (a) Kindly refer the explanation of the above Ans. (c) According to Article 88 of the Constitution, the
question. Attorney General of India shall have the right to speak
5. How is the Attorney General of India in and to take part in the proceedings of either House,
appointed? any joint sitting of the Houses and any committee of
(a) By the Parliament Parliament of which he may be named a member but
(b) By the President shall not be entitled to vote in Lok Sabha.
(c) By the President on the recommendation of 8. Who among the following can attend the
the Parliament meeting of both Houses of Parliament while not
(d) By the Chief Justice being a member of either?
(e) By the Speaker of the Lok Sabha (a) Solicitor General of India
st
Chhattisgarh PSC (Pre)1 Paper G.S., 2013 (b) Vice-President of India
Ans. (b) Kindly refer the explanation of the above (c) Comptroller and Auditor General of India
question. (d) Attorney General of India
6. Who among the following holds his/her office Punjab PSC (Pre) G.S 2015
during pleasure of the President? Ans. (d) Kindly refer the explanation of the above
(a) Comptroller and Auditor General of India question.
(b) Chief Election Commissioner
9. Which one of the following is legal advisor to
(c) Speaker of Lok Sabha
the State Government?
(d) Attorney General of India
(a) Attorney General
UPPCS (Pre) G.S., 2016
(b) Advocate General
Ans. (d) Attorney General of India holds his/her office
(c) Solicitor General
during pleasure of the President. Comptroller and Auditor
(d) Comptroller and Auditor General
General of India and Chief Election Commissioner of
UPPCS (Main) G.S. IInd 2014
India can be removed from the office like manner and on
the like grounds as a Judge of the Supreme Court. The Ans. (b) According to Article 165 of the Constitution,

Speaker of the Lok Sabha can be removed from office the Governor of each State shall appoint a person who is

only on a resolution of the House passed by a majority of qualified to be appointed a Judge of a High Court to be
all the then members of the House. Advocate General for the State. Further it shall be the
duty of the Advocate General to give advice to the
7. Consider the following statements:
Government of the State upon such legal matters and to
Attorney General of India can
perform such other duties of a legal character.
1. Take part in the proceedings of the Lok Sabha
2. Be a member of a committee of the Lok 10. Who appoints the Advocate General for the

Sabha state?

3. Speak in the Lok Sabha (a) President

4. Vote in the Lok Sabha (b) Prime Minister


Which of the statements given above is/are (c) Governor
correct? (d) Chief Justice of Supreme Court
(a) 1 only (b) 2 and 4 TNPSC (Pre) G.S. 2014
(c) 1, 2 and 3 (d) 1 and 3 only Ans. (c) : Kindly refer the explanation of the above
IAS (Pre) G.S, 2001 question.
Indian Polity & Constitution 161 YCT
28.
Panchayati Raj and Municipalities
1. Which part of the Constitution has the 4. When was the third tier added to Indian
provisions for Panchayati Raj System? Federal System?
(a)IX (b) VI (a) 1990 (b) 1991
(c) III (d) IVA (c) 1992 (d) 1993
UPPCS (Pre) G.S 2020 UPPCS (Pre) G.S 2020
Ans. (a) The Constitution (73rd Amendment) Act, 1992 Ans. (c) Local Governance in India has been formalized
has added a new Part IX (consisting of 16 Articles) and under the Panchayati Raj system since 1992. The
the Eleventh Schedule to the Constitution. The 73rd Panchayati Raj system is a three-tier system with elected
Amendment envisages the Gram Sabha as the bodies at the village, taluk and district levels. The modern
foundation of the Panchayat Raj System to perform system is based in part on traditional panchayat
functions and powers entrusted to it by the State governance in part on the vision of Mahatma Gandhi and
Legislatures. The amendment provides for a three tier in part by the work of various committees to harmonize
Panchayat Raj System at the village, block and district the highly centralized Indian governmental
levels. The basic objective of the Panchayati Raj is to administration with a degree of local autonomy. The
evolve a system of democratic decentralization and result was intended to create greater participation in local
devolution of people with a view to ensuring rapid government by people and more effective
implementation of rural development programmes.
socio-economic progress and justice.
5. Local self-government can be best explained as
2. The fundamental object of Panchayati Raj
an exercise in
system is to ensure which among the following?
(a) Federalism
1. People's participation in development
(b) Democratic decentralisation
2. Political accountability
(c) Administrative delegation
3. Democratic decentralization
(d) Direct democracy
4. Financial mobilization
IAS (Pre) G.S, 2017
Select the correct answer using the code given
Ans. (b) Kindly refer the explanation of the above
below.
question.
(a) 1, 2 and 3 only (b) 2 and 4 only
6. The fundamental objective of Panchayati Raj
(c) 1 and 3 only (d) 1, 2, 3 and 4
System is to ensure which of the following?
IAS (Pre) G.S., 2015
1. People's participation in development
Ans. (c) Kindly refer the explanation of the above
2. Political accountability
question.
3. Democratic decentralization
3. The Panchayati Raj System was adapted to
4. Financial mobilization
(a) Make people aware of politics.
Select the correct answer from the codes given
(b) Decentralise the power of democracy. below
(c) Educate the peasants. (a) 1, 2 and 3 only
(d) None of the above (b) 2 and 4 only
UPPCS (Pre) G.S., 2016 (c) 1 and 3 only
Ans. (b) Kindly refer the explanation of the above (d) 1, 2, 3 and 4
question. UPPSC ACF-RFO Mains (Ist Paper), 2019
Indian Polity & Constitution 162 YCT
Ans. (c) Kindly refer the explanation of the above 11. The Panchayati elections are scheduled in the
question. month of September 2018 in Punjab. Consider
7. Which Article of the Constituion of India gives the following statements about the Panchayati
provisions to set up panchayats? Raj Institutions. Which of them is incorrect?
(a) The term of the office to the Panchayat at
(a) Article 15
every level is 5 years.
(b) Article 25
(b) The direction and control of preparation of
(c) Article 243
electoral roles and conduct of all elections to
(d) Article 42
Panchayats shall be vested in the Election
Maharashtra PSC (Pre) G.S 2011 Commission of India.
Ans. (c) Article 243 under Part IX of the Constitution (c) The State legislature may make provisions
gives provision to set up village panchayats. with respect to all matters relating to elections
8. Which of the following was constituted under to the Panchayats.
the Panchayati Raj system? (d) The 73rd Amendment Act provides for the
(a) Khap Panchayat reservation of not less than one-third of total
(b) Caste Panchayat number of seats for women (including
(c) Gram Panchayat number of seats reserved for women
(d) Jan Panchayat belonging to SC/ST).
(e) None of the above/ More than one of the above Punjab PSC (Pre) G.S 2018
Ans. (b) According to Article 234(E) of the
Ans. (c) Kindly refer the explanation of the above
Constitution, the term of the office of the Panchayat at
question.
every level is 5 years. Hence first statement is correct.
9. Consider the following statements: According to Article 243 (K) of the Constituion,
1. The minimum age prescribed for any person elections for Panchayats shall be conducted by the State
to be a member of Panchayat is 25 years. Election Commission. Hence second statement is
2. A Panchayat reconstituted after premature incorrect. According to Article 243 (K) (4) of the
dissolution continues only for the remainder Constitution, the State legislature may make provisions
period. with respect to all matters relating to elections to the
Which of the statements given above is/are Panchayats. Therefore statement three is correct. The
correct? 73rd Amendment Act, 1992 added Part IX titled “The
(a) 1 only (b) 2 only Panchayats” through which Article 243 (D) was added
(c) Both 1 and 2 (d) Neither 1 nor 2 by which one-third of the total number of seats was
IAS (Pre) G.S. 2016 reserved for women. Further one-third of the seats
reserved for SCs and STs also reserved for women.
Ans. (b) According to Article 243(F) (1) (a) of the
Hence statement fourth is also correct.
Constitution, minimum age for the election as a member of
a Panchayat is 21 years. Hence statement 1 is incorrect. 12. With reference to Panchayats which of the
The members of a reconstituted Panchayat shall enter upon statements is/are correct?
their offices on the date fixed for the reconstitution of the 1. Fifty percent seats shall be reserved for the
Panchayat and shall continue only for the remainder of the women of SC/ST categories
period for which the dissolved panchayat would have 2. Gram sabha is a body consisting of all
continued.Hence statement 2 is correct. persons living in the village
3. There will be a Panchayat at intermediate
10. A person can contest election for Panchayat if
level
he completes the age of
Select the correct answer using the codes given
(a) 25 years (b) 30 years
below:
(c) 21 years (d) 18 years
Codes:
Uttarakhand PCS (Pre) 1st 2014
(a) 1 Only (b) 2 and 3
Ans. (c) Kindly refer the explanation of the above (c) Both 1 and 2 (d) 3 Only
question.
UPPCS (Pre) G.S, 2019
Indian Polity & Constitution 163 YCT
Ans: (d) According to Article 243(D)(3) of the Ans (a) Kindly refer the explanation of the above
Constituion, not less than one third (including the number question.
of seats reserved for women belonging to the Scheduled 17. What is the minimum population below which
Castes and the Scheduled Tribes) of the total number of
at the intermediate level the Panchayats are not
seats to be filled by direct election in every Panchayat shall
constituted as per provisions of the Indian
be reserved for women. Hence statement 1 is incorrect.
Constitution (73rd Amendment Act)?
According to Article 243(b), Gram Sabha means a body
(a) 25Lakh (b) 20Lakh
consisting of persons registered in the electoral rolls
relating to a village comprised within the area of Panchayat (c) 35Lakh (d) 30Lakh
at the village level. Hence statement 2 is also incorrect. UPPCS (Main) G.S IInd 2017
According to Article 243(B)(1), there shall be constituted Ans. (b) According to Article 243(b)(2) of the
in every State, Panchayats at the village, intermediate and Constitution, Panchayats at the intermediate level may
district levels. Hence statement 3 is correct. not be constituted in a State having a population not
13. If a Panchayat is dissolved, elections are to be exceeding twenty lakhs.
held within: 18. The members of the Pachayat Samiti are
(a) 1 month (b) 3 months (a) Nominated by the Block Development
(c) 6 months (d) 1 year Officer
UPPCS (Pre) G.S, 2002 (b) Nominated by the Zila Panchyat Chairman
Ans. (c) Duration of Panchayats is five year. Fresh (c) Directly elected by the people
election to constitute a Panchayat shall be completed
(d) Indirectly elected by the members of the
before the expiry of its term or in case of dissolution,
Village Panchyat
before the expiry of a period of 6 months from the date
of its dissolution. UPPCS (Pre) G.S., 2013

14. Constitutional status has been provided to Ans. (d) Panchayat Samiti also called Taluka Panchayats
Panchayats- or Block Panchayats is the intermediate level in Panchayati
(a) Under the Article 226 Raj Institutions. The Panchayat Samiti acts as the link
(b) Under the Article 243 between Gram Panchayat (Village) and Zilla Panchayat
(c) Under the Article 239 (District). These blocks do not hold elections for the
(d) Under the Article 219 Panchayat Samiti council seats. Rather, the block council
consists of all of the Sarpanchas and the Upa Sarpanchas
Rajasthan (RAS/RTS) (Pre), 2006
from each Gram Panchayat along with members of the
Ans. (b) Panchayats have been provided Constitutional
legislative assembly (MLA), Members of Parliament
status by the 73rd Constituion Amendment Act, 1992
(MPs), associate members (like a representative from a
under Article 243 of the Constitution. Several
provisions for Panchayats have been made under Part cooperative society) and members from the Zilla Parishad
IX (Article 243 to 243I) of the Constitution. who are a part of the block. The Gram Panchayat members
nominate their Sarpanch and Upa Sarpanch amongst their
15. Article________ was inserted into Constituion
ranks, which extend to the selection of the Chairperson and
under the 73rd Constituional Amendment.
(a) 245 B (b) 244 B Vice-Chairperson as well.
(c) 243 B (d) 242 B 19. Members of Panchayat Samiti are elected-
TNPSC (Pre) G.S. 2019 (a) Directly elected by the public
Ans (c) Kindly refer the explanation of the above (b) Elected by the members of the Panchayat
question. (c) Nominated by the District Magistrate
16. In which part of the Indian Constitution has (d) Appointed by the open competition
the provision for Panchayats been made? Jharkhand PSC (Pre) G.S. 2010
(a) Part IX (b) Part IV IAS (Pre) G.S, 2002
(c) Part III (d) Part X Ans. (b) Kindly refer the explanation of the above
Himanchal PSC (Pre), G.S, 2011 question.
Indian Polity & Constitution 164 YCT
20. In Panchayati Raj system the Panchayat Samiti iv. The meeting of the Gram Sabha can be
is constituted at the adjourned in the absence of Quorum.
(a) Village level (b) Block level v. Quorum is essential in adjourned meeting.
(c) District level (d) State level vi. Budget can be passed in the adjourned
UPPCS (Main) G.S. IInd Paper 2015 meeting where Quorum is not present
Ans. (b) Kindly refer the explanation of the above vii. Audit report can be passed in the adjournment
question. meeting where Quorum is not present
21. A Panchayat Samiti at the block level in India viii. Sarpanch can be dismissed if Quorum is not
is only a/an present in five subsequent meeting of Gram
(a) Advisory Body Sabha
(b) Consultative Committee (a) i ii v
(c) Co-ordinating and Supervisory Authority (b) iii vi viii
(d) Administrative Authority (c) iv vi vii
Ans. (d) Kindly refer the explanation of the above (d) v vi vii
question. (e) vi vii viii
22. The decision to hold Panchayat Elections under Chattisgarh PCS (Pre) G.S., 2015
the Constitutional Provisions is taken by Ans. (d) 1/10 Quorum is essential in the meeting of the
(a) Central Government Gram Sabha. The presence of 1/3 woman is essential in
(b) State Government the meeting of the Gram Sabha. The meeting of the
(c) The Election Commission India Gram Sabha is postponed in case Quorum is not met. If
(d) The State Election Commission meeting of Gram Sabha is postponed continuously for
UPPCS (Main) G.S. IInd 2008 the five times then Sarpanch can be dismissed. Both
Budget and Audit report cannot be passed if Quorum is
BPSC (Pre) 2004
not present.
Ans. (b) The decision to hold Panchayat elections under
the constitutional provisions is taken by State 25. When the Sarpanch of the Gram Panchayat is
Government. The Panchayat elections are conducted by elected indirectly? (According to Chhattisgarh
State Election Commission. Panchayati Raj Act, 1993, Article 13)
(a) When the Sarpanch tenders his resignation
23. The superintendence, direction and control of
all elections to the Panchayats are vested in (b) When the Sarpanch is recalled
(a) The Governor (c) When the Sarpanch is dismissed
(b) The Election Commission of India (d) When the election of Sarpanch is pending
(c) The District Panchayat Raj Officer (e) All the above
(d) The State Election Commission Chattisgarh PSC (Pre) G.S., 2017
UPPCS (Pre) G.S 2020 Ans. (d) According to Article 13 of the Chhattisgarh
Ans. (d) According to Article 243K of the Constituion, Panchayati Raj Act, 1993, Sarpanch of the Gram
The superintendence, direction and control of the Panchayat is elected indirectly if election of the
preparation of electoral rolls for and the conduct of all Sarpanch is scheduled to be held in near future.
elections to the Panchayats shall be vested in a State 26. Which of the following taxes is levied by the
Election Commission consisting of a State Election Gram Panchayats?
Commissioner to be appointed by the Governor.
(a) Sales Tax
24. What is not correct about Gram Sabha? (b) Land Revenue Tax
i. 1/10 Quorum is essential in the meeting of the (c) Tax on Local
Gram Sabha
(d) None of the above
ii. The presence of 1/3 woman is essential in the
UPPCS (Pre) G.S, 2018
meeting of the Gram Sabha
Ans: (c) Taxes on local fairs is levied by the Gram
iii. Sarpanch and Panch are responsible for
Panchayats.
Quorum.
Indian Polity & Constitution 165 YCT
27. Consider the following statements: independent elected bodies at the village, taluk and
1. Part IX of the Constitution of India contains district levels. Hence first statement is incorrect. Rest of
provisions for Panchayats and was inserted by the given options are correct.
the Constitution (73rd Amendment) Act, 1992 30. Who among the following is authorised to
2. Part IX A of the Constitution of India make provisions with respect to the
contains provisions for municipalities and the composition of Panchayats?
Article 243 Q envisages two types of (a) Governor of State
municipalities - Municipal Council and a (b) Legislature of State
Municipal Corporation for every state. (c) Parliament of India
Which of the statements given above is/are (d) President of India
correct? UPPCS (Pre) G.S., 2016
(a) 1 only (b) 2 only
Ans. (b) According to Article 243(C)(1) of the
(c) Both 1 and 2 (d) Neither 1 nor 2 Constitution, Legislature of a State by law make
IAS (Pre) G.S., 2005 provisions with respect to the composition of the
Ans. (a) The Constitution (73rd Amendment) Act, 1992 Panchayats.
has added a new part IX and the Eleventh Schedule to the 31. How many subjects have been earmarked for
Constitution. The 73th Amendment envisages the the Panchayats in the Eleventh Schedule of the
foundation of the Panchayat Raj System. Hence Constitution?
statement 1 is correct. Article 243Q of the Constitution is (a) 27 (b) 28
regarding the constitution of municipalities. It asserts that
(c) 29 (d) 30
there shall be three type of municipalities namely Nagar
UPPCS (Pre) Opt. Pol. Science 2010
Panchayat for a transitional area (an area in transition
from a rural area to an urban area), Municipal Council for Ans. (c) Eleventh Schedule was added by the
a smaller urban area and Municipal Corporation for a Constitution (Seventy-Third Amendement) Act, 1992.
larger urban area. Hence statement 2 is incorrect. Eleventh Schedule contains 29 subjects.

28. Which part of the Constitution has the 32. Which one of the following was NOT proposed
provisions for Panchayati Raj System? by the 73rd Constitutional Amendment in the
area of Panchayati Raj?
(a) IX (b) VI
(a) Thirty per cent seats in all elected rural local
(c) III (d) IV (a)
bodies will be reserved for women candidates
UPPCS (Pre) G.S., 2016
at all levels
Ans. (a) Kindly refer the explanation of the above
(b) The States will constitute their Finance
question.
Commsission to allot resources to Panchayati
29. Which one of the following is incorrect in Raj Institutions
respect of Local Government in India? (c) The Panchayati Raj elected functionaries will
(a) According to the Indian Constitution, local be disquailified to hold their offices if they
government is not an independent tier in the have more than two children
federal system (d) The elections will be held in six months time
(b) 30% of the seats in local bodies are reserved if Panchayati Raj bodies are superceded or
for women dissolved by the State government
(c) Local government finances are to be provided IAS (Pre) G.S.,1997
by a Commission
Ans. (c) The Panchayati Raj elected functionaries will be
(d) Elections to local bodies are to be determined
disqualified to hold their offices if they have more than
by a Commission
two children was not proposed in 73rd Constitution
IAS (Pre) G.S., 1995 Amendment, 1992, regarding Panchayati Raj. Later after
Ans. (a) Local Governance in India has been formalized 1991 census, National Development Council set up a
under the Panchayati Raj System since 1992. The committee on population which recommended legislation
Panchayati Raj system is a three-tier system with in Parliament prohibiting persons with more than two
Indian Polity & Constitution 166 YCT
children from holding any post from Parliament to Ans. (c) On the basis of the report of the Bhuria
Panchayats. A number of States have adopted this norm Committee submitted in 1995, the Parliament enacted the
for elected Panchayats, urban local bodies, cooperatives. Panchayats (Extension to Scheduled Areas) Act, 1996
Rajasthan was the first State to implement such (PESA). This is a legislation which extends the
recommendations. Rest of the given options were part of
provisions of Panchayats to the Fifth Schedule
the 73rd Constitutional Amendment.
Areas. Fifth Schedule areas exist in 10 States namely
33. Under the Scheduled Tribes and Other Andhra Pradesh, Chhattisgarh, Gujarat, Himachal
Traditional Forest Dwellers (Recognition of
Pradesh, Jharkhand, Madhya Pradesh, Maharashtra,
Forest Rights) Act, 2006, who shall be the
Odisha, Rajasthan and Telangana. The salient feature of
authority to initiate the process for
the Panchayats (Extension to the Scheduled Areas) Act,
determining the nature and extent of individual
1996 (PESA) are legislation on Panchayats shall be in
or community forest rights or both?
conformity with the customary law, social and religious
(a) State Forest Department
practices and traditional management practices of
(b) District Collector/Deputy Commissioner
community resources, habitation or a group of habitations
(c) Tahsildar/Block Development Officer/
or a hamlet or a group of hamlets comprising a
Mandai Revenue Officer
community and managing its affairs in accordance with
(d) Gram Sabha
traditions and customs and shall have a separate Gram
IAS (Pre) G.S., 2013
Sabha. Every Gram Sabha has to safeguard and preserve
Ans. (d) Scheduled Tribes and Other Traditional Forest the traditions and customs of people, their cultural
Dwellers (Recognition of Forest Rights) Act, 2006 is an identity, community resources and the customary mode
Act to recognize and vest the forest rights and occupation
of dispute resolution. The Gram Sabha have roles and
in forest land in forest dwelling scheduled tribes and
responsibilities in approving all development works in
other traditional forest dwellers who have been residing
the village, identify beneficiaries, issue certificates of
in such forests for generations but whose rights could not
utilization of funds, powers to control institutions and
be recorded. Whereas the recognized rights of the forest
functionaries in all social sectors and local plan. Gram
dwelling scheduled tribes and other traditional forest
Sabha or Panchayats at appropriate level shall also have
dwellers include the responsibilities and authority for
powers to manage minor water bodies, power of
sustainable use, conservation of biodiversity and
mandatory consultation in matters of land acquisition,
maintenance of ecological balance and thereby
strengthening the conservation regime of the forests resettlement and rehabilitation and prospecting
while ensuring livelihood and food security of the forest licenses/mining leases for ‘minor’ minerals, power to
dwelling scheduled tribes and other traditional forest prevent alienation of land and restore alienated land,
dwellers. As per the provisions provided by the Act, the regulate and restrict sale/consumption of liquor, manage
forest land to be diverted for the development purposes village markets, control money lending to schedule tribes
should be less than one hectare and the clearance of such and ownership of minor forest produce. Only four States
developmental projects shall be subject to the condition have framed their rules for implementation of PESA.
that the same is recommended by the Gram Sabha. They are, Andhra Pradesh, Himachal Pradesh,
34. The Government enacted the Panchayat Maharashtra and Rajasthan.
Extension to Scheduled Areas (PESA) Act in 35. Consider the following statements:
1996. Which one of the following is not (1) The provisions of Part IX of the Constitution
identified as its objective? relating to the Panchayats are not applicable
(a) To provide self-governance to the Fifth Schedule Areas.
(b) To recognize traditional rights (2) At present seven states have Fifth Schedule
(c) To create autonomous regions in tribal areas Areas.
(d) To free tribal people from exploitation (3) The Parliament has enacted the PESA Act in
IAS (Pre) 2000 1996 for such areas.
Indian Polity & Constitution 167 YCT
Which of the above statements is/are correct? 37. Reservation of seats in Panchayat elections for
(a) 1 and 2 (b) 1 and 3 Scheduled Castes shall not apply to the State of
(c) Only 1 (d) All of the above (a) Uttar Pradesh (b) Assam
Maharashtra PSC (Pre) G.S 2018 (c) Arunachal Pradesh (d) Haryana
Ans. (b) Article 244 (1) of the Constitution defines Fifth UPPCS (Pre) G.S., 2016
Schedule Area as such area as the President may decrare Ans. (c) According to the Constitution (Eighty- Third
to be Schedule Area after consultation with the Governor Amendment) Act, 2000, since no scheduled castes exist
of that State under the criteria; preponderance of tribal in the State of Arunachal Pradesh, it is proposed to insert
population compactness and reasonable size of the area a new clause (3A) in Article 243M of the Constitution to
and economic backwardness of the area as compared to exempt the State of Arunachal Pradesh from the
the neighbouring areas. The provisions of Part IX of the application of Article 243D relating to the reservation of
Constituion relating to the Panchayats are not applicable seats in Panchayats for the Scheduled Castes.
to the Fifth Schedule Area. However the Panchayats 38. The number of seats reserved for woman in
(Extension to the Scheduled Areas) Act, 1996 (PESA) Panchayati Raj Institutions in U.P is
has extended the Part IX of the Constitution to the Fifth (a) One- third of the total seats
Scheduled Area.Hence statement 1 and 3 are correct. At (b) In proportion to the woman’s population
present Fifth Schedule Area exist in ten States. Therefore (c) One- fourth of the total seats
statement 2 is incorrect. (d) As per the requirement of the circumstances
36. In the areas covered under the Panchayat UPPCS (Pre) G.S., 2016
(Extension to the Scheduled Areas) Act, 1996, Ans. (a) According to Article 243(D) of the
what is the role/power of Gram Sabha? Constitution, not less than one third (including the
1. Gram Sabha has the power to prevent number of seats reserved for women belonging to the
alienation of land in the Scheduled Areas. Scheduled Castes and the Scheduled Tribes) of the total
2. Gram Sabha has the ownership of minor number of seats to be filled by direct election in every
forest produce. Panchayat shall be reserved for women.
3. Recommendation of Gram Sabha is required 39. Consider the following sources of Revenue of
for granting prospecting licence or mining the Panchayats:
lease for any mineral in the Scheduled Areas. 1. Local Authority Grant by Finance
Which of the statements given above is/are Commission
correct? 2. Assistance by Central Cooperative Banks
(a) 1 only (b) 1 and 2 only 3. Allocations for centrally sponsored schemes
(c) 2 and 3 only (d) 1, 2 and 3 4. NABARD
IAS (Pre) G.S., 2012 5. Allocation from State Finance Commission
Ans. (b) As per the provisions of the Panchayats Of the above, the correct sources of Finance for
(Extension to the Scheduled Areas) Act, 1996, every Panchayats would be
Gram Sabha has the power to prevent alienation of land (a) 1, 2, 3 and 5 (b) only 1 and 2
in the scheduled areas and to take appropriate action to (c) 1, 2 and 5 (d) 1, 2. 4 and 5
restore any unlawfully alienated land of a scheduled UPPCS (Main) G.S. IInd Panper 2007
tribe. Gram Sabha has the ownership of minor forest Ans. (a) Panchayati Raj Institutions receive funds by
produce. The recommendations of the Gram Sabha or the grants from the Union Government based on the
the Panchayats at the appropriate level shall be made recommendations of the Union Finance Commission,
mandatory prior to grant of prospecting licence or grants/loans by the State Government based on the
mining lease for ‘minor’ minerals (all nontimber forest recommendations of the State Finance Commission,
produce of plant origin including bamboo, brush wood, assistance by central cooperative banks and programme-
stumps, cane, tussar, cocoons, honey-wax, lac, tendu specific allocation under centrally sponsored schemes.
leaves, medicinal plants, herbs, roots, tubers, etc.) in the National Bank for Agriculture and Rural Development
scheduled areas. Hence statement 3 is incorrect. (NABARD) provides credits to boost rular economy.

Indian Polity & Constitution 168 YCT


40. Who presides the meeting of Gram Sabha in (a) Only III (b) Only II and III
Schedule Areas? (c) II, III and IV (d) I, II , III and IV
(a) Sarpanch (e) None of the above
(b) Deputy Sarpanch BPSC (Pre) G.S., 2018
(c) Sarpanch or Deputy Sarpanch Ans. (d) Currently 20 States in India provide 50%
(d) Schedule Tribe member present reservation for woman in Panchayati Raj Institutions.
These States are Assam, Andhra Pradesh, Bihar,
Chattisgarh PCS (Pre) 2015
Chhattisgarh, Gujarat, Himachal Pradesh, Jharkhand,
Ans. (d) The meeting of Gram Sabha in scheduled areas
Kerala, Karnataka, Madhya Pradesh, Maharashtra,
are presided by the schedule tribe member present at Odisha, Rajasthan, Sikkim, Tamil Nadu, Telangana,
that particular meeting. Tripura, Uttarakhand, West Bengal and Punjab.
41. Which of the following is related to the theme 45. Which one of the following functions is not the
of ‘Grass Root Democracy’? concern of Local Self-Government?
(a) Lokpal (a) Public Health
(b) Panchayati Raj System (b) Sanitation
(c) Inter-State Council (c) Public Utility Services
(d) Regional Politics (d) Maintenance of Public Order
UPPCS (Main) G.S. IInd Paper 2013 (e) None of the above
Ans. (b) Grass root democracy has been established in BPSC (Pre) G.S., 2017
India through 73rd Amendment of the Constitution Ans. (d) Maintenance of Public Order is State subject.
which is related to the Panchayati Raj Institutions. Rest of the given options are subjects related to
Panchayats.
42. Nyaya Panchayats in Panchayati Raj system
46. Which one of the following is not concerned
have no powers of awarding imprisonment
with the Panchayats?
except in the State of-
(a) The State Election Commission will conduct
(a) Bihar (b) Jammu and Kashmir
Panchayat elections
(c) Maharashtra (d) West Bengal
(b) The Constitution (74th Amendment) Act
UPPCS (Main) G.S. IInd Paper 2005 (c) There shall be a fixed five year term for all
Ans. (a) Under the Bihar Panchayati Raj Act, 2006, the the Panchayats.
Gram Courts have power of awarding imprisonment. (d) Fresh elections would have to be held within
43. Which one of the following Committees/Com- six months of the dissolution of a Panchayat.
missions recommended the creation of Nyaya UPPCS (Main) G.S. IInd Paper 2010
Panchayats? Ans. (b) The Constitution (74th Amendment) Act gave
(a) Balwant Rai Mehta Committee Constitutional status to Municipalities. Rest of the given
(b) Ashok Mehta Committee options are related to Panchayats.
(c) G.V.K. Rao Committee 47. The system of Panchayati Raj is mentioned in
(d) Sarkaria Commission (a) The Union List
(e) None of the above (b) The State List
(c) the Concurrent List
BPSC (Pre) G.S., 2018
(d) None of the above
Uttarakhand-PSC (Pre) 2016
(e) None of the above
Ans. (b) In 1978, Ashok Mehta Committee recomm-
BPSC (Pre) G.S., 2018
ended the creation of Nyay Panchayats.
Ans. (b) Panchayats have been provided Constitutional
44. 50% reservation for women Panchayati Raj status under Article 243 of the Constitution. Several
Institutions is legalized in certain States such as provisions for Panchayats have been made under Part
I. Bihar II. Uttarakhand IX (Article 243 to 243I) of the Constitution. The system
III. Madhya Pradesh IV. Himachal Pradesh of Panchayati Raj is mentioned in the State List.
Indian Polity & Constitution 169 YCT
48. Which one of the following Articles provides Ans. (d) According to the Chhattisgarh Panchayat Raj
reservation of seats for the Scheduled Castes Act, 1993, in the event that the Sarpanch of Gram
and Scheduled Tribes in the Panchayats? Panchayat/ President of Zilla Panchayat is suspended
,the Secretary or the Chief Executive Officer of the
(a) 243 (D) (b) 243 (C)
concerned Panchayat shall cause to be called a special
(c) 243 (B) (d) 243 (A) meeting of the Panchayat immediately (but not later
Uttarakhand-PSC (Pre)-2016 than fifteen days from the date of receipt of information
Chhattisgarh PSC (Pre) 2016 from prescribed authority) and the members shall elect
from amongst themselves, a person to hold the office of
Ans. (a) Article 243 (D) of the Constitution provides
Sarpanch or President temporarily and such officiating
reservation of seats for the Scheduled Castes and
Sarpanch or President shall perform all the duties and
Scheduled Tribes in the Panchayats. 243(A) is regarding exercise all the powers of Sarpanch or President during
power and functions of the Gram Panchayats, 243 (B) is the period for which such suspension continues.
about Constitution of the Panchayats and 234 (C) is 52. If a candidate expired before polling in
regarding composition of the Panchayats. Panchayat election, then what shall be
49. The tenure of every Panchayat in India is five happened?
years from- i. Polling shall be postponed
(a) The scheduled date of its first meeting ii. Polling shall not be postponed
iii. If remained only one candidate, then polling
(b) The date of notification of election
will be postponed
(c) The declaration of the election results iv. If remained more than one, then polling will
(d) The date of oath of office by members not be postponed
UP Lower (M) G.S., 2013 (a) i.ii (b) ii, iii
Ans. (a) According to Article 243 E of the Constitution, (c) iii, iv (d) ii, iv
every Panchayat, unless sooner dissolved, shall continue (e) None of the above
for five years from the date appointed for its first Chhattisgarh PSC (Pre) G.S., 2017
meeting. Ans. (c) If a candidate expired before polling in
Panchayat election and if only one candidate remained
50. State Election Commissioner for the election of
for election, polling will be postponed. Whereas if there
Panchayat would be appointed by- is more than one candidate, polling will not postponed
(a) Chief Election Commissioner of India and process of election would continue.
(b) President of India 53. In which of the following States Panchayati Raj
(c) Governor of the State was introduced first?
(d) None of the above (a) U.P (b) Bihar
UP Lower (M) G.S., 2013 (c) Rajasthan (d) Gujarat
Punjab PSC (Pre) G.S 2015
Ans. (c) According to Article 243 K of the
IAS (Pre) G.S., 1998
Constitution, all elections to the Panchayats shall be
UPPCS (Pre) G.S., 2012
vested in a State Election Commission consisting of a
UPPCS (Main) G.S., 2003
State Election Commissioner to be appointed by the BPSC (Pre) 1999
Governor of the State. RAS/RTS (Pre) G.S., 1996
51. If the President of the Zilla Panchayat is Uttarakhand PCS (Pre) 2006
suspended, what will be happened? Ans. (c) In 1957, the Balwant Rai Mehta Committee
(a) The Vice President of the Zilla Panchayat recommended for the introduction of a three-tier
will be act as acting President Panchayati Raj System in India. Rajasthan has the
distinction of being a pioneer in accepting the scheme of
(b) The Vice President will become President
democratic decentralization called Panchayati Raj
(c) The office of the President will be vacant envisaging a three-tier system of representative bodies at
(d) The members will elect President temporarily the village, block and district levels. Panchayati Raj was
(e) None of the above inaugurated by the then Prime Minister Pandit Jawaharlal
Chattisgarh PSC (Pre) G.S., 2017 Nehru on 2nd October 1959 at Nagour in Rajasthan.
Indian Polity & Constitution 170 YCT
54. Balwant Rai Mehta committee in its report Ans. (c) Balwant Rai Mehta Committee - 1957
suggested Ashok Mehta Committee - 1977
(a) Three tier system (b) One tier system G.V.K. Rao Committee - 1985
(c) Four tier system (d) Two tier system L.M. Singhvi Committee - 1986
TNPSC (Pre) G.S. 2019
59. Who is considered as the “Architect of
Ans. (a) : Kindly refer the explanation of the above Panchayati Raj” in India?
question. (a) Acharya Narendra Deo
55. Which State first adopted the panchayati Raj (b) G.V.K. Rao
in India in 1959? (c) B.R. Mehta
(a) Rajasthan (b) Kanataka (d) L.M. Singhavi
(c) Gujrat (d) Panjab
UPPCS (Pre) G.S., 2016
Himanchal PSC (Pre) G.S. 2019
Ans. (c) Balwantrai Mehta, the second Chief Minister
Ans. (a) : Kindly refer the explanation of the above of Gujarat is considered as the 'Architect of Panchayati
question. Raj' in India.
56. Balwant Rai Mehta Committee is related to: 60. Which of the following Committee
(a) Panchayati Raj System recommended for according a constitutional
(b) Centre-State relations Position to Panchayati Raj?
(c) Rural Development (a) L. M. Singhvi Committee
(d) Electoral Reforms (b) Balwant Rai Mehta Committee
Himanchal PSC (Pre) G.S, 2010 (c) Rao Committee
Ans. (a) Kindly refer the explanation of the above (d) Ashok Mehta Committee
question. UPPCS (Main) G.S. IInd Paper 2008
57. Arrange the folowing committees on Ans. (a) In 1986 L.M. Singhvi Committee recommends
Panchayati Raj in chronological order- that the Pnachayati Raj institution should be
(a) L.M. Singhvi Committee constiutionally recognised, protected and preserved. For
(b) B.R. Mehta Committee this purpose a new chapter should be added in the
(c) Ashok Mehta Committee constiution.
(d) Thungan Committee Basically It was primarily suggested by Ashok Mehata
Select the correct answer from the codes given Committee in 1977 but after the collapse of the Janta
below- Government it lead to no action on its
Codes - recommendations.
(a) 1, 3, 4 and 2 61. Constitution Status to the Panchyat Raj
(b) 3, 4, 1 and 2 Institution was initiated based on the
(c) 4, 3, 2 and 1 recommendation of.
(d) 2, 3, 1 and 4 (a) G.V. K. Rao Committee
UPPSC ACF-RFO Mains (IInd Paper), 2019 (b) L.M. Singhvi Committee
Ans. (c) Balwant Rai Mehta Committee - 1957 (c) Ashok Mehta Committee
Ashok Mehta Committee - 1977 (d) Balwant Rai Mehta Committee
L.M. Singhvi Committee - 1986 TNPSC (Pre) G.S. 2017
P.K.Thungan Committee - 1989 Ans. (b) : Kindly refer the explanation of the above
58. Arrange these committees related to question.
Panchayati Raj in chronological order: 62. Which of the following are the
1. Rao Committee recommendation/recommendations of the L.M.
2. L.M. Singhvi Committee Singhvi Committee?
3. B.R. Mehta Committee (I) Nyaya Panchayats should be created for a
4. Ashok Mehta Committee cluster of villages
Select the correct answer using given codes: (II) Consitutional recognition for Panchayati Raj
(a) 2, 3, 1 and 4 (b) 1, 3, 4 and 2 institutions
(c) 3, 4, 1 and 2 (d) 4, 3, 2 and 1 (III) More financial resources for village
UPPCS (Main) 2017 G.S. IInd Paper panchayts
Indian Polity & Constitution 171 YCT
(a) II & III (b) I & III Ans. (d) Article 243 ZE of the Constitution is regarding
(c) I & II (d) All of these committee for metropolitan planning. Article 243ZE (3)
Himanchal PSC (Pre) G.S. 2019 asserts that every metropolitan planning committee shall
prepare draft development plans for metropolitan area.
Ans. (d) : In 1986, Rajiv Gandhi Government appointed
This article further asserts that the metropolitan planning
a committee on the ‘Revitalisation of Panchayati Raj
committee has sole responsibility to implement
institutions for democracy and development’ under the
government sponsored schemes in metropolitan area.
chairmanship of L.M.Singhvi. L.M.Singhvi committee
Hence all the statements given are correct.
recommended providing more financial resources and
66. In which one of the following areas does the
constitutional status to the panchayats to strengthen
State Government NOT have control over its
them. The Nyaya Panchayat should be estbilised for a
local bodies?
cluster of villages.
(a) Citizen's grievances (b) Financial matters
63. Which committee was appointed in 1986 to
(c) Legisation (d) Personal matters
deal with ‘Revitalisation of Panchayati Raj
IAS (Pre) G.S., 2001
institutions for democracy and development’?
Ans. (a) State Government controls the functioning of
(a) Ashok Mehta Committee
the local bodies in the areas related to legislation,
(b) G.V.K.Rao Committee
financial, personnel etc. Citizen’s grievances are
(c) L.M.Singhvi Committee monitored and addressed by the local bodies.
(d) Santhanam Committee 67. What is correct about the tenure of municipal
TNPSC (Pre) G.S. 2019 council?
Ans: (c) Kindly refer the explanation of the above (i) The tenure of municipal council is counted
question. from the date of declaration of election result.
64. National Panchayati Raj Day in India is (ii) The tenure of municipal council is counted
celebrated on from the date of its first meeting.
(a) 26 January (b) 2 October (iii) The tenure of municipal council is 5 years.
(c) 21 April (d) 24 April (iv) If municipal council is dissolved in two years,
UPPCS (Pre) G.S, 2019 then new municipal council will be organised
th for three years.
Ans: (d) In India, 24 April is celebrated as National
(v) If municipal council is dissolved in two years,
Panchayati Raj Day every year. The day marks the
then new municipal council will be organised
passing of the Constitution Act (73rd Amendment),
for five years.
1992, which came into existence with effect from 24th
(a) (i) (iii) (v)
April, 1993.
(b) (ii) (iii) (iv)
65. Consider the following statements:
(c) (i) (iii) (iv)
In India, a metropolitan planning committee:
(d) (ii) (iii) (v)
1. Is constituted under the provisions of the (e) None of the above
constitution of India.
Chhattisgarh PSC (Pre) G.S., 2017
2. Prepapares the draft development plans for
Ans. (b) Constitution (Seventy Forth Amendment) Act,
metropolitan area.
1992 has introduced a new Part IXA in the Constitution
3. Has the sole responsibility for implementing
which deals with Municipalities from Article 243P to
government sponsored schemes in the 243ZG. The duration of Municipality has been fixed at
metropolitan area. five years from the date of its first meeting. According
Which of the statements given above is/are to Article 243U(4), a Municipality constituted upon the
correct? dissolution of a Municipality before the expiration of its
(a) 1 and 2 only (b) 2 only duration shall continue only for the remainder of the
(c) 1, 3 only (d) 1, 2 and 3 period for which the dissolved Municipality would have
IAS (Pre) G.S., 2011 continued.

Indian Polity & Constitution 172 YCT


28.
Election Commission
1. Consider the following statements: party symbol and how to classify the resulting parties in
1. The Election Commission of India is a five- terms of national and State parties. The Election
member body. Commission of India has to resolve these disputes
2. Union Ministry of Home Affairs decides the although its decisions can be challenged in the
election schedule for the conduct of both courts.Hence statement 3 is correct.
general elections and bye-elections. 2. Which one of the following is not a faction of
3. Election Commission resolves the disputes Chief Election Commission of India?
relating to splits/mergers of recognised (a) Conduct of elections to the office of the State
political parties. Governor.
Which of the statements given above is/are (b) Conduct of elections to the offices of the
correct? President and Vice-President.
(a) 1 and 2 only (b) 2 only (c) Conduct of elections to parliament
(c) 2 and 3 only (d) 3 only (d) Conduct of elections to the State legislatures.
IAS (Pre) G.S, 2017 TNPSC (Pre) G.S. 2015

Ans. (d) The Election Commission of India was Ans. (a) : Kindly refer the explanation of the above
established in accordance with the Constitution of India question.
on 25th January, 1950. The Election Commission of 3. The Election Commission was converted into
India is an autonomous constitutional authority ‘Three Members Commission’ in the following
responsible for administering Parliament and State year:
Legislatures election processes in India. The Election (a) 1987 (b) 1988
Commission of India administers elections to the Lok (c) 1989 (d) 1990
Sabha, Rajya Sabha, State Legislative Assemblies and UPPCS (Main) G.S. IInd 2006
the offices of the President and Vice President of India. Ans. (c) Kindly refer the explanation of the above
Originally the commission had only one Chief Election question.
Commissioner. It currently consists of Chief Election 4. Which one among the following commissions
Commissioner and two Election Commissioners. For was set up in pursuance of a definite provision
the first time two additional Commissioners were under an Article of the Constitution of India?
th
appointed on 16 October 1989 but they had a very (a) University Grants commission
short tenure till 1st January 1990. On 1st October 1993, (b) National Human Rights Commission
two additional Election Commissioners were appointed. (c) Election Commission
The concept of multi-member Commission has been in (d) Central Vigilance Commission
operation since then with decision making power by IAS (Pre) G.S., 2006
majority vote. Hence statement 1 is incorrect. Election Ans. (c) According to Article 324 (1) of the
Commission of India decides the election schedule for Constitution; superintendence, direction and control of
the conduct of both general elections and bye-elections. the preparation of the electoral rolls and conduct of all
Hence statement 2 is also incorrect. Splits, mergers and elections to the Parliament and to the Legislature of
alliances have frequently disrupted the compositions of every State and of elections to the offices of President
political parties. This has led to a number of disputes and Vice President shall be vested in Election
over which section of a divided party gets to keep the Commission.

Indian Polity & Constitution 173 YCT


5. Election Commission of India is Ans. (d) Kindly refer the explanation of the above
(a) Statutory body question.
(b) Administrative body 9. Consider the following tasks:
(c) Enacted body 1. Superintendence, direction and conduct of
(d) Constitutional body free and fair elections.
Uttarakhand RO/ARO (M) 2016 2. Preparation of electoral rolls for all election
Ans. (d) Kindly refer the explanation of the above to the parliament, State Legislatures and the
question. Office of the President and the Vice
President.
6. Which one of the following functions is not
3. Giving recognition to political parties and
related to the Election Commission?
allotting election symbols to political parties
(a) Direction and control of the preparation of the
and individuals constesting the election.
electoral rolls.
4. Proclamation of final verdict in the case of
(b) Conduct of all elections to the Parliament and
election disputes.
Legislatures of every States.
Which of the above are the functions of the
(c) To conduct the election of the offices of
Election Commission of India?
President and Vice President.
(a) 1, 2 and 3 (b) 2, 3, and 4
(d) To make provision with respect to elections
(c) 1 and 3 (4) 1 ,2 and 4
to Legislatures.
IAS (Pre) G.S., 2003
UPPCS (Main) G.S. IInd Paper 2009
Ans. (a) The functions of the Election Commission of
Ans. (d) Kindly refer the explanation of the above India are superintendence, direction and control of the
question. preparation of the electoral rolls for the conduct of all
7. While deciding any question relating to the elections to the Parliament and to the Legislature of
disqualification of a Member of Parliament, the every State and of elections to the offices of President
President shall obtain the option of and Vice President. Election Commission of India
(a) Chief Justice of India prepares electoral rolls for all above said elections.
(b) Election Commission of India Election Commision recognizes political parties and
(c) Attorney General of India provides them election symbols. Proclamation of final
(d) Speaker of the Lok Sabha verdict in case of election dispute lies with President
which in turn shall obtain the opinion of the Election
UPPCS (Pre) G.S., 2015
Commission and shall act according to such opinion.
Ans. (b) According to Article 103 of the Constitution, if
10. Consider the following statements with
any question arises as to whether a member of either
reference to India :
House of Parliament has become subject to any of the
1. The Chief Election Commissioner and other
disqualifications, the question shall be referred for the
Election Commissioners enjoy equal power
decision of the President and his decision shall be final.
but receive unequal salaries
Before giving any decision on any such question, the
2. The Chief Election Commissioner is entitled
President shall obtain the opinion of the Election
to the same salary as in provided to the judge
Commission and shall act according to such opinion.
of the Supreme Court
8. Who decides disputes regarding
3. The Chief Election Commissoner shall not be
disqualification of Members of Parliament?
removed from his office except in like
(a) The President
manner and on like grounds as a judge of the
(b) The Concerned House Supreme Court
(c) The Election Commission 4. The term of office of the Election
(d) The President in consultation with the Commissioner is five years from the date he
Election Commission assumes his office or till the day he attains the
UPPCS (Main) G.S., IInd, 2006, 2009 age of 62 years, whichever is earlier
Indian Polity & Constitution 174 YCT
Which of these statements are correct? removed from his office except in like manner and on
(a) 1 and 2 (b) 2 and 3 the like grounds as a Judge of the Supreme Court and
(c) 1 and 4 (d) 2 and 4 any other Election Commissioner or a Regional
IAS (Pre) G.S., 2002 Commissioner shall not be removed from office except
on the recommendation of the Chief Election
Ans. (b) According to Article 324 (5) of the
Commissioner. Hence statement (a) is correct.
Constitution, Chief Election Commissioner shall not be
Originally the Election Commission had only a Chief
removed from his office except in like manner and on
Election Commissioner. It currently consists of Chief
the like grounds as a Judge of the Supreme Court and
Election Commissioner and two Election
any other Election Commissioner or a Regional
Commissioners. The concept of multi-member
Commissioner shall not be removed from office except
Commission has been in operation till date with
on the recommendation of the Chief Election
decision making power by majority vote. So Chief
Commissioner. The salary of the Chief Election
Election Commissioner is merely first among equals.
Commissioner and other Commissioners is same as that
Hence both statement (c) and (d) are correct. As per the
of Judge of the Supreme Court. The term of the Election
official answer key provided by the commission,
Commissioner is six years from the day he assumes
statement (b) is wrong but the Chief Election
office or till the day he attains the age of 65 years,
Commissioner enjoys the same status and receives
whichever is earlier.
salary and perks as available to Judges of the Supreme
11. The Election Commissioner can be removed by Court. The Chief Election Commissioner can be
the removed from office only through impeachment by
(a) Chief Election Commissioner Parliament (same as Supreme Court Judges). Hence
(b) Prime Minister statement (b) is also correct.
(c) President on the recommendation of the Chief 13. Study the following statements and select the
Election Commissioner correct answer from the code given below:
(d) Chief Justice of India Assertion (A) The State Election Commission is a
UPPCS (Pre) Re-exam. G.S., 2015 constitutional authority.
Ans. (c) Kindly refer the explanation of the above Reason (R) Elections to rural local bodies are
question. overseen by the Election Commission of India.
12. Which one of the following statements about Codes:
Election Commission of India is false? (a) Both (A) and (R) are true and (R) is correct
(a) Only the Chief Election Commissioner has a explanation of (A).
security to tenure but not to the other Election (b) Both (A) and (R) are true, but (R) is not the
Commissioners. correct explanation of (A).
(b) The Chief Election Commissioner holds the (c) (A) is false, but (R) is true.
position or status of a judge of the Supreme (d) (A) is true, but (R) is false.
Court. UPPCS (Pre) G.S., 2014
(c) The Chief Election Commissioner is merely Ans. (d) Election Commission of India and State
first among equals. Election Commissions are constitutional bodies. Hence
(d) In case of difference of opinion amongst the Assertion A is correct. Superintendent, direction and
Chief Election Commissioner and other control of the preparation of electoral roles and conduct
Election Commissioners, the matter is of all elections to the Gram Panchayat and Zilla
decided by majority. Pachayat shall be vested in the State Election
Maharashtra PSC (Pre) G.S, 2019 Commission consisting of a State Election
Ans. (*) : According to Article 324 (5) of the Commissioner to be appointed by the Governor of the
Constitution, Chief Election Commissioner shall not be State. Hence Reason R is incorrect.
Indian Polity & Constitution 175 YCT
14. Which one of the following is correct in respect Delimitation Commissions have been constituted four
of the commencement of the election process in times in 1952, 1963, 1973 and 2002. Under Article 82 of
India? the Constitution, the Parliament by law enacts a
(a) The recommendation for election is made by Delimitation Act after every census and after the
the government and the notification for commencement of the Act; the Central Government
election is issued by the Election Commission constitutes a Delimitation Commission. The present
(b) The recommendation for election is made by delimitation of constituencies has been done on the basis
the Election Commission and the notification of 2001 census figures under the provisions of
for election is issued by the Home Ministry at Delimitation Act, 2002. The Constitution of India was
the Centre and Home Deparments in the specifically amended in 2002 not to have delimitation of
States constituencies till the first census after 2026. The
(c) The recommendation for election is made by Delimitation Commission in India is a high power body
the Election Commission and the notification whose orders have the force of law and cannot be called
for election is issued by the President and in question before any court. These orders come into
Governors of the States concerned force on a date to be specified by the President of India in
(d) Both the exercises of making a this behalf. The copies of its orders are laid before the
recommendation for election and that of Lok Sabha and the State Legislative Assembly concerned
issuing a notification in respect of it are done but no modifications are permissible therein by them.
by the Election Commission 16. Voting right by the youths at the age of 18
IAS (Pre) G.S., 1995 years was exercised for the first time in the
Ans. (c) As per the Representation of the People Act, General Election of
1951, the President issues notification for the election (a) 1987
on the recommendation of the Election Commission. (b) 1988
15. With reference to the Delimitation (c) 1989
Commission, consider the following statements: (d) 1990
1. The orders of the Delimitation Commission UPPCS (Pre) G.S., 2011
cannot be challenged in a Court of Law.
Ans. (c) The Constitution (Sixty-First Amendment) Act,
2. When the orders of the
Delimitation 1988 proposed to reduce the voting age from 21 years to
Commission are laid before the Lok Sabha or
18 years. By the Constitution (Sixty-First Amendment)
State Legislative Assembly, they cannot
Act, 1988, Article 326 of the Constitution was amended
effect any modifications in the orders.
to reduce the voting age from 21 years to 18 years and
Which of the statements given above is/are implemented on March 18th, 1989.
correct?
17. Who recognises the political parties in India?
(a) 1 only
(a) President of India
(b) 2 only
(b) Election Commission of India
(c) Both 1 and 2
(c) Ministry of Law and Justice
(d) Neither 1 nor 2
(d) Speaker of the Lok Sabha
IAS (Pre) G.S., 2012
UK-PSC (Pre) 2016
Ans. (c) The Delimitation Commission demarcates the
Ans. (b) Election Commission of India recognises the
boundaries of the Parliamentary Constituencies as per
political parties in India.
provisions of the Delimitation Act. In India, such
Indian Polity & Constitution 176 YCT
29.
Finance Commission
1. Finance Commission is constituted after every Codes :
(a) 3 years (b) 2 years (a) only 1
(c) 5 years (d) 4 years (b) 2 and 3
nd
UPPCS (Main) G.S. II Paper 2008 (c) 3 and 4
Ans. (c) According to the Article 280 of the (d) 1, 2 and 4
Constitution, the President at the expiration of every IAS (Pre) G.S., 2010
fifth year or at such earlier time as the President Ans. (b) It is the duty of the Finance Commission to
considers necessary, by order constitute a Finance make recommendations to the President as to—
Commission which shall consist of a Chairman and four (i) The distribution of the net proceeds of taxes
other members to be appointed by the President. between the Union and the States which are to be
2. State Finance Commission is constituted under or may be divided between them and the allocation
the Article of the Constitution of India- between the States of the respective shares of such
(a) According to Article 243(H) proceeds.
(b) According to Article 243(I) (ii) The principles which should govern the grants-in-
(c) According to Article 243(J) aid of the revenues of the States out of the
(d) According to Article 243(K) Consolidated Fund of India.
Uttarakhand UDA (Pre) 2003 (iii) The measures needed to augment the Consolidated
Ans. (b) According to Article 243(I) of the Fund of a State to supplement the resources of the

Constitution, the Governor of a State shall, at the Panchayats in the State on the basis of the

expiration of every fifth year, constitute a State Finance recommendations made by the Finance

Commission to review the financial position of the Commission of the State.

Panchayats and to make recommendations to the (iv) The measures needed to augment the Consolidated
Governor of the State. Fund of a State to supplement the resources of the
Municipalities in the State on the basis of the
3. Consider the following statements
recommendations made by the Finance
The function(s) of the Finance Commission
Commission of the State.
is/are
(v) Any other matter referred to the Commission by the
1. To allow the withdrawal of money out of the
President in the interests of sound finance.
Consolidated Fund of India.
4. With reference to the finance commission of
2. To allocate between the States the shares of
India, which of the following statements is
proceeds of taxes.
correct?
3. To consider applications for grants-in-aid
(a) It encourages the inflow of foreign capital for
from States.
infrastructure development.
4. To supervise and report on whether the Union
(b) It facilities the proper distributor of finances
and state governments are levying taxes in
among the public section undertakings.
accordance will the budgetary provisions
Indian Polity & Constitution 177 YCT
(c) It ensures transparency in financial admini- (c) The Prime Minister of India
stration. (d) The Union Finance Minister
(d) None of the statements (a), (b) and (c) given IAS (Pre) G.S., 2003
above is correct in his context. Ans. (a) According to Article 281 of the Constitution,
IAS (Pre) G.S., 2011 the President shall cause every recommendation made
Ans. (d) Kindly refer the explanation of the above by the Finance Commission together with an

question. explanatory memorandum as to the action taken thereon


to be laid before each House of Parliament.
5. Which of the following statements is correct in
reference to the Finance Commission of India? 8. Which among the following is responsible for
revenue sharing between the State Government
(a) It facilitates the judicial distribution of
and Local Government?
finance among PSU’s.
(a) Chief Minister
(b) It ensures transperancy in financial
(b) Governor
administration
(c) State Finance Commission
(c) It deals with the financial relations between
(d) None of the above
States and Center
UPPCS (Pre) G.S., 2013
(d) It ensures financial prudence of State
Ans. (c) State Finance Commission is mentioned in
Government.
Article 243(I) of the Constitution. The functions of
Himanchal PSC (Pre) G.S 2016
State Finance Commission are-
Ans. (c) : Kindly refer the explanation of the above
(i) The distribution of the net proceeds of the taxes,
question.
duties, tolls and fees liveable by the State between
6. Which one of the following authorities the State and the Panchayats, which may be divided
recommends the principle governing the between them and the allocation between the
grants-in-aid of the revenues to the states out of Panchayats at all levels of their respective shares of
the Consolidated Fund of India? such proceeds.
(a) Finance Commission (ii) The determination of the taxes, duties, tolls and
(b) Inter-State Council fees which may be assigned as or appropriated by
(c) Union Ministry of Finance the Panchayats.
(d) Public Accounts Committee (iii) The grants-in-aid to the Panchayats from the
IAS (Pre) G.S, 2001 Consolidated Fund of the State.

Ans. (a) Kindly refer the explanation of the above 9. The State Finance Commission is a
question. (a) Legal body

7. Who of the following shall cause every (b) Non-statutory body

recommendation made by the Finance (c) Constitutional body

Commission to be laid before each House of (d) None of the above

Parliament? UPPCS (Pre) G.S., 2016

(a) The President of India Ans. (c) Kindly refer the explanation of the above
(b) The Speaker of Lok Sabha question.
Indian Polity & Constitution 178 YCT
30
Comptroller and Auditor
General of India
1. The financial control exercised by the 3. The Comptroller and Auditor General is
Parliament over the executive through appointed by the President of India under
(a) Comptroller and Auditing General of India which Article
(b) The Auditor General (a) Article 162
(c) The Finance Secretary (b) Article 148
(d) The Accountant General (c) Article 153
TNPSC (Pre) G.S. 2019 (d) Article 174
Ans. (a) The financial control exercised by the TNPSC (Pre) G.S. 2019
Parliament over the executive through Comptroller and Ans. (b) Kindly refer the explanation of the above
Auditor General of India. The Comptroller and Auditor question.
General of India, a statutory authority under the
4. Comptroller and Auditor General have been
Constituion of India acts as a watch dog of the
appointed by the President. He can be removed
Parliament and conducts audit to see that the
from his post
expenditure incurred by the Executive is for the purpose
(a) By President
voted by the Parliament and is within the sanctioned
(b) On the recommendation of both the Houses
grants.
of the Parliament
2. The Office of the Comptroller and Auditor
(c) Supreme Court
General of India was created
(d) By the Supreme Court on the recomm-
(a) Through an Act of Parliament
endation of President
(b) By the Constitution
UPPCS (Pre) G.S., 2006
(c) Through a Cabinet Resolution
Ans. (b) Kindly refer the explanation of the above
(d) None of the above
question.
UPPCS (Pre) G.S., 2012
5. The term in office for the Comptroller and
Ans. (b) According to Article 148 of the Constitution,
Auditor General of India is-
there shall be a Comptroller and Auditor General of
(a) 4 Years
India (CAG), who shall be appointed by the President
and shall only be removed from office in like manner (b) 5 Years

and on the like grounds as a Judge of the Supreme (c) 6 Years


Court. Salary and expenses of CAG is Charged (not (d) 7 Years
Voted) to the Consolidated Fund of India. Constituion Ans. (c) The term in office for the Comptroller and
disallow his holding of any other Government office Auditor General of India is 6 years or 65 years
after his term expires. (whichever is earlier).

Indian Polity & Constitution 179 YCT


6. In which year was accounting separated from performance audit reports covering revenue collection
auditing and Comptroller and Auditor and expenditure of Government, separate audit reports
General’s task only remained confined to on the functioning of certain autonomous bodies as
auditing the Government Accounts? provided by legislation, reports on the Financial
(a) 1975 (b) 1977 position of Central and State Governments and reports
(c) 1976 (d) 1981 on the adherence to the Appropriation Acts passed by

UPPCS (Pre) G.S, 2019 Parliament and Legislatures. The CAG also submits the
certified annual accounts of the States known as the
Ans: (c) The CAG was both the audit and accounting
Finance and Appropriation Accounts to the State
authority for the Centre as well as the States. The
Legislatures.
accounting functions were taken away in the case of the
9. In India, other than ensuring that public funds
Centre in 1976 and handed over to the Controller
are used efficiently and for intended purpose,
General of Accounts (the Principal Accounting
what is the importance of the office of the
Authority of the Government of India) while accounts
Comptroller and Auditor General (CAG)?
of the States continue to be compiled by the CAG.
1. CAG exercises exchequer control on behalf
Several entitlements functions in the States were also
of the Parliament when the President of India
transferred from the CAG to State Governments
declares national emergency/financial
between 1976 and 1989.
emergency.
7. Who is called the Gurdian of Public Purse?
2. CAG reports on the execution of projects or
(a) President
programmes by the ministries are discussed
(b) Comptroller and Auditor General
by the Public Accounts Committee.
(c) Parliament
3. Information from CAG reports can be used
(d) Council of Ministers
by investigating agencies to press charges
UPPCS (Main) G.S. IInd Paper 2008 against those who have violated the law while
Ans. (b) Comptroller and Auditor General is called managing public finances.
Gurdian of Public Purse. 4. While dealing with the audit and accounting
8. Comptroller and Auditor General of India is a of government companies, CAG has certain
friend and guide of: judicial powers for prosecuting those who
(a) Select Committee violate the law.
(b) Estimates Committee Which of the statements given above is/are
(c) Prime Minister correct?
(d) Public Accounts Committee (a) 1, 3 and 4 only

Uttarakhand UDA/LDA (M) 2007 (b) 2 only

Ans. (d) Comptroller and Auditor General of India is a (c) 2 and 3 only

friend and guide of Public Accounts Committee. Public (d) 1, 2, 3 and 4


Accounts Committee examines the report of accounts of IAS (Pre) G.S., 2012
the Union Government submitted by the Auditor and Ans. (c) Under Article 148 of the Constitution, the
Comptroller General of India to the President. The Comptroller and Auditor General (CAG) of India is an
Audit Reports of the CAG submitted to the Parliament authority to audit all receipts and expenditures of the
and State Legislatures consist of compliance and
Indian Polity & Constitution 180 YCT
Government of India and State Governments • All receipts into and spending from the
irrespective of national emergency/financial emergency. Consolidated Fund (also called as coffers) of the
Hence statement 1 is incorrect. Public Account Union and State Governments.
Committee examines the report of accounts of the • All transactions relating to the Emergency expenses

Union Government submitted by the Auditor and called Contingency Funds and relating to the
monies of the public held by the Government e.g.
Comptroler General of India. Hence statement 2 is
Postal savings, Vikas Patras (called Public
correct. While dealing with the audit and accounting of
Accounts) at Central as well as State levels.
government companies, if CAG finds out any violation
• All trading, manufacturing, profit and loss
of law, it is duty of the CAG to submit its report to the
accounts, balance sheets and other subsidiary
concerned authority and its reports can be used by
accounts kept in any Government department.
investigating agencies to press charges against those
• All stores and stock accounts of all Government
who have violated the law while managing public
offices and departments.
finances. Hence statement 3 is correct. As per the • Accounts of all Government companies and
Constitution of India, CAG has no judicial power to Corporations e.g. ONGC, SAIL etc.
prosecute. Hence statement 4 is incorrect. • Accounts of all autonomous bodies and authorities
10. Which one of the following duties in NOT receiving Government money e.g. municipal
performed by the Comptroller and Auditor bodies, IIM's, IIT's, State Health societies.
General of India? • Accounts of any body or authority on request of the

(a) To audit and report on all expenditure from President/Governor or on his own initiative.

the Consolidated Fund of India. • The Act also provides for compilation of accounts
of the State Governments from the subsidiary
(b) To audit and report on all expenditure from
accounts maintained by the State Governments.
the Contigency Funds and Public Accounts
11. The ultimate responsibility of taking due action
(c) To audit and report on all trading,
on the comments of the Comptroller and
manufacturing, profit and loss accounts
Auditor General vests with
(d) To control the receipt and issue of public
(a) President of India
money and to ensure that the public revenue
(b) Supreme Court
is lodged in the exchequer
(c) Parliament
IAS (Pre) G.S., 2001 (d) National Development Council
Ans. (d) Articles 149 to 151 of the Constitution UP RO/ARO (Main) G.S., 2014
prescribe the unique role of the CAG. As envisaged in Ans. (c) According to Article 151(1) of the
Article 149 of the Constitution, the Parliament enacted a Constitution, the reports of the Comptroller and Auditor
detailed legislation called the CAG’s Duties, Powers General of India relating to the accounts of the Union
and Conditions Act, 1971, which describes CAG’s shall be submitted to the President, who shall cause
mandate and puts almost every spending, revenue them to be laid before each House of Parliament. So it is
collecting or aid/grant receiving unit of the Central and the ultimate responsibility of the Parliament to take
State Governments under his audit domain. CAG duties action on the comments of the Comptroller and Auditor

are to audit and report upon: General.

Indian Polity & Constitution 181 YCT


31.
Union and State Public Service
Commission
1. Which of the following is correct? Public Service Commission for that group of States and if
Union Public Service Commission is- a resolution to that effect is passed by the House of each
(a) Regulatory Body of those States, Parliament may by law provide for the
(b) Statuary Body appointment of a Joint State Public Service Commission
(c) Established by Parliamentary Ordinance to serve the needs of those States. The Chairman and
(d) Constitutional Body other members of Union Public Service Commission and
Joint Public Service Commission shall be appointed by
UPPCS (Main) G.S. IInd Paper 2014 the President and in the case of a State Commission, by
Ans. (d) According to Article 315 of the Constitution, the Governor of the State.
there shall be a Public Service Commission for the 4. Under which Article of the Constitution a
Union and a Public Service Commission for each State. member of the Public Service Commission can
Hence Union Public Service Commission is a be removed?
Constitutional body.
(a) 315 (b) 316
2. Which one of the following is not a statuary (c) 317 (d) 318
function of UPSC?
(a) To advice the government on the methods of Chhattisgarh PSC (Pre) G.S., 2010
recruitment, promotion and control of public Ans. (c) Article 317 of the Constituion is regarding
services. removal and suspension of a member of a Public
(b) To look after the interest and rights of civil Service Commission.
servants. 5. Who can remove the Chairman of the State
(c) To hear appeals from civil servants and Public Service Commission?
redress their grievances. (a) The President on the recommendation of the
(d) To supervise the functioning of the State Supreme Court
Public Service Commission. (b) The Governor on the recommendation of the
Punjab PSC (Pre) G.S 2015 High Court
Ans. (b) Under Article 320 of the Constitution, the (c) The Chief Justice of the High Court
Public Service Commission is required to be consulted (d) The Governor of the concerned State on his
on all matters relating to recruitment to civil services own
and posts. The functions of the Public Service Maharashtra PSC (Pre) G.S 2013
Commission are- Ans. (a) According to Article 317 (1) of the
• Conduct examinations for appointment to the Constituion, the Chairman or any other member of a
services of the Union. Public Service Commission shall only be removed from
• Direct recruitment by selection through interviews. his office by order of the President on the ground of
• Appointment of officers on promotion / deputation misbehaviour after the Supreme Court, on reference
/ absorption. being made to it by the President has on inquiry held in
• Framing and amendment of recruitment rules for accordance with the procedure prescribed in that behalf
various services and posts under the Government. under article 145 reported that the Chairman or such
• Disciplinary cases relating to different Civil other member, as the case may be, ought on any such
Services. ground to be removed.
• Advising the Government on any matter referred to 6. Union Public Service Commission submits its
the Commission by the President of India. annual report to
3. Choose the correct alternative: (a) Prime Minister
The Chairman of a Joint Public Service (b) President
Commission for two or more States is (c) Speaker of the Lok Sabha
appointed by (d) Home Minister
(a) The Governor of the largest state UP Lower (Pre) G.S., 2013
(b) The President of India UPUDA/LDA Special (Pre) G.S., 2010
(c) A Committee of Governors of all the States Ans. (b) According to the Article 323 of the
Concerned Constitution, it shall be the duty of the Union Public
(d) The President of India on the Service Commission to present annually a report to the
recommendation of the Governors concerned President as to the work done by the Commission and it
RAS/RTS (Pre) G.S., 2016 shall be the duty of a State Commission to present
Ans: (b) According to Article 315(2) of the Constitution, annually a report to the Governor of the State as to the
two or more States may agree that there shall be one work done by the Commission.

Indian Polity & Constitution 182 YCT


32.
National Commission for Schedule
Caste and Schedule Tribes
1. Provision for appointment of National 5. The provisions of the sixth Schedule of Indian
Commission for SC’s and ST’s has been made Constitution are applicable in which of the
in the Constitution under Article following states?
(a) 338 and 338A (b) 337 (a) Tripura (b) Sikkim
(c) 334 (d) 339 (c) Nagaland (d) All of the above
UPPCS (Pre) G.S., 2013 UPPCS (Pre) G.S, 2019
Ans: (a) Kindly refer the explanation of the above
Ans. (a) Article 338 of the Constitution makes
question.
provision for appointment of National Commission for
Schedule Caste. The National Commission for 6. Under which Act the Schedule Caste and
Schedule Tribes Atrocity Removal Law
Scheduled Tribes was constituted on 19th February,
implemented?
2004, consequent to the amendment of Article 338 of
(a) Act 1990 (b) Act 1989
the Constitution and insertion of a new Article 338A by
(c) Act 1992 (d) Act 1991
the Constitution (Eighty-Ninth Amendment) Act, 2003.
MPPSC (Pre) G.S, 2015
2. Under which Article of the Constitution the
Ans. (b) The Scheduled Castes and the Scheduled
Separate Scheduled Tribe Commission was
Tribes (Prevention of Atrocities) Act, 1989 is an Act to
established? prevent the commission of offences of atrocities against
(a) 337 (b) 338 the members of the Scheduled Castes and the Scheduled
(c) 338-A (d) 340 Tribes and to provide for special courts for the trial of
Uttarakhand-PSC (Pre) 2016 such offences.
Ans. (c) Kindly refer the explanation of the above 7. Under which Article Scheduled Caste/
question. Schedule Tribe has granted fundamental,
3. National Commission for Schedule Tribes socio- economic, political and cultural rights?
(a) Article 20 (b) Article 19
recommends that the National Education
(c) Article 18 (d) Article 17
Commission must be setup for every.
MPPSC (Pre) G.S, 2015
(a) One year (b) three year
Ans. (d) Article 17 of the Constitution is regarding
(c) Two year (d) Five year
abolition of untouchability. The enforcement of any
TNPSC (Pre) G.S 2015 disability arising out of untouchability shall be an offence
Ans. (d) : National Commission for Schedule Tribes punishable in accordance with law. Article 17 provides
recommends that the National Education Commission fundamental, socio- economic, political and cultural
must be setup for every five years. rights to Scheduled Caste / Schedule Tribes in India.
4. The Sixth Schedule of the Constitution of India 8. The causes are not atrocity of scheduled caste
deals with the administration of tribal areas of and scheduled tribe according to the report of
which of the following States? National Commission of Scheduled Caste and
Scheduled Tribe in the Year 1990.
(a) Bihar, Chhattisgarh, Goa
(a) Land alienation (b) Bonded labour
(b) Meghalaya, Tripura and Mizoram
(c) Indebtedness (d) Religious causes
(c) Uttarakhand, Manipur, Jharkhand
MPPCS (Pre) G.S, 2015
(d) Nagaland, Arunanchal, Tripura
Ans. (d) According to the report of National
UPPCS (Main) G.S., IInd Paper, 2016 Commission of Schedule Caste and Scheduled Tribes,
Ans. (b) Sixth Schedule of the Constitution of India is 1990; land alienation, bonded labour and indebtedness
regarding administration of tribal areas in the States of are the causes of atrocity but religious causes are not
Assam, Meghalaya, Tripura and Mizoram. one of them.

Indian Polity & Constitution 183 YCT


34.
Miscellaneous
1. Which of the following gave legal basis to extra 3. Chairman of the Union Public Service
ordinary powers exercised by the Government Commission
of India during Lockdown? 4. Comptroller and Auditor General of India
1. Mental Health Care Act, 2017 Choose the correct answer from the codes
given below:
2. Disaster Management Act, 2005
Codes:
3. Enemy Property Act,1968
(a) 1 and 2 only
4. Epidemic Disease Act, 1897 (b) 3 and 4 only
Select the correct answer from the codes given (c) 1,2 and 3 only
below: (d) 3 only

(a) 1,2 and 3 (b) 2, 3 and 4 UPPCS (Pre) G.S 2020

(c) 2 and 4 (d) 1,3 and 4 Ans: (d) The process of removal of Judges of the
Supreme Court and High Courts is mentioned in Article
UPPCS (Pre) G.S. 2020
124(4) of the Constitution which asserts that the Judges
Ans. (c) Besides the some provisions of the
of the Supreme Court and High Courts shall not be
Constitution, the Government of India took recourse to
removed from his office except by an order of the
two laws to enforce national lockdown. The Center took
President passed after an address by each House of
recourse to Article 256 of the Constitution, which
Parliament supported by a majority of the total
stipulates that the Center can give directions on how to
membership of that House and by a majority of not less
implement laws made by Parliament.The Center also
than two third of the members of that House present and
took recourse to two other laws which provide the
voting has been presented to the President in the same
Center and the States the statutory basis for acting
session for such removal on the ground of proved
against the pandemic- the Epidemic Diseases Act, 1897
misbehaviour or incapacity. According to Article 148 of
and the Disaster Management Act, 2005. The Epidemic
Diseases Act, 1897 is an Act to provide for the better the Constitution, there shall be a Comptroller and

prevention of the spread of Dangerous Epidemic Auditor General of India (CAG), who shall be

Diseases. Disaster Management Act, 2005 mandates appointed by the President and shall only be removed

setting up a three-tier Disaster Management Authority from office in like manner and on the like grounds as a
at the National, State and District level to formulate a Judge of the Supreme Court. Article 317 (1) of the
disaster plan for its level. Constitution asserts that the Chairman or any other

2. In the removal of which of the following member of a Public Service Commission shall only be
officials does the Parliament NOT play any removed from his office by order of the President on the
role? ground of misbehavior. Hence Parliament does not play
1. Judges of the High Court any role on the removal of Chairman of the Union
2. Judges of the Supreme Court Public Service Commission.

Indian Polity & Constitution 184 YCT


3. Which Article of the Indian Constitution 5. Consider the following statements
describes the Central Vigilance Commission? 1. According to Constitution of India, A person
(a) Article 268 (b) Article 280 who eligible to vote can be made a minister in a
(c) Article 276 (d) None of the above state for six months even if he/she is not a
UPPCS (Pre) G.S 2020 member of the legislature of that state
Ans: (d) The Central Vigilance Commission (CVC) 2. According to the representation of people act,
was set up by the Government of India in February, 1951, a person convicted of a criminal offence
1964 on the recommendations of the Committee on and sentenced to imprisonment for at least five
Prevention of Corruption headed by Shri K. Santhanam years is permanently disqualified from
to advise and guide Central Government agencies in the contesting an election even after his release
field of vigilance. Central Vigilance Commission is from prison.
conceived to be the apex vigilance institution, free of Which of the above given statements is/are
control from any executive authority, monitoring all correct?
vigilance activity under the Central Government and (a) 1 only (b) 2 only
advising various authorities in Central Government (c) Both 1 and 2 (d) Neither 1 nor 2
organizations in planning, executing, reviewing and IAS (Pre) G.S. 2020
reforming their vigilant work. Consequent upon Ans. (a) A person who eligible to vote can be made a
promulgation of an Ordinance by the President, the minister in a state for six months even if he/she is not a
Central Vigilance Commission has been made a multi member of the legislature of that State provided he/she
member Commission with "statutory status" with effect should be get elected to the House within 6 months of
from 25th August, 1998. The CVC Bill was passed by his/her appointment. Hence statement 1 is correct.
both the houses of Parliament in 2003 and the President According to Section 8 in The Representation of the
gave its assent on September 11, 2003. Thus the Central People Act, 1951; A person convicted of any offence
Vigilance Commission Act 2003 came into effect from and sentenced to imprisonment for not less than two
that date. years [other than any offence referred to in sub-section
4. In the context of India, which one of the (1) or sub-section (2)] shall be disqualified from the
following is the characteristic appropriate for date of such conviction and shall continue to be
bureaucracy? disqualified for a further period of six years since his
(a) An agency for widening the scope of release. Hence statement 2 is not correct.
parliamentary demoracy 6. A constitutional government by definition is a
(b) An agency for strengthening the structure of (a) Government by legislature
federalism (b) Popular government
(c) An agency for facilitating political stability
(c) Multi party government
and economic growth
(d) Limited government
(d) An agency for the implementation of public
policy IAS (Pre) G.S 2020

IAS (Pre) G.S. 2020 Ans. (d) A Constitutional Government is one which is
Ans. (a) While the policies are made by the government organized, controlled and limited by the Constituion of
but their implementation on ground is done by the country and not operated by the views of any
bureaucracy through bureaucrats. These bureaucrats indivisual or any political party but regulated only by
works as a bridge between government and people. the law of the land.
Indian Polity & Constitution 185 YCT
7. Given below are two statements, one is labeled 9. The Lokpal bill was First introduced in the Lok
Sabha in which year?
as Assertion (A) and the other as Reason (R):
(a) 1968 (b) 1967
Assertion (A): Indian federalism is called
(c) 1965 (d) 1964
‘Quasi-federal’.
TNPSC (Pre) G.S 2015
Reason (R): India has an independent judiciary
Ans. (a) : The concept of constitutional ombudsman
with power of Judicial Review.
was first proposed by the then law minister Ashok
Select the correct answer from the codes given
Kumar Sen in Parliament in the early 1960s. The term
below.
lokpal and lokayukta were coined by Dr.L.M.Singhvi
(a) Both A and R are correct and R is the correct as the Indian model of ombudsman for the redressal of
explanation of A public grievances. The Lokpal Bill was introduced in
(b) Both A and R are correct but R is not the Parliament in 1968 but was not passed. Eight attempts
correct explanation of A were made till 2011 to pass the Bill. In 2011, the
(c) A is true, but R is false government formed a Group of Ministers, chaired by
(d) A is false, but R is true Pranab Mukherjee to suggest measures to tackle
corruption and examine the proposal of a Lokpal Bill.
UPPCS (Pre) G.S. 2020
In 2013, Lokpal and Lokayuktas Bill, 2013, were
Ans. (b) Quasi-federalism means an intermediate form passed in both Houses of Parliament. In 2016, Lok
of State between a unitary State and a Federation. Sabha agreed to amend the Lokpal Act and Bill was
sent to Standing Committee for review. The Act allows
Indian Constitution contains both features of a federal
setting up of anti corruption ombudsman called Lokpal
constitution and unitary constitution. Hence it is called
at the Centre and Lokayukta at the State level.
Quasi-Federal. India has an independent judiciary with
The Lokpal will cover all categories of public servants
power of judicial review. Judicial review is the power of
including the Prime Minister but the armed forces do
the Supreme Court and the High Courts to examine the
not come under the ambit of Lokpal.
constitutionality of the Acts of the Parliament and State
In 2019, retired Supreme Court judge Pinaki
Legislatures, executive orders of both Centre and State
Chandra Ghose was appointed as the first Lokpal of
Governments. Hence both A and R are correct but R is
India.
not the correct explanation of A.
10. Which among the following is NOT correctly
8. Creamy layer concept was first introduced in matched?
the case of Bodies Establish
(a) Lakshmikant Sukla Vs State of Uttar Pradesh ment
(b) Indra Sawhney Vs Union of India (a) Inter-state Council – 1990
(c) K.K. Roy Vs State of Tripura (b) National Development – 1954
(d) Jitendra Kumar Vs State of Uttar Pradesh Council
TNPSC (Pre) G.S. 2019 (c) Central Vigilance – 1964

Ans. (b) The term “creamy layer" was first used during commission

the 1992 Supreme Court judgment (Indira Sawhaney Vs (d) National Human Rights – 1993

Union of India) on the Mandal Commission Commission

recommendations asking for 27% reservations for other UPPSC RO/ARO (Pre), 2016
backward classes (OBCs) in Central Government jobs. (Held on 20/09/2020)

Indian Polity & Constitution 186 YCT


Ans: (b) National Development Council was 12. Which of the following statements is/are
th
constituted on August 6 , 1952. Rest is correctly included as provision of the Indian
matched. Constitution?

11. With reference to the Constitution of India, 1. The person to be appointed as Governor of a
State should be a domicile of some other
Prohibitions or limitations or provisions
State of the Indian Union.
contained in ordinary laws cannot act as
2. There should be at least one Muslim judge in
prohibitions or limitations on the constitutional
the Supreme Court so as to ensure its secular
powers under Article 142 it could mean which character.
one of the following? 3. The Chairman of the Public Accounts
(a) The decisions taken by the Election Committee of the parliament should belong to
Commission of India while discharging its a party in the opposition.
duties cannot be chalenged in any court of (a) Only 1

law. (b) Only 2


(c) None of the above
(b) The Supreme Court of India is not
(d) 1 and 2
constrained in the exercise of its powers by
Maharashtra PSC (Pre) G.S, 2019
laws made by the Parliament.
Ans: (c) According to Article 155 of the Constituion,
(c) In the event of grave financial crisis in the
the Governor of a State shall be appointed by the
country, the President of India can declare
President by warrant under his hand and seal. There is
Financial Emergency without the counsel
no constitutional provision regarding the Governor of a
from the Cabinet. State shall be domicile of some other State but this is a
(d) State Legislatures cannot make laws on practice developed over the years and may be treated as
certain matters without the concurrence of emerging conventions of the Indian Constituion. The
Uninon Legislature. Chief Justice of India and the Judges of the Supreme
Court are appointed by the President under clause (2) of
IAS (Pre) G.S, 2019
Article 124 of the Constitution. There is no provision in
Ans: (b) According to Article 142 of the Constituion,
the Constituion regarding Muslim Judge in Supreme
the Supreme Court in the exercise of its jurisdiction Court to ensure its secular character but again this is a
may pass such decree or make such order as is practice developed over the years. The Committee on
necessary for doing complete justice in any cause or Public Accounts is the oldest Parliamentary committee
matter pending before it and any decree so passed or that examines the accounts showing the appropriation of

orders so made shall be enforceable throughout the sums granted by Parliament for expenditure of
Government of India and such other accounts of
territory of India in such manner as may be prescribed
autonomous and semi-autonomous bodies. Since 1967,
by or under any law made by Parliament. Under Article
a convention has been established whereby an eminent
142, Supreme Court of India has powers to pass an
leader from the opposition in Lok Sabha is appointed as
order (under extraordinary circumstances) which its chairperson but it is not a provision which has been
overrides the laws made by the Parliament. provided in the Constituion.

Indian Polity & Constitution 187 YCT


13. Regarding the Sovereignty of India, which of
Ans. (c) The National Human Rights Commission
the following statements are correct?
(NHRC) of India was established on 12th October, 1993.
1. India is member of Commonwealth. The statue under which it is established is the Protection
2. Membership of Commonwealth puts the of Human Rights Act (PHRA), 1993 as amended by the
sovereignty of India at stake Protection of Human Rights (Amendment) Act, 2006.
Select the correct answer from the code given Therefore National Human Rights Commission is a
below- statury body. It is in conformity with the ‘Paris
Code: Principles’ for the promotion and protection of human
(a) Only 1 rights which was endorsed by General Assembly of
(b) Only 2 United Nations in 1991. The National Human Rights
(c) 1 and 2 Commission consists of a Chairperson, four full-time
(d) None of the above members and four deemed members. The Chairperson

UPPCS (Main) 2017 G.S. IInd Paper is a retired Chief Justice of the Supreme Court. Among
the four full time members, one member who is or has
Ans. (a) Commonwealth, also called Common-wealth
been a Judge of the Supreme Court of India, one
of Nations is a free association of Sovereign States
member who is or has been the Chief Justice of a High
comprising United Kingdom and a number of its former
Court, two members from amongst persons having
dependencies, who have chosen to maintain ties of
knowledge of or practical experience in matters relating
friendship and practical cooperation. These nations
to human rights. Deemed members are Chairpersons of
acknowledge the British monarch as symbolic head of
the National Commission for Minorities, National
their association. India is a member of Commonwealth
Commission for Scheduled Castes, National
nations since 1947. The word “Sovereign” means that
Commission for Scheduled Tribes and National
State has the power to legislate on any subject in
Commission for Women.
conformity with constitutional limitations. India is
internally and externally sovereign i.e. externally free
from the control of any foreign power and internally, it
has a Government which is directly elected by the
people and makes laws that govern the people. No
external power can dictate the Government of India. So
the membership of Commonwealth nations does not put
the sovereignty of India at stake. Source: nhrc.nic.in

14. Who amongst the following can be the 15. The National Human Right Commission is a
Chairman of the National Human Rights (a) Constitutional body
Commission? (b) Extra Constitutional body
(a) Any serving judge of the Supreme Court (c) Statutory Body
(b) Any serving judge of the High Court (d) Public-Private Body
(c) A retired Chief Justice of the Supreme Court Himanchal PSC (Pre) G.S. 2019
(d) A retired Chief Justice of High Court
Ans. (c) : Kindly refer the explanation of the above
UPPCS (Main) 2015 G.S. IInd Paper question.

Indian Polity & Constitution 188 YCT


16. Which of the following statements are correct
about the National Human Rights Commission?
(1) The National Human Rights Commission is
an independent body.
(2) It is not constitutional body.
(3) It is a statutory body.
(4) It is a multi-member body.
Select the correct answer from the codes given
Source: nhrc.nic.in
below:
18. The Chairman and Members of the National
Codes:
Human Rights Commission are appointed on
(a) (1) and (2)
(b) (1), (2) and (4) the recommendation of a committee

(c) (1), (2) and (3) consisting of :


(4) (1), (2), (3) and (4) (i) Speaker of the Lok Sabha
Himanchal PSC (Pre) G.S, 2014 (ii) Depty Chairman of the Rajya Sabha
Ans. (d) Kindly refer the explanation of the above
(iii) Leader of opposition in the Lok Sabha
question.
(iv) Leader of opposition in the Rajya Sabha
17. The Chairman of National Human Rights
(v) Prime Minister
Commission is appointed by the President on
the recommendation of a committee of certain (vi) Union Home Minister
persons. Who among the following is/are not Select the correct answer from the codes given
part of that committee?
below:
(1) The Prime Minister
(a) i, iii, v, vi
(2) The Home Minister
(b) i,iii and v
(3) Leader of Opposition party in the Lok Sabha
(4) Leader of the Opposition party in the Rajya (c) i,iii,iv and v
Sabha (d) i,ii,iii,iv,v and vi
(5) The Speaker of the Lok Sabha Maharashtra PSC (Pre) G.S 2014
(6) The Chairman of the Rajya Sabha
Ans. (d) Kindly refer the explanation of the above
(a) Only 2 (b) Only 2 and 4
question.
(c) Only 5 and 6 (d) Only 6
Maharashtra PSC (Pre) G.S 2017 19. Consider the following statements:

Ans. (d) Chairman and other members of the National 1. As per the Right to Education (RTE) Act, to
Human Rights Commission are appointed by the be eligible for appointment as a teacher in a
President on the recommendation of a committee State, a person would be required to possess
chaired by Prime Minister and other members which
the minimum qualification laid down by the
includes Speaker of the Lok Sabha, Union home
concerned State Council of Teacher
minister, leader of opposition in both the Houses of
Parliament and Deputy Chairperson of the Rajya Sabha. Education.

Indian Polity & Constitution 189 YCT


2. As per the RTE Act, for teaching primary 20. The Environmental (Protection) Act was
classes, a candidate is required to pass a enacted in 1986 was last amended in;
Teacher Eligibility Test conducted in (a) 1992
accordance with the National Council of (b) 2001
Teacher Education guidelines. (c) 2011
3. In India, more than 90% of teacher education (d) None of the above
institutions are directly under the State Maharastra PSC (Pre) G.S 2017
Governments. Ans. (d) The aim of the Environment Protection
Which of the statements given above is/are Act, 1986 is to improve and protect the
correct? human environment, most importantly to prevent
(a) 1 and 2 hazards from happening and causing damage to the
(b) 2 only ecology. On November 19th, 1986, the Environment
(c) 1 and 3 Protection Act, 1986 came into force and was extended
(d) 3 only to the whole India. The Environment Protection
IAS (Pre) G.S, 2018 Act of 1986 was last amended in 2015 as Environment
Ans: (b) The Constitution (Eighty-Sixth Amendment) (Protection) (Fifth Amendment) Rules, 2014.
Act, 2002 inserted Article 21-A in the Constitution to 21. The Government of India enacted Wild Life
provide free and compulsory education of all children in (Protection) Act 1972 with the obiective of:
the age group of six to fourteen years as a Fundamental (a) Effectively protecting the protected wildlife
Right. The Right of Children to Free and Compulsory of this country and to control smuggling and
Education (RTE) Act, 2009, represents the to lllegalize trade in wildlife and its
consequential legislation envisaged under Article 21-A derivatives
which means that every child has a right to full time (b) Effectively protecting the wildlife to move
elementary education. Article 21-A and RTE Act came from one place to another into inhabited
into effect on 1 April 2010. In accordance with the
places
provisions of sub-section (1) of section 23 of the Right
(c) Effectively protecting the wildlife of this
of Children to Free and Compulsory Education (RTE)
country and to control poaching, smuggling
Act, 2009, the National Council for Teacher Education
and illegal trade in wildlife and its derivatives
(NCTE) has laid down the minimum qualifications for a
(d) None of the above
person to be eligible for appointment as a teacher for
Haryana PSC (Pre) G.S 2014
class I to VIII. Hence statement 1 is not correct. As per
the RTE Act, for appointment as a teacher in any of the Ans. (c) Wildlife Protection Act 1972 was passed on

school, he/she should pass the Teacher Eligibilty Test August 21, 1972 but was later implemented on

(TET) which will be conducted by the appropriate September 9, 1972.This Act prohibits the capturing,

government in accordance with the guidelines framed killing, poisoning or trapping of wild animals. The
by the NCTE. Hence statement 2 is correct. In India, objectives of Wild Life (Protection) Act, 1972 are
more than 90% of teacher education institutions are prohibition of hunting, protection and management of
directly under the State Governments is not a correct wildlife habitats, establishment of protected areas,
statement as since RTE Act has been implemented; regulation and control of trade in parts and products
situation has changed due to the compulsion of TET. derived from wildlife, management of zoos.

Indian Polity & Constitution 190 YCT


22. Consider the following statements: Ans. (d) A political party shall be treated as a
(1) According to Article 262 of the Indian recognized national political party if and only if the
Constitution, the Parliament may by law
political party fulfills any of the following conditions:
provide for the adjudication of interstate river
water dispute. • The party wins 2% of seats in the Lok Sabha (11

(2) Article 262 empowers the President of India seats) from at least 3 different States.
to set up an interstate river water dispute • At a General Election to Lok Sabha or Legislative
tribunal.
Assembly, the party polls 6% of votes in four
Select the incorrect statements:
States and in addition it wins 4 Lok Sabha seats.
(a) Both 1 and 2
(b) Only 1 • A party gets recognition as State Party in four or

(c) Only 2 more States.


(d) Both 1 and 2 are incorrect For recognition as a State political party:
Maharashtra PSC (Pre) G.S. 2017
• At General Elections or Legislative Assembly
Ans. (c) Article 262 of the Constitution is regarding
elections, the party has won 3% of seats in the
adjudication of disputes relating to waters of inter State
rivers or river valleys which asserts that the Parliament legislative assembly of the State (subject to a

may by law provide for the adjudication of any dispute minimum of 3 seats).
or complaint with respect to the use, distribution or • At a Lok Sabha General Elections, the party has
control of the waters of, or in, any inter State river or
won 1 Lok sabha seat for every 25 Lok Sabha seat
river valley. Hence statement 2 is incorrect. The
allotted for the State.
Parliament has enacted Inter-State River Water Disputes
(Amendment) Bill, 2019 to constitution of a single • At a General Election to Lok Sabha or Legislative
tribunal with different benches and the setting up of Assembly,the party has polled minimum of 6% of
strict timelines for adjudication.
votes in a State and in addition it has won 1 Lok
23. Which of the following is not the essential
Sabha or 2 Legislative Assembly seats.
condition for a political party to be recognized
as a national party? • At a General Election to Lok Sabha or Legislative

(a) If a party gets six percent of the total valid Assembly, the party has polled 8% of votes in a
votes polled in the election for Lok Sabha in State.
four or more States.
24. The number of Judges in the International
(b) If a party sends four elected members to the
Lok Sabha. Court of Justice is

(c) If a party gets two percent of the total number (a) 15


of seats in Lok Sabha from not less than three (b) 10
States.
(c) 11
(d) If a party is recognized as a State Party in at
(d) 14
least two States.
Punjab PSC (Pre) G.S 2018 Punjab PSC (Pre) G.S 2015

Indian Polity & Constitution 191 YCT


Ans. (a) The International Court of Justice (ICJ) is the
Ans. (c) : Planning Commission - March 1950
principal judicial organ of the United Nations. It was
National Development - August 1952
established in June 1945 by the Charter of the United
Zonal Council - 1956
Nations and began work in April 1946. The seat of the

Court is at the Peace Palace in The Hague National Integration Council - October 1961

(Netherlands). Of the six principal organs of the United 26. Arrange the following Committee in

Nations(General Assembly, Security Council, Chronological order.

Economic and Social Council, Trusteeship Council, (a) Gorwala Committee

International Court of Justice and UN Secretariat ), it is (b) Ayyangar Committee


the only one not located in New York (United States of (c) Appleby Report
America). The Court’s role is to settle in accordance
(d) Administrative Reforms Commission
with international law, legal disputes submitted to it by
Code :
States and to give advisory opinions on legal questions
(a) I, III, II, IV
referred to it by authorized United Nations organs and
(b) II, III, I, IV
specialized agencies. The Court is composed of 15
(c) II, I, III, IV
Judges, who are elected for terms of office of nine years
(d) II, I, IV, III
by the United Nations General Assembly and the
TNPSC (Pre) G.S. 2016
Security Council.
Ans. (c) :
25. Arrange the following in ascending order of

their formation. 1. Ayyangar Committee- Criminal

(I) Planning commission Tribes Act enquiry - 1949-50

(II) Zonal Councils 2. Gorwala Committee on proposed

(III) National Integration Council legislation for the regulation of

(IV) National development Council Stock Exchanges and Contracts in

Code : Securities - 1951

(a) I II III IV 3. Appleby Report on public

(b) I IV III II administration in India - 1953

(c) I IV II III Administrative Reforms

(d) I III IV II 4. Commission to examine the public

TNPSC (Pre) G.S. 2016 administration of the country - 1966

Indian Polity & Constitution 192 YCT

You might also like